Sei sulla pagina 1di 561

FACULTY BASED

GOVT. BANK
Solution
2020

Join us
Bank Recruitment
Exam Boost
All Exam Taken by AUST

1. Basic Bank (Assistant Manager)-2018


2. ICB Assistant Programmer-2017
3. Probashi Kallayan Bank SEO- 2018
4. Probashi Kallayan Bank SO -2018
5. Sonali Bank (Assistant Programmer)-2016
6. Sonali Bank (Cash Officer)-2018
7. Karmasangsthan Bank DEO-2018
Please Join Us
8. Janata bank EO (civil)-2017
9. Bangladesh Krishi Bank DEO-2018 Bank
10. Combined 4 Bank SO (IT) - 2018
Recruitment
11. Combined 8 Bank Senior officer-2018
12. Combined 2 bank Officer(General)-2018 Exam Boost
13. Combined 3 Bank Senior Officer-2018
14. Combined 3 Bank Officer(Cash)-2018
ফ োকোস রোইট িং,
15. Combined 5 Bank Engineer Exam 2018 ফে োর রোইট িং,
16. Sonali Bank Officer Cash (FF)-2018 ট্রোন্সলেশন এবিং
17. Sonali Bank Senior Officer (FF)- 2019 ফেলকোলনো বইলের
18. Sonali Bank Officer (FF)- 2019 আপলে কপপ ফপলে
19. Combines 2 Bank Officer (IT) - 2019
Bank Recruitment
20. Combined 5 Bank Officer Cash -2019
Exam Boost গ্রুলপ
21. Combined 8 Bank Senior Officer -2019
জলেন কলরন।
22. Combined 4 Bank Officer -2019
23. Probashi Kallayan Bank Programmer - 2019
24. Probashi Kallayan Bank Assistant Programmer - 2019
25. Janata & Rupali Bank Officer- 2019
26. Janata Bank AEO- 2019
27. Janata Bank officer ( Teller)- 2019
28. Probashi kallyan Bank EO ( Cash)- 2019
29. Probashi kallyan Bank EO- 2019
30. Rupali Bank Senior Officer- 2019
BANGLA
01. ?
(A)
(B)
(C)
(D)
02.
(A)
(B)
(C)
(D)
03. ?
(A)
(B)
(C)
(D)
04. ?
(A)
(B)
(C)
(D)
05. ?
(A)
(B)
(C)
(D)
06. ?
(A)
(B)
(C)
(D)
07. ?
(A)
(B)
(C)
(D)
08. ?
(A)
(B)
(C)
(D)
09. ?
(A)
(B)
(C)
(D)
10. ?
(A)
(B)
(C)
(D)
11. ?
(A)
(B)
(C)
(D)
12. ?
(A)
(B)
(C)
(D)
13. ?
(A)
(B)
(C)
(D)
14. ?
(A)
(B)
(C)
(D)
15. ?
(A)
(B)
(C)
(D)
16.
(A)
(B)
(C)
(D)

ENGLISH
(17-20): Select the pair that best expresses a relationship
SIMILAR to that expressed in the original pair.
17. OSTRICH: BIRD
(A) bud : leaf
(B) tiger : cat
(C) guest : storm
(D) lizard : frog
18. CURDLE: MILK
(A) flow : water
(B) change : oil
(C) brew: coffee
(D) clot: blood
19. CONSCIOUS : CARELESS
(A) careful : ugly
(B) graceful: ugly
(C) generous : unkind
(D) careful : indifferent
20. LIGHT : BLIND
(A) tongue : sound
(B) voice : vibration
(C) speech : dumb
(D) None of them

Q. (21 24): Fill in the blank with right option.


21. Complete shutdown ___ observed today against new law.
(A) is being
(B) can be
(C) is been
(D) was been
22. The officer has stayed away from ____ populism such as raising the income tax exemption limit.
(A) restricted
(B) classified
(C) overt
(D) furtive
23. Mohit was advised to take ______ since he broke his leg in the accident.
(A) medicine
(B) coffee
(C) leftover
(D) rest
24. We must Endeavour to _______ the miseries of poor.
(A) mitigate
(B) elaborate
(C) discourage
(D) Augment

Q. (25 28): Choose the correct word spelling from the given options.
25.
(A) Lassivious
(B) Lascivious
(C) Lacivoius
(D) Lasivious
26.
(A) Velnerable
(B) Vulnarable
(C) Vulnerable
(D) Valnerable
27.
(A) Adulterate
(B) Adeldurate
(C) Adulterat
(D) Adultarate
28.
(A) Dicotomy.
(B) Dicotemy
(C) Dichotomy
(D) Dechotomy

MATHEMATICS
29. In a container, there are 2 green marbles and 2 red marbles. You randomly pick the marbles. What is the probability that both of
them are green?
(A) 1/2
(B) 1/3

(C) ¼
(D) 1/6
Solution:
green ball = 2
Red ball = 2
From 4 balls, possibility of 2 green = 4C2 = 6
From 2 balls, possibility of 1 green = 2C2 = 1.
From rest 2 balls, possibility of being green = 2C2=1
Possibility of 2 green = (2C2 x 2C2)/4C2= (1×1)/6= 1/6
30. To represent a family budget on a circle graph, how many degrees of the circle should be used to represent an item that is 20% of
the total budget?
(A) 76°
(B) 72
(C) 60°
(D) 20°
Solution:
Total circle = 360°
20% used 20% of 360° = (20/100) × 360° = 72°
31. A boat sailing against a stream of river tanks 6 hours to travel 24tm, while sailing with the stream it takes 4 hours to travel the
same distance. What is the speed of the stream?
(A) 2.5 km/hr
(B) 1.5 km/hr
(C) 1 km/hr
(D) 0.5 km/hr
Solution:
Let,
Speed of stream =y &
Speed of boat = x
ATQ,
x-y= 24/6 = 4 (i)
x+y = 24/4 = 6 (ii).
From,
(ii) (i),
x+y-x+y = 6-4
Or, 2y = 2
Or, y = 1
32. A milkman purchases the milk at Tk. x per liter and sells it Tk. 2x per liter still he mixes 2 liters water with every 6 liters of pure
milk. What is the profit percentage?
(A) 116%
(B) 166.66%
(C) 60%
(D) 100%
Solution:
Let,
He purchases 6liters milk.
So, cost of 6 liters = 6x tk.
After mixing 2 liters waters, he sells , (6+2)= 8 liters
Now, selling price of 8 liters = 8 × 2x = 16x tk.
Profit = 16x 6x = 10x tk.
Profit percentage = (10x/6x) × 100 = 166.66%
33. A man could buy a certain number of notebooks for Tk. 300. If each notebook cost is Tk. 5 more, he could have bought 10
notebooks less for the same amount. Find the price of each notebook?
(A) 15
(B) 20
(C) 10
(D)
Solution:
Let,
Cost of 1 Note book = x tk.
ATQ,
300/x = {300/(x+5)} + 10
Or, (300/x) 300/(x+5) = 10
Or, (300x+1500-300x)/{x(x+5)} = 10
Or, x(x+5)10 = 1500
Or, x2 +5x-150 = 0
Or, (x+15)(x-10) =0
Or, x≠ -15, x=10
34. 4 men and 6 women can complete a work in days, while 3 men and 7 women can complete it in 10 days. In how many days will
10 women complete it?
(A) 40 days
(B) 36 days
(C) 32 days
(D) 34 days
Solution:
4men + 6 women complete in 1 day = 1/8 part -(1)
3 men + 7 women complete in 1 day = 1/10 part (2)
From, (1)×3 (2) ×4
12men + 18 women complete in 1 day = 3/8 part -(1-
12 men + 28 women complete in 1 day = 4/10 part (2)
-10 women complete in 1 day = 3/8 4/10 = -1/40
So, 10 women complete in 1 day = 3/8 4/10 = 1/40 part
So, they complete the work in 40 days
35. Two-fifth of one-fourth of three seventh of a number is 15. What is the half of the number?
(A) 75
(B) 157
(C) 175
(D) 57
Solution:
Let,
The number = x
ATQ,
2/5 × ¼ × 3/7 × x =15
Or, 16x/ 140 = 15
Or, x = 350

½ of 350 =175
36. Length of a train is 170 meters and speed of train is 63 km/hour. This train can pass a bridge in 30 seconds, then find the length of
the bridge.
(A) 355m
(B) 325m
(C) 365m
(D) 312m
Solution:
Let,
Length of bridge = x m
ATQ,
63000/(60×80) = (170x+x)/30
Or, x+170 = 105 × 5
Or, x =525- 170
Or, x = 355
37. A water tank has two taps (Tap-1 and Tap 2). Tap-1 can fill a tank in 8 hours and Tap-2 can empty the tank in 16 hours. How long
will they fill the tank if both taps are opened simultaneously but Tap-2 is closed after 8 hours?
(A) 10
(B) 12
(C) 14
(D) 16
Solution:
Tap-1, fills in 1 hr = 1/8
Tap-2, empties in 1 hr = 1/16
When both taps are open, the tank fills in 1 hr = (1/8 1/16) = 1/16
In 8 hrs, the tank fills = 1/16 × ½
Remaining = 1 ½=½
½ is filled by only Tap 1
1/8 is filled by Tap-1 in 1 hr
So, ½ is filled by = (1×8)/2= 4 hr
Total time = 8+4 = 12 hr
38. A cylindrical rod of iron, whose height is equal to its radius, is melted and cast into spherical balls whose radius is half the radius of
the rod. Find the number of balls.
(A) 3
(B) 4
(C) 5
(D) 6
Solution:
Volume of cylinder = πr2h [Here h = r]
= πr2×r = πr3
Volume of sphere = 4/3 πr3 [Here r = r/2)
=(4/3) π× (r/2)3 = (πr3)/6
Number of balls = πr3/( πr3/6)= 6
39. Rahim borrowed Tk. 800 at 6% per annum and Karim borrowed Tk. 600 at 10% per annum. After how much time, will they both
have equal debts?
(A) (50/3)×yr
(B) (83/3)×yr
(C) (44/3)×vr
(D) (20/3)×yr
Solution:
Let,
After x years their debts will be equal.
ATQ,
{800 × (6 × x)/ 100}+800 = {600 × (10 × x)/ 100}+600
⇒ 48x + 800 = 60x + 600
⇒ 60x 48x = 800 600
⇒12x = 200
⇒ x = 50/3
40. The average of 6 numbers is 25. If 3 more numbers, with an average of 22 are added to these numbers, what will be the average of
the combined 9 numbers?
(A) 24
(B) 25
(C) 26
(D) 28
Solution:
Sum of 6 numbers = (6×25) = 150.
Sum of 3 additional numbers = (3 × 22) = 66.
Sum of (6 + 3) =9 numbers = (150+66)=216
.:. average of 9 numbers = 216/9 = 24
41. The difference of two numbers is 1365. On dividing the larger number by the smaller, we get 6 as quotient and the 15 as
remainder. What is the smaller number?
(A) 270
(B) 1270
(C) 350
(D) 720
Solution:
Let,
Smaller number = x,
difference =1365
So, the larger number = (x +1365)
ATQ,
6x +15 = x+ 1365
Or, 5x = 1350
Or, x= 270
42. The product of two positive numbers is p. If each of the number is increased by 2, the new product is how much greater than twice
the sum of the two original number?
(A) p times
(B) 2p times
(B) (p + 4) times
(D) (2p + 3) times
Solution:
Let,
One number is = x; other = y
xy = P (i)
(x + 2) (y + 2) = xy + 2y + 2x + y
xy + 2 (x + y) + 4 = xy + 4 + 2(x + y)
= P + 4 + 2(x + y)
The new result is (P+4) greater.
43. If a > b > 1, then which of the following is true?
(A) a2>b2
(B) a2> ab
(C) b + a> 2a
(D) a b<0
Solution:
Let,
a = 3 & b=2
Option a) a2> b2 = (3)2> (2)2 = 9 > 4 True
Option b) a2 < ab = (3)2 < 3 x 2 =9<6 false
Option c) a b<0= 3-2<0 = 1< 0 false
Option d) b +a > 2a = 2+3> 2 x 3 = 5 > 6 false
44. A and B together can do a piece of work in 40 days. A having worked for 20 days, finishes the remaining work alone in 60 days. In
how many days shall B finish the whole work alone?
(A) 60 days
(B) 70 days
(C) 80 days
(D) 90 days
Solution:

Then,
x + y = 1/40 (1)
20x + 60y = 1 -(2)
From (2) (1)×20,
20x + 60y 20x -20y = 1 ½
Or, 40y = ½
Or, y = 1/80

Hence, B alone finishes the work in 80 days


45. A jar contains white, red and green marbles in the ratios 2:3:5. Six more green marbles are added to the jars, and then the ratio
becomes 2 : 3:7. How many white marbles are there in the jar?
(A) 5
(B) 6
(C) 9
(D) 10
Solution:
Ratio of W : R : G= 2: 3:5
If 6 Green is added, Ratio becomes W: R:G = 2: 3:7
Difference of ratio for 6 marbles = 7 5=2
For 1 ratio = 3 marbles
White marbles = 2×3 = 6
46. The length of a rectangular plot is 3 folds its width. Half the area of the plot is covered by a playground whose area is 363 square
meter. What is the length of the plot?
(A) 18.50m
(B) 13.61m
(C) 17.21m
(D) 15.51m
None
Solution:
Let, width = x
.:. The length of the plot = 3x
ATL,
x × 3x = 363 × 2
⇒3×2 = 363 × 2
⇒ x = 15.56
The length of the plot = 3×15.56 = 46.67 m
47. A box contains 10 electric bulbs from which 2 bulbs are defective. Two bulbs are chosen at random. What is the probability that one
of them is defective?
(A) 3/10
(B) 16/45
(C) 25/68
(D) 8/33
Solution:
Defective 2; Normal
Probability that one of them is defective will be,
= (8C1×2C2)/10C2
= (8×2)/45
= 16/45
48. Three pipes A, B and C can fill a tank in 6 hours. After working at it together for 2 hours, C is closed and A and B can fill the
remaining part in 6 hours. The number of hours taken by C alone to fill the tank is:
(A) 8 hours
(B) 10 hours
(C) 14 hours
(D) 18 hours
Solution:
A + B+C fill the tank in 6 hrs
A+B+C fill in 2 hrs = 2/6 = 1/3
Remaining = 1- 1/3 = 2/3
In 6 hrs A + B fill 2/3
In 1 hr A + B fill 2/3 × 1/6 = 1/9
Now C alone fill in 1 hr = 1/6 1/9 = 1/18
So, C needs 18 hrs to fill the tank.
49. If x=7 4√3 then find the value of x+1/x?
(A) 3√3
(B) 8√3
(C) 14
(D) 14 + 8√3
Solution:
x =7 4√3
1/x = 1/(7 4√3) = (7 + 4√3)/ (7 4√3)( 7 + 4√3) = (7 + 4√3)/49-48 =(7 + 4√3)
x+1/x = 7 4√3 + 7 + 4√3 = 14
50. An employer pays 3 workers X, Y and Z a total of Tk. 36,600 a week. X is paid 125% of the amount Y is paid and 80% of the
amount Z is paid. How much does X make a week?
(A) 9,000
(B) 12,000
(C) 10,800
(D) 11,700
Solution:
X= 125% of y = 125y/100 = 5y/4
Or, y = 4x/5
Again,
X= 80% of z = 80z/100 = 4z/5
Or, z= 5x/4
X:y:z = x:4x/5:5x/4 = 20:16:25
Sum of ratio = 20+16+25=61
So, x makes a week = 20/61 × 36600 = 20×600=12000
51. The volume of a rectangular solid is to be increased by 50% without altering its base. To what extent the height of the solid must be
changed.
(A) 50%
(B) 40%
(C) 30%
(D) 20%
Solution:
Let,
Volume = LBH
ATC,
LBH + 50% of LBH
=LBH + LBH/2
Let, H is to bbe increased by x%
So,
LB(H+Hx/100) = LBH + LBH/2
Or, LBH + LBHx/100 = LBH + LBH/2
Or, LBHx/100 = LBH/2
Or, x/100= ½
Or, x = 50%
52. If is added to the square of a number, the answer so obtained is 16202. What is the 1/26 of that number?
(A) 5.65
(B) 2.70
(C) 3.50
(D) 6.66
Solution:
ATQ,
(89)2 + x2 = 16202
Or, x2 = 16202 (89)2= 16202 7921 = 8281
.::x=√(8281) = ±91
Now, 1/26 of 91 = 3.50

GENERAL KNOWLEDGE
53.
(A) 5
(B) 6
(C) 7
(D) 8
54. Gravity setting chambers are used in industries to remove
(A) NOX
(B) SOX
(C) CO
(D) Suspended particulate matter
55. Brown Waterfalls are situated in:
(A) Australia
(B) New Zealand
(C) Canada
(D) Switzerland
56. The length of the day is determined in
(A) astronomical units
(B) solar terms
(C) length of the hours
(D) none of the above
57. Which of the following is not an E-Banking product?
(A) ATM
(B) SMS banking
(C) Plastic money
(D) School banking
58. Guarantee to an exporter that the importer ill pay immediately for the goods ordered by him, is known as
(A) Letter of Credit
(B) Inflation
(C) Laissez-faire
(D) None of these
59.
(A) Environment
(B) Economics
(C) History
(D) None of these
60. Detroit in USA is known as the city of
(A) Rockets
(B) Motor Cars
(C) Lights
(D) Aero-planes
61. What kind of organization is NATO?
(A) Military
(B) Political
(C) Social
(D) Environmental
62. Which of the following organization of the World Bank is known as soft loan window?
(A) IDA
(B) MIGA
(C) IDB
(D) IFC
63. Which of the following is correct as the name of a reputed organization?

(B) icddr,b
(C) iccddr.b
(D) icddrb
64. Which one is not direct tax?
(A) Income tax
(B) Wealth tax
(C) VAT
(D) All of these
65. The process of removing unwanted part of an image is called
(A) Hiding
(B) Bordering
(C) Cutting
(D) Cropping

COMPUTER
66. Which of the following is not the Section Break Option?
(A) Next Page
(B) Previous Page
(C) Odd Page
(D) Even Page
67. Portrait and Landscape are
(A) Page Orientation
(B) Pager Size
(C) Page Layout
(D) All of above
68. Which of the followings is word processing software?
(A) WordPerfect
(B) WordPad
(C) MS Word
(D) All of above
69. Any data or instruction entered into the memory of a computer is considered as
(A) Storage
(B) Output
(C) Input
(D) Information
70. Graphical pictures that represent an object like file, folders etc, are:
(A) Icons
(B) Desktop
(C) Task bar
(D) Windows

ENGLISH
Questions (01 04) Fill in the blank with right option.
There are blank spaces in the following passage which have been numbered. These numbers printed below the passage and against
each some words are suggested. Find out the word that best fit the blank spaces.
Global strategies to control infective disease have historically included the erection of barriers to international travel and ________ (01).
Keeping people with such deadly disease outside national borders has ________ (02) as an important public health policy in the human
immunodeficiency virus (HIV) epidemic. Between 29 and 50 countries ________ (03) to have introduced border restrictions on HIV-
positive foreigners, usually those planning an __________ (04) stay in the country, such as students, workers or seamen.

01.
(A) migration
(B) immigrate
(C) emigration
(D) immigration
02.
(A) reemerged
(B) returned
(C) merged
(D) restrained
03.
(A) is reported
(B) are
(C) are reported
(D) reports
04.
(A) extended
(B) intended
(C) unplanned
(D) interested

Questions (05 07). Select the word or phrase that best completes the sentence.
05. You can reach your goal only when you aspire.
(A) dream
(B) attempt it
(C) try
(D) aim high
06. Read the abridge version of the novel to get an overall idea of the book.
(A) condensed
(B)
(C) summarized
(D) accurate
07. Employers look for agility in workers in small-part production plants.
(A) alertness
(B) dexterity
(C) flexibility
(D) liveliness

Questions (08 10). Select the pair that best expresses a relationship similar to that expressed in the original pair.
08. CURDLE: MILK
(A) Flow: Water
(B) Change: Oil
(C) Brew: Coffee
(D) Clot: Blood
09. VINDICTIVE: MERCY
(A) Avaricious: Greed
(B) Skeptical: Trustfulness
(C) Insightful: Hope
(D) Modest: Dignity
10. CONSCIOUS: CARELESS
(A) Careful: Indifferent
(B) Graceful: Ugly
(C) Generous: Unkind
(D) Careful: Ugly

BANGLA
?
(A)
(B)
(C)
(D)
?
(A)
(B)
(C)
(D)
?
(A)
(B)
(C)
(D)

(A)
(B)
(C)
(D)
?
(A)
(B)
(C)
(D)

(A)
(B)
(C)
(D)

(A)
(B)
(C)
(D)
?
(A)
(B)
(C)
(D)
?
(A)
(B)
(C)
(D)

(A)
(B)
(C)
(D)
MATHEMATICS
A manufacturer sells three products i.e. A, B and C. Product A costs 200 and sells for 250, Product B costs 150 and sells for 180,
Product C costs 100 and sells for 110. On which product, he has maximum percentage of profit?
(A) B Only .
(B) A and B both
(C) A Only
(D) C Only
Answer: A Only
Solution:
Profit from A = 250-200=50tk
Percentage = (50×100)/200=25%
Profit from B= 180-150=30tk
Percentage = (30×100)/150= 20%
Profit from C= 110-100=10tk
Percentage= (10×100)/100=10%
he areas
of the three plots increased?
(A) 125%
(B) 150%
(C) 200%
(D) 375%
Answer: 125%
Solution:
Let, the length of each side of the garden = 100
Area = 1002 = 10000
Area of 3 square = 3×10000 = 30000
50% increase of length = 100+100×50/100 = 150
Area = 1502 = 22500
Area of 3 square = 3×22500 = 67500
Increasing area = 67500-30000 =37500
Percentage = (37500×100)/30000=125%
If a, b and c are 3 consecutive integers and a>b>c, which of the following has the maximum value?
(A) b+(c/a)
(B) c+(a/b)
(C) c+(b/c)
(D) a+(b/c)
Answer: b+(c/a)
Solution:
As a is the greater number, adding a to any fraction of the given choice will have the maximum value.
Three boys have marbles in the ratio of 19 : 5 : 3. If the boy with the least number has 9 marbles, how many marbles does the boy
with the highest number have?
(A) 57
(B) 15
(C) 76
(D) 38
Answer: 57
Solution:
Let the boy has 3x marbles
So, 3x = 9
x = 9/3 = 3
The boy with the highest number of marbles = 19×3 = 57
A rectangular fish tank 25″ by 9″ has water in it to a level of 2″. This water is carefully poured into a cylindrical container with a
diameter of 10″. How high (in terms of π) will the water reach in the cylindrical container?
(A) 9/2π
(B) π/18
(C) 18/π
(D) 18π
Answer: 18/π
Solution:
The volume of the fish tank = 25×9×2 = 450 square inch
Let height of the cylindrical container is = h
πr2h = 450
h = 450/ πr2 = 450/π52 = 18/π
k. 42360,
how much is the total profit?
(A) Tk. 169,440
(B) Tk. 183,000
(C) Tk. 196,700
(D) Tk. 168,440
Answer: Tk. 169,440
Solution:
As the profit distributed according to the investment ratio the profit and investment ratio will be equal

5x = 42360
x = 8472

Total profit = 43260+59304+67776=169440


A square carpet with an area of 169 cm2 must have 2 cm cut off one of its edges in order to be a perfect fit for a rectangular room.
What is the area (in cm2) of this rectangular room?
(A) 117
(B) 143
(C) 145
(D) 165
Answer: 143
Solution:
Let, the carpet length = x
x2 = 169
x = 13
width of the room = (13-2) cm = 11 cm
area of the room = (13×11)2 = 143 cm2
To represent a family budget on a circle graph, how many degrees of the circle should be used to represent an item that is 20% the
total budget?
(A) 60°
(B) 62°
(C) 70°
(D) 72°
Answer: 72°
Solution:
In circle graph 100% is equals to 360°
In circle graph 20% is equals to = (360×20)/100 = 72°
The average of 6 numbers is 25. If 3 more numbers, with an average of 22 are added to these numbers, what will be the average of
the combined 9 numbers?
(A) 20
(B) 24
(C) 28
(D) 32
Answer: 24
Solution:
The average of 6 numbers is 25
The Total of 6 numbers is =25×6=150
The average of 3 numbers is 22
The Total of 3 numbers is =22×3=66
The Total of (6+3)=9 numbers is =(150+66)=216
The average of 9 numbers is=216/9=24
In a club 50% of the male voters and 80% of the female voters voted for candidate A. If candidate A received 70% of the total
votes, what is the ratio of male to female voters?
(A) 1/5
(B) ¼
(C) ½
(D) 1/3
Answer: ½
Solution:
Male voter = x
Female voter = y
50% of x + 80% of y = 70% of (x+y)
50x/100 + 80y/100 = 70(x+y)/100
50x + 80y = 70x + 70y
80y 70y= 70x 50x
10y = 20x
x/y = 10/20 = ½
If x+y>5 and x y>3, then which of the following gives all possible values of x and only possible values of x?
(A) x>4
(B) x>3
(C) x > 2
(D) x<4
Answer: x>4
Solution:
The interest charged on a loan is p dollars per $1,000 for the first month and q dollars per $1,000 for each month after the first
month. How much interest will be charged during the first three months on a loan of $10,000?
(A) 10p+20q
(B) 30q
(C) 30p
(D) 20p+10q
Answer: 10p+20q
Solution:
Interest = 1st month + 2nd month + 3rd month
= p×(10000/1000) + q×(10000/1000) + q×(10000/1000)
=10p +10q + 10q
= 10p + 20q
The length and breadth of a square are increased by 40% and 30% respectively. The area of the resulting rectangle exceeds the area
of the square by?
(A) 62%
(B) 42%
(C) 82%
(D) None of the above
Answer: 82%
Solution:
Let, length is = x so, area is = x2
40% increase in length = x+2x/5 =7x/5
30% increase in breadth = x+3x/10 =13x/10
Area = (7x/5)×( 13x/10) = 91×2/50
Increasing area = (91×2/50)- x2 = 41×2/50
Percentage = {(41×2/50) x2}×100 = (41/50)×100 = 82%
If an integer y is subtracted from an integer x, and the result is greater than x, then y must be
(A) equal to x
(B) less than x
(C) less than 0
(D) greater than 0
Answer: less than 0
Solution:
ATQ,
x-y>x
Or, -y > 0
Or, y < 0
A box contains 12 poles and 7 pieces of net. Each piece of net weighs 0.2 gm; each pole weighs 1.1 gm. The box and its contents
together weigh 16.25 gm. How much does the empty box weigh?
(A) 1.25 gm
(B) 1.65 gm
(C) 2.25 gm
(D) 2.65 gm
Answer: 1.65 gm
Solution:
Weight of 7 nets = (7×0.2)gm =1.4gm
Weight of 12 poles = (12×1.1)gm = 13.2gm
Total weight of net and pole = (1.4+13.2)gm = 14.6gm
Box weight is = (16.25 14.6) = 1.65gm

SUBJECTIVE QUESTIONS
Questions 36 80: Choose the correct answer.
Which type of cable is used to connect the COM port of a host to the COM port of a router or switch?
(A) Crossover
(B) Straight-through
(C) Rolled
(D) Shielded twisted pair
Which protocol can cause overload on a CPU of a managed device?
(A) Netflow
(B) WCCP
(C) IP SLA
(D) SNMP
Which of the following is a group of one or more attributes that uniquely indenfies a row?
(A) Key
(B) Determinant
(C) Tuple
(D) Relation
For every relationship, how many possible sets of minimum cardinalities are there?
(A) Two
(B) Three
(C) Four
(D) Six
The SQL statement that queries or reads data from a table is
(A) SELECT
(B) READ
(C) QUERY
(D) None of the above
Which of the following protocols uses both TCP and UDP ports?
(A) SMTP
(B) telnet 6
(C) FTP
(D) DNS
Two dimensional arrays are also called
(A) tables arrays
(B) matrix arrays
(C) both a and b
(D) None of the above
Which of the following is an SQL trigger supported by Oracle?
(A) BEFORE
(B) INSTEAD OF
(C) AFTER
(D) None of the above
k iternations by Newton-Raphson method of finding square root of a number increases accuracy in number of digits.
(A) k
(B) 2k
(C) 3k
(D) 4k
Bubble sort algorithm sorts n data items using
(A) O(n2) comparison
(B) O(n√n) comparison
(C) O(n log n) comparison
(D) O(n) comparison
Which control statement can be executed at least once?
(A) while
(B) for
(C) do..while
(D) All of the above
An algorithm that calls itself directly or indirectly is known as
(A) Sub algorithm
(B) Recursion
(C) Polish notation
(D) Traversal algorithm
Two main measures for the efficiency of an algorithm are-
(A) Processor and memory
(B) Complexity and capacity
(C) Time and space
(D) Data and space
Which of the following correctly shows the hierarchy of arithmetic operations in C?
(A) / + *
(B) *- / ÷
(C) + /*
(D) / * ÷
The complexity of Binary search algorithm is:
(A) O(n)
(B) O(log n)
(C) O(n2)
(D) O(n log n)
-
(A) Lower bound
(B) Upper bound
(C) Range
(D) Extraction
Which one of the following is not a valid identifier?
(A) _mysun
(B) Mysun
(C) my-sun
(D) my_sun
Consider the following code:
#include<stdio.h>
int main(int argc, char *argve[]){
return 0;
}
What is the minimum length of character array argv in the above code?
(A) 0
(B) 1
(C) Undefined
(D) -1

double d = math.round
(2.5+Math.random() );
(A) 2
(B) 2.5
(C) 3
(D) 4
Which of the following TCP/IP addresses constitute the loopback address?
(A) 1.1.1.1
(B) 25.255.255.255
(C) 127.0.0.0
(D) 127.0.0.1
Which one is modern light weight message exchange format?
(A) XML
(B) MX
(C) JSON
(D) HTML
Which data structure allows deleting data elements from front and inserting at rear?
(A) Stacks
(B) Queues
(C) Qeques
(D) Binary search tree
58. Consider the following code segment.
What will be content of str[4]?

(A) 4
(B) 6
(C) 0
(D) 2
What will be the output of the following code?
void main ()
\
(A) 0.000
(B) 1.000
(C) 0.111
(D) None of the above
The only language understood by a digital computer is called
(A) Assembly language
(B) High level language
(C) Binary language
(D) None of the above
Which of the following name does not relate to stacks?
(A) FIFO list
(B) LIFO list
(C) Piles
(D) Push-down lists
Which of the following will produce an answer that is closest in value to a double, d, while not being greater than d?
(A) (int)Math.min(d);
(B) (int)Math.max(d);
(C) (int)Math.abs(d);
(D) (int)Math.floor(d);
Which keyword must be used to inherit a class in Java?
(A) extends
(B) super
(C) this
(D) extent
Which of the following is the
(A) *Vehicle0
(B) *Vehicle(int value)
(C) ~Vehicle0
(D) ~Vehicle(int value)
A class that is inherited in Java is called a-
(A) subclass
(B) super class
(C) static class
(D) implement class
Which one of these interfaces is implemented by Thread class?
(A) Set
(B) Connections
(C) Runnable
(D) None of the above
Which one is less costly for insertion operation at a particular position?
(A) array
(B) queue
(C) link list
(D) stack
Register Circuit is not used in-
(A) digital clocks
(B) computers
(C) RAM
(D) Amplifier
Which one is a universal logic gate?
(A) NAND
(B) AND
(C) OR
(D) NOT
The data structure required to evaluate a postfix expression is:
(A) queue
(B) stack
(C) array
(D) linked-list
Which of the declaration is correct?
(A) int length;
(B) char int;
(C) int long;
(D) float double;
In Java, which operator is used to create an object?
(A) class
(B) scanf
(C) print
(D) None of the above
The operator that cannot be overloaded is:
(A) ++
(B) ::
(C) ~
(D) 0
How many instances of an abstract class can be created?
(A) 0
(B) 1
(C) 2
(D) 13
Which of the following cannot be checked in a switch-case statement?
(A) Character
(B) Integer
(C) Float
(D) None of the above
Which of the following is a design pattern?
(A) List
(B) Queue
(C) Factory
(D) None of the above
The main threat of Cloud based provisioning is:
(A) Cost
(B) Security
(C) Virtualization
(D) All of the above
Link list can be implemented by using-
(A) Array
(B) pointers
(C) both a and b
(D) None of the above
Which of the following data structure is linear type?
(A) Strings
(B) Lists
(C) Queues
(D) All of the above
In Java, which one will be used for comparing whether the two String object str1 and str2 are same?
(A) str1==str2
(B) str1.equalslgnoreCase(str2)
(C) str1==str2
(D) All of the above

BANGLA
01. ?
(A)
(B)
(C)
(D)
02. ?
(A)
(B)
(C)
(D)
03. ?
(A)
(B)
(C)
(D)
04.
(A)
(B)
(C)
(D)
05.
(A)
(B)
(C)
(D)
06. ?
(A)
(B)
(C)
(D)
07.
(A)
(B)
(C)
(D)
08. ?
(A)
(B)
(C)
(D)
09.
(A)
(B)
(C)
(D)
10. ?
(A)
(B)
(C)
(D)
11. ?
(A)
(B)
(C)
(D)
12. ?
(A)
(B)
(C)
(D)
13. ?
(A)
(B)
(C)
(D)
14. ?
(A)
(B)
(C)
(D)
15.
(A)
(B)
(C)
(D)
16. ?
(A)
(B)
(C)
(D)

ENGLISH
Q. (17 20): Choose the word that conveys the same meaning as the given word.
17. IMPUTE
(A) blame
(B) attribute
(C) apply
(D) expert
18. EXIGENCY
(A) dilemma
(B) constraint
(C) demand
(D) occasion
19. ANXIETY
(A) worry
(B) comfort
(C) faith
(D) None of them
20. RUTHLESS
(A) Mindful
(B) Cruel
(C) Merciful
(D) None of them

Q. (21 23). Select the word or phrase that best completes the sentence.
21. Employers look for agility in workers in small-part production plants.
(A) flexibility
(B) dexterity
(C) alertness
(D) liveliness
22. You can reach your goal only when you aspire.
(A) aim high
(B) attempt it
(C) try
(D) dream
23. Writing a beautiful sonnet is as much an achievement as to finish a 400 page novel.
(A) finishing
(B) it is to finish
(C) to have finished
(D) if to finish
Q. (24 27): There are blank spaces in the following passage which have been numbered. These numbers are printed below the
passage and against each some words are suggested. Find out the word that best fit the blank.
Global strategies to control infective disease have historically included the erection of barriers to international travel and _ (24) _
Keeping people with such deadly disease outside national borders has_ (25) _ as an important public health policy in the human
immunodeficiency virus (HIV) epidemic. Between 29 and 50 countries (26) to have introduced border restrictions on HIV-positive
foreigners, usually those planning an (27) stay in the country, such as students, workers or seamen.
24.
(A) migration
(B) immigrate
(C) emigration
(D) immigration
25.
(A) reemerged
(B) returned
(C) merged
(D) restrained
26.
(A) is reported
(B) are
(C) are reported
(D) reports
27.
(A) extended
(B) intended
(C) unplanned
(D) interested

Q. (28 32): Read the following passage and select the most appropriate answer.
Nine books are to be arranged from the left to right on a shelf. Of these, four books are leather-bounds, three are cloth-bounds and
remaining books are paperbacks. The leather bound books must be together. The paperback books must also be together. The
clothbound books need not to be together.
28. The cloth-bound books must be next to each other, if a paperback book is in which of the following position?
(A) the ninth
(B) the first
(C) the fifth
(D) the seventh
29. If the second book is a clothbound and the third book is a paperback, which of the following can be a clothbound book?
(A) the second
(B) the ninth
(C) the seventh
(D) none of these
30. If no clothbound book is next to another clothbound book, which one can be a paperback book?
(A) the fifth
(B) the ninth
(C) the third
(D) the sixth
31. If the first and seventh books have the same kinds of binding, which of the following must be a leather-bound books?
(A) the second
(B) the fourth
(C) the first
(D) the sixth
32. If the sixth book is a leather-bound and eighth book is a cloth-bound, which of the following must be a paperback?
(A) the second
(B) the first
(C) the third
(D) the fourth

MATHEMATICS
33. Triangle ABC has the following vertices: A(1,0), B(5,0) and C(3, 4). Which of the following is true?
(A) CA = CB
(B) AB = AC
(C) AC < BC
(D) AB = BC
Solution:
Distance of (x1,y1) & (x2,y2) = √{(x1 x2)2 + (y1 y2)2}
AB/BA = √{(1 5)2 + (0 0)2 }= √(-4)2 = √16 = 4
BC/CB = √{(5 3)2 + (0 4)2 }= √22 + 16 = √20 = 2√5
AC/CA=√{(1-3)2 + (0 4)2 }= √4 + 16 =√20 = 2√5
34. Increasing the original price of an item by 10%, then decreasing by 20% and then again increasing the price by 10% is equivalent:
(A) 4.4% increase
(B) 3.2% decrease
(C) 3.5% decrease
(D) None of these
Solution:
Let,
Selling price = 100
After 10% increase = (100+10)=110
After 20% decrease = (110 20% of 110) = 88
After 10% increase = (88 + 10% of 88) = 96.8
Final decrease = (100-96.8)=3.2%
35. (2√27-√75+√12. is equal to
(A) 4√3
(B) √3
(C) 2√3
(D) 3√3
Solution:
Given,
(2√27 √75 + √12)
= 2√(32.3)-√(3× 25) + √(3 × 4)
= 6√3 5√3 + 2√3
= √3 5√3
= 3√3
36. A two-digit number has 3 in its unit digit. The sum of its digits is one seventh of the number itself. What is the number?
(A) 73
(B) 53
(C) 63
(D) 83
Solution:
Let,
The number = 10x + 3
ATQ,
7(x + 3) = 10x + 3
⇒7x + 21 = 10x + 3
⇒21-3 = 10x 7x
⇒3x = 18
⇒x= 6
The number = 10 × 6 + 3 = 63.
37. A student loses 1 mark for every wrong answer and scores 2 marks for every correct answer. If he answers all the 60 questions in
an exam and scores 39 marks, how many of them were correct?
(A) 33
(B) 31
(C) 27
(D) 37
Solution:
Let,
Number of wrong answers = x
ATQ,
2(60-x) -x = 39
⇒120 -2x-x = 39
⇒-3x = 39-120
⇒-3x = -81
⇒x = 27
correct answer = (60 27) = 33
38. The triangular base of a prism is a right triangle of sides a and b = 2a. The height h of the prism is equal to 10 mm and its volume
is equal to 40 mms, what will be the lengths of the sides a and b of the triangle?
(A) 2 mm and 3 mm
(B) 1 mm and 4 mm
(C) 2 mm and 2 mm
(D) 2 mm and 4 mm
Solution:

½ × abc = 40
Or, abc = 80
Or, a × 2a × 10 = 80
Or, 20a2 = 80
Or, a2 = 2
Or, a = 2
And b = 2a = 2×2 = 4
39. The sum of principal and simple interest of a certain amount of money would be Tk. 460 after 3 years from now and Tk. 500 after 5
years from now. What is the total interest rate?
(A) 5%
(B) 12%
(C) 15%
(D) 20%
Solution:
(5 3) yr = 2 yrs interest = (500 460) = 40 tk
1 year interest = 40/2 = 20 tk.
3 year interest = (20 × 3) = 60 tk
Principal amount= (460 60) = 400 tk
Interest rate = 60/(400×3) = 0.05 = 5%
40. A wheel rotates 10 times per minutes and moves 20m during each rotation. How many feet does the wheel move in 1 hour?
(A) 10,000
(B) 12,000
(C) 18,000
(D) 20,000
None
Solution:
In 1 min the wheel passes = (20 × 10) = 200m
In 60 min the wheel passes = (200 x 60) = 12,000m
We know,
1m =3.28 feet
.: 12,000m=(3.28 × 12,000) = 39360 feet
41. If x and y are consecutive positive integers, which of the following must be an even integer?
(A) X
(B) y
(C) xy/2
(D) xy
Solution:
As X and Y are consecutive positive integers, one will be even and the other odd.
odd × even= even
42. There are n students in a school. If r% among the students are 12 years or younger, which of the following expressions represents
the number of students who are older than 12?
(A) n(1-r)
(B) 100(1-r)n
(C) n(1-r)/100
(D) n(100-r)/100
Solution:
Students above 12,
In 100, there are (100-r)
In 1, there are (100-r)/100
In n, there n(100-r)/100
43. The ratio between the perimeter and the breadth of a rectangular is 5:1. If the. area of the rectangle is 216cm, what is the length of
the rectangle?
(A) 16cm
(B) 18cm
(C) 20cm
(D) 22cm
Solution:
Let,
Length and breadth of the rectangle= x, y
So, area = xy
Perimeter = 2(x+y)
ATQ,
2(x+y)/y = 5
Or, 2x + 2y = 5y
Or, 2x = 3y
Or, y = 2x/3
Now,
xy = 216
Or, x × 2x/3 = 216
Or, 2×2 = 648
Or, x2 = 324
Or, x = 18
44. jf (x + y) > 5 and (x y) > 3, then which of the following gives all and only possible value of x?
(A) x<3
(B) x> 3
(C) x> 4
(D) x < 5
Solution:
Given,
(x + y) > 5 -(1)
(x y) > 3 (2)
(1)+(2),
2x >8
Or, x>4
45. Three boys have marbles in the ratio of 19: 5: 3. If the boy with the least number has 9 marbles, how many marbles does the boy
with the highest number have?
(A) 57
(B) 15
(C) 76
(D) 38
Solution:
Smaller ratio 3 = 9
So, Smaller ratio 1 = 9/3 = 3
Now,
Highest ratio 19 = 19×3 = 57
46. The area of a rectangle R with width 4 ft is equal to the area of a square S, which has a perimeter of 24 ft, the perimeter of the
rectangle R, in feet, is
(A) 9
(B) 16
(C) 24
(D) 26
Solution:
Given,
Perimeter of the square = 24
So, side of the square = 24/4=6
Area = 62 = 36
Let,
Length of the rectangle = x
ATQ,
4x = 36
Or, x= 9
So, perimeter of the rectangle = 2(9+4) = 26
47. In a class of 120 students, 70 percent can speak only Bengali and the rest can speak English. If 25 percent of those in the class who
can speak English can also speak Bengali, how many of the students in the class can speak Bengali?
(A) 39
(B) 48
(C) 84
(D) 93
Solution:
Only speak bangla = 120 × 70/100 = 84
Speak English = 120× 30/100 = 36
Now,
25% of 36 = 36× 25/100 =9 can also speak bangla
So, total bangla speaking student =(84 +9) = 93
48. A lamp is manufactured to sell for $35.00, which yields a profit of 25% of cost. If the profit is to be reduced to 15% of cost, what
will be the new retail price of the lamp?
(A) $31.50
(B) $28.00
(C) $21.00
(D) $32.20
Solution:
In 25% profit,
If selling price 125 then cost = 100
So, when selling price 35 then cost = (100×35)/125 = 28
In 15% profit,
If cost is 100 then selling price = 115
So, when cost is 28, then selling price = (115×28)/100 = 32.20
49. In triangle ABC, AB = AC. All of the following statements are true except
(A) AB < AC + BC
(B) AC< AB + BC
(C) BC+ AC> AB + BC
(D) AC+ BC= AB + BC
Solution:
Given,
AB = AC
Or, BC + AB = BC + AC [Adding BC to both sides]

50. If an inspector reject 0.08% of a product as defective, how many units of the product will he examined in order to reject 2?
(A) 500
(B) 1,500
(C) 2,000
(D) 2,500
Solution:
0.08 product is rejected from = 100
So, 2 product is rejected from = (100 × 2 × 100)/0.08 = 2500
51. There are 15 balls in a box: 8 balls are green, 4 are blue and 3 are white. Then 1 green and 1 blue balls are taken from the box
and put away. What is the probability that a blue ball is selected at random from the box?
(A) 3/15
(B) 4/15
(C) 3/13
(D) 4/13
Solution:
green balls = (8 1) = 7
blue balls = (4 1) = 3
white balls = 3
possibility of getting a blue ball = 3/(7+3+3) = 3/13
52. A short distance athlete has taken 60 seconds to cover 100 meters. If he makes 30 steps in 9 seconds, how many steps has he
taken in that time?
(A) 130
(B) 170
(C) 173
(D) None of these
Solution:
9 sec = 30 steps
60 sec = (30×60)/9= 200 steps
53. Three angles of a triangle are in proportion 5: 6:7. Then what is the difference in degrees between the biggest and the smallest
angles?
(A) 10°
(B) 20°
(C) 25°
(D) 30°
Solution:
Sum of 3 angles of a triangle = 180°
Sum of ratio = 5+6+7 = 18
So, the larger angle = 180° × 7/18 = 70°
And, the smaller angle = 180° × 5/18 = 50°
Difference = 70° 50° = 20°
54. he areas
of the three plots increased?
(A) 375%
(B) 200%
(C) 150%
(D) 125%
Solution:
Let,
Side of a square = 100
So, area = (100)2 = 10000
Area of 3 square = 3× 10000 = 30000
After 50% increase side of a square becomes = (100 + 50% of 100) = 150
So, area = 1502 = 22500
Area of 3 square = 22500 × 3 = 67500
Area increases = (67500 30000) = 37500
So, percentage of increase = (37500×100)/30000 = 125%
55. A manufacturer sells three products i.e. A, B and C. Product A costs 200 and sells for 250. Product B costs 150 and sells for 180.
Product C costs 100 and sells for 110. On which product, he has maximum percentage of profit?
(A) B only
(B) A and B both
(C) A only
(D) C only
Solution:
-200)/200}×100 = 25%
-150)/150}×100 = 20%
-100)/100}×100 = 10%
56. A farmer has two rectangular fields. The larger field has twice the length and four times the width of the smaller field. If the smaller
field has area K, then the area of the larger field is greater than the area of the smaller field by what amount?
(A) 2K
(B) 5K
(C) 6K
(D) 7K
Solution:
Let,
Length of smaller field = x
So, length of larger field = 2x
Again,
Width of smaller field = y
So, width of larger filed = 4y
Area of smaller field = xy = K
Area of larger field = 2x × 4y = 8xy = 8K
Difference = 8K K = 7K

GENERAL KNOWLEDGE & COMPUTER


57. The radiant energy of the Sun results from
(A) Nuclear fusion
(B) Nuclear fission
(C) Cosmic radiation
(D) Combustion
58. Fathometer is used to measure
(A) Sound intensity
(B) Earthquakes
(C) Rainfall
(D) Ocean depth
59. Which of the following is called Blue Planet?
(A) Saturn
(B) Earth
(C) Jupiter
(D) Mars
60. Radio link is a tool of banking communication in a
(A) WAN area
(B) LAN area
(C) Remote area
(D) VAN area
61. Stress test is related to assess health of which institution?
(A) Bank
(B) Multinational Firm
(C) Private Firm
(D) None of them
62. Which one of the following is not the official language of United Nation Organization?
(A) Spanish
(B) Portuguese
(C) Chinese
(D) None of them
63. Fathometer is used to measure
(A) Ocean depth
(B) Earthquakes
(C) Rainfall
(D) Sound intensity
64. Which one of the following countries is not a member of the OPEC?
(A) Algeria
(B) Indonesia
(C) Malaysia
(D) Nigeria
65.
(A) Thomas Hardy
(B) Thomas Mann
(C) Thomas Moore
(D) Thomas Paine
66. Which one of the following is not an official language of the United Nations?
(A) Arabic
(B) Chinese
(C) Portuguese
(D) Spanish
67. On July ___man walked on the Moon for the first time.
(A) 19th
(B) 20th
(C) 18th
(D) 21st
68. When cutting and pasting, cutting section is temporarily stored in
(A) Dashboard
(B) Hard drive
(C) Diskette
(D) Clipboard
69. If you need to change the typeface of a document, which menu will you choose?
(A) Edit
(B) View
(C) Format
(D) Tools
70. You can move between two or more Excel files opened by using the
(A) ctrl + tab
(B) ctrl + page up
(C) ctrl + page down
(D) ctrl + F9
71. The basic unit of a worksheet into which you enter data in Excel is called a
(A) column
(B) box
(C) table
(D) cell
72. The process of transforming files from a computer on the Internet to your computer is called
(A) Forwarding
(B) Downloading
(C) FTP
(D) Uploading
73. You can detect spelling and grammar errors by
(A) Press Shift + F7
(B) Press Ctrl + F7
(C) Press Alt + F7
(D) Press F7
74. Verification of a login name and password is known as:
(A) configuration
(B) authentication
(C) accessibility
(D) logging in
75. Which can be used for quick access to commonly used commands and tools?
(A) Status bar
(B) Tool bar
(C) Menu bar
(D) Title bar
76. ________ are software which is used to do particular task.
(A) Operating system
(B) Data
(C) Software
(D) Program
77. All of the following are examples of real security and privacy risks EXCEPT:
(A) Viruses
(B) Spam
(C) Hackers
(D) None of them
78. The program compresses large files into a smaller file
(A) WinZip
(B) WinShrink
(C) WinStyle
(D) None of them
79. Which can be used for quick access to commonly used commands and tools?
(A) Status bar
(B) Tools bar
(C) Menu bar
(D) Title bar
80. Operating System is the most common type of software.
(A) System
(B) Application
(C) Communication
(D) None of these

BANGLA
01. ?
(A)
(B)
(C)
(D)
02.
(A)
(B)
(C)
(D)
03.
(A)
(B)
(C)
(D)
04. ?
(A)
(B)
(C)
(D)
05.
(A)
(B)
(C)
(D)
06. ?
(A)
(B)
(C)
(D)
07. ?
(A)
(B)
(C)
(D)
08. ?
(A)
(B)
(C)
(D)
09. , ?
(A)
(B)
(C)
(D)
10.
(A)
(B)
(C)
(D)
11. ?
(A)
(B)
(C)
(D)
12. ?
(A)
(B)
(C)
(D)
13. ?
(A)
(B)
(C)
(D)
14.
(A)
(B)
(C)
(D)
15. ?
(A)
(B)
(C)
(D)
16. ?
(A)
(B)
(C)
(D)

ENGLISH
Question: 17 19 Choose the correct spelling from the given options
17.
(A) Questionere
(B) Questionaire
(C) Questionnaire
(D) Questionare
18.
(A) Ridiculus
(B) Rcdiculous
(C) Ridiculas
(D) Ridiculous
19.
(A) Lisense
(B) Lisence
(C) License
(D) Licents

Questions (20-22). Select the word or phrase that best completes the sentence
20. Every individual is responsible for upholding the moral of society.
(A) keeping up
(B) developing
(C) judging
(D) maintaining
21. In comparison to the history of the world, human life is very transient.
(A) Meaningless
(B) short-lived
(C) poignant
(D) arbitrary
22. Computer has revolutionized office procedures more than any machine of modern time
(A) has any machine
(B) any other machine
(C) other machine
(D) None of these

Questions (23-24): Fill in the blank with right option


23. The view ___the open window is very pretty.
(A) through
(B) by
(C) along
(D) to
24. Spring is ___ it will be warmer soon.
(A) to be coming
(B) coming
(C) prepared now
(D) on the way
25. Lack of exercise and high fat diets have ____to be factors in heart attacks.
(A) long known been
(B) known been long
(C) been long known
(D) long been known

Questions (26-28): Select the pair that best expresses a relationship similar to that expressed in the original pair.
26. WOOD: SAND
(A) metal : burnish
(B) coal : burn
(C) brick : lay
(D) oil : polish
27. HARM : DAMAGE
(A) sweet : sour
(B) stout : weak
(C) hook : crook
(D) injure : incapacitate
28. YOLK : EGG
(A) nucleus : cell
(B) duck : fowl
(C) web : spider
(D) rind : melon

Questions (29-32): In each of the following lists, one word is out of sequence from the other words. Find the word that is not similar to
the other words in each list.
29.
(A) Height
(B) Huge
(C) Large
(D) Massive
30.
(A) Inflation
(B) Discourse
(C) Interests
(D) Investment
31.
(A) Strategy
(B) Aim
(C) Goal
(D) Objective
32.
(A) Temporary
(B) Short-lived
(C) Transitory
(D) Steady

MATHEMATICS
33. In a club 50% of the male voters and 80% of the female voters voted for candidate A. If candidate A received 70% of the total

(A) 1/3

(B) ½
(C) ¼
(D) ¾
Solution:
Let, total member of the club = 100 and male member =x
Female member = (100-x)
Now,
x×50% + (100-x)80% = 70×100%
5x +(100-x)8 = 70×10
5x + 800 8x = 700
3x = 100
X = 33.33
Now, female: Male = 33.33 : (100-33.33) = 1:2
34. Four liters of milk are to be poured into a 2 liter bottle and a 4 liter bottle. If each bottle is to be filled to the same fraction of its
capacity, how many liters of milk should be poured into the 4 liter bottle?
(A) 7/3
(B) 2/3
(C) 8/3
(D) 4/3
Solution:
Let, milk in 4 litter bottle is x litter
So, milk in 2 litter bottle is (4-x) litter
x/4 = (4-x)/2
x = 8-2x
x = 8/3
35. One dozen eggs and ten pounds of apples are currently of the same price. If the price of a dozen eggs rises by 10% and that of
apples rises by 2%, how much more will it cost to buy a dozen of eggs and ten pounds of apples?
(A) 2%
(B) 6%
(C) 10%
(D) 12%
Solution:
Say, Cost of eggs = Cost of apples = 100
Total = 200
New egg cost = 110
New apple cost = 102
New total = 110 + 102 = 212
Percentage increase ={(212−200)/200}×100= 6%
36. An iron rod that weighs 24 kg is cut into two pieces so that one of these pieces weighs 16 kg and is 34m long. If the weight of each
piece is proportional to its length, how long is the other piece?
(A) 11m
(B) 17m
(C) 34m
(D) 68m
Solution:
16 kg rod = 34m
24 kg rod = (34×24)/16 = 51m
Length of the other part is = 51-34 = 17m
37. A wholesaler sells goods to a retailer at a profit of 20%. The retailer sells to the customer, who pays 80% more than the cost of the

(A) 30%
(B) 40%
(C) 50%
(D) 60%
Solution:
Let cost of wholesaler = 100tk
At 20% profit he sells = 120tk = the cost of retailer
Retailer sell it for 80% profit on cost of wholesaler
= 100 + 100×(80/100) = 180 tk
Profit of retailer = 180- 120 = 60 tk
Percentage = (60×100)/120 = 50%
38. The price of a pen is 25% more than the price of a book. The price of a pen holder is 50% more than the price of the book. How
much is the price of the pen holder more than the price of the pen?
(A) 20%
(B) 25%
(C) 50%
(D) 37.5%
Solution:
Let price of book = 100tk
Price of pen = 100+100×25% = 125 tk
Price of penholder = 100 + 100×50% = 150 tk
Difference is = 150 -125 = 25 tk
Percentage = (25×100)/125 = 20%
39. Three individuals contributed Tk 8,000 each toward the purchase of a computer. If they bought the computer on sale for Tk. 19500
plus 10% sales tax, how much money should be refunded to each individual?
(A) Tk. 650
(B) Tk. 850
(C) Tk.1,500
(D) Tk1,950
Solution:
Contribution of three person = 8000×3 = 24000 tk
Cost of the computer = 19500+19500×10% = 19500+1950 = 21450 tk
3 person get back = 24000 21450 = 2250 tk
1 person get back = 2250/3 = 850 tk
40. A 10% monthly salary increase resulted in a Tk. 9,000 per year increase in salary for an employee. What was his monthly salary
before the increase?
(A) Tk. 7,500
(B) Tk. 9,000
(C) Tk. 5,000
(D) Tk. 12,000
Solution:
Yearly increase = 9000 tk
Monthly increase = 9000/12 = 750 tk
10% of Salary is = 750 tk
100% of Salary is = (750×100)/10 = 7500 tk
41. When a certain number is divided by 7, the remainder is 0. If the remainder is not 0 when the number is divided by 14, then the
remainder must be
(A) 7
(B) 5
(C) 3
(D) 8
Solution:
If a number is divisible by 7, remainder will be 0. So, any number divisible by any multiple of 7 will leave remainder 7 or 0.
42. A mixture of 20 kg of sprit and water contains 10% water. How much water must be added to this mixture to raise the percentage
of water to 25%
(A) 2
(B) 4
(C) 5
(D) 6
Solution:
Water contains in mixture = 20×10% = 2 kg
Sprit in the mixture = 20-2 =18 kg
x litter water need to mix in the mixture.
So, (2+x)/18 = 25/75
(2+x)/18 = 1/3
6+3x = 18
x=4
43. How much interest will Tk. 1,000 earn in one year at an annual interest rate of 20% if interest rate is compounded every 6 months?
(A) 200
(B) 205
(C) 208
(D) 210
Solution:
We know,
n×m
C= P(1+r/m)
1×2
=1000{1+20/(100×2)}
2
= 1000{1+1/10} = 1210
Interest = 1210-1000 = 210 tk
44. A garden of 100% length and 60m width has a walkway of 2m width on every side. What is the area of the garden, in square meter,
excluding the walkway?
(A) 5684
(B) 6000
(C) 5376
(D) 5123
Solution:
Length of the garden excluding road = 100-2×2 =96m
Width of the garden excluding road = 60-2×2 =56m
Area garden excluding road = 96×56 =5376 sq. m
45. The total age of A and B is 12 years more than the total age of B and C. C is how many years younger than A?
(A) 12
(B) 24
(C) 26
(D) None of these
Solution:
ATQ,
A + B = 12 + B +C
A = 12 +C
That is, C is 12 years younger than A.
46. An article when sold at a gain of 5% yields Tk.15 more than when sold at a loss of 5%. Its cost price would be
(A) Tk. 100
(B) Tk. 150
(C) Tk. 200
(D) Tk. 250
Solution:
At 5% profit, selling price = 100+5=105 tk.
At 5% loss, selling price = 100-5= 95 tk.
Difference between selling price = 105-95=10 tk.
If difference 10, then cost = 100tk
If difference 1, then cost = 100/10 tk
If difference 15, then cost = (100×15)/10 tk = 150tk.
47. If ax = b, by = c, and cz = a, then the value of xyz is
(A) 0
(B) 1
(C) 1/abc
(D) abc
Solution:
Given,
ax = b
Or, (cz)x = b
Or, (by) z x = b
Or, (b) xyz = b1
Or, xyz = 1
48. For what value of x is 82x-4 =16x?
(A) 2
(B) 3
(C) 4
(D) 6
Solution:
Given,
82x-4 =16x
Or, 23(2x-4) = 24x
Or, 6x -12 = 4x
Or, 2x = 12
Or, x = 6
49. A is 30% more efficient than B. How much time will they, working together, take to complete a job which A alone could have done
in 23 days?
(A) 13 days
(B) 15 days
(C) 10 days
(D) None of these
Solution:
Ratio of times taken by A and B = 100 : 130 = 10 : 13.
Suppose, B takes x days to do the work.
Then,
10 : 13 = 23 : x
Or, x = (23 x 13)/10
Or, x = 299/10

=(1/23 +10/299)= 23/299 =1/13


Therefore, A and B together can complete the work in 13 days.
50. The lengths of the sides of a triangle are in the ratio of 3 to 5 to 6. If the perimeter of the triangle is 70, what is the length of the
longest side?
(A) 10
(B) 15
(C) 25
(D) 30
Solution:
3x+5x+6x=70
Or, 14x=70
Or, X=5
Length of the longest side= 5×6=30
51. If for integer x, 5 <x<10 and y = x + 5, what is the greatest possible value of x + y?
(A) 32
(B) 22
(C) 23
(D) 27
Solution:
Given,
5 < x < 10 and y =x+5
Possible value of x = 6, 7, 8, 9
When x =9,
y =9+5 = 14
Now,
x + y = 9 + 14 = 23
52. Square of a number plus two times the number equal 63. What is the number?
(A) 7
(B) 9
(C) 9
(D)
Solution:
x2 + 2x = 63
x2 + 2x 63 = 0
2
x + 9x 7x 63 = 0
x (x + 9) 7 (x + 9) = 0
(x + 9) (x 7) = 0
x = 9 and x = 7
53. If the perimeter of a certain rectangle is 76m and its area is 360m2, then what is the length of its shortest side?
(A) 18
(B) 15
(C) 13
(D) 10
Solution:
Let, length = x and width = y
Now, 2(x+y) = 76

xy = 360

(i)=
360/y + y =38
360 + y2 = 38y
y2 -38y + 360 = 0
y2 -20y 18y + 360 = 0
y(y -20) 18(y 20) = 0
(y -20) (y 18) = 0
y= 20 and 18
Width = 18m
54. If x=1+ √2 and y =1-√2 find the value of (x2 + y2)
(A) 12
(B) 10
(C)
(D) 6
Solution:
x2 + y2
=(1+ √2) 2 + (1-√2) 2
= 1 + 2√2 + 2 + 1 2√2 + 2
=6
55. If x and y are consecutive positive integers, which of the following must be an even integer?
(A) X
(B) Y
(C) xy/2
(D) xy
Solution:
As x & y are consecutive positive integers, one of them must be odd and the other even.
Now, Odd×Even= Even.
56. If two planes leave the same airport at 1:00 PM, how many miles apart will they be at 3:00 PM, if one travels directly north at 150
mph and the other travels directly west at 200 mph?
(A) 50 miles
(B) 100 miles
(C) 500 miles
(D) 700 miles
Solution:
Plane travels for 2 hours
1st one passes =150×2=300mile
2nd one passes =200×2=400mile
Distance between them
d2 = 3002 + 4002
Or, d = √(90000 + 160000) = √250000 = 500

GENERAL KNOWLEDGE
57. Which one is not a member country G-7?
(A) Japan
(B) Sweden
(C) Italy
(D) Canada
58. Transparency International is based in
(A) London
(B) Paris
(C) Berlin
(D) Washington D.C.
59. Which of the following countries has
(A) Switzerland
(B) Russia
(C) India
(D) France
60. Which of the following organization is concerned with environmental issues?
(A) OIC
(B) WMO
(C) MIGA
(D) IPCC
61. OTC stands for
(A) Over that Counter
(B) Over the Court
(C) Over the Counter
(D) Over that Court
62. The Harry Potter series is written by
(A) JK Rowling
(B) KK Rowling
(C) KJ Rowling
(D) A K Rowling
63. Profits of a firm that are distributed or given out to its investors are called
(A) Bonds
(B) Equity
(C) Dividends
(D) None of them
64. Point of sale (POS) machine is widely used by the
(A) Tellers
(B) Merchants
(C) Government
(D) Central bank
65. The great Victoria Desert is located in
(A) Canada
(B) West-Africa
(C) North-America
(D) Australia
66. Money-market is a market for
(A) short-term fund
(B) long-term fund
(C) hedge fund
(D) risk free assets
67. Which of the following is not a part of the World Bank Group?
(A) IDA
(B) IMF
(C) IBRD
(D) IFC
68. Stress test is related to assess health of which institution?
(A) Bank
(B) Multinational Firm
(C) Private Firm
(D) None of them

COMPUTER
69. A light sensitive device that converts drawing, printed text or other images into digital form is
(A) Keyboard
(B) Plotter
(C) Scanner
(D) None of these
70. Which of the following memory is non-volatile?
(A) ROM
(B) DRAM
(C) SRAM
(D) None of these
71. Which network protocol is used to send e-mail?
(A) SMTP
(B) FTP
(C) SSH
(D) POP3
72. The operating system manages.
(A) Memory
(B) Processor
(C) Devices
(D) All of the above
73. By default, your documents print in mode.
(A) Page setup
(B) Portrait
(C) Landscape
(D) Print preview
74. Ctrl + N is used to
(A) Save Document
(B) Open Document
(C) New Document
(D) Close Document
75. By default, on which page the header or the footer is printed?
(A) on first page
(B) on alternate page
(C) on every page
(D) none of the above
76. Which of these toolbars allows changing of Fonts and their sizes?
(A) Standard
(B) Formatting
(C) Print Preview
(D) None of these
77. Background color or effects applied on a document is not visible in
(A) Web layout view
(B) Print Layout view
(C) Reading View
(D) Print Preview
78. What is the default font size of a new Word document based on Normal template?
(A) 9 pt
(B) 12 pt
(C) 14 pt
(D) None of above
79. A Microsoft Windows is an
(A) Operating system
(B) Graphic program
(C) Word Processing
(D) Database program
80. Which of the following Operating System does not implement multitasking truly?
(A) Windows
(B) Windows NT
(C) MS DOS
(D) Windows XP

ENGLISH
1. The view _______ the open window is very pretty.
(A) By
(B) To
(C) Along
(D) Through
2. __________ his parents allow him or not, Anis intends to go to the party.
(A) Whether
(B) Although
(C) Despite
(D) While

Questions (03-05): Choose the correct word spelling from the given options.
3.
(A) Lisense
(B) License
(C) Lisence
(D) Licents
4.
(A) Extantion
(B) Extention
(C) Extansion
(D) Extension
5.
(A) Collateral
(B) Colateral
(C) Colleteral
(D) Collataral

Read the following passage and answer the question (06-08) :


There are eight seats in a small plain. These seats are arranged in four rows, numbered I through 4 and each row has two seats. Six
seats are assigned to six passengers N,P,Q,R,S and T. Seat assignments are made according to the following conditions.
N must sit alone in a row.
P must sit in the same row as R.
Q cannot sit in the same row as S.
The rows with only one passenger must be row 1 and row 3.

6. Which of the following passengers could be assigned to sit in the same row as Q?
(A) T
(B) S
(C) R
(D) P
7. If P and R are in row 2, which of the following must be true?
(A) N is in row 2
(B) Q is in row 1
(C) S is in row 3
(D) None
Answer:
8. How many passengers could be assigned to sit in the same row as T?
(A) 1
(B) 2
(C) 3
(D) 4

Questions (09-10): Select the word or phrase that best completes the sentence.
9. Writing a beautiful sonnet is as much an achievement as to finish a 400-page novel.
(A) it is to finish
(B) if to finish
(C) finishing
(D) to have finished
10. Today, this is totally different world than we have seen in the last decade.
(A) since we have seen
(B) from what we have seen
(C) from what we seen
(D) None of these

BANGLA
11.
(A)
(B)
(C)
(D)
12. ?
(A)
(B)
(C)
(D)
13. ?
(A)
(B)
(C)
(D)
14. ?
(A)
(B)
(C)
(D)
15. ?
(A)
(B)
(C)
(D)
16. ?
(A)
(B)
(C)
(D)
17. ?
(A)
(B)
(C)
(D)
18. ?
(A)
(B)
(C)
(D)
19. ?
(A)
(B)
(C)
(D)
20. ?
(A)
(B)
(C)
(D)
MATHEMATICS
21. A short distance athlete has taken 30 seconds to cover 100m. If he makes 30 steps in 9 seconds, now many steps he taken in that
time?
(A) 130
(B) 170
(C) 173
(D) None
Solution:
9 sec = 30 steps
60 sec = (30×60)/9= 200 steps
22. A car goes 15 km on a gallon of octane when it is driven at 50 km/h. When the car is driven at 60 km/h, it only goes 80% as far.
How many gallons of octane are needed to travel 200 km if half the distance is traveled at 50 km/h and the rest at 60 km/h?
(A) 15
(B) 16.67
(C) 10.60
(D) 14
Solution:
At 50km/h
15 km needs 1 gallon
So, 100 km needs 100/15 gallon
At 60km/h,
It goes 80% far = 80% of 15 = 12 km
12 km needs 1 gallon
So, 100 km needs 100/12 gallon
So, (100+100) = 200 km needs
= (100/15 + 100/12) gallon
= (400+500)/60 gallon
= 900/60 gallon
= 15 gallon
23. A manufacturer sells three products i.e. A, B and C. Product A cost 200 and sells for 250, Product B cost 150 and sells for 180,
Product C cost 100 and sells for 110. On which product, he has maximum percentage of profit?
(A) B only
(B) A and B both
(C) A only
(D) C only
Solution:
Profit from A = 250-200=50tk
Percentage = (50×100)/200=25%
Profit from B= 180-150=30tk
Percentage = (30×100)/150= 20%
Profit from C= 110-100=10tk
Percentage= (10×100)/100=10%
24. Tk. 42,360,
how much is the total profit?
(A) Tk. 169,440
(B) Tk. 183,000
(C) Tk. 196,700
(D) Tk. 168,440
Solution:
As the profit distributed according to the investment ratio the profit and investment ratio will be equal
5x = 42360
x = 8472

Total profit = 43260+59304+67776=169440


25. One third of the faculty members of a department are female. Sixteen of the male teachers are unmarried while 60% of them are
married. The total number of faculty members in the department is
(A) 72
(B) 60
(C) 30
(D) 90
Solution:
Let,
Total number = x
Number of female = 1/3 of x = x/3
Number of male = x-x/3 = 2x/3
Married male = 60% of 2x/3 = 2x/5
ATQ,
2x/3 2x/5 = 16
Or, 4x/15 = 16
Or, x = 60
26. A wholesaler sells goods to a retailer at a profit of 20%. The retailer sells to the customer, who pays 80% more than the cost of the

(A) 40%
(B) 50%
(C) 60%
(D) 70%
Solution:
Let cost of wholesaler = 100tk
At 20% profit he sells = 120tk = the cost of retailer
Retailer sell it for 80% profit on cost of wholesaler
= 100 + 100×(80/100) = 180 tk
Profit of retailer = 180- 120 = 60 tk
Percentage = (60×100)/120 = 50%
27. If an integer y is subtracted from an integer x, and the result is greater than x, then y must be
(A) Equal to x
(B) less than 0
(C) less than x
(D) greater than 0
Solution:
ATQ,
x-y>x
Or, -y > 0
Or, y < 0
28. A train went 300 km from City X to City Y at an average speed of 100 km/h. At what speed did it travel on the way back if its
average speed for the whole trip was 120 km/h?
(A) 120 km/h
(B) 125 km/h
(C) 130 km/h
(D) 150 km/h
Solution:
Let,
Way back speed = x
ATQ,
2×x×100/(x+100) = 120
Or, 200x/(x+100) = 120
Or, 200x = 120x+12000
Or, 80x = 12000
Or, x= 150
29. If a, b and c are 3 consecutive integers and a >b>c, which of the following has the maximum value?
(A) b+(c/a)
(B) c+(a/b)
(C) c+(b/a)
(D) a+(b/c)
Solution:
As a is the greater number, adding a to any fraction of the given choice will have the maximum value.
30. Sam can mow a lawn in 20 min, while Mark takes 10 min longer to mow the same lawn. How long will they take to mow the lawn if
they work together?
(A) 12 min
(B) more than 15 min
(C) 15 min
(D) 14 min
Solution:
In 1 minute, Sam mows 1/20 of the lawn
In 1 minute, Mark mows 1/(20+10) = 1/30 of the lawn
(1/20 + 1/30) = 5/60=1/12 of the lawn takes 1 minute
So, whole(1) lawn takes 12 minutes
31. If two planes leave the same airport at 1:00 pm, how many km apart will they be at 3:00 pm, if one travels directly north at 150
km/h and the other travels directly west at 200 km/h?
(A) 50km
(B) 500km
(C) 400km
(D) 600km
Solution:
Difference of time = 3:00 pm 1:00 pm = 2 hrs
First plane travels in 2 hrs = 2×150=300 km
Second plane travels in 2 hrs = 2×200=400 km
The directions of the planes forms a right angle triangle
So, the direct distance = √(3002 + 4002) = √250000=500 km
32. If an inspector rejects 0.08% of a product as defective, how many units of the product will he examined in order to reject 2?
(A) 500
(B) 1500
(C) 2000
(D) 2500
Solution:
0.08% product is rejected from = 100
So, 2 product is rejected from = (100 × 2 × 100)/8 = 2500
33. A two-digit number has 3 in its unit digit. The sum of its digits is one seventh of the number itself. What is the number?
(A) 53
(B) 63
(C) 73
(D) 83
Solution:
Let,
The number = 10x + 3
ATQ,
7(x + 3) = 10x + 3
⇒7x + 21 = 10x + 3
⇒21-3 = 10x 7x
⇒3x = 18
⇒x= 6
The number = 10 × 6 + 3 = 63.
34. A trader market the price of an article 30% above the cost price and gave the buyer 10% discount on marked price, thereby gaining
Tk. 340. The cost of the article is?
(A) 3000
(B) 2000
(C) 1900
(D) 1800
Solution:
35. The length and breadth of a square are increased by 40% and 30% respectively. The area of the resulting rectangle exceeds the area
of the square by?
(A) 62%
(B) 42%
(C) 82%
(D) None
Solution:
Let, length is = x so, area is = x2
40% increase in length = x+2x/5 =7x/5
30% increase in breadth = x+3x/10 =13x/10
Area = (7x/5)×( 13x/10) = 91×2/50
Increasing area = (91×2/50)- x2 = 41×2/50
Percentage = {(41×2/50) x2}×100 = (41/50)×100 = 82%
36. There boys have marbles in the ratio of 19:5:3. If the boy with the least number has 9 marbles, how many marbles does the boy
with the highest number have?
(A) 57
(B) 15
(C) 76
(D) 38
Solution:
Smaller ratio 3 = 9
So, Smaller ratio 1 = 9/3 = 3
Now,
Highest ratio 19 = 19×3 = 57
37. A circular wheel 28 inches in diameter rotates the same number of inches per second as a circular wheel 35 inches in diameter. If
the smaller wheel makes x revolutions per second, how many revolutions per minutes does the larger wheel make in terms of x?
(A) 12x
(B) 24x
(C) 36x
(D) 48x
Solution:
For smaller wheel,
Circumference = 2×π×28/2 = 28π
Revolve in 1 second = x times
Revolve in 60 seconds = 60x times
So, distance = 28π×60x
For larger wheel,
Circumference = 2×π×35/2 = 35π
Let,
Revolve per minute = n
So, distance = 35πn
Now,
28π×60x = 35πn
Or, n=(28π × 60x)/35π
Or, n= 48x
38. Of two groups of tourists, each has 60 people. If three-fourth (i.e. 75%) of the first group and two-third of the second group board
buses to travel to a museum. How many more people of the first group board buses than that of the second group?
(A) 3
(B) 5
(C) 10
(D) 15
Solution:
¾ of first group = ¾ of 60 = 45
2/3 of 2nd group = 2/3 of 60 = 40
Difference = 45-40 = 5
39. Six consecutive whole numbers are given. The sum of the first three numbers is 27. What is the sum of the last three numbers?
(A) 30
(B) 32
(C) 36
(D) 38
Solution:
Let,
Numbers are n, n+1, n+2
ATQ,
n+n+1+n+2 =27
Or, 3n+3=27
Or, 3n=24
Or, n=8
So, the numbers are-8,9,10
Now, the last 3 consecutive numbers are 11,12,13
Sum of them 11+12+13 =36
40. he areas
of the three plots increased?
(A) 125%
(B) 150%
(C) 200%
(D) 375%
Solution:
Let, the length of each side of the garden = 100
Area = 1002 = 10000
Area of 3 square = 3×10000 = 30000
50% increase of length = 100+100×50/100 = 150
Area = 1502 = 22500
Area of 3 square = 3×22500 = 67500
Increasing area = 67500-30000 =37500
Percentage = (37500×100)/30000=125%

ENGLISH
Questions (01-05): Fill in the blank with right option
01. The environmental damage caused by the oil spill will likely last -
(A) to several years
(B) severally years
(C) for several years
(D) year several
02. The man was in ______ health that the family began to consider whether he could continue to live in his home.
(A) so bad
(B) such worse
(C) so badly
(D) such bad
03. After suspended for misbehavior the student requested reconsideration.
(A) having been
(B) having
(C) have
(D) was
04. The oncologist studied the results of the biopsy and decided _____ additional tests.
(A) should order
(B) to order
(C) he should ordering
(D) ordering
05. One should be careful about ______ duty.
(A) their
(B) one
s
(D) the

Questions (06-09): Select the word or phrase that best completes the sentence.
06. Salt has been used for centuries as a method of preserving foods.
(A) displaying
(B) cooking
(C) conserving
(D) seasoning
07. The presentation by Dr. Toha was very illuminating.
(A) enlightening
(B) bright
(C) disheartening
(D) boring
08. Assertive people are generally more decisive.
(A) Docile
(B) Articulate
(C) Confident
(D) Compliant
09. abundant.
(A) inadequate
(B) substandard
(C) excellent
(D) plentiful

Questions (10-13): Select the pair of best expresses a relationship similar to that expressed in the pair
10. NATURAL : SYNTHETIC
(A) Cotton: wool
(B) Fragrance: perfume
(C) Boast: Humble
(D) None of these
11. MONEY: PECULATION
(A) Radiation : Bomb
(B) Bank: Cashier
(C) Comment: Insult
(D) Writing : Plagiarism
12. EYES : TEARS
(A) Sea : Water
(B) Volcano : Lava
(C) Heart: Artery
(D) Hunger : Bred
13. INSULT : HUMILIATE
(A) Shoot : Kill
(B) Abuse : Disrespect
(C) Injury : Pungent
(D) None of these

Questions (14-18): Choose the correct spelling from the given options.
14.
(A) pretyier
(B) prettyer
(C) pretiyer
(D) prettier
15.
(A) scurryd
(B) scurried
(C) scurryied
(D) scuried
16.
(A) ugliest
(B) ugliiest
(C) ugleest
(D) uggliest
17.
(A) Leiutenant
(B) Luietenent
(C) Lieutanent
(D) Lieutenant
18.
(A) Heiritage
(B) Heritage
(C) Heratage
(D) Heretage

BANGLA
19.
(A)
(B)
(C)
(D)
20.
(A)
(B)
(C)
(D)
21.
(A)
(B)
(C)
(D)
22. ?
(A)
(B)
(C)
(D)
23. ?
(A)
(B)
(C)
(D)
24. ?
(A)
(B)
(C)
(D)
25. ?
(A)
(B)
(C)
(D)
26. ?
(A)
(B)
(C)
(D)
27. ?
(A)
(B)
(C)
(D)
28. ?
(A)
(B)
(C)
(D)
29.
(A)
(B)
(C)
(D)
30. ?
(A) >
(B) >
(C) >
(D)
31. ?
(A)
(B)
(C)
(D)
32. ?
(A)
(B)
(C)
(D)
33. ?
(A)
(B)
(C)
(D)
34.
(A)
(B)
(C)
(D)

MATHEMATICS
Questions (35-60): Choose the correct answer
35. Karim has 40% more stamps than Rahim if he gives 45 of his stumps to Rahim, then Rahim will have 10% more stamps than Karim.
How many stamps did Karim begin with?
(A) 175
(B) 200
(C) 220
(D) 245
Solution:
Let,
Rahim has = x stamps
So Karim has = 140% of x = 1.4x
ATQ,
110% of (1.4x-45) = x+45
Or, {11(1.4x 45)/10} = x + 45
or, 15.4x-495 = 10x+450
or, 5.4x = 945
Or, x= 175
So, Karim has 175+ 40% of 175 = 175+70 = 245
36. P & Q start a business with initial investments in the ratio of 13:8. Their corresponding profits are in the ratio of 7:5. If P invested
his money for 7 months, find the time period for what Q invested his money.
(A) 8 months
(B) 9 months
(C) 10 months
(D) 11 months
Solution:
Let,
Q invested for x months
ATQ,
(13×7) : (8×x) = 7:5
Or, 91/8x = 7/5
Or, 8x = 91×5/7
Or, x = 65/8=8.125 =8
37. Compound interest on a certain sum for 2 years at 10% per annum is Tk.420. What would be the simple interest at the same rate
and for the same time?
(A) Tk100
(B) Tk.200
(C) Tk.300
(D) Tk.400
Solution:
Let,
Principal =P
ATQ,
P × (|1+ 10/100)2 P = 420
Or, P(11/10)2 P = 420
Or, P(1.1)2 P = 420
Or, 1.21P P = 420
Or, P(1.21 1) = 420
Or, 0.21P = 420
Or, P = 2000
Simple interest = 2000 × 2 × 10/100 = 400
38. Ravi leaves home for stadium which is 12 km from his house. After the stadium, he goes to his club which is 7 km from his
stadium. If his home, stadium and club all fall in a line, then what is the minimum distance he has to travel to get back home?
(A) 19 km
(B) 7 km
(C) 5 km
(D) 12 km
Solution:
His last destination was club. This club may be either side of stadium and in a straight line. So, minimum distance = 12-7 = 5km
39. If a man cycles at 10 km/hr, then he arrives at a certain place at 1 pm. If he cycles at 15 km/hr, arrive at the same place at 11 am.
At what speed must he cycles to go there at noon?
(A) 11 km/hr
(B) 12 km/hr
(C) 13 km/hr
(D) 14 km/hr
Solution:
Let,
distance = x
Time difference(Same Day) = 1 PM. 11 AM. = 2 hours.
ATQ,
x/10 x/15 = 2
Or, x/30 = 2
Or, x = 60
30
.:.x = 60
When speed is 10km/hr then time taken = 60 ÷10 = 6 hours.
So, at 10km/hr, his starting time was = 1PM. 6 hours = 7 AM(Same Day)
So, to reach at noon or 12 P.M, total time needed = 12 P.M. 7 A.M = 5hours.
Thus, speed = 60 ÷ 5 = 12km/hr.
41. Three measuring rods are 64 cm. 80 cm and 96 cm length. What is the least of cloth that can measure exact number of rods using
any one of these rods?
(A) 9.60 m
(B) 8.0 m
(C) 9.60 cm
(D) 96 m
Solution:
L.C.M of 64 80 and 96 = 960 cm
100 cm = 1 m
or 960 cm = 9.6 m
7. Which of the following equals (√2 -√8)(2√2 +√8)?
(A) -2√2
(B) -4
(C) 4√2
(D) -8
Solution:
(√2 -√8)(2√2 +√8)
=(√2 -2√2)(2√2 +2√2)
=4√2(√2 -2√2)
=4√2.√2(1 2)
= 4√2.√2
= 8
42. A tank can be filled by a tap in 20 minutes and by another tap in 60 minutes both the taps are kept open for 10 minutes and then
the first tap is shut off. After this, the tank will completely filled in what time?
(A) 10 min
(B) 15 min
(C) 20 min
(D) 25 min
Solution:
In 1 min both taps fill = 1/20 + 1/60 = 4/60 = 1/15
So, in 10 minutes taps fill = 10 × 1/15 = 2/3
Remaining part = 1 2/3 = 1/3
After shutting off 1st tap,
2nd tap fills 1/3 part in 60 × 1/3 minutes = 20 minutes
43. A box contains 20 electric bulbs, out of which 4 are defective. Two bulbs are chosen at random from this box. The probability that
at least one of these is defective is
(A) 7/19
(B) 6/19
(C) 5/19
(D) 3/19
Solution:
Here, Total bulbs = 20
defective bulbs = 4
so, non defective bulbs = 20-4 = 16
Probability = 16C2 / 20C2 = (16 × 15)/2 ÷ (20 × 19)/2 = 120/190 = 12/19
.:. at least 1 defective = 1 12/19 = 7/19
44. There is a rectangular parking lot with a length of 2x and a wide of x. What is the ratio of the perimeter of the parking lot to the
area of the parking lot, in terms of x ?
(A) 2/x
(B) 3/x
(C) 4/x
(D) 5/x
Solution:
Perimeter =2(2x+x) = 6x
Area = 2x × x = 2×2
Ratio = 6x:2×2 = 3:x = 3/x
45. The ratio of income of A to that of B is 7:5 and expenditure of A to that B is 3 : 2. If at the end of the year, each saves Tk. 500,
find the income of A?
(A) Tk. 2500
(B) Tk. 300
(C) Tk. 3200
(D) Tk. 3500
Solution:
Let, income of A and B = 7x and 5x
And expenditure = 3y and 2y respectively
ATQ,
7x-3y = 500 (i)
5x-2y = 500 (ii)
From (ii) × 3 (i) × 2,
15x 6y -14x + 6y = 1500 1000
Or, x = 500
So, income of A = 7×500 =Tk. 3500
46. An empty bucket (Foto) being filled with paint at a constant rate takes 6 min to be filled 7/10 of its capacity. How much more time
will it take to fill the bucket to full capacity
(A) 18/7
(B) 6/19
(C) 20/14
(D) 18/14
Solution:
7/10 tool 6 m
So, 1 took 6 × 7/10 m
So, 3/10 took 6 × 7/10 × 3/10 m = 18/7 m
47. In distributing milk at a summer camp it is found that a quart of milk will fill either 3 large glass tumblers or 5 small glass tumblers.
How many small glass tumblers can be filled with one large glass tumbler?
(A) 11/5
(B) 7/5
(C) 5/3
(D) 7/3
Solution:
3 large = 5 small
So, 1 large = 5/3 small
48. A bank offers 5% interest compounded half yearly. A customer deposits Tk.1600 each on 1st January and 1st. July of a year. At the
end of the year the amount he would have gained by the way of interest is
(A) Tk. 121
(B) Tk. 122
(C) Tk. 123
(D) Tk. 124
Solution :
Total amount = 1600(1+5/2 %)2 + 1600(1+5/2 %)
=1600{1+ 5/(2×100)}2 + 1600{1+ 5/(2×100)}
= 1600(1+1/40)2 +1600(1+1/40)
= 1600 × 41/40 × 41/40 + 1600 × 41/40
= 1600 × 41/40 (41/40 + 1)
= 1600 × 41/40 × 81/40 =3321
Gain = 3321 (1600+1600) = 121
49. A box is made in the form of a cube. If a second cubical box inside dimensions three times those of the first box, how many times
as much does the second box contain?
(A) 6
(B) 9
(C) 12
(D) 27
Solution:
Let,
one side of first cube = a
So, Volume of first cube = a3
ATQ,
One side of the 2nd cube = (3a)3 = 27a3
Times = 27a3 / a3 = 27
50. A can do a job in 24 days, B in 9 days and C in 12 days. B and C together start the work but leave after 3 days. How much time
was taken A to complete remaining work?
(A) 7 days
(B) 9 days
(C) 10 days
(D) 12 days
Solution:
B+C do in 1 day = 1/9 + 1/12 = 7/36
B+C do in 3 days = 3 × 7/36 = 7/12
Remaining = 1 7/12 = 5/12
A does 1 part in 24 days
So, A does 5/12 part in 24 × 5/12 = 10 days
51. In a group of 15, 7 can speak Spanish 8 can speak French and 3 can speak neither.
What fraction of the group can speak both French and Spanish?
(A) 1/5
(B) 4/15
(C) 1/3
(D) 7/15
Solution:
n(SUF) = n(S) + n(F) n(S∩F) + none
Or, 15 = 7 + 8 n(S∩F) + 3
Or, n(S∩F) = 3
The fraction = 3/15 = 1/5
52. A, B and C enter into partnership by making investments in the ratio 3: 5:7. After a
year, C invests another Tk.337600 while A withdraws Tk.45600. The ratio of investments then changes to 24:59:67. How much does A
invest initially?
(A) Tk. 140600
(B) Tk.141600
(C) Tk.131600
(D) Tk.140500
Solution:
Let,
investment ratio A, B & C = 3x, 5x, 7x
ATQ,
3x-45600 : 5x = 24 : 59
Or, (3x 45600)/5x = 24/59
Or, 177x 59×45600 = 120x
Or, 57x = 59×45600
Or, x = 47200

53. A rectangular fish tank 25m by 9m has water in it to a level of 2m. This water is carefully poured into a cylindrical container with a
diameter of 10m. How high will the water reach in the cylindrical container?
(A) 18/π
(B) 18
(C) 7/18
(D) 9/27
Solution:
Given,
Diameter 10 m
So radius = 10/2= 5m
ATQ,
πr2h = 25 × 9 × 2
Or, π52h = 25 × 9 × 2
Or, h = (25 × 9 × 2) / 25 π
Or h= 18/π
54. At a symposium, 20% of the professors are psychologists, 60% are biologists and the remaining 12 professors are economists. If 20
of the professors wear glasses, what percent of the professors do not wear glass?
(A) 20%
(B) 33,%
(C) 50%
(D) 66%
Solution:
Let,
Total professors = x
ATQ,
x = 20% of x + 60% of x + 12
Or, x = x/5 + 3x/5 + 12
Or, x x/5 3x/5 = 12
Or, x/5 = 12
Or, x = 60
20 = 60 20 = 40
Required percentage = (40/60) × 100 = 66 2/3% = 66%
55. In a department, 3/5 of the workers are men and the rest are women. If 1/2 of the men and 3/7 of the women in the department
are over 35, what fraction of all the workers in the department are over 35
(A) 33/70
(B) 66/70
(C) 33/140
(D) 35/140
Solution:
Men = 3/5
Women = 1 3/5 = 2/5
Workers over 35 are = ½ of 3/5 + 3/7 of 2/5 = 3/10 + 6/35 = 33/70
56. The ratio of investments of two partners P and Q is 7:5 and the ratio of their profits is 7:10. If P invested the money for months,
find for how much time did Q invest the money?
(A) 7 months
(B) 10 months
(C) 9 months
(D) 11 months
Answer: (Wrong question)
Solution:
Let,
Q invested for x months
ATQ,
7×y : 5×x = 7:10
Or, 7y/5x = 7/10
Or, x= (10/7) × 7y × 1/5 = 2y
y
57. many times will it strike in 24 hours?
(A) 78
(B) 136
(C) 156
(D) 196
Solution:

= 12(l2 + 1)/2 = 156


58. The perimeter of a rectangle is 26 cm. The rectangle is converted to a square by tripling the width and taking a quarter of the
length, What is the perimeter of the resulting square?
(A) 9 cm
(B) 12 cm
(C) 20 cm
(D) 26 cm
Solution:
2(length + breadth) = perimeter
Or, 2(L + B) = 26
Or, L + B = 13
Or, B = 13 L
New L of square = 3L & B = (13-L) × ¼
ATQ,
3L = (13-L) × ¼
Or, 12L = 13 L
Or, L = 1
So, length of the square = 3×1 = 3
So, perimeter of the square = 4×3 =12
59. In an examination, 34% of the students failed in mathematics and 42% failed in English. If 20% of the students failed in both the
subjects, then find the percentage of students who passed in both the subject.
(A) 22%
(B) 32%
(C) 44%
(D) 54%
Solution:
n(MUE) = n(M) + n(E) n(M∩E) + passed in both
Or, 100% = 34% + 42% 20% + passed in both
Or, passed in both = 44%
60. 125 gallons of a mixture contains 20% water. What amount of additional water should be added such that water content is raised to
25%?
(A) 15/2 gallons
(B) 17/2 gallons
(C) 19/2 gallons
(D) 25/3 gallons
Solution:
Water in the mixture = 20% of 125 = 25 gallons
So, other = 125-25 = 100
let, water be added = x
ATQ,
25+x = 25% of (125+x)
Or, 25 + x = (125+x)/4
Or, 100+4x =125+x
Or, 3x = 25
Or, x = 25/3.

GENERAL KNOWLEDGE AND COMPUTER


Questions (61-80): Choose the correct answer
61. The process of transferring files from a computer on the Internet to your computer is called
(A) Forwarding
(B) FTP
(C) Downloading
(D) Uploading
62. Verification of a login name and password is known as:
(A) configuration
(B) authentication
(C) accessibility
(D) logging in
63. A special type of memory chip that holds software that can be read but not written to
(A) RAM
(B) Mother Board
(C) ROM
(D) CPU
64. The set of instructions that tells the computer what to do is
(A) Softcopy
(B) Hardware
(C) Software
(D) Hardcopy
65. Which of the following will not protect you from spam?
(A) spam blockers
(B) e-mail rules
(C) filters
(D) popup blocker
66. ________ controls the way in which the computer system functions and provides a means by which users can interact with the
computer.
(A) The operating system
(B) The motherboard
(C) The platform
(D) Application software
67. All of the following are examples of real security and privacy risks EXCEPT.
(A) Viruses
(B) Hackers
(C) Spam
(D) Identity theft
68. Which of the following is used for close a tab on a browser?
(A) Ctrl+Y
(B) Ctrl+W
(C) Ctrl + A
(D) Ctrl+T
69. When cutting and pasting, cutting section is temporarily stored in
(A) Dashboard
(B) Hard drive
(C) Diskette
(D) Clipboard
70. Which of the following is a correct format of Email address?
(A) care@website.com
(B) care@website@com
(C) carewebsite.com
(D) care.website.com
71. The Finance Commission is appointed after every
(A) 2 years
(B) 3 years
(C) 4 years
(D) 5 years
72. The system of dual citizenship exists in
(A) India
(B) France
(C) United Kingdom
(D) USA
73. Point of sale (POS) machine is widely used by the
(A) Tellers
(B) Merchants
(C) Government
(D) Central bank
74. Which of the following organization is concerned with environmental issues?
(A) OIC
(B) IPCC
(C) MIGA
(D) WMO
75. The device used for measuring altitudes is
(A) altimeter
(B) ammeter
(C) audiometer
(D) galvanometer
76. The value of Gold is determined in
(A) Rome
(B) Washington
(C) Teheran
(D) London
77. Where is the headquarters of Asian Development Bank (ADB) situated?
(A) Manila
(B) Delhi
(C) Bangkok
(D) Hong Kong
78. Which one of the following glasses is used in bulletproof screens?
(A) Soda glass
(B) Pyrex glass
(C) Jena glass
(D) Reinforced glass
79. Which is the Land of the Rising Sun?
(A) Japan
(B) Australia
(C) China
(D) Taiwan
80. The longest mountain range in the world is
(A) The Alps
(B) The Himalayas
(C) The Andes
(D) The Rockies
ENGLISH
Questions (01-05): Fill in the blank with right option
1. The department of heavy industries plans to ______ a vibrant ecosystem for the capital goods sector to ______ manpower issues.
(A) organize, unfold
(B) export, resolve
(C) create, address
(D) start, decide
2. The emergency services ______ the accident site as soon as the news of the train collision was _______ to them.
(A) reached, relayed
(B) suspended, omitted
(C) carried. postponed
(D) marked, transferred
3. Market leaders usually want to ____ their market share even further, or at least to protect their current market share.
(A) decrease
(B) dominate
(C) increase
(D) establish
4. _________ some employers oppose the very existence of unions, many theorists stress the necessity of unions.
(A) Because
(B) Due to
(C) However
(D) Although
5. She stands a good chance, _____ only two people are contesting the seat and the other candidate is very unpopular.
(A) while
(B) since
(C) but
(D) and

Questions (06-10): Find the correctly spelt word


06.
(A) Waitres
(B) Waitress
(D) Waitres
(C) Waittress
07.
(A) Tentamount
(B) Tantramount
(C) Tantemount
(D) Tantamount
08.
(A) Quotient
(B) Quoteient
(C) Quetient
(D) Quetient
09.
(A) Vertebrete
(B) Vertebratte
(C) Vertebrate
(D) Verrtebrate
10.
(A) Judiceary
(B) Judiciary
(C) Judeacry
(D) Judiciery

Questions (11-15): Read the following passage and answer the questions given below:
Eight persons A, B, C, D, E, F, G and H have their birthdays on different months of the year viz. January, April, May and July, such that
not more than two persons have their birthdays in the same months. All the birthdays are either on 14th or 23rd of the month. No two
persons have their birthdays on the same day of the same month. The following information is also known about them:

number of persons who have their birthdays between the birthdays of G and H is equal to the number of persons who have their
birthdays between the birthdays of B and D. D was born in July. Birthdays of both E and B are in the same month. There are three
birthdays between the birthdays of F and C.
11. Who among the following were born in the same month?
(A) A and D
(B) B and C
(C) C and F
(D) D and G
Exp:

Jan April May July

14 23 14 23 14 23 14 23

G F E B H C A D

12. Who was born on 23rd July?


(A) A
(B) B
(C) C
(D) D
Exp:

Jan April May July

14 23 14 23 14 23 14 23

G F E B H C A D

13. Which of the following Birthday Person combination is correct?


(A) January 23rd G
th
(B) April 14 E
(C) May 23rd H
(D) July 14″ C
Exp:

Jan April May July

14 23 14 23 14 23 14 23

G F E B H C A D

14. In which month C has his birthday?


(A) January
(B) April
(C) May
(D) July
Exp:

Jan April May July

14 23 14 23 14 23 14 23

G F E B H C A D

15. How many people have birthdays before G?


(A) 1
(B) 3
(C) 5
(D) None of these
Exp:

Jan April May July

14 23 14 23 14 23 14 23

G F E B H C A D

Questions (16-20): Choose the word that is closest in meaning to the underlined word.
16. She came to the meeting late on purpose so she would miss the introductory speech.
(A) aiming at
(B) intentionally
(C) reasonably
(D) with a goal
17. Travel insurance is sometimes mistaken for temporary health insurance, but the two are different.
(A) transitory
(B) passing
(C) mutable
(D) permanent
18. Innocuous looking mushroom you see around.
(A) harmful
(B) harmless
(C) innocent
(D) conspicuous
19. When the leadership changed, his position in the organization became precarious.
(A) secure
(B) exalted
(C) important
(D) uncertain
20. There was no doubt that the judgment was fair.
(A) insincere
(B) biased
(C) upright
(D) inconsiderate

BANGLA
Question (21-40): Choose the correct answer
21. ?
(A)
(B)
(C)
(D)
22. ?
(A)
(B)
(C)
(D)
23. ?
(A)
(B)
(C)
(D)
24. ?
(A)
(B)
(C)
(D)
25. ?
(A)
(B)
(C)
(D)
26. ?
(A)
(B)
(C)
(D)
27. ?
(A)
(B)
(C)
(D)
28. ?
(A)
(B)
(C)
(D)
29. ?
(A)
(B)
(C)
(D)
30. ?
(A)
(B)
(C)
(D)
31. ?
(A)
(B)
(C)
(D)
32. ?
(A)
(B)
(C)
(D)
33. ?
(A) √
(B) √
(C) √
(D) √
34. ?
(A)
(B)
(C)
(D)
35. ?
(A)
(B)
(C)
(D)
36. ?
(A)
(B)
(C)
(D)
37. ?
(A)
(B)
(C)
(D)
38. ?
(A)
(B)
(C)
(D)
39. ?
(A)
(B)
(C)
(D)
40. ?
(A)
(B)
(C)
(D)
MATHEMATICS
Questions (41-60): Read the following questions carefully and choose the right answer.
41. There are 3 green, 4 orange and 5 white color bulbs in a bag. If a bulb is picked at random, what is the probability of having either
a green or a white bulb?
(A) 3/4
(B) 2/3
(C) 4/3
(D) 2/9
Solution:
Total bulbs = 3+4+5 = 12
green + white bulbs = 3+5 = 8
So, probability of having green or white bulbs = 8/12 = 2/3
42. A motor cycle covers 40 km with a speed of 20 km/hr. Find the speed of the motorcycle for the next 40 km journey so that the
average speed of the whole journey will be 30 km/hr.
(A) 70 km/hr
(B) 52.5 km/hr
(C) 60 km/hr
(D) 60.5 km/hr
Solution:
Average speed,
2xy/(x+y) = 30
Or, 2×20×y(20+y) = 30
Or, 40y = 600+30y
Or, 10y = 600
Or, y = 60
43. With a uniform speed, a car covers a distance in 8 hours. Had the speed been increased by 4 km/hr, the same distance could have
been covered in 7 hours and 30 minutes. What is the distance covered?
(A) 420 km
(B) 480 km
(C) 520 km
(D) 640 km
Solution:
Let,
Distance = x km
7 hours and 30 minutes = 7.5 hr
ATQ,
x/7.5 x/8 = 4
Or, 10x/75 x/8 = 4
Or, (80x 75x)/600 = 4
Or, 5x/600 = 4
Or, x= (4×600)/5 = 480 km
44. The respective ratio between the speed of a car, a train, and a bus is 5:9:4. The average speed of the car, bus and train is 72
km/hr together. What is the average speed of the car and the train together?
(A) 82 km/h
(B) 72 km/h
(C) 67 km/h
(D) 84 km/h
Solution:
Let the speed of car, train and bus = 5x, 9x , 4x km/hr
ΑΤQ,
(5x+9x+4x)/3=72
Or, 18x = 216
Or, x=216/18
Or, x= 12
Average speed of car and train = (5x+9x)/2=7x=7×12=84 km/hr
45. A, B and C are partners of a company. During a particular year A received one-third of the profit. B received one-fourth of the
profit and C received the remaining tk.5000. How much did A receive?
(A) Tk. 5000
(B) Tk 4000
(C) Tk 3000
(D) Tk. 1000
Solution:
Total Profit = x
So,
A received = x/3
B received = x/4
ATQ,
x x/3 x/4 = 5000
Or, (12x 4x 3x)/12 = 5000
Or, 5x/12 = 5000
Or, x = 12000
A received = 12000/3 = 4000
46. The average age of a group of 15 employees is 24 years. If 5 more employees join the group, the average age increases by 2 years.
Find the average age of the new employees.
(A) 35
(B) 30
(C) 24
(D) 32
Solution:
Total age of 15 employees = 15×24 = 360
After 5 more employees joining the group, the average age increases by 2 years.
That means, total age =(20×26) = 520
Total age of 5 employees =(520 360) = 160
So, average = 160/5 = 32
47. By selling 32 guavas for tk. 30 at the rate of Tk. 1.066 per guava a man loss 25%. How many guavas should be sold for Tk. 18 to
gain 20% of profit in the transaction?
(A) 24
(B) 12
(C) 18
(D) 36
Solution:
At 25% loss,
cost price of 32 guavas= 30×100/75 = 40 tk
at 20% profit,
selling price of 32 guavas=40×120/100=48 tk
So,
48 tk is the selling price of 32 guava
1 =32/48
18 =(32×18)/48=12 guavas
48. A sold a watch to B at a gain of 20% and B sold it to C at a loss of 10%. If C bought the watch for Tk.216, at what price did A
purchase it?
(A) Tk. 200
(B) TK. 216
(C) TK. 250
(D) Tk. 176
Solution:
At 10% loss,
Cost price to B = (216×100)/90 = 240 tk
At 20% gain,
Cost price to A = (240×100)/120 = 200 tk
49. A square is inscribed in a circle of diameter 2a and another square is circumscribing circle. The difference between the areas of
outer and inner squares is
(A) a2
(B) 2a2
(C) 3a2
(D) 4a2
Exp:
ATQ,
Diagonal of inner square = 2a
So, one hand of the inner square = 2a/√2 = √2a
So, area of the inner square = (√2a)2 = 2a2
Again,
One hand of the outer square = 2a
So, area of the outer square = (2a)2 = 4a2
So, difference = 4a2 2a2 = 2a2
50. The average of the three numbers x, y and z is 45. X is greater than the average of y and 2 by 9. The average of y and z by 9. The
average of y and z is greater than y by 2. Then the difference of x and z is
(A) 3
(B) 5
(C) 7
(D) 11
Solution:
Sum of x,y and z = 45×3
So, ∴x+y+z =135 (i)
Again,
X = (y+z)/2 +9
Or, 2x = y+z+18 -(2)
Again,
(y+z)/2 = y+2
Or, 2y+4 = y+z
Or, y = z-4
From (2),
2x = z-4+z+18
Or, 2x 2z = 14
Or, x-z = 7
51. A boat travel with a speed of 10 km/hr in still water. If the speed of the stream is 3 km/hr then find time taken by boat to travel 52
km downstream.
(A) 2 hrs
(B) 4 hrs
(C) 6 hrs
(D) 9 hrs
Solution:
Speed in downstream = 10+3 = 13km/hr
So, time taken to travel 52km downstream =52/13= 4hrs
52. Rahima can row 16 km/hr in still water. It takes her thrice as long to row up as to row down the river. Find the difference between
her speed in still water and that of the stream.
(A) 8 km/hr
(B) 16 km/hr
(C) 24 km/hr
(D) 12 km/hr
Solution:
Let, the speed of Stream = x,
so, downstream speed = 16+x and upstream speed = 16-x
ATQ,
16+x = 3(16-x)
Or, 16+x = 48 -3x
Or, 4x = 32
Or, x =8
So speed of stream = 8 and
difference of speed of still water & stream = 16 8 = 8km/hr
53. A train leaves a station A at 7 am and reaches another station B at 11:00 am. Another train leaves B at 8 am and reaches A at
11:30 am. The two ruins cross one another at
(A) 8:36 am
(B) 8:56 am
(C) 9.00 am
(D) 9:24 am
Solution:
Let, distance between A and B = x mile
So, Speed of first train = x/4 km/hr
Speed of second train = x/3.5 km/hr
Let after y hrs, two train will meet.
So,
x/4 ×(y+1) + x×y/3.5 = x
Or, (y+1)/4 = y/3.5 = 1
Or, (3.5y + 3.5 + 4y)/14 = 1
Or, 7.5y + 3.5 = 14
Or, 7.5y = 10.5
Or, y = 7/5 = 1 hr 24 min
So, the train will meet = 8+1hr 24 min = 9:24 Am
54. A train 150 m long crosses a milestone in 15 seconds and crosses another train of the same length travelling in the opposite
direction in 12 seconds. The speed of the second train in km/hr is-
(A) 52 km/hr
(B) 56 km/hr
(C) 54 km/hr
(D) 58 km/hr
Solution:
Speed of 1st train=150/15=10 m/s
Relative speed of two train = (150+150)/12 = 25 m/s
So speed of 2nd train = 25-10=15 m/s = 15×(18/5)=54 kmh
55. There are 2 numbers in the ratio of 4:5. If 4 is subtracted from both numbers the ratio becomes 3:4. What will be the ratio if 4 is
added in the both numbers?
(A) 1:2
(B) 2:3
(C) 5:6
(D) 1:4
Solution:
Let the numbers are 4x and 5x
ATQ,
(4x-4)/(5x-4) = ¾
Or, 16x 16 = 15x 12
Or, 16x 15x = 4
Or, x = 4
So, Required ratio = (16+4) : (20+4) = 20:24 = 5:6
56.
income?
(A) 7%
(B) 8%
(C) 9%
(D) 10%
Solution:
= 100

-25 = 75

A is less than C = C-A = 133.33-120 = 13.33


less % = (13.33×100)/133.33 = 10%
57. The ratio of two numbers is 7: 4. If 8 is added to both the numbers ratio becomes 13:8. What is the smaller number?
(A) 40
(B) 56
(C) 38
(D) 52
Solution:
Let, the number be 7x and 4x
ATQ,
(7x+8)/(4x+8) = 13/8
Or, 56x + 64 = 52x + 104
Or, 4x = 40
Or, x = 10
So, smaller number = 4x = 4×10 = 40
58. A and B can do a work in 12 days. B can do the same work in 18 days. In how many days A can complete the 2/3 of the same
work?
(A) 36 days
(B) 24 days
(C) 16 days
(D) 27 days
Solution:
A alone can do (1/12 1/18) = 1/36 part of the work
So, A alone can do 1/36 part of the work in 1 day
So, A alone can do 2/3 of the work in 36 × 2/3 day = 24 days
59. A is twice as good as B and together they finish a piece of work in 16 days. The number of days taken by A alone to finish the work
is
(A) 20 days
(B) 21 days
(C) 22 days
(D) 24 days
Solution:
Let, A needs = x days so, B needs 2x days
ATQ,
1/x + 1/2x = 1/16
Or, 3/2x = 1/16
Or, 2x = 48
Or, x = 24
60. A box contains 5 pink, 3 green and 2 yellow balls. Three balls are picked up randomly. What is the probability that none of the ball
drawn is green?
(A) 3/16
(B) 7/24
(C) 5/18
(D) 4/24
Solution:
Total number of balls = 5+3+2 = 10
total number of non green balls = 5+2 = (pink+yellow)
10
Total number of ways of selecting 3 balls from 10 balls = C3 = 120
Number of ways of selecting 3 non green balls from 7 non green balls = 7C3 = 35
So, required probability = 35/120 = 7/24

GENERAL KNOWLEDGE AND BASIC COMPUTER


Questions (61-80): Read the following questions carefully and choose the right answer.
61. Which of the following country has recently topped the E-Government Development Survey 2018?
(A) Sweden
(B) Australia
(C) Denmark
(D) United States
62. Which one of the following has recently launched reusable rocket to send a communication satellite for Indonesia into the orbit?
(A) ISRO
(B) Space X
(C) NASA
(D) JAXA
63. Which of the following city has recently topped the Global Liveability Index 2018?
(A) Melbourne
(B) Osaka
(C) Calgary
(D) Vienna
64. Where i
(A) Greenwich
(B) Indira Point
(C) New York
(D) Shriharikota
65. Head office of Coca Cola company is in-
(A) New York
(B) Atlanta
(C) New Jersey
(D) Florida
66. In absorption of insolation, the most significant part is played by
(A) Ozone
(B) Oxygen
(C) Haze
(D) Carbon dioxide
67. During periods of inflation, tax rates should
(A) Increase
(B) Decrease
(C) Remain constant
(D) Fluctuate
68. Who was the Founder of Bluetooth?
(A) Martin Cooper
(B) Ericson
(C) Steve Jobs
(D) Apple
69. Which country has adopted Bengali language the status of second state language?
(A) India
(B) Nepal
(C) Sierra Leone
(D) Uganda
70. Who is not a member of World Trade Organization?
(A) Bangladesh
(B) Morocco
(C) Mexico
(D) Russia
Answer: All are members
71. Information on a computer is stored as what?
(A) analog data
(B) modem data
(C) digital data
(D) None of these
72. A word in a web page that, when clicked, opens another document,
(A) Hyperlink
(B) Anchor
(C) Reference
(D) URL
73. Which of the following is not term of information technology?
(A) Cyber space
(B) Light storage
(C) Upload
(D) Modem
74. A collection of related information sorted and dealt with as a unit is a
(A) Disk
(B) Data
(C) File
(D) None of these
75. Documents, Movies, Images and Photographs etc. are stored at a
(A) Application Sever
(B) File Server
(C) Web Sever
(D) Print Server
76. USB is which type of storage device?
(A) Primary
(B) Secondary
(C) Tertiary
(D) None of these
77. Which function key is used to check spellings?
(A) F2
(B) F5
(C) F7
(D) FH
78. What is a portion of a document in which you set certain page formatting options?
(A) Section
(B) Page
(C) Document
(D) Page Setup
79. Which of the following is graphics solution for Word Processors?
(A) WordArt
(B) Clipart
(C) Drop Cap
(D) All of above
80. By default, on which page the header or the foot is printed?
(A) on first page
(B) on every page
(C) on alternate page
(D) None of these

English
Questions (01-05): Fill in the blank with right option
1. A defect can be caused _______ negligence by one of the members of a team.
(A) to
(B) by
(C) at
(D) in
2. _______ his being innocent of the crime, the judge sentenced him to one year imprisonment.
(A) In spite of
(B) In case of
(C) On account of
(D) In the event of
3. _______ pollution control measures are expensive; many industries hesitate to adopt them.
(A) Despite
(B) However
(C) Although
(D) Because
4. At times he gets very angry, and then no one can ______ him.
(A) prevent
(B) satisfy
(C) mollify
(D) humor
5. Google is one of the most popular search engines; it is ______ by the internet users.
(A) utilized
(B) effected
(C) examined
(D) flabbergasted

(06-10): Read the following passage and answer the questions given below it in the context of the passage.
A, B, C, D, E, G and I are seven friends who study in three different standards namely 5th, 6th and 7th such that not less than two
friends study in the same standard. Each friend also has a different favorite subject namely History, Civics, English, Bangla, Science,
Maths and Economics but not necessarily in the same under.
A takes Maths and studies in the 5th standard with only one other friend who likes Bangla. I studies with two other friends. Both the
friends who study with I like languages (Here languages include only Science, English and Bangla). D studies in the 6th standard with
only one person and does not like Civics. E studies with only one friend. The one who likes History does not study in 5th or 6th
standard. E does not like languages. C does not like English, Science or Civics.
6.
(A) Civics and 7th
(B) Economics and 5th
(C) Civics and 6th
(D) History and 7th
7.
(A) History
(B) Civics
(C) Bangla
(D) All are incorrect
8. Who amongst the following studies in the 7th standard?
(A) C
(B) G
(C) E
(D) Either D or B
9. Which of the following combinations is definitely correct?
(A) I and Science
(B) G and English
(C) C and Bangla
(D) B and Science
10. Which of the following subjects does G like?
(A) Either Math or Bangla
(B) Either Science or Civics
(C) Either Science of Bangla
(D) Either Science or English

Questions (11-15). Select the pair that best expresses a relationship similar to that expressed in the original pair
11. EXPLORE : DISCOVER
(A) read : skim
(B) research : learn
(C) write : print
(D) sleep : wake
12. JAUNDICE : LIVER
(A) rash : skin
(B) dialysis: kidney
(C) smog: lung
(D) valentine : heart
13. WAITRESS : RESTAURANT
(A) Doctor : Diagnosis
(B) Actor : Role
(C) Driver : Truck
(D) Teacher : School
14. LIGHT : BLIND
(A) Language : Deaf
(B) Speech : Dumb
(C) Tongue : Sound
(D) voice: vibration
15. BINDING : BOOK
(A) Nail : Hammer
(B) Criminal : Gang
(C) Frame : Picture
(D) Artist : Carpenter

Question (16-20): Select the word that is most closely similar in meaning to the original word.
16. ABSOLUTE
(A) Division
(B) Complete
(C) Small
(D) Half
17. AGGRAVATE
(A) Decline
(B) Acquire
(C) Excited
(D) Irritate
18. LATENT
(A) Apparent
(B) Manifest
(C) Concealed
(D) Obvious
19. Unfold
(A) Conceal
(B) Elaborate
(C) Withhold
(D) Maintain
20. Eccentric
(A) Unconventional
(B) Destructive
(C) Deceitful
(D) Conformist
BANGLA
Question (21-40): Choose the correct answer
21. ?
(A)
(B)
(C)
(D)
22.
(A)
(B)
(C)
(D)
23. ?
(A)
(B)
(C)
(D)
24.
(A)
(B)
(C)
(D)
25. ?
(A)
(B)
(C)
(D)
26.
(A)
(B)
(C)
(D)
27. ?
(A)
(B)
(C)
(D)
28. ?
(A)
(B)
(C)
(D)
29. ?
(A)
(B)
(C)
(D)
30. ?
(A)
(B)
(C)
(D)
31. ?
(A) -
(B) -
(C) -
(D) -
32. ?
(A) +
(B) +
(C) +
(D) +
33. ?
(A) +
(B) +
(C) +
(D) +
34.
(A)
(B)
(C)
(D)
35. ?
(A)
(B)
(C)
(D)
36. ?
(A)
(B)
(C)
(D)
37. , , ?
(A)
(B)
(C)
(D)
38. ?
(A)
(B)
(C)
(D)
39. ?
(A)
(B)
(C)
(D)
40.
(A)
(B)
(C)
(D)

MATHEMATICS
Questions (41-60): Read the following questions carefully and choose the right answer.
41. If the numerator of fraction is increased by 200% and the denominator is increased by 350%. The resultant fraction is 5/12. What
was the original fraction?
(A) 5/7
(B) 5/8
(C) 7/32
(D) 11/82
Solution:
Let numerator of the fraction is X and denominator is Y, then fraction will be X/Y.
Numerator increased 200%, so new numerator would be,
[X + (200 of X)/100] = 3X ----------------------------- (1)
New denominator,
[Y + (350Y)/100 ] = 45Y/10 --------------------------------- (2)
Now, accordibng to question,
3X /(45Y/10) = 5/12
30X / 45Y = 5/12
360X = 225Y
X/Y = 225/360
X/Y = 5/8
So, Original fraction was 5/8.
42. In a mixture, the ratio of the milk and water is 6: 5. When 22 liter mixture is replaced by water, the ratio becomes 9:13. Find the
quantity of water after replacement.
(A) 62 liter
(B) 50 liter
(C) 40 liter
(D) 52 liter
Solution:
Let milk = 6x, water = 5x
According to the question,
6x-22 × 6/11 : 5x 22 × 5/11 + 22= 9 : 13
6x 12 : 5x 10 + 22 = 9 : 13
13 (6x 12) = 9 (5x + 12)
78x 156 = 45x + 108
78x 45x = 156 + 108
33x = 264
x=8
Water after replacement = 5 × 8 10 + 22 = 40 + 22 = 52 litre
43. A man covers a total distance of 100 km on bicycle. For the first 2 hours, the speed was 20 km/hr and for the rest of the journey, it
came down to 10 km/hr. The average speed will be
(A) 12.5 km/hr
(B) 13 km/hr
(C) 14.5 Km/hr
(D) 20 Km/hr
Solution:
Distance covered in 1st 2 hr.
= 2×20 = 40 km
Time taken to cover 60 km
distance =60/10=6 hr.
Average speed=Total distance/Total time
=100/(2+6)=100/8=12.5 km/hr
44. A man can reach a certain place in 40 hours. If he reduces his speed by 1/15th, he goes 5 km less in that time. Find the total
distance covered by him.
(A) 60 km
(B) 85 km
(C) 52 km
(D) 75 km
Solution:
Let the speed be x km / hr. Then,
40x - 40 × 14x/15 = 5
x/15 =1/8
x=15/8
So distance covered = 40 × 15/8 = 75km
45. A shopkeeper gains 17% after allowing a discount of 10% on the marked price of an article. Find his profit percent if the article is
sold at marked price allowing no discount.
(A) 30%
(B) 37%
(C) 23%
(D) 27%
Solution:
Let the cost price = 100tk
Therefore, Selling price = 100×117/100=117tk
Therefore, Marked price = 117×100/90=130
Therefore, Profit Percentage = [(130−100)/100]×100=30%
46. Alam sold two vehicles for Tk 46000 each. If he gained 10% on the first and lost 10% on another, then what is his percentage profit
or loss in this transaction?
(A) 2% loss
(B) 1% profit
(C) 1% loss
(D) None of these
Solution:
Let the profit be X% and loss be Y% . So,
Net profit or loss% = X +(-Y) + X×(-Y)/100
(Negative sign denotes that their is a loss)
Their is 10% loss and 10% profit then
Net profit or loss% = 10+(-10)+10×(-10)/100 =10 10 100/100 = 0 1 = -1
( negative sign here shows that there is a net loss )
So the net loss percentage is 1% .
47. The profit earned after selling an article for Tk.3,362 is the same as the loss incurred after selling the article for Tk.2,346. At what
selling price will a trader make a 20% profit on this article?
(A) 4639.4
(B) 4769.6
(C) 4830.8
(D) None of these
Solution:
Here , Cost price = (3362-2346)/2 = 2854
So , Selling price at 20% profit = 2854*(6/5) = 3424.8 Rs (Ans.)
48. An article costs TK 500 and the marked price is mentioned as Tk.800. What is the profit % for the seller if he sells and offers a
discount of 10% on the marked price?
(A) 30%
(B) 44%
(C) 56%
(D) 64%
Solution:
CP = TK. 500
MP = TK. 800
Discount = 80 TK.
SP = (800 - 80) = 720 Tk.
Profit % = (720 -500)/500 × 100
= 220/500 × 100
= 44%
49. The average of the first and the second of three numbers is 15 more than the average of the second and the third of these
numbers. What is the difference between the first and the third of these three numbers?
(A) 15
(B) 45
(C) 60
(D) None of these
Solution:
x + z/2 = 15 + (y + z )/2
x + z = 30+(y + z )
x-y =30
50. 30 pens and 75 pencils altogether were purchased for Tk.510. If the average price of a pencil was Tk 2, what was the average price
of a pen?
(A) Tk. 9
(B) Tk. 10
(C) Tk.11
(D) Tk. 12
Solution:
According to question
30 pens + 75 pencils = TK. 510
Average price of a pencil = TK. 2
Price of 75 pencils = 2 × 75 = TK. 150
Price of 30 pens = 510 - 150 = TK. 360
Average price of pen = 360/30 = TK. 12
51. A boat travel with a speed of 10 km/hr in still water. If the speed of the stream is 3 km/hr then find time taken by boat to travel 52
km downstream.
(A) 2 hrs
(B) 4hrs
(C) 8hrs
(D) 9 hrs
Solution:
Downstream speed = 10 + 3 = 13 km/hr
So time needed = 52/13 = 4 hours
52. The ratio of the speed of boat in still water to the speed of stream is 16:5. A boat goes 16.5 km in 45 minute upstream, find the
time taken by boat to cover the distance of 17.5 km downstream.
(A) 30 minutes
(B) 25 minutes
(C) 50 minutes
(D) 45 minutes
Solution:
Let the speed of boat in still water = 16x, speed of stream =5x
Upstream speed = 16x 5x = 11x
S =D/t
11x = 16.5/45 × 60
x=2
speed of boat in still water = 32 km/h,
speed of stream = 10 km/h
Downstream speed = 32 + 10 = 42 km/h
Distance = 17.5 km
time = 17.5/42
= 5/12 hour
or , 5/12 × 60 = 25 minutes
53. A train passes a platform in 40 sec and a woman standing on the platform in 30 sec. If the speed of the train is 108 km/hr, what is
the length of the platform?
(A) 100 m
(B) 300 m
(C) 900 m
(D) 1020 m
Solution:
Let the length of train L and length of platform P
Speed of train 108 km/h =108 * 5/18 m/s =30 m/s
It passes woman on platform in 30 sec so
L=30×30=900m (distance =speed*time)
now when it crosses the platform effective distance becomes L+P
hence , L+P= 40×30 =1200 m
P = 1200-900 = 300
Lenth of platform =300 m
54. A train 125 m long passes a person, running at 8 km/hr in the same direction in which the train is going in 25 seconds. The speed
of the train is:
(A) 22
(B) 36
(C) 30
(D) 26
Solution:
Speed of the train relative to man = 125/25 m/sec
=( 5 × 18/5 ) km/hr
=18 km/hr
Let the speed of the train be x km/hr. Then, relative speed = (x - 8) km/hr.
x - 8 = 18
x = 26 km/hr.
55. There are two numbers. 1st number is 12 more than the 2nd number. The average of the two numbers is 19. If 2 is added in both
numbers, find the ratio of the numbers.
(A) 5:9
(B) 11:9
(C) 4:9
(D) 9:5
Solution:
Sum of two numbers = 2×19 = 38
Let the 1st number = x
So, 2nd number = x-12
Atq,
X+x-12 =38
X= 25
Required ratio = (x+2)/(x-12+2) = 27 : 15 = 9 : 5
56. Sum of 4 consecutive even numbers is greater than three consecutive odd numbers by 81. If the sum of the least odd and even
numbers is 59 then find the sum of largest odd and even numbers
(A) 69
(B) 53
(C) 65
(D) 72
Solution:
4 consecutive even numbers are X + (X+2) + (X+4) + ( X+6) = 4X + 12
3 consecutive odd numbers are Y + (Y+2) + (Y+3) = 3Y + 6
ATq,
4X + 12 = 81 + 3Y + 6
4X - 3Y = 75
Again,
X + Y = 59
By solving the equations 4X - 3Y = 75 and X + Y = 59 we get X = 36
from X + Y = 59 we get Y = 23
so numbers are 36, 38, 40, 42 and 23, 25, 27
and the sum of largest odd and even numbers = 42 + 27 = 69
57. A number is as much greater than 36 as is less than 86. Find the number.
(A) 68
(B) 43
(C) 61
(D) 73
Solution:
Let the number = x
ATQ,
X 36 = 86 x
X= 61
58. X, Y and Z can do a piece of work in 6 hours. X and Y can do the work in 8 hours and Y and Z can do the same work in 12 hours.
In how many time will X and Z do the work?
(A) 6 hours
(B) 8 hours
(C) 10 hours
(D) 12 hours
Solution:
X+Y+Z 1 hours work = 1/6
X+Y 1 hours work = 1/8
Y+Z 1 hours work = 1/12
1/6 = 1/24
1/24 = 3/24
Hence X and Z will complete the work in = 24/3 = 8 hours
59. If A can do ¼ of a work in 3 days and B can do 1/6 of the same work in 4 days, how much will A get if both work together and are
paid Tk. 180 in all?
(A) Tk. 60
(B) Tk. 120
(C) Tk. 90
(D) Tk. 180
× 1/4 = 1/12
× 1/6 = 1/24
Ratio of work A : B = 1/12 : 1/24 = 2 : 1
So, A will get = 180 × 2/3 = Tk. 120
60. In a box, there are 10 apples and 2/5th of the apples are rotten. If three apples are taken out from the box, what will be the
probability that at least one apple is rotten.
(A) 3/4
(B) 5/6
(C) 5/8
(D) 8/13
Solution:
2
Let rotten apples = 10 × = 4, others = 6
5
If 1 apple is rotten + 2 apples are other
4
= C1 × 6C2 = 60
If 2 apples are rotten + 1 apple is other
4
= C2 × 6C1 = 36
If 3 apples are rotten
4
= C3 = 4
10
Total outcomes = C3 = 120

60 + 36 + 4
Probability =
120

100 5
= =
120 6

GENERAL KNOWLEDGE
Questions(61-70): Study the following questions carefully and answer the questions given below
61. Which of the following countries as has successfully test launched its first hypersonic aircraft?
(A) India
(B) China
(C) Russia
(D) United States
62. Which of the following countries has recently topped the Global Innovation Index for the eighth consecutive year in 2018?
(A) Switzerland
(B) Sweden
(C) Netherlands
(D) Singapore
63. Which of the following has recently hosted the opening ceremony of the 2018 Asian Games?
(A) Vietnam
(B) India
(C) Malaysia
(D) Indonesia
64. Which of the following countries does not have a border with Afghanistan?
(A) Iran
(B) Georgia
(C) Turkmenistan
(D) Uzbekistan
65. Which one among the following African countries is not landlocked?
(A) Zambia
(B) Uganda
(C) Angola
(D) Zimbabwe
66.
(A) Term Deposit
(B) Saving Bank Deposit
(C) Current deposit
(D) Demand deposit
67. The branch of science dealing with the study of collection of coins, banknotes, and medals is known as-
(A) Aphnology
(B) Numismatics
(C) Chrematistics
(D) Accountancy
68. The Uber Cup is associated with which sports?
(A) Cricket
(B) Football
(C) Badminton
(D) Tennis
69. -
(A) USA
(B) South Africa
(C) England
(D) Russia
70. When did World War 2 start?
(A) 1937
(B) 1938
(C) 1939
(D) 1940

COMPUTER
71. What is the main folder on a storage device called?
(A) Platform
(B) Interface
(C) Root Directory
(D) Device driver
72. Which of the following refers to a small single site network?
(A) LAN
(B) DSL
(C) RAM
(D) USB
73. The process of a computer receiving information from a server on the internet is known as
(A) Pulling
(B) Pasting
(C) Transforming
(D) Downloading
74. The portion that shows all the choices you can make while working in a window is called the
(A) Options
(B) Menu bar
(C) Table
(D) None of these
75. WWW stands for
(A) World Whole Web
(B) Wide World Wave
(C) World Wide Web
(D) Web World Wide
76. Which device among following is used for sending digital data over a phone line?
(A) USB
(B) Modem
(C) Scanner
(D) Printer
77. Which of the following is spreadsheet program?
(A) Ms word
(B) Ms power point
(C) Ms excel
(D) Ms Access
78. What is the short cut key for line break?
(A) Shift + Enter
(B) Space + Enter
(C) Alt + Enter
(D) CTRL + Enter
79. How is data organized in a spreadsheet?
(A) Rows and columns
(B) Layers and planes
(C) Lines and spaces
(D) Height and width
80. Once information is input into a computer it becomes
(A) objects
(B) ideas
(C) data
(D) facts

ENGLISH
Questions (01-05): Select the word or phrase that best completes the sentence
01. Computer has revolutionized office procedures more than any machine of modern time
(A) has any machine
(B) any other machine
(C) other machine
(D) none of these
02. Writing a beautiful sonnet is as much an achievement as to finish a 400-page novel.
(A) finishing
(B) it is to finish
(C) to have finished
(D) if to finish
03. My mother takes the responsibility for running the household.
(A) moving quickly
(B) going
(C) managing
(D) none of these
04. People of diverse backgrounds now go to different places for pleasure, business or education
(A) distinctive
(B) isolated
(C) distant
(D) different
05. Organizing some extra-curricula activities has occupied a lot of my time recently
(A) taken in
(B) taken up
(C) taken over
(D) taken after

Questions (06-10): Choose the correct spelling from the given options
06.
(A) Presinct
(B) Precinct
(C) Pricinct
(D) Prescinct
07.
(A) Faxsimile
(B) Fachsimile
(C) Facsimile
(D) Factsimile
08.
(A) Acquaintance
(B) Aquaintance
(C) Aquantence
(D) Aquaintence
09.
(A) Transeint
(B) Trancient
(C) Transient
(D) Transent
10.
(A) Collosal
(B) Collossal
(C) Colosal
(D) Colossal

Questions (11-15) : Fill in the blank with right option


11. Voting took place peacefully most of the country.
(A) across
(B) besides
(C) into
(D) for
12. His name rings a but I am unable to remember him
(A) toll
(B) bell
(C) cymbal
(D) chime
13. She usually the baby down for sleep at this time
(A) lies
(B) lied
(C) lay
(D) lays
14. I promise to you in all circumstances
(A) stand up to
(B) stand with
(C) stand off
(D) stand by
15. I caught the bus if I had hurried.
(A) could have
(B) can have
(C) cannot have
(D) could not have

Questions (16-20): Select the pair that best expresses a relationship similar to that expressed in the original pair:
16. HARM : DAMAGE
(A) Sweet : Sour
(B) Stout : Weak
(C) Hook : Crook
(D) Injure: Incapacitate
17. HACK: CARVE
(A) Gouge : Engrave
(B) Snip : Mince
(C) Hew: Fell
(D) Grind : Polish
18. ARTICULATE : UNCLEAR
(A) Explain : Lucid
(B) Elaborate : Sketchy
(C) Grieve : Somber
(D) Assign: Agile
19. VINDICTIVE: MERCY
(A) Insightful : Hope
(B) Modest Dignity
(C) Avaricious : Greed
(D) Skeptical: Credulity
20. QUARRY : MARBLE
(A) Metal: Silver
(B) Ore: Cold
(C) Mine Coal
(D) None of these
BANGLA
Question (21-35): Choose the correct answer
21. ?
(A)
(B)
(C)
(D)
22. ?
(A)
(B)
(C)
(D)
23. ?
(A) >
(B) >
(C) >
(D)
24. ?
(A)
(B)
(C)
(D)
25.
(A)
(B)
(C)
(D)
26.
(A)
(B)
(C)
(D)
27. ?
(A)
(B)
(C)
(D)
28. ?
(A)
(B)
(C)
(D)
29. ?
(A)
(B)
(C)
(D)
30. ?
(A)
(B)
(C)
(D)
Answer:
31. ?
(A)
(B)
(C)
(D)
Exp:
32. ?
(A)
(B)
(C)
(D)
33. ?
(A)
(B)
(C)
(D)
34. ?
(A)
(B)
(C)
(D)
35.
(A)
(B)
(C)
(D)

MATHEMATICS
36. In June a baseball team that played 60 games had won 30% of its games played. After a phenomenal winning streak this team
raised its average to 50% How many games must the team have won in a row to attain this average?
(A) 30
(B) 45
(C) 20
(D) 24
Solution:
Let, Additional match = x
Now,
30% of 60+ x = 50% of (60+x)
Or, 18+x = 30+0.5x
Or, x = 24
37. A rectangular fish tank 25m by 9m has water in it to a level of 2m. This water is carefully poured into a cylindrical container with a
diameter of 10m. How high will the water reach in the cylindrical container?
(A) 18π
(B) 18/π
(C) π/18
(D) 9/2π
Solution:
The volume of the fish tank = 25×9×2 = 450 square inch
Let height of the cylindrical container is = h
πr2h = 450
h = 450/ πr2 = 450/π52 = 18/π
38. In a club 50% of the male voters and 80% of the female voters voted for candidate A. If candida A received 70% of the total votes,
what is the ratio of male to female voters?
(A) 1/3
(B) 3/4
(C) 1/4

(D) ½
Solution:
Male voter = x
Female voter = y
50% of x + 80% of y = 70% of (x+y)
50x/100 + 80y/100 = 70(x+y)/100
50x + 80y = 70x + 70y
80y 70y= 70x 50x
10y = 20x

x/y = 10/20 = ½
39. An iron rod that weighs 24 kg is cut into two pieces so that one of these pieces weighs 16 kg and is 34 m long. If the weight of each
piece is proportional to its length, how long is the other piece?
(A) 11m
(B) 17m
(C) 34m
(D) 68m
Solution:
16 kg rod = 34m
24 kg rod = (34×24)/16 = 51m
Length of the other part is = 51-34 = 17m
40. The price of a pen is 25% more than the price of a book. The price of a pen holder is 50% more than the price of the book. How
much is the price of the pen holder more than the price of the pen?
(A) 50%
(B) 25%
(C) 20%
(D) 37.5%
Solution:
Let price of book = 100tk
Price of pen = 100+100×25% = 125 tk
Price of penholder = 100 + 100×50% = 150 tk
Difference is = 150 -125 = 25 tk
Percentage = (25×100)/125 = 20%
41. A 10% monthly salary increase resulted in a Tk.9000 per year increase in salary for an employee. What was his monthly salary
before the increase?
(A) Tk. 7500
(B) Tk. 8500
(C) Tk. 5000
(D) Tk. 9500
Solution:
Yearly increase = 9000 tk
Monthly increase = 9000/12 = 750 tk
10% of Salary is = 750 tk
100% of Salary is = (750×100)/10 = 7500 tk
42. A garden of 100 m length and 60m width has a walkway of 2 m width on every side. What is the area of the garden, in square
meter, excluding the walkway?
(A) 5684
(B) 6000
(C) 5376
(D) 5123
Solution:
Area of the garden excluding walkway = {(100-2×2) × (60-2×2)}= 96×56=5376m2
43. Three angles of a triangle are in proportion 5:6:7. Then what is the difference in degrees between the biggest and the smallest
angles?
(A) 10°
(B) 20°
(C) 25°
(D) 30°
Solution:
Sum of 3 angles of a triangle = 180°
Sum of ratio = 5+6+7 = 18
So, the larger angle = 180° × 7/18 = 70°
And, the smaller angle = 180° × 5/18 = 50°
Difference = 70° 50° = 20°
44. Increasing the original price of an item by 10%, then decreasing by 20% and then again increasing the price by 10% is equivalent:
(A) 4.4% increase
(B) 3.2% decrease
(C) 3.5% decrease
(D) None of these
Solution:
Let,
Selling price = 100
After 10% increase = (100+10)=110
After 20% decrease = (110 20% of 110) = 88
After 10% increase = (88 + 10% of 88) = 96.8
Final decrease = (100-96.8)=3.2%
45. The sum of principal and simple interest of a certain amount of money would be Tk.460 after 3 years from now and Tk. 500 after 5
years from now. What is the total interest rate?
(A) 5%
(B) 12%
(C) 15%
(D) 20%
Solution:
(5 3) yr = 2 yrs interest = (500 460) = 40 tk
1 year interest = 40/2 = 20 tk.
3 year interest = (20 × 3) = 60 tk
Principal amount= (460 60) = 400 tk
Interest rate = 60/(400×3) = 0.05 = 5%
46. The ratio between the perimeter and the breadth of a rectangular is 5:1. If the area of the rectangle is 216 cm2, what is the length
of the rectangle?
(A) 16cm
(B) 18 cm
(C) 20cm
(D) 22cm
Solution:
Let,
Length and breadth of the rectangle= x, y
So, area = xy
Perimeter = 2(x+y)
ATQ,
2(x+y)/y = 5
Or, 2x + 2y = 5y
Or, 2x = 3y
Or, y = 2x/3
Now,
xy = 216
Or, x × 2x/3 = 216
Or, 2x2 = 648
Or, x2 = 324
Or, x = 18
47. An article when sold at a gain of 5% yields Tk. 15 more than when sold at a loss of 5%. Its cost price would be
(A) Tk. 100
(B) Tk. 150
(C) Tk. 200
(D) Tk. 250
Solution:
At 5% profit, selling price = 100+5=105 tk.
At 5% loss, selling price = 100-5= 95 tk.
Difference between selling price = 105-95=10 tk.
If difference 10, then cost = 100tk
If difference 1, then cost = 100/10 tk
If difference 15, then cost = (100×15)/10 tk = 150tk.
48. If for integer x, 5<x<10 and y=x+5, what is the greatest possible value of x+y?
(A) 32
(B) 22
(C) 27
(D) 23
Solution:
Given,
5 < x < 10 and y =x+5
Possible value of x = 6, 7, 8, 9
When x =9,
y =9+5 = 14
Now,
x + y = 9 + 14 = 23
49. A water tank has two taps (Tap-1 and Tap-2). Tap-1 can fill a tank in 8 hours and Tap.2.can empty the tank in 16 hours. How long
will they take to fill the tank if both taps are opened simultaneously but Tap-2 is closed after 8 hours?
(A) 10 hours
(B) 12 hours
(C) 14 hours
(D) 16 hours
Solution:
Tap-1, fills in 1 hr = 1/8
Tap-2, empties in 1 hr = 1/16
When both taps are open, the tank fills in 1 hr = (1/8 1/16) = 1/16
In 8 hrs, the tank fills = 1/16 × ½
Remaining = 1 ½=½
½ is filled by only Tap 1
1/8 is filled by Tap-1 in 1 hr
So, ½ is filled by = (1×8)/2= 4 hr
Total time = 8+4 = 12 hr
50. The interest charged on a loan is p dollars per $1,000 for the first month and q dollars per $1000 for each month after the first
month. How much interest will be charged during the first three months on a loan of $10,000?
(A) 10p+20q
(B) 30q
(C) 30p
(D) 20p+10q
Solution:
Interest = 1st month + 2nd month + 3rd month
= p×(10000/1000) + q×(10000/1000) + q×(10000/1000)
=10p +10q + 10q
= 10p + 20q
51. Triangle ABC has the following vertices: A (1,0), B (5,0) and C (3.4). Which of the following is true?
(A) AB = BC
(B) CA = CB
(C) AB = AC
(D) AC < BC
Solution:
Distance of (x1,y1) & (x2,y2) = √{(x1 x2)2 + (y1 y2)2}
AB/BA = √{(1 5)2 + (0 0)2 }= √(-4)2 = √16 = 4
BC/CB = √{(5 3)2 + (0 4)2 }= √22 + 16 = √20 = 2√5
2 2
AC/CA=√{(1-3) + (0 4) }= √4 + 16 =√20 = 2√5
52. If a>b>l, then which of the following is true?
(A) a2> b2
(B) a2 < ab
(C) a b<0
(D) b +a > 2a
Solution:
Let,
a = 3 & b=2
Option a) a2> b2 = (3)2> (2)2 = 9 > 4 True
Option b) a2 < ab = (3)2 < 3 x 2 =9<6 false
Option c) a b<0= 3-2<0 = 1< 0 false
Option d) b +a > 2a = 2+3> 2 x 3 = 5 > 6 false
53. When 6 gallons of gasoline are put into a car, the indicator goes from 1/4 to 5/8. What is the total capacity of the gasoline tank?
(A) 12
(B) 14
(C) 16
(D) 18
Solution:
(5/8 ¼) = 3/8 means 6 gallons
So, Total capacity = 6×8/3 = 16 gallons
54. A square carpet with an area of 169 cm2 must have 2 cm cut off one of its edges in order to be a perfect fit for a rectangular room.
What is the area (in cm) of this rectangular room?
(A) 117
(B) 143
(C) 145
(D) 165
Solution:
Let, the carpet length = x
x2 = 169
x = 13
width of the room = (13-2) cm = 11 cm
area of the room = (13×11)2 = 143 cm2
55. A box contains 12 poles and 7 pieces of net. Each piece of net weighs 0.2 gm: each pole weighs 1.1gm. The box and its contents
together weigh 16.25 gm. How much does the empty box weigh?
(A) 1.2gm
(B) 1.65gm
(C) 6.15gm
(D) 2.75gm
Solution:
Weight of 7 nets = (7×0.2)gm =1.4gm
Weight of 12 poles = (12×1.1)gm = 13.2gm
Total weight of net and pole = (1.4+13.2)gm = 14.6gm
Box weight is = (16.25 14.6) = 1.65gm
56. The average of 6 numbers is 25. If 3 more numbers, with an average of 22 are added to these numbers, what will be the average of
the combined 9 numbers?
(A) 20
(B) 24
(C) 26
(D) 32
Solution:
Sum of 6 numbers = (6×25) = 150.
Sum of 3 additional numbers = (3 × 22) = 66.
Sum of (6 + 3) =9 numbers = (150+66)=216
average of 9 numbers = 216/9 = 24
57. In a container, there are 2 green marbles and 2 red marbles. You randomly pick the marbles. What is the probability that both of
them are green?
(A) 1/2
(B) 1/4
(C) 1/6
(D) 1/3
Solution:
green ball = 2
Red ball = 2
From 4 balls, possibility of 2 green = 4C2 = 6
From 2 balls, possibility of 1 green = 2C2 = 1.
From rest 2 balls, possibility of being green = 2C2=1
Possibility of 2 green = (2C2 x 2C2)/4C2= (1×1)/6= 1/6
58. Three boys have marbles in the ratio of 19:5:3. If the boy with the least number has 9 marbles. how many marbles does the boy
with the greatest number have?
(A) 57
(B) 33
(C) 27
(D) 81
Solution:
Smaller ratio 3 = 9
So, Smaller ratio 1 = 9/3 = 3
Now,
Highest ratio 19 = 19×3 = 57
59. If an inspector rejects 0.08% of a product as defective, how many units of the product will he examined in order to reject 2?
(A) 1500
(B) 500
(C) 2000
(D) 2500
Solution:
0.08 product is rejected from = 100
So, 2 product is rejected from = (100 × 2 × 100)/8 = 2500
60. A box is made in the form of a cube. If a second cubical box has inside dimensions three times those of the first box, how many
times as much does the second box contain?
(A) 12
(B) 27
(C) 9
(D) 6
Solution:
Let,
one side of first cube = a
So, Volume of first cube = a3
ATQ,
One side of the 2nd cube = (3a)3 = 27a3
Times = 27a3 / a3 = 27

GENERAL KNOWLEDGE AND COMPUTER


(61-80) : Choose the correct answer
61. The headquarter of International Atomic Energy Agency (IAEA) are situated at
(A) Vienna
(B) Geneva
(C) Rome
(D) Paris
62. Profits of a firm that are distributed or given out to its investors are called
(A) Bonds
(B) Equity
(C) Dividends
(D) None of these
63. Money market is a market for
(A) short-term fund
(B) long-term fund
(C) hedge fund
(D) Risk free assets
64. Fathometer is used to measure
(A) Ocean depth
(B) Earthquakes
(C) Rainfall
(D) Sound intensity
65. India has largest deposits of ______ in the world.
(A) gold
(B) copper
(C) mica
(D) None of the above
66. In cricket, a run taken when the ball passes the batsman without touching his bat or body is called
(A) leg bye
(B) bye
(C) bosic
(D) drive
67. Of the various non-conventional energy sources that are being tapped, which has the largest potential?
(A) Water power
(B) Wind power
(C) Solar power
(D) Biomass power
68. The chief constituent of gobar ( gas is
(A) ethane
(B) methane
(C) hydrogen
(D) carbon dioxide
69. The longest mountain range in the world is
(A) The Alps
(B) The Himalayas
(C) The Andes
(D) The Rockies
70. The main object of which of the following UN agency is to help the underdeveloped countries in the task of raising their living
standards?
(A) IMF
(B) UNICEF
(C) UNDP
(D) IDA
71. By default, your documents print in ________ mode.
(A) Page setup
(B) Landscape
(C) Portrait
(D) Print preview
72. Which of these toolbars allows changing of Fonts and their sizes?
(A) Formatting
(B) Standard
(C) Print Preview
(D) None of these
73. All of the following are examples of real security and privacy risks EXCEPT:
(A) Viruses
(B) Spam
(C) Hackers
(D) none of them
74. The basic unit of a worksheet into which you enter data in Excel is called a
(A) column
(B) box
(C) table
(D) cell
75. You can detect spelling and grammar errors by
(A) Press Shift + F7
(B) Press Ctrl + F7
(C) Press Alt+ F7
(D) Press F7
76. Verification of a login name and password is known as:
(A) configuration
(B) authentication
(C) accessibility
(D) logging in
77. A light sensitive device that converts drawing, printed text or other images into digital form is
(A) Keyboard
(B) Plotter
(C) Scanner
(D) None of these
78. Graphical pictures that represent an object like file, folders etc. are:
(A) Icons
(B) Desktop
(C) Task bar
(D) Windows
79. Computer Virus is a __________ .
(A) Hardware
(B) Software
(C) Bacteria
(D) Freeware
80. Which of the followings is word processing software?
(A) Perfect
(B) Wordpad
(C) MS Word
(D) All of these

ENGLISH
Questions (01-05): Read the following passage and answer the questions given below it in the context of the passage.
P.O.R.S.T. V and W are seven friends. Each of them likes a particular fruit, viz. Apple, Banana, Pear. Guava, Orange, Mango and
Watermelon and each of them has a favorite city, viz. Mumbai, Pune, Delhi, Kolkata, Chennai, Hyderabad and Cochin. The choices of
fruit and favorite city of the seven friends aren't necessarily in the same order. O likes Mango and his favorite city is Chennai. The one
whose favorite city is Pune likes Watermelon. TS favorite city is Kolkata. R likes Guava and his favorite is not Mumbai. W's favorite city is
Cochin and he does not like either Banana or Pear. The favorite city of the one who likes Orange is Hyderabad. T does not like Pear. P's
favorite city is neither Pune nor Hyderabad. S does not like Watermelon.
01. Who likes Apple?
(A) W
(B) T
(C) V
(D) P
02. Which fruit does P like?
(A) Apple
(B) Orange
(C) Pear
(D) Watermelon
03. Which is R's favorite city?
(A) Mumbai
(B) Delhi
(C) Hyderabad
(D) Pune
04. Which of the following combinations of Person-Fruit-City is incorrect?
(A) R-Guava-Kolkata
(B) T-Banana-Cochin
(C) S-Guava-Delhi
(D) All are incorrect
05. Which is V's favorite city?
(A) Pune
(B) Hyderabad
(C) Mumbai
(D) Data inadequate

Questions (06-10): Fill in the blank with right option


06. Obviously, objectives occasionally.... be modified or changed.
(A) have to
(B) must to
(C) shouldn't
(D) ought
07. The emergency services - the accident site as soon as the news of the train collision was to them.
(A) suspended, emitted
(B) carried, postponed
(C) reached, relayed
(D) marked, transferred
08. The ....... chosen for construction of the building is in the heart of the city.
(A) cite
(B) site
(C) slight
(D) sight
09. He went ... to oblige his superior.
(A) on his way
(B) in his way
(C) with his way
(D) out of his way
10. The teachers said that they were no longer prepared to the ways of the new Headmaster.
(A) put up with
(B) put up to
(C) put on with
(D) put over with

Questions (11-15): Select the word or phrase that is most closely opposite in meaning to the original word
11. Abundant
(A) Scarce
(B) Acrimonious
(C) genuous
(D) Heavenly
12. Gaudy
(A) Sumptuous
(B) Ostentatious
(C) Modest
(D) Blatant
13. Accumulate
(A) Amass
(B) Produce
(C) Gather
(D) Scatter
14. Costly
(a) Frugal
(b) Economical
(C) Thrifty
(D) Expensive
15. Forbid
(A) Forgive
(H) Refuse
(C) Allow
(D) Deprive

Questions 16-20 Each sentence has four Parts marked (A) (B) (C) and (D). Identify the incorrect part of the sentence.
16. Roshni was found (A) of pets and she was a Proud (b) owner of many unique (C) animals (D)
(A) found
(B) proud
(C) unique
(D) animals
17. The book is full (A) of updated (B) contemparary (C) commentary.(D)
(A) full
(B) updated
(C) contemparary
(D) commentary
18. The pump (A) shut off (B) as a result(c) of mecanical (D) failure.
(A) pump
(B) shut off
(C) result
(D) mecanical
19. The book is full (A) of updated (B) contemparary (C) commentary.(D)
(A) full
(B) updated
(C) contemparary
(D) commentary
20. The first thing that catches your eye (A) in this building(B) is the flour(C) with laser lights (D).
(A) eye
(B) building
(C) flour
(D) lights

BANGLA
21. ?
(A)
(B)
(C)
(D)
22. ?
(A)
(B)
(C)
(D)
23. ?
(A)
(B)
(C)
(D)
24. '
(A)
(B)
(C)
(D)
25. ' ' ?
(A)
(B)
(C)
(D)
26. ?
(A)
(B)
(C)
(D)
27. ' ' ?
(A)
(B)
(C)
(D)
25. ?
(A)
(B)
(C)
(D)
29. ?
(A)
(B)
(C)
(D)
30. ?
(A)
(B)
(C)
(D)
31. ?
(A)
(B)
(C)
(D)
32. ' ?
(A) √ +
(B) √
(C) √ +
(D) √
33. ' ' ?
(A) +
(B) +
(C) +
(D)
34. ?
(A) +
(B) + =
(C)
(D) + =
35. ' ' ?
(A)
(B)
(C)
(D)
36. ' ?
(A)
(B)
(C)
(D)
37. ?
(A)
(B)
(C)
(D)
38. ' ?
(A)
(B)
(C)
(D)
39. ' ?
(A)
(B)
(C)
(D)
40.
(A)
(B)
(C)
(D)

MATHEMATICS
41. In a business A and C invested amounts in the ratio 2:1 whereas A and B invested amounts in the ratio 3:2. If their annual profit be
Tk. 157300, t
(A) Tk. 24200
(B) Tk. 24400
(C) Tk. 48200
(D) Tk. 48400
Solution:
A:B=3:2
A:C=2:1
A:B:C=6:4:3
48400.
42. The average age of a group of 15 employees is 24 years. If 5 more employees join the group, the average age increases by 2 years.
Find the average age of the new employees.
(A) 35
(B) 30
(C) 24
(D) 32
Solution:
Sum of the ages of (15+5) = 20 employees => 20×(24+2) = 520 years
Sum of the ages of 15 employees => 15×24 = 360
Sum of the ages of new 5 employees = (520-360) = 160
Average = 160/5 = 32.
43. A person travels a certain distance at 3 km/hr and reachers 15 min late. If he travels at 4 km/hr, he reaches 15 min earlier. The
distance he has to travel is:
(A) 4.5 km
(B) 6 km
(C) 7.2 km
(D) 12 km
Solution:
D/4 D/3 = (15+15)/60
D/12 = 1/2
D = 6 km
44. The difference between the radii of bigger circle and smaller circle is 14 cm and the difference between their areas is 1056 cm².
Radius of the smaller circle is :
(A) 7 cm
(B) 5 cm
(C) 9 cm
(D) 3 cm
Solution:
π(x+14)² πx² = 1056
π(x²+28x+196-x²) = 1056
28x+196 = 1056/π = 1056×7/22 = 336
x = (336-196)28 = 5 cm
45. A sum fetches a simple interest of Tk. 6000 at the rate of 5% p.a. in 6 years. What would be the compound interest earned at the
same rate of interest and the same principal in 2 years?
(A) Tk. 2050
(B) Tk. 2500
(C) Tk. 2125
(D) Tk. 2245
Solution:
The sum = Tk. (6000×100)/(5×6) = Tk. 20000
Compound Interest = (20000 × 105/100 × 105/100) 20000 = Tk. 2050.
46. Alam sold an item for Tk. 6384 and incurred a loss of 30%. At what price should he have sold the item to have gained a profit of
30%?
(A) Tk. 14656
(B) Tk. 11856
(C) Tk. 13544
(D) None
Solution:
Required price = Tk. 6384 × (100+30)/(100-30) = Tk. 11856.
47. Kiran purchased a scooter for Tk. 52000. He sold it at loss of 10%. With that money he purchased another scooter and sold it at
profit of 20%. What is his overall loss/profit?
(A) Tk. 2060 profit
(B) Tk. 2560 loss
(C) Tk. 13400 loss
(D) Tk. 4160 profit
Solution:
Selling price of the first scooter = Tk. 52000 × (100-10)/100 = Tk. 46800.
Loss on the first scooter = Tk. 52000×10% = Tk. 5200.
Profit on the second scooter = Tk. 46800×20% = Tk. 9360.
Net profit = Tk. (9360-5200) = Tk. 4160.
48. A square is inscribed in a circle of diameter 2a and another square is circumscribing circle. The difference between the areas of
outer and inner squares is:
(A) a2
(B) 2a2
(C) 3a2
(D) 4a2
Solution:
Length of a side of the outer square = 2a.
Area = (2a)² = 4a².
Length of the diagonal of the inner square = 2a.
Length of a side of the inner square = 2a/√2
Area = (2a/√2)² = 4a²/2 = 2a²
Difference = 4a²-2a² = 2a².
49. The average of nine numbers is 50. The average of the first five numbers is 54 and thatof the last three numbers is 52. Then the
6th number is:
(A) 30
(B) 34
(C) 44
(D) 24
Solution:
The 6th number is = (50×9) (5×54 + 3×52) = 450 426 = 24.
50. A batsman has a certain average of runs for 12 innings. In the 13th inning, he scores 96 runs threby increasing his average by 5
runs. What is his average after the 13 innings?
(A) 36
(B) 48
(C) 24
(D) 31
Solution:
Average of 12 innings = x
According to the question:
(12x+96)/13 = x+5
x = 96-65 = 31.
Average in 13 innings = 31+5 = 36
51. A man takes 3 hours 45 minutes to row a boat 22.5 km downstream of a river and 2 hours 30 minutes to cover a distance of 10 km
upstream. Find hte speed of the river current in km/hr.
(A) 1 km/hr
(B) 2 km/hr
(C) 3 km/hr
(D) 4 km/hr
Solution:
Speed of the boat = x kmph, the river current = y kmph.
3H45M = (3+ 45/60) hr = 15/4 hr
2H30M = (2+ 30/60) hr = 5/2 hr.
According to the question:
x+y = 22.5/(15/4) = 6
x-y = 10/(5/2) = 4
2x = 2
x = 1 km/hr.
52. A man can row 6 km/hr in still water. If the speed of the current is 2 km/hr, it takes 3 hrs more in upstream than in the
downstream for the same distance. The distance is:
(A) 30 km
(B) 24 km
(C) 20 km
(D) 32
Solution:
D/(6-2) D/(6+2) = 3
D = 24 km
53. A train 150 m long passes a pole in 15 seconds and crosses another train of the same length travelling in opposite direction in 8
seconds. The speed of the second train in (km/h) is:
(A) 60 km/hr
(B) 66 km/hr
(C) 72 km/hr
(D) 99 km/hr
Solution:
Speed of the first train = 150/15 = 10 mps.
speed of the second train = x mps.
According to the question
x+10 = (150+150)/8
x = 27.5 mps = 27.5 × 3600/1000 = 99 kmph.
54. The ratio of the speeds of a train and a man is 6:1. The length of the train is 650 m and crosses a pole in 1 minute 5 seconds. In
how much time will hte man cross 240m long platform?
(A) 1 minutes 24 sec
(B) 2minutes 30 sec
(C) 2minutes
(D) 2 minutes 24 sec
Solution:
Speed of the train = 650/65 = 10 mps.
Speed of the man = 10/6 = 5/3 mps.
Time for the man to cross the platform = 240/(5/3) = 144 seconds = 2 minutes 24 seconds.
55. If the numerator of a fraction is increased by 150% and the denominator of a fraction is increased by 200% fraction becomes 10/19.
Find the fraction.
(A) 12/17
(B) 10/16
(C) 12/19
(D) 9/11
Solution:
(x + 150% of x)/(y + 200% of y) = 10/19
2.5x/3y = 10/19
x/y = 10×3/19×2.5 = 12/19.
56. A number when divided by 627 leaves a remainder 43. By dividing the same number by 19, the remainder will be :
(A) 32
(B) 43
(C) 13
(D) 5
Solution:
627 is a multiple of 19. So, if a number divided by 627 leaves a remainder 43, then the same number divided by 19 will leave the same
remainder as 43 divided by 19 leaves.
43 = 19×2 + 5.
So, the remainder will be 5.
57. A can finish a work in 5 days and B takes 4 days to do the same work. If the work is increased by 8 times, how many days will it
take for both of them to finish the work if they work together?
(A) 10 days
(B) 15 days
(C) 20 days
(D) 25 days
Solution:
new work size = 1+8 = 9 works.
A & B can do in 1 day (working together) = 1/5 + 1/4 = 9/20
Required number of days = 9/(9/20) = 20 days.
58. A computer can perform 30 identical tasks in 6 hour. At that rate, what is the minimum number of computers that should be
assigned to complete 80 of the tasks within 3 hours?
(A) 12
(B) 7
(C) 6
(D) 16
Solution:
Number of computers = (80×6/30×3) = 5.33 = minimum 6 .
59. A and B together can complete a piece of work in 12 days, B and C can do it in 20 days and C and A can do it in 15 days. A, B and
C together can complete it in:
(A) 12 days
(B) 6 days
(C) 8 days
(D) 10 days
Solution:
A & B can do in 1 day = 1/12 work
B & C can do in 1 day = 1/20 work
C & A can do in 1 day = 1/15 work
2(A&B&C) can do in 1 day = 1/12 + 1/20 + 1/15 = 12/60 = 1/5 work.
A&B&C can do in 1 day = 1/(5×2) = 1/10 work.
Required number of days = 1/(1/10) = 10 days.
60. There are 15 boys and 10 girls in a class. If three students are selected at random, what is the probability that 1 girl and 2 boys are
selected?
(A) 1/40
(B) 1/2
(C) 21/46
(D) 7/41
Solution:
1 girl & 2 boys can be selected in = (10C1)×(15C2) ways = 10×105 ways = 1050 ways.
3 students can be selected from 25 students = 25C3 = 2300 ways
Required probability = 1050/2300 = 21/46.

GENERAL KNOWLEDGE & COMPUTER


61. Which of the following countries has recently launched amnesty program for foreign workers overstaying permits?
(A) Iran
(B) UAE
(C) Israel
(D) Jordon
62. Which of the following countries is planning to create a Space Force by 2020?
(A) China
(B) Russia
(C) India
(D) United States
63. Name the singer who was 18-time Grammy Award winner, passed away recently.
(A) Aretha Franklin
(B) Whitney Houston
(C) Patti Labelle
(D) Tina Turner
64. The civilian Airport of highest altitude is in
(A) Tibet
(B) Chaina
(C) Nepal
(D) India
65. The humidiy of air measured in percentage is called
(A) Absolute humidity
(B) Relative humidity
(C) Specific humidity
(D) All Of These
66. The exchange of commodities between two countries is referred as
(A) Balance of trade
(B) Bilateral trade
(C) Volume of trade
(D) Multilateral trade
67. Luanda is the capital of which country?
(A) Guyana
(B) Liberia
(C) Armenia
(D) Angola
68. Which one among the following is responsible for formation of 'Ozone Holes in the stratosphere?
(A) Chloro Fluoro Carbons
(B) Benzophyrene
(C) Hydrocarbons
(D) UV Radiation
69. The regulations related to intellectual property is known as (A) IPRS
(B) TRIM
(C) TRIPS
(D) GATT
70. I which year American Independence was acknwoledged by England
(A) 1782
(B) 1783
(C) 1784
(D) 1785
71. Which of the following functions is not performed by Server?
(A) Email-Processing
(B) Database Sharing
(C) Word processing
(D) Storage
72. A device that connects to a network without the use of cables is said to be
(A) Distributed
(B) Wireless
(C) Centralized
(D) None of these
73. The blinking point which shows your position in the text is called
(A) Blinker
(B) Causer
(C) Cursor
(D) Pointer
74. In excel, charts are created using which option?
(A) Chart wizard
(B) Pivot table
(C) Pie chart
(D) Bar chart
75. Which among following is secondary Storage device?
(A) Ram
(B) Diode
(C) Semi-conductor
(D) Hard Disk
76. By pressing which key we can move to beginning of a page? (A) Window key
(B) Shift key
(C) Tab key
(D) Home key
77. Which among following is an image name extension?
(A) gif
(B) docx
(C) ppt
(D) lib
78. Typeface option will come under which menu?
(A) Edit
(B) View
(C) Format
(D) Tools
79. Programs designed to perform specific tasks is known as
(A) System software
(B) Application software
(C) Utility programs
(D) Operating system
80. Pressing F8 key for three times selects
(A) A paragraph
(B) Entire document
(C) A sentence
(D) A word

ENGLISH
Questions (01-03): Fill in the blanks with right options
1. The trade deficit has been growing rapidly, and the capital inflows _____ to fund it could get disrupted in case US monetary policy gets
tightened ______ than expected.
(A) Mandate, equal
(B) Needed, more
(c) Required, higher
(D) Both B and D
2. The _____ to e-buses would not lower the level of emissions but merely ____ their place of origin.
(A) Evolution, vary
(B) Transition, change
(C) Shift, altered
(D) Altercation, destroy
3. It is not easy for a country with high fiscal deficits, high inflation, _______ banks and one that is _____ to macro panics to completely
open up its capital account.
(A) week, privy
(B) failing, level
(C) weak, prone
(D) None of these

Questions (04-07): Select the word or phrase that best completes the sentence.
4. The supply of food and beverage for the program was abundant.
(A) inadequate
(B) substandard
(C) excellent
(D) plentiful
5. Assertive people are generally more decisive.
(A) Docile
(B) Articulate
(C) Confident
(D) Compliant
6. People of diverse backgrounds now go to different places for pleasure, business or education.
(A) distinctive
(B) isolated
(C) distant
(D) different
7. Computer has revolutionized office procedures more than any machine of modern time.
(A) has any machine
(B) any other machine
(C) other machine
(D) none of these

Questions (08-10): Select the pair that best expresses a relationship similar to that expressed in the original pair.
8. DEBATE: SOLILOQUY
(A) Elegant : Regal
(B) Ovation : Silence
(C) Group: Hermit
(D) None of these
9. ARENA: CONFLICT
(A) Forum: Discussion
(B) Mirage: Reality
(C) Asylum: Pursuit
(D) Utopia: Place
10. TORRENT: DROPLET
(A) water: eddy
(B) swamp: desert
(C) avalanche : pebble
(D) hurricane : wreckage

BANGLA
Question (11-20): Choose the correct answer.
11. ?
(A)
(B)
(C)
(D)
12. ?
(A) -
(B) -
(C) -
(D) -
13. ?
(A)
(B)
(C)
(D)
14. ?
(A)
(B)
(C)
(D)
15. ?
(A)
(B)
(C)
(D)
16. ?
(A)
(B)
(C)
(D)
17. √ + ?
(A)
(B)
(C)
(D)
18. ?
(A) + =
(B) + =
(C) + =
(D) + =
19. ?
(A) >
(B) >
(C) >
(D)
20. ?
(A)
(B)
(C)
(D)

MATHEMATICS
Questions (21-30): Read the following questions carefully and choose the right.
21. Alam sold two vehicles for Tk 46000 each. If he gained 10% on the first and lost 10% on another. then what is his percentage profit
or loss in this transaction?
(A) 2% loss
(B) 1% profit
(C) 1% loss
(D) None of these
Solution:
Let the profit be X% and loss be Y% . So,
Net profit or loss% = X +(-Y) + X×(-Y)/100
(Negative sign denotes that their is a loss)
Their is 10% loss and 10% profit then
Net profit or loss% = 10+(-10)+10×(-10)/100 =10 10 100/100 = 0 1 = -1
( negative sign here shows that there is a net loss )
So the net loss percentage is 1% .
22. A square is inscribed in a circle of diameter 2a and another square is circumscribing circle. The difference between the areas of
outer and inner squares is
(A) a2
(B) 2a2
(C) 3a2
(D) 4a2
Solution:

Area or outer square = 2a x 2a = 4a2


In△BAD
(BD)2=(AD)2+(AB)2
(2a)2=x2+x2
4a2=2x2
2a2=x2
x=2 √a.
Area of inner square =√2a x √2a=2a2
Difference between the areas of the outer and inner square =4a2−2a2=2a2
23. 12 marbles are selected at random from a large collection of white, red, green and yellow marbles. The number of marbles of each
color is unlimited. Find the probability that the selection contains at least one marble of each color?
(A) 34/91
(B) 23/31
(C) 36/91
(D) 33/91
Solution:
Number of ways of slecting 12 marbels is equal to the number of non negative intrigal solution of
White Red Green Yellow = 12
Total number of ways = C (12 4-1,4-1) = C(15,3)
The number of selection that contains at least one marbels of each colour is equal to the number of positive intrigal solution of White
Red Green Yellow= 12.
=C(12-1,4-1) = C(11,3)
Required probably = C(11,3)/C(15,3)= 33/91.
24. A motor-cycle covers 40 km with a speed of 20 km/hr. Find the speed of the motorcycle for the next 40 km journey so that the
average speed of the whole journey will be 30 km/hr.
(A) 70.0 km/hr
(B) 52.5 km/hr
(C) 60.0 km/hr
(D) 60.5 km/hr
Solution:
Let, speed of the motorcycle for the next 40 km = x km/hr
Average speed,
30 = 2.x.20/(20+x)
Or, 600 + 30x = 40x
Or, x = 60
25. A man can row 6 km/hr in still water. If the speed of the current is 2 km/hr, it takes 3 hours more in upstream than in the
downstream for the same distance. The distance is
(A) 30 km
(B) 24 km
(C) 20 km
(D) 32 km
Solution:
Let, d be the distance.
Then:
d/(6 2)kph = d/(6+2)kph +3
Or, d/4 = d/8 +3
Or, d/8 =3
Or, d=24 km distance
26. The respective ratio between the speed of a car, a train, and a bus is 5:9:4. The average speed of the car, bus and train is 72 km/hr
together. What is the average speed of the car and the train together?
(A) 82 km/hr
(B) 72 km/hr
(C) 67 km/hr
(D) 84 km/hr
Solution:
5x + 9x + 4x = 72 × 3
Or, 18x = 72 × 3
Or, x = 12
Avg. speed of the car and the train,
= (5x+9x)/2 = 7x = 7X12 = 84 km/hr
27. Three angles of a triangle are in proportion 5:6.7. Then what is the difference in degrees between the biggest and the smallest
angles?
(A) 20o
(B) 10o
(C) 30°
(D) 25o
Solution:
5x°+6x°+7x°=180°
Or, 18x°=180°
Or, x=10
Largest angle =7x° = 7(10)°= 70°
Smallest Angle
5x°=5(10)°=50°
Difference = 70° - 50° = 20°
28. A water tank has two taps (Tap-1 and Tap-2). Tap-I can fill a tank in 8 hours and Tap-2 can empty the tank in 16 hours. How long
will they take to fill the tank if both taps are opened simultaneously but Tap-2 is closed after 8 hours?
(A) 10
(B) 12
(C) 14
(D) 16
Solution:
Tap-1, fills in 1 hr = 1/8
Tap-2, empties in 1 hr = 1/16
When both taps are open, the tank fills in 1 hr = (1/8 1/16) = 1/16
In 8 hrs, the tank fills = 1/16 × = ½
Remaining = 1 ½=½
½ is filled by only Tap 1
1/8 is filled by Tap-1 in 1 hr
So, ½ is filled by = (1×8)/2= 4 hr
Total time = 8+4 = 12 hr
29. The average of the three numbers x, y and z is 45. x is greater than the average of y and z by 9. The average of y and z is greater
than y by 2. Then the difference of x and z is
(A) 3
(B) 5
(C) 7
(D) 11
Solution:
(x+y+z)/3 = 45
Or, x+y+z=135-----(1)
Again,
X=(y+z)/2 + 9
Or, 2x-y-z = 18 -----(2)
From (1) + (2),
3x = 153
Or, x = 51
Again,
(y+z)/2= y+2
Or, y = z-4
From (1)
x+y+z=135
Or, 51+z-4+z=135
Or, z = 40
So, x-z = 51-44 = 7
30. A can finish a work in 5 days and B takes 4 days to do the same work. If the work is increased by 8 times, how many days will it
take for both of them to finish the work if they work together?
(A) 20 days
(B) 15 days
(C) 10 days
(D) 25 days
Solution:
A takes 5 days and B takes 4 days to compete the work.
Let's say total work is 20 units (which can be easily cancelled by 5 & 4)
So the rate of work of A= 20/5 =4 units per day
Rate of work for B= 20/4 = 5 units per day
Together they can do 5+4= 9 units per day.
Now we assumed total work as 20 units in beginning. Just increase it 8 times.
New total work= 20*8 = 160 units
Therefore, they will take 160/9 days or 17.77 days or 17 days and 18 hours or 18 days approx.

GENERAL KNOWLEDGE
Questions (31-40): Study the following questions carefully and answer the questions given below.
31. The civilian Airport of highest altitude is in
(A) Tibet
(B) China
(C) Nepal
(D) India
32. Money market is a market for
(A) short-term fund
(B) long-term fund
(C) hedge fund
(D) risk free assets
33. Which of the following city has recently topped the Global Liveability Index 2015?
(A) Melbourne
(B) Osaka
(C) Vienna
(D) Calgary
34. In absorption of insolation, the most significant part is played by
(A) Ozone
(B) Oxygen
(C) Haze
(D) Carbon dioxide
35. Which of the following countries as successfully launched its first hypersonic aircraft?
(A) India
(B) China
(C) Russia
(D) United States
36. The exchange of commodities between two countries is referred as
(A) Bilateral trade
(B) Balance of trade
(C) Volume of trade
(D) Multilateral trade
37.
(A) Greenwich
(B) Indira Point
(C) New York
(D) Shriharikota
38. The branch of science dealing with the study or collection of banknotes and medals is known as
(A) Aphnology
(B) Accountancy
(C) Chrematistics
(D) Numismatics
39. Which one among the following is responsible for formation of Ozone Holes in the stratosphere?
(A) Chloro Foro Carbon
(B) Benzopyrene
(C) Hydrocarbons
(D) UV radiation
40.
(A) China
(B) Japan
(C) UAE
(D) None of these

SUBJECTIVE
Questions (41-80): Study the following questions carefully and answer the questions given below.
41. A host machine is unable to communicate with Google server. Which command is the most appropriate to run at host machine to
determine which intermediary device is responsible for this failure?
(A) telnet 192.168.1.
(B) ping 192.168.1.1
(C) tracert 192.168.1.1
(D) ftp 192.168.1.1
42. Which one is modern light weight message exchange format
(A) JSON
(B) MX
(C) HTML
(D) XML
43. A shared lock allows which of the following types of transactions to occur?
(A) Delete
(B) Insert
(C) Read
(D) Update
44. Which is not a feature of object oriented programming?
(A) Inheritance
(B) Recursion
(C) Encapsulation
(D) Abstraction
45. Which of the following protocols uses both TCP and UDP ports?
(A) SMTP
(B) Telnet
(C) FTP
(D) DNS
46. To divide a class C network into a maximum of 14 subnets - each capable of having up to 14 hosts, the subnet mask used should
be
(A) 255.255 255 240
(B) 255.255.255.192
(C) 255.255.255.78
(D) 255.255.255.0
47. The result of a SQL SELECT statement is a
(A) report
(B) form
(C) file
(D) table
48. Email is a protocol of the following layer
(A) Physical Layer
(B) Data Link layer
(C) Application layer
(D) TCP layer
49. Which protocol is used for secured web browsing
(A) www
(B) RSA
(C) ftp
(D) http
50. Which software is mostly used for virtualization?
(A) VMWare
(B) Oracle DBMS
(C) Windows XP
(D) Linux
51. To remove duplicate rows from the results of an SQL SELECT statement, the ----- qualifier specified must be included.
(A) ONLY
(B) DISTINCT
(C) UNIQUE
(D) SINGLE
52. The complexity of Bubble sort algorithm is
(A) O(n)
(B) O(log n)
(C) O(n2)
(D) O{n log n)
53. Which control statement can be executed at least once?

(B) for
(C) while
(D) All of these
54. The operation of processing each element in the list is known as
(A) Sorting
(B) Merging
(C) Inserting
(D) Traversal
55. Which of the following data structures are indexed structures?
(A) linear arrays
(B) linked lists
(C) both a and b
(D) None of these
56. The term push and pop is related to the
(A) array
(B) lists
(C) stacks
(D) All of these
57. ______ is a statement that is executed automatically by the system
(A) trigger constraint
(B) assertion
(C) durability
(D) integrity
58. What does a COMMIT statement do to a CURSOR:
(A) Open the Cursor
(B) Fetch the Cursor
(C) Close the Cursor
(D) None of these
59. Which data structure is used for indexing-
(A) Binary Tree
(B) B-Tree
(C) stack
(D) Link List
60. Trojan can be introduced through?
(A) Internet browsing
(B) Database accessing
(C) emails
(D) trouble shooting
61. How many layer Internet Protocol suite?
(A) 7
(B) 5
(C) 3
(D) 4
62. The step by step instructions that solve a problem are called
(A) An algorithm
(B) A list
(C) A plan
(D) None of the above
63. An acronym for conference on Data systems languages is
(A) Collute
(B) CODASYL
(C) CCD
(D) DASD
64. According to Boolean algebra the value of (A +AB)(B+ AB) is
(A) A
(B) B
(C) AB
(D) 1
65. Which of the following correctly shows the hierarchy of arithmetic operation
(A) /+*-
(B) *-/+
(C) +-/*
(D) /*+-
66. Consider the following code:
#include <stdio.h>
int main(int arge, char *argv[]){
return 0;
}
What is the minimum length of character array argv in the above code?
(A) 0
(B) 1
(C) undefined
(D) -1
67.
double d =Math.round (2.5+ Math.random():
(A) 2
(B) 25
(C) 3
(D) 4
68. Which O/S is recommended for real time systems?
(A) Windows
(B) Unix
(C) Oracle
(D) µ/OS
69. According to Boolean algebra the value of (A + AB). (B + AB) is
(A) AB
(B) B
(C) A
(D) 1
70. Which one of the following is not an example redundancy?
(A) Data Center
(B) Aero plane
(C) Motor Car
(D) None of these
71. Compared to CISC, RISC processors (at the same clock) are
(A) faster
(B) slower
(C) similar
(D) undefined
72. Which of the following is not a type of Web search engine?
(A) Computer-created
(B) Human-organized
(C) Hybrid
(D) Portal
73. Where can a JavaScript code be placed in an html page?
(A) <head>
(B) <body>
(C) both a and b
(D) none of the above
74.
(A) 34
(B) 7
(C) 0
(D) Undefined
75. In Java, which operator is used to create an object?
(A) class
(B) scanf
(C) print
(D) None of these
76. Distributed Queue Dual Bus is a standard for
(A) MAN
(B) LÀN
(C) Wireless LAN
(D) WAN
77. Which of the following will produce an answer that is closest in value to a double, d, while not being greater than d?
(A) (int) Math.min(d);
(B) (int)Math.max(d);
(C) (int)Math.abs(d);
(D) (int)Math.floor(d):
78. Which one is a universal logic gate?
(A) NAND
(B) AND
(C) OR
(D) NOT
79. Domain name to IP address mapping is done by -----
(A) Web server
(B) Database server
(C) DNS server
(D) Proxy server
80.
12+64+128+256+1024
(A) 10
(B) 8
(C) 6
(D) 9

ENGLISH
Questions (01-04) Select the word that is most closely similar in meaning to the original word.
1. STALEMATE
(A) Degeneration
(B) Deadlock
(C) Exhaustion
(D) Settlement
2. PERPETUAL
(A) Never ending
(D) Perfect
(C) Confused
(D) Seasonal
3. ZEALOUS
(A) Ardent
(B) Jealous
(C) Jealous
(D) Impatient
4. UNBEARABLE
(A) Tolerable
(B) Acceptable
(C) Undefeated
(D) Unpleasant

Questions (05-08): Find the correctly spelt word


5.
(A) Omineous
(B) Ommineous
(C) Omerous
(D) Ominous
6.
(A) Acquariam
(B) Aquarium
(C) Acquerium
(D) Aquarim
7.
(A) Amatuer
(B) Amature
(C) Amateur
(D) Ameteur
8.
(A) Adulation
(B) Adlation
(C) Aduletion
(D) Addulation

Questions (09-12): Each sentence has four parts, marked (A), (B), (C) and (D). Identify the incorrect part of the sentence
9. Canals are (A) artificial waterways, often constructed (B) either to transport heavy loads or to (C) delivering water (D) to cities and
farms.
(A) artificial
(B) either
(C) delivering
(D) to cities
10. (A) The pineapple, a fruit (B) grow in tropical climates (C) throughout the world, (D) is native to parts of South America.
(A) The
(B) grow
(C) throughout
(D) is
11. A large (A) number of automobile (B) part are now (C) made of plastic (D) instead of steel.
(A) number
(B) part
(C) made
(D) instead
12. Although a kangaroo (A) normally uses (B) its large feet and strong legs (C) for hopping, (D) but it can also swim.
(A) normally
(B) its large feet
(C) for hopping
(D) but it can

Questions (13-16): Fill in the blank with right option.


13. Many modern medicines are derived _____ plants and animals.
(A) from
(B) on
(C) for
(D) in
14.
(A) idea
(B) opinion
(C) attitude
(D) thinking
15. AIDS is not a dieses that can be ______ through the air or by insects.
(A) circulated
(B) transferred
(C) transmitted
(D) disseminated
16. I shall ring you up as soon as ________.
(A) I will arrive
(B) I arrive
(C) I shall arrive
(D) I shall be arriving

BANGLA
Question (17-32): Choose the correct answer
17. ?
(A)
(B)
(C)
(D)
18. ?
(A)
(B)
(C)
(D)
19. ?
(A)
(B)
(C)
(D)
20. ?
(A) √
(B) √
(C) √
(D)
21. ?
(A)
(B)
(C)
(D)
22. ?
(A)
(B)
(C)
(D)
23. ?
(A)
(B)
(C)
(D)
24. ?
(A)
(B)
(C)
(D)
25. ?
(A)
(B)
(C)
(D)
26. ?
(A)
(B)
(C)
(D)
27. ?
(A)
(B)
(C)
(D)
28. ?
(A)
(B)
(C)
(D)
29. ?
(A)
(B)
(C)
(D)
30. ?
(A)
(B)
(C)
(D)
31. ?
(A)
(B)
(C)
(D)
32. ?
(A)
(B)
(C)
(D)

MATHEMATICS
Questions (33-56): Read the following questions carefully and choose the right answer.
33. In an office, 44% of the workers prefer coffee and 72% prefer tea. If each of them prefers coffee or tea and 40 like both, the total
number of workers in the office is
(A) 200
(B) 250
(C) 240
(D) 210
Solution:
(72%+44%)−100%=16%
16%=40
100%=(40×100)/16=250
34. 100 kg of solution A is mixed with 60 kg of solution B. If solution A has tin and Copper in the ratio 1: 4 and solution B has lead
and tin in the ratio 3:2, then what is the amount of tin in the new solution
(A) 70 kg
(B) 36 kg
(C) 44 kg
(D) 56 kg
Solution:
Tin in 100kg of A = ( 100 x 1/5) = 20kg.
Tin in 60kg of B = ( 60 x 2/5) = 24kg.
Amount of tin in new alloy = ( 24 + 20 )kg = 44kg.
35. The difference between the length and the breadth of a blackboard is 8 cm. If the breadth is decreased by 4 cm and the length is
increased by 7 cm, the area remains the same. Find the dimensions of the blackboard?
(A) 30, 22
(B) 28, 20
(C) 34, 26
(D) 56. 48
Solution:
Let, breadth = x
Length = x+8
ATQ,
(x+8)x = (x+15)(x-4)
Or, X2 + 8x = x2 + 11x -60
Or, x = 20
Breadth = 20. So, Length = (20+8) = 28
36. X can complete a certain work in the same time in which Y and Z together can do it. If X and Y together can finish it in 10 days and
Z alone in 15 days, then Y alone can do it in?
(A) 22 days
(B) 18 days
(C) 20 days
(D) 25 days
Answer
Solution:
X+Y can do in 1 day = 1/10 part
Z can do in 1 day = 1/15 part
Now, X=Y+Z
=>X+Y=2Y+Z
=>1/10=2Y+1/15
Y=1/2*(1/10-1/15)=1/60
So, Y can do it in 60 days
37. The difference between the radii of bigger circle and smaller circle is 14 cm and the difference between their areas is 1056 cm 2.
Radius of the smaller circle is
(A) 7 cm
(B) 5 cm
(C) 9 cm
(D) 3 cm
Solution:
R-r=14
3.14 (R^2-r^2)=1056
22/7 (R-r)(R+r)=1056
(22×14)/7(R+r)=1056
44 (R+r)=1056
R+r=1056/44
R+r=24
on solving R-r=14 and R+r=24
we get, R=38/2=19cm
r=5cm
38. A jar contains a mixture of oil and water in the ratio 22:3. 50 litres of the mixture was taken out and 25 litres of water was added
to it. If water was 34% in the resultant mixture, what was the initial quantity of the mixture (in litres) in the jar?
(A) 175
(B) 150
(C) 75
(D) 125
Solution:
Let initial quantity of the mixture is x+50 litres. Oil : water=22:3
in x litres mixture Oil =22x/(22+3)=22x/25 , water=3x/25
25 litres water added ,
water=25+(3x/25)=(625+3x)/25
total volume x+25 litres
% of water =(625+3x)×100/{25×(x+25)}=34
or 4(625+3x)/(x+25)=34
or, 2500+12x=34x+850
or, 34x-12x=2500 850
or, 22x=1650
or, x=1650/22= 75 litres
initial quantity of mixture =x+50 =75+50=125 litres .
39. The ratio of the length to the breadth of a rectangular park is 3:2, if a man cycling along the boundary of the park at the speed of
12 Km/hr completes one round in 8 minutes, then what is the area of the park?
(A) 153600 m2
(B) 154000 m2
(C) 307400 m2
(D) None of these
Solution:
Perimeter = Distance travelled in 8 minutes,
=> Perimeter = 12000/60 * 8 = 1600 meter. [because Distance = Speed * Time]
As per question length is 3x and width is 2x
We know perimeter of rectangle is 2(L+B)
So, 2(3x+2x) = 1600
=> x = 160
So Length = 160*3 = 480 meter
and Width = 160*2 = 320 meter
Finally, Area = length * breadth
= 480 * 320 = 153600
40. A boat takes 19 hours to travel from A to point B and coming back to a point C midway between A and B. If the speed of the
stream is 4 km/hr and the speed of the boat in still water is 14 km/hr, what is the distance between A and B?
(A) 160 km
(B) 200 km
(C) 180 km
(D) 220 km
Solution:
Speed in downstream = (14 + 4) km/hr = 18 km/hr;
Speed in upstream = (14 4) km/hr = 10 km/hr.
Let the distance between A and B be x km. Then,
x/18 + (x/2)/10 = 19 ⇔ x/18 + x/20 = 19 ⇒ x = 180 km.
41. Given that 24 carat gold is pure gold, 18 carat gold is 3/4th of pure gold and 20 carat gold is 5/6th of pure gold, the ratio of the
pure gold in 18 carat gold to the pure gold in 20 carat gold is
(A) 5:8
(B) 10:9
(C) 8:5
(D) 9:10
Solution:
ATQ,
18 carat gold = ¾ pure gold
=3/4 X 24 = 18
20 carat gold = 5/6 pure gold
= 5/6 X 24 = 20
So, Required ratio = 18:20 = 9:10
42. On reducing the entry fee by 35% in a park, the number of people coming to the park increased by 40%, then the percent increase
or decrease in the income from the entry fee is
(A) 9% decrease
(B) 7% decrease
(C) 9% increase
(D) 5% decrease
Solution:

Total income = a × b = ab
Now, reducing the entry fee by 35% in a park, the number of people coming to the park increased by 40%
∴ New entry fee = a (35% of a) = 0.65a
New number of people = b + (40% of b) = 1.4b
New total income = 0.65a × 1.4b = 0.91ab
Decrease in income = ab 0.91ab = 0.09ab
% decrease in income =(0.09ab/ab)×100%
⇒ % decrease in income = 9%
43. Three numbers are added in pairs, the sums so obtained are 20, 27, 23. What are those three numbers?
(A) 6, 4 and 15
(B) 9, 11 and 14
(C) 8, 12 and 15
(D) 10, 4 and 17
44. After distributing the chocolates equally among 25 kids, 8 chocolates remain. Had the number of children been 28, 22 chocolates
would have been left after equally distributing. Find the total number of chocolates.
(A) 315
(B) 358
(C) 368
(D) 322
Solution:
Let total no. of chocolates = 25x + 8
Then (25x+8)-22 will be divisible by 28
=25x-14 is divisible by 28
= 28x-(3x+14) is divisible by 28
= 3x+14 is divisible by 28
x=14
So total no. of chocolates = 25(14)+8= 358
45. If radius of a circle is increased by 30% then its area is increased by
(B) 40%
(B) 70%
(C) 50%
(D) 69%
Solution:
So, Percentage change in area of the Circle is 30 + 30 +(30X30)/100 = 69%
46. An amount of Tk. 10,000 becomes T 20,736 in 2 years. If the rate of interest is compounded half yearly, what is the annual rate of
interest?
(A) 25%
(B) 20%
(C) 40%
(D) 30%
Solution:
nt
We know, p(1+r/n) =C
10000(1+r/200)4=20736
(1+r/200)4=20736/10000
=>1+r/200=12/10
r=(6/5-1)200
r=1/5*200=40%
47. The difference between two positive numbers is 3 and the sum of the squares is 369. Then the sum of the numbers is
(A) 27
(B) 33
(C) 20
(D) 81
Solution:
Let, the numbers be x & x-3
So,
X2 + (x-3)2 = 369
Or, x2 + x2 6x + 9 = 369
Or, x2 -3x 180 = 0
Solving, x = 15
Sum of the numbers are = 15 + (15-3) = 27
48. A hall, 20m long and 15m broad, is surrounded by a verandah of uniform width of 2.5m. The cost of flooring the verandah at Tk.
3.50 per square meter is
(A) Tk. 500
(B) Tk. 700
(C) Tk. 600
(D) Tk. 800
Solution:
Area of the hall = 20 ⨉ 15 = 300m2
Area of the hall with verandah = [{20+(2.5⨉2)}+{15+(2.5⨉2)}]=500m2
Area of the verandah = 500 300 = 200m2
The cost of flooring the verandah = 200 ⨉ 3.5 = 700 TK.
49. of the
two digits is 6, the number is-
(A) 51
(B) 24
(C) 15
(D) 42
Solution:
Let unit digit = x
And ten digit =y

Solving 1 and 2 we get,


X=5 and y=1
SO the number is = 15
50. In your bookshelf, you have five favorite books. If you decide to arrange these five books in every possible combination and moved
just one book in every half a minute. How much time it will take you to arrange?
(A) 3 hours
(B) 1 hours
(C) 2 hours
(D) 30 minutes
51. A number when divided by 3 leaves a remainder 1. When the quotient is divided by 2, it leaves a remainder 1. What will be the
remainder when the number is divided by 6?
(A) 2
(B) 3
(C) 4
(D) 5
Solution:
Let x be the number.when it is divided by 3 the remainder is 1.Let the quotient be y. Then x=3y+1.
When the quotient is divided by 2 the remainder is 1. So the quotient should be of the form 2z+1 for some integer z. ie y=2z+1
So x = 3y + 1 = 3(2z+1) +1= 6z+4.
So when x is divided by 6 the remainder will be 4.
52. A train starts from city Y at 2 PM and travels towards city X at 75 km/hr. Another train starts from city X at 1 PM and travels
towards Y at 60 km/hr. If the distance between these two cities is 330 km then at what time will they meet?
(A) 4:00 PM
(B) 5:00 PM
(C) 5:30 PM
(D) 6:00 PM
Solution:
Relative speed of the trains = 75+60 = 135 km/h
Remaining distance at 2pm = 330 60 = 270 km
So the train will meet after = 270/135 = 2 hours
So they will meet at = 2 + 2 = 4.00 pm
53. Two trains P and Q are moving in opposite direction at a rate of 36 km/hr and 45 km/hr respectively. A passenger is sitting in train
P, finds that his train passes train Q in 8 seconds. What is the length of train Q?
(A) 195 m
(B) 175 m
(C) 190 m
(D) 180 m
Solution:
Relative speed of the train = 36 + 45 = 81 km/h
P passes Q in 8 sec
So length of train Q= 81 ⨉ 8/3600 =0.18km = 180m
54. A boat takes 4 hours to cover a certain distance running downstream, while it requires 8 hours 48 minutes to cover the same
distance. Find the ratio between the speed of the current and the speed of the boat?
(A) 1:2
(B) 3:8
(C) 2:3
(D) 4:3
Solution:
Let the rate upstream of the boat =x kmph
and the rate downstream of the boat =y kmph
Distance travelled upstream in 8 hrs 48 min = Distance travelled downstream in 4 hrs.
Since distance = speed × time, we have
4
x×8 =y×4
5
44
X× =y×4
5
11
X× = y ⋯(equation 1)
5
Hence, speed of the boat = (y+x)/2
speed of the water = (y−x)/2
Required Ratio
=(y−x)/2 : (y+x)/2
= (y−x) : (y+x)
=(11x/5 − x) : (11x/5 + x) (∵ Substituted value of y from equation 1)
=(11x−5x) : (11x+5x) =6x:16x = 3:8
55. Lubana purchased 20 kg of pulses at a rate of Tk. 14.25 per kg and 30 kg of pulses at a rate of Tk 11.50 per kg. She decided to mix
the two and sold the mixture. To make a profit of 30%, what price per kg should he sell the mixture?
(A) 15.60
(B) 14.80
(C) 16.38
(D) 18.20
Solution:
20 kg pulses *14.25tk =285 tk
30 kg pulses *11.50tk= 345 tk
Total 630 tk for 50 kg pulses
Average cost of rice after he mixed =
630/50=12.6tk
pulses he bought is 12.6 rs per kilo..
To get 30%profit per kilo = 12.6+(12.6*30/100)
= 12.6+3.78
= 16.38
So he should sell his pulses @ 16.38 tk/kilo to earn 30% profit
56. Rahim invested in all Tk. 2600 in three different schemes at 8%, 4%, and 6% per annum simple interest. At the end of the year, he
received the same interest in all the three schemes. What is the money invested in scheme having 4% rate of interest?
(A) Tk. 700
(B) Tk. 800
(C) Tk 750
(D) Tk. 1200
Solution:
Let say Money invested at 4 % per annum = Tk. 100X
Let say Money invested at 6 % per annum = Tk. 100Y
Money invested at 8 % per annum = 2600 - 100 ( X + Y) Tk.
Interest at Tk. 100X = 100X * 4 * 1 /100 = Tk. 4X
Interest at Tk. 100Y = 100Y * 6 * 1 /100 = Tk. 6Y
4X = 6Y
2X = 3Y
Y = 2X/3
Interest at Tk. 2600 - 100 ( X + Y) ;
= (2600 - 100 ( X + Y)) * 8 * 1 /100
=( 26 - (X + Y) ) * 8
= 208 - 8X - 8Y
208 - 8X - 8Y = 4X
208 = 12X + 8Y
208 = 12X + 8(2X/3)
624 = 36X + 16X
52X = 624
X = 12
Money invested at 4 % per annum = 100X = 1200 Tk.

GENERAL KNOWLEDGE AND COMPUTER


Questions (57-80): Study the following questions carefully and answer the questions given below.
57. Which country bans on the oldest and most polluting diesel cars to protect climate?
(A) France
(B) Germany
(C) China
(D) None of these
58. Fathometer is used to measure
(A) Rainfall
(B) Earthquakes
(C) Ocean depth
(D) Sound intensity
59. The longest mountain range in the world is
(A) The Alps
(B) The Himalayas
(C) The Rockies
(D) The Andes
60. Which of the following organization is concerned with environmental issues?
(A) OIC
(B) IPCC
(C) MIGA
(D) WMO
61.
(A) Greenwich
(B) Indira Point
(C) New York
(D) None of these
62. Which of the following country has recently topped the E-Government Development Survey 2018?
(A) Denmark
(B) Australia
(C) Sweden
(D) United States
63. Which of the following is the largest and the deepest ocean of the world?
(A) Arctic
(B) Atlantic
(C) Pacific
(D) Indian
64. ASEAN is headquartered at
(A) Jakarta
(B) Male
(C) Kathmandu
(D) Kuala Lumpur
65. Which country is stand on second country in the world according to most roads?
(A) China
(B) India
(C) Afghanistan
(D) South Korea
66. Number of temporary members of the UN Security Council is
(A) 5
(B) 15
(C) 10
(D) None of these
67. Regional Cooperation Development Headquarter located in:
(A) Tehran
(B) Islamabad
(C) Istanbul
(D) None of these
68. Which country is developing a 750-acre Industrial zone in Bangladesh?
(A) Japan
(B) India
(C) Malaysia
(D) China
69. What is a portion of a document in which you set certain page formatting options?
(A) Section
(B) Page
(C) Document
(D) Page Setup
70. From where we can change the name of a User Account?
(A) Settings
(B) Computer Name
(C) Code Panel
(D) Control Panel
71. What is the shortcut key to open the Open dialog box?
(A) F12
(B) Alt + F12
(C) Shift +F12
(D) Ctrl+F12
72. Which one of the following is not a search engine?
(A) Bing
(B) Google
(C) Windows
(D) Yahoo
73. A number of letters that appears little below the normal text is called:
(A) Subscript
(B) Superscript
(C) Supertext
(D) Toptext
74. The rectangular area of the screen that displays a program, data and/or information is a
(A) Title bar
(B) Button
(C) Dialog box
(D) Window
75. Which of the following peripheral devices displays information to a user?
(A) Keyboard
(B) Monitor
(C) Both A and B
(D) None of these
76. Which of the following is not term of information technology?
(A) Cyberspace
(B) Modem
(C) Light storage
(D) Upload
77. USB is which type of storage device?
(A) Tertiary
(B) Secondary
(C) Primary
(D) None of these
78. Which of the followings is word processing software?
(A) WordPerfect
(B) Wordpad
(C) MS Word
(D) All of these
79. All of the following are examples of real security and privacy risks EXCEPT:
(A) Viruses
(B) Spam
(C) Hackers
(D) None of these
80. Which of the following is used for close a tab on a browser?
(A) Ctrl+Y
(B) Ctrl + A
(C) Ctrl + W
(D) Ctrl+T

ENGLISH
Questions (01-04): Fill in the blank with right option
1. Cellular phone service has ______ a new phase of communication.
(A) called
(B) started
(C) paved
(D) ushered
2. Children must be ____ with love and care.
(A) brought about
(B) brought as
(C) brought for
(D) brought up
3. The ____ to e-buses would not lower the level of emissions but merely ____ their place of origin.
(A) Transition, Change
(B) Shift, Altered
(C) Evolutionary, Vary
(D) None of these
4.
(A) up
(B) off
(C) up with
(D) through to

Questions (05-08): Select the word or phrase that is most closely opposite in meaning to the original word.
5. ACCUMULATE
(A) Amiss
(B) Produce
(C) Amass
(D) Scatter
6. EFFERVESCENT
(A) Vivacious
(B) Dull
(C) Mediocre
(D) Generous
7. HOSTILE
(A) Friendly
(B) Negative
(C) Positive
(D) Inimical
8. ILLUSTRIOUS
(A) Conspicuous
(B) Venerable
(C) Unknown
(D) Resplendent

Questions (09-12): Each sentence has a highlighted word or phrase. Choose the one word or phrase that best keeps the meaning of the
original sentence.
9. It is difficult to get young people to plan for their old age, which seems very distant to them.
(A) far away
(B) impossible
(C) observable
(D) fearful
10. Passenger ships and aircraft are often equipped with ship to shore or air-to-land radio telephones.
(A) highways
(B) railroads
(C) sailboats
(D) airplanes
11. It is not possible for people to recall everything that they have thought, felt, or done.
(A) appreciate
(B) remember
(C) repeat
(D) discuss
12. Receptors for the sense of smell are located at the top of the nasal cavity.
(A) upper end
(B) inner edge
(C) mouth
(D) division

Questions (13-16): Read the following passage and answer the questions given below it in the context of the passage.
Our eyes and ears might be called transformers because they sense the light and sound around us and turn them into electrical
impulses that the brain can interpret. These electrical impulses that have been transformed by the eye and ears reach the brain and are
turned into messages that we can interpret. For the eye, the process begins as the eye admits light waves, bends them at the cornea
and lens, and then focuses them on the retina. At the back of each eye, nerve fibers bundle together to form optic nerves, which join
and then split into two optic tracts. Some of the fibers cross, so that part of the input from the right visual field goes into the left side
of the brain, and vice versa. The process in the ear is carried out through sensory cells that are carried in fluid-filled canals and that are
extremely sensitive to vibration. Sound transformed into electricity travels along nerve fibers in the auditory nerve. These fibers form a
synod neurons that carry the messages to the auditory cortex on each side of the brain.
13. Which of t
(A) selects
(B) interprets
(C) lets in
(D) focuses on
14.
(A) group
(B) grow
(C) branch
(D) settle
15.
(A) tear
(B) fracture
(C) separate
(D) crack

MATHEMATICS
37. An inspector notices a thief from a distance of 200 meters after this thief starts running and the inspector chases him. The inspector
and the thief run at the speed of 11 km/hr and 10 km/hr respectively. The distance between them after 6 minutes is?
(A) 100 m
(B) 90 m
(C) 110 m
(D) 120 m
Solution:
Relative speed of policeman with respect to thief = (11 10) = 1 Km/hr.
Now the relative distance covered by policeman in 6 min
= Speed X Time = 1×(6/60)
= 1/10km = 100 m
The distance between the policeman and thief after 6 min = 200 100 = 100 m.
38. A man is running at a speed of 3 km/hr in the direction of the train whose length is 50 meters. If the train is moving at a speed of
63 km/hr then how many seconds will this train take to cross the man?
(A) 38 sec
(B) 35 sec
(C) 34 sec
(D) 30 sec
Answer
Solution:
Speed of train relative to man
= (63 3) km/hr = 60 km/hr
= (60 X 5/18) = 50/3 m/sec
So, Time taken to pass the man = (50 X 3/50) = 3 sec
39. A train left 30 minutes later than the Schedule time and to reach its destination in time it increased its speed by 250 km/hr. If the
target is 1500 km away, find the speed of the train?
(A) 800 km/hr
(B) 750 km/hr
(C) 725 km/hr
(D) 850 km/hr
Solution:
Let the usual time taken by the train = x km/hr
Distance to the destination = 1500 km
Case (i)
Speed = Distance / Time = (1500 / x) Hrs
Case (ii)
Time taken by the train = (x 1/2) Hrs
Distance to the destination = 1500 km
Speed = Distance / Time = 1500 / (x 1/2) Hrs
Increased speed = 250 km/hr
⇒ [1500 / (x 1/2)] [1500 / x] = 250
⇒ 1/(2×2 x) = 1/6
⇒ 2x2 x=6
⇒ (x 2)(2x + 3) = 0
⇒ x = 2 or -3/2
Since, the time cannot be negative,
The usual time taken by the train = 2 hrs
and the usual speed = (1500 / 2) = 750 km/hr.
40. In still water, a boat can travel at 5 km/hr. It takes 1 hour to row to a place and come back. If the velocity of the stream is 1
km/hr, how far is the place?
(A) 3.5 km
(B) 2.6 km
(C) 2.4 km
(D) None of these
Solution:
Speed downstream = (5 + 1)= 6 kmph;
Speed upstream = (5 1)= 4 kmph.
Let the required distance be x km.
Then,
x/6 + x/4 = 1
Or, 5x = 12
Or, x = 2.4
41. Three varieties of rice costing Tk 30, TK, 50, and Tk. 20 are mixed in the ratio 3:2:4 in terms of weight. A retailer sells the mixture
at Tk. 33 per kg. What percentage of profit does he make?
(A) 5%
(B) 10%
(C) 9%
(D) None of these
Solution:
Let,
Quantity of rice = 3x, 2x & 4x
Costing of rice = (30X3x + 50X2x + 20X4x) = 270x
Selling price of rice = 33(3x + 2x + 4x) = 279x
Profit = (297x 270x) = 27x
Profit percentage = (27x/270x)X100 = 10%
42. P charges at 7% p.a. simple interest to Q and R and lends a certain sum to R and Tk 2500 to Q. After 4 years, P completely receives
Tk. 1120 as interest from Q and R. Find the sum lent to R?
(A) TK 1500
(B) Tk. 3100
(C) Tk. 6400
(D) Tk. 900
Solution:
Amount lent to R = x.
Total amount of Q & R = (2500 + x)
ATQ,
(2500+x)X4X(7/100) = 1120
Or, x+ 2500 = 4000
Or, x = 1500
43. If 25% of a number is subtracted from a second number, the second number reduces to the five-sixth. The ratio of the first number
to the second number is
(A) 1:2
(B) 3:2
(C) 2:3
(D) None of these
Solution:
Let,
First Number = A
Second Number = B
B 1/4A = 5/6B
Or, B = 1/4A + 5/6B
Or, 1/6B = 1/4A
Or, 4B = 6A
Or, 2B = 3A
Or, 2/3B = A
Or, A:B = 2:3
44. Seats for Computer Science, Mechanical and Electronics in a university are in the ratio 7:5:8. There is a planning to increase these
seats by 50%, 40%, and 75% respectively. Find the ratio of increased seats?
(A) 6:7:8
(B) 8:6:9
(C) 3:4:5
(D) 3:2:4
Solution:
Let,
the seats for Computer Science = 7x
Mechanical = 5x and
Electronics = 8x
After increase,
Seats for Computer Science = 150% of 7x = 21x/2
Seats for Mechanical = 140% of 5x = 7x
Seats for Electronics = 175% of 8x = 14x
New ratio will be = 21x/2 : 7x : 14x = 3 : 2 : 4
45. The breadth of a rectangular field is increased by (r + 5)%, and the length decreased by r%. If the area of the field is unchanged,
what is the value of r?
(A) 20
(B) 15
(C) 8
(D) 10
Solution:
Let original length = x and original breadth = y,
Then , original area = xy
New area = [(100−r)x /100][(105+r)y /100]
=[(10500−5r−r2)/10000)]xy
∴ [(10500−5r−r2)/10000)]xy =xy
Or, r2+5r−500=0
Or, (r+25)(r−20)=0
Or, r=20
46. A piece of wire 78 cm long is bent in the form of an isosceles triangle. If the ratio of one of the equal sides to the base is 5:3, then
what is the length of the base?
(A) 16 cm
(B) 20 cm
(C) 18 cm
(D) 30 cm
Solution:
Given, it is isosceles triangle
So, 5x+5x+3x=78
Or, 13x=78
Or, x=78/13=6
base is 3x so 3×6=18cm
47. A train takes 50 sec to cross a boy travelling at 6 km/h in the opposite direction to it. Another train which is half as long as and 25%
faster take 30 sec to cross the stationary pole. Find the approximate length of the second train.
(A) 100 m
(B) 125 m
(C) 190 m
(D) 148 m
Solution:
Let the length and speed of the first train = P km and x km/h
So the length and speed of the 2nd train = P/2 km 5x/4 km/h

ATq,
(x + 6) * 50/3600 = 5x/4 * 60/3600
x + 6 = 3x/2
x = 12 km/h
From eq (1) We get P = 1/4km = 250 m
So the legth of the second train = 125 m
48. A bag contains 30 balls numbered I to 30. Two balls are drawn at random. What is the probability that the balls drawn contain a
number which is multiple of 4 or 6 but not a multiple of both.
(A) 1/8
(B) ¼
(C) 1/3
(D) None of these
Solution:
Total no. of balls = 30
Multiple of 4 or 6 but not a multiple of both is-
We know from 1 to 30 there is 8 balls that are 4, 6, 8, 16, 18, 20, 28, 30 [ 12 and 24 are multiple of both ]
30
Number of ways of drawing 2 balls out of 30 = C2 = 435
Number of ways of drawing multiple of 4 or 6 = 8C2 = 28
So the probability is = 28/435
49. A, B and C start running around a circular field having circumference 144 m at the same time from the same point. Speeds of A, B
and C are 6 m/min. 8 m/min and 12 m/min. Find after how much time, they will meet again at the same point for the first time.
(A) 72 min
(B) 36 min
(C) 144 min
(D) 18 min
Solution:
Time taken by A = (144/6) = 24 min
Time taken by B = (144/8) = 18 min
Time taken by C = (144/12) = 12 min
L.C.M. of 24, 18 & 12 = 144
50. Compound interest on a certain sum at the rate of 12% per annum after 2 years is Tk. 142464. Find the simple interest on that sum
at the rate of 15% per annum for 7 years.
(A) Tk. 568000
(B) Tk. 586000
(C) Tk. 588000
(D) Tk. 566000
Solution:
We know, C = P (1 + r/100)n
And C = I + P
ATq,
142464 + P = P(1 + 12/100)2
P = 142464/0.2544
P = 560000
Now simple interest I = pnr = 560000 x 7 x 15/100 = 588000
51. If px = q, qy = r and rz = p then the value of xyz is
(A) 1
(B) 1
(C) 0
(D) pqr
Solution:
Given,
rz = p
Or, (qy)z = p
Or, (px)yz = p
Or, pxyz = p
Or, xyz = 1
52. In a city, the fare of a cab consists of fixed charge plus the charge for the distance covered. For a journey of 8 km, the charge paid
is Tk. 300 and for 15 km, the charge paid is Tk. 335. Determine the charge a person will have to pay for 24 km?
(A) Tk. 236
(B) Tk. 248
(C) Tk. 346
(D) Tk. 380
Solution:
Let,
the fixed charge = x km and
the running charges be y/km.
Then, according to given condition,
x+8y = 300 (1)
x+15y = 335 (2)
From, (2) (1)
7y = 35
Or, y = 5
So, x = 300 8X5 = 260
For 24 km charge have to be paid
= 260+(24X5) = 380
53. Surface area of hollow cylinder
(A) 2πr-h
(B) 2πrh
(C) 2πh
(D) πr2
54. Rahim can do a piece of work in 20 days. Karim is 25% more efficient than Rahim. The number of days taken by Karim to do the
same piece of work is
(A) 15
(B) 19
(C) 18
(D) 16
Solution:

Being 25% more efficient, Karim can finish 1.25/20 work in one day.
So the number of days taken by Karim is 20/1.25 = 16 days.
55. I walk a certain distance and ride back taking a total time of 37 minutes. I could walk both way 55 minutes. How long would it take
me to ride both ways?
(A) 30 minutes
(B) 19 minutes
(C) 37 minutes
(D) 20 minutes
Solution:
Time taken in walking both ways = 55 minutes
Time taken in walking one ways=55/2=27.5 min
Time taken walking and riding back = 37 min
Time taken in riding on one way = 37 27.5 = 9.5 minutes
Time taken in riding both ways= 9.5×2=19 min

GENERAL KNOWLEDGE AND COMPUTER


61. Commonwealth of Nations headquarter situated in ?
(A) London
(B) Washington Dc
(C) Geneva
(D) None of these
63.
(A) China
(B) UAE
(C) Saudi Arabia
(D) None of these
64. Which country has adopted Bengali language the status of second state language?
(A) India
(B) Nepal
(C) Sierra Leone
(D) Uganda
65. Money market is a market for
(A) short-term fund
(B) long-term fund
(C) hedge fund
(D) Risk free assets
66. The value of Gold is determined in
(A) Rome
(B) Washington
(C) Teheran
(D) London
67. Head office of Coca Cola company is in-
(A) New York
(B) Atlanta
(C) New Jersey
(D) Florida
68. In absorption of insolation, the most significant part is played by
(A) Ozone
(B) Oxygen
(C) Haze
(D) Carbon dioxide
69. A light sensitive device that converts drawing, printed text or other images into digital form is
(A) Keyboard
(B) Plotter
(C) Scanner
(D) None of these
70. Documents, Movies, Images and Photographs etc. are stored at a
(A) Application Sever
(B) File Server
(C) Web Sever
(D) Print Server
71. Which of these toolbars allows changing of Fonts and their sizes?
(A) Formatting
(B) Standard
(C) Print Preview
(D) None of these
72. The basic unit of a worksheet into which you enter data in Excel is called a
(A) column
(B) box
(C) table
(D) cell
73. The process of transforming files from a computer on the Internet to your computer is called
(A) Forwarding
(B) Downloading
(C) FTP
(D) Uploading
74. Which of the following will not protect you from spam?
(A) spam blockers
(B) e-mail rules
(C) filters
(D) popup blocker
75. Which software is not a file compression utility ?
(A) Compress
(B) 7-zip
(C) Winrar
(D) Win zip
76. Superscript, subscript, strikethrough are known as ?
(A) Font Face
(B) Font style
(C) Font Format
(D) Font effects
77. In internet terminology IP means
(A) Internet provider
(B) Internet protocol
(C) Internet procedure
(D) Internet processor
78. The space left between a margin and start of a paragraph is called-
(A) Spacing
(B) Gutter
(C) Indentation
(D) Alignment
79. The portion that shows all the choice you can make while working in a window is called the-
(A) Menu bar
(B) Option
(C) Table
(D) Item bar
80. Which of the following is not an output device ?
(A) Printer
(B) Monitor
(C) Mouse
(D) Both A and B

ENGLISH
Questions (01-04): Fill in the blank with right option
1. Someone who is____ is hopeful about the future or the success of something in particular
(A) powerful
(B) pessimistic
(C) stagnant
(D) optimistic
2. While on a tour, tourists is only given a _______ time at each place of attraction
(A) particular
(B) limited
(C) confined
(D) certain
3. I just had to _______ the orders of my employer.
(A) carry away
(B) carry out
(C) carry through
(D) carry on
4. Many forms of cancer can be cured if _____ early
(A) Detected
(B) Detecting
(C) Detect
(D) None of these

Questions (05-08): Select the word that is most closely similar in meaning to the original word
5. BARBARIAN
(A) Unkind
(B) Unlikeness
(C) Impolite
(D) Uncivilized
6. CONCEAL
(A) Reveal
(B) Unfold
(C) Open
(D) Discover
No Answer.
7. ABSOLUTE
(A) Division
(B) Hair
(C) Small
(D) Complete
8. AMBITION
(A) Plan
(B) Proclamation
(C) Desire
(D) Decision

Questions (09-12): Read the following page and answer the questions given below it in the context of the passage.
A new atomic clock being developed for navigation satellites will perform better than previous devices. The Clock will use a new
microwave cavity design to provide a compact and lightweight package and new electronic techniques to maintain long-term stability.
The clock can provide precise navigation information because it is stable to one second in three million years. The differences in the
time when signals from four satellites arrive at one location can be used to calculate that position to within a few yards.
9. It can be inferred from the passage that the new clock will be-
(A) long-lasting
(B) harmful to humans
(C) Attractive looking
(D) None of these
10. According to the passage signals from how many satellites will be used to calculate a position?
(A) 1
(B) 4
(C) 3
(D) 2
11. What is the primary purpose of the passage?
(A) To teach a lesson
(B) To sell a product
(C) To support a theory
(D) To provide information
12. From the passage, it can be inferred that which of the following characteristics of the clock mentioned will be most impressive?
(A) Its compact size
(B) Its weight
(C) Its accuracy
(D) All of these

Questions (13-16) Select the pair that best expresses a relationship similar to that expressed in the original pair.
13. INSULT: HUMILIATE
(A) Dog : Bark
(B) Shoot : Kill
(C) Injury : Pungent
(D) Abuse : happy
14. ARENA : CONFLICT
(A) Mirage: Reality
(B) Asylum: Pursuit
(C) Utopia: Place
(D) Forum : Discussion
15. FANS: BLEACHERS
(A) Team: Goalposts
(B) Conductor : Podium
(C) Audience : Seats
(D) Referee : Decision
16. LAWN: GRASS
(A) Pelt : Fur
(B) Rice : Farm
(C) Skin : Gout
(D) Wool : Sheep

BANGLA
(17-32): Choose the correct answer
17.
(A)
(B)
(C)
(D)
18. ?
(A)
(B)
(C)
(D)
19. ?
(A)
(B)
(C)
(D)
20. ?
(A)
(B)
(C)
(D)
21. ?
(A)
(B)
(C)
(D)
22. ?
(A)
(B)
(C)
(D)
23. ?
(A)
(B)
(C)
(D)
24. ?
(A)
(B)
(C)
(D)
25. ?
(A)
(B)
(C)
(D)
26. ?
(A)
(B)
(C)
(D)
27. ?
(A)
(B)
(C)
(D)
28. ?
(A)
(B)
(C)
(D)
29. ?
(A)
(B)
(C)
(D)
30. - ?
(A)
(B)
(C)
(D)
31.
(A)
(B)
(C)
(D)
32. ?
(A)
(B)
(C)
(D)

MATHEMATICS
Questions (33-56): Read the following questions carefully and choose the right answer.
33. The length of a rectangle is 20% more than its breadth. Find the ratio of the area of the rectangle to that of the square whose side
is equal to the breadth of the rectangle.
(A) 8:9
(B) 3:2
(C) 6:5
(D) 2:1
Solution:
Let the breadth of the rectangle be x
∴ According to question,
Length = 1.20x
∴ Area of rectangle = 1.20x × x = 1.20x2
Area of square = x × x = x2
∴ Required ratio = 1.20x2/x2
=12/10= 6/5
34. A tank has two leakages. The first leakage alone can empty the tank in 9 minutes and the second alone would have done it in 6
minutes. If water leaks out at a constant rate, how long does it take both the leakage together to empty the tank?
(A) 3.6 min
(B) 3.1 min
(C) 3.5 min
(D) 4.0 min
Solution:
1st case,
In 1 min tank is emptied 1/9 part
2nd case,
In 1 min tank is emptied 1/6 part
So, together in 1 min, they empties = (1/9 + 1/6) = 5/18 part
So, both take time = 18/5 min = 3.6 min.
35. Worker P is 50% as efficient as worker Q. Worker R does half of the work done by P and Q together. If R alone does the work in 40
days, then P, Q and R together can do the work in
(A) 20 1/3 days
(B) 25 days
(C) 15 days
(D) 13 1/3 days
Solution:
day
work)
s one day work is 5% + 2.5 % = 7.5 %

It means that they will do 100% work in 100%/ 7.5% = 1000/75 = 40/3 = 13 1/3 days
36. The sum of the circumference of a circle and the perimeter of a rectangle is 130 cm. The area of the rectangle is 104cm2 and the
length of the rectangle is 13 cm. What is the area of the circle?
(A) 516 cm2
(B) 616 cm2
(C) 816 cm2
(D) 216 cm2
Solution:
Length of rectangle = L = 13 cm
breadth = B = 104/13 = 8 cm
Perimeter = 2(13+8) = 42
ATQ,
2πr + 42 = 130
Or, 2πr = 88
Or, r = 14
Area of circle = πr2 =22/7 X 142 = 616
37. A box is made in the form of a cube. If a second cubical box has inside dimensions three times those of the first box, how many
times as much does the second box contain?
(A) 27
(B) 12
(C) 9
(D) 6
Solution:
If the second box has each dimension three times that of the first box, then its volume is 3 × 3 × 3 = 27 times as great.
38. If 2 kg of metal, of which 1/3 is zinc and the rest is copper, be mixed with 3 kg of metal, of which ¼ is zinc and the rest is copper,
then what will be the ratio of zinc to copper in the mixture?
(A) 17:43
(B) 13:42
(C) 19:43
(D) 15:42
Solution:
Quantity of zinc in the mixture = 2(1/3) +3(1/4) = 17/12
Quantity of copper in the metal = 3 + 2 17/12 = 43/12
So ratio = 17/12 : 43/12 = 17 : 43
39. One filling pipe P is three times faster than another filling pipe Q, if P can fill tank in 24 hours, then what is the time taken to
completely fill the tank if both the pipes are opened together?
(A) 12 hours
(B) 16 hours
(C) 18 hours
(D) 14 hours
Time taken by P = 24 hours
Time taken by Q = 24X3 = 72 hours
Required time taken = (24X72)/(24+72) = 18 hours
40. A box contains 4 tennis ball, 6 season and 8 dues balls. 3 balls are randomly drawn from the box. What is the probability that the
balls are different?
(A) 2/17
(B) 4/17
(C) 4/11
(D) 3/13
Solution:

There are a total of 18 balls.


18
Total ways = 3 balls can be chosen in C3 ways
= 18!/(3!X15!) = (18 × 17 × 16)/(3X2X1) = 816
Favorable ways = 1 tennis ball, 1 season ball, and 1 dues Ball = 4 × 6 × 8 = 192
Probability = 192/816 = 4/17
41. On dividing an individual number by 342, we get 47 as remainder. If the same number is divided by 18, what will be the
remainder?
(A) 19
(B) 15
(C) 13
(D) 11
Solution:
Consider the number be 342 + 47 = 389.
Therefore 389 divided by 18 leave remainder 11

42. Rahim and Karim donated Tk. 100 each in charity. Karim gives each Tk. 1 more than Rahim and Rahim distributes money to 5 more
people than Karim. How many people are there in this charity?
(A) 45
(B) 72
(C) 90
(D) 60
Solution:
Let, Karim gives x taka each.
Rahim gives = x+1 taka each
ATQ,
100/x 100/(x+1) = 5
Or, x2 + x -20 = 0
Or, (x-4)(x+5) = 0
Taken, x = 4
Karim gives to = 100/4 = 25 people
Rahim gives to = 100/5 = 20 people
Total people = 20+25 = 45 people
43. A square is inscribed in a circle of diameter 2a and another square is circumscribing circle. The difference between the areas of
outer and inner squares is
(A) a2
(B) 2a2
(C) 3a2
(D) 4a2
Solution:
Area or outer square = 2a x 2a = 4a2
In△BAD
(BD)2=(AD)2+(AB)2
(2a)2=x2+x2
4a2=2x2
2a2=x2
x=2 √a.
Area of inner square =√2a x √2a=2a2
Difference between the areas of the outer and inner square =4a2−2a2=2a2
44.
income?
(A) 7%
(B) 8%
(C) 9%
(D) 10%
Solution:
Let,
C = 100
So, B = 100-25 = 75
And,
A = 75 + 75 X 20/100 = 75+15 = 90

= {(100-90)/100}X100 = 10%
45. (2√27 -√75 + √12) is equal to
(A) 4√3
(B) √3
(C) 3√3
(D) 2√3
Solution:
(2√27 -√75 + √12)
=(6√3 5√3 + 2√3)
=3√3
46. There are n students in a school. If r% among the students are 12 years or younger, which of the following expressions represents
the number of students who are older than 12?
(A) n(1-r )
(B) n(100-r)/100
(C) n(1- r)/100
(D) 100(1-r)n
47. 125 gallons of a mixture contains 20% water. What amount of additional water should be added such that water content be raised
to 25%?
(A) 15/2 gallons
(B) 13/2gallons
(C) 9/2 gallons
(D) 8 ⅓ gallons
Solution:
In the original 125 gallons of mixture, 20% is water, thus 80% is wine. Hence, no. of gallons of others:
125 X 80% = 100 gallons
In the new mixture, water makes up 25%, thus 75% is others. As no. of gallons of others is unchanged, 100 gallons = 75% the new
mixture volume.
The total volume of the new mixture is : 100 / 75% = 100/ 0.75 = 133.3 gallons.
No. of gallons of added water = 133.3 125 = 8.3 = 8 ⅓
48. The sum of squares of two numbers is 80 and the square of difference between the two number is 36. Find the product of two
numbers
(A) 11
(B) 22
(C) 33
(D) 26
Solution:
Two numbers x & y
ATQ,
x2 + y2 = 80
And,
(x y)2 = 36
Or, x2 + y2 2xy = 36
Or, 80 2xy = 36
Or, 2xy = 44
Or, xy = 22
The product of two numbers is: 22
49. A student loses 1 mark for every wrong answer and scores 2 marks for every correct answer. If he answers all the 60 questions in
an exam and scores 39 marks, how many of them were correct?
(A) 27
(B) 31
(C) 33
(D) 37
Solution:
Let us assume that he answered x question correctly. Marks scored by him in x question = 2x
Wrong answer would be = 60 x
Marks lost by him in 60 x question = 60 x
ATQ,
2x (60 x) = 39
Or, 3x = 99
Or, x = 33
50. There are two stations of length 162 meters and 120 meter respectively. A train takes 15 seconds to pass first station and 18
seconds to pass another station. Determine the length of the train.
(A) 90 m
(B) 70 m
(C) 100 m
(D) 95 m
Solution:
Let length of train = L
In first case, train covers 162 + L m in 15 seconds and in second case train covers 120 + L m in 18 seconds.
Speeds are (162 + L)/18 and (120 + L)/15 respectively.
But they are equal.
So,
(162 + L)/18 = (120 + L)/15
Or, (162 + L)15 = (120 + L)18
Or, 2430 2160 = 3L
Or, L = 270/3 = 90 m
51. A man rows a certain distance along the stream and against the stream in 1 hour and 1.5 hours respectively. If the velocity of the
current is 3 km/hr, what is the speed of the man in still water?
(A) 12 km/hr
(B) 15 km/hr
(C) 18 km/hr
(D) 13 km/hr
Solution:
Let the speed of the man in still water = x
Given that, speed of the stream = 3 kmph
Speed downstream = (x+3) kmph
Speed upstream = (x 3) kmph
ATQ,
(x+3)X1 = (x-3)X1.5
Or, x + 3 = 1.5x 4.5
Or, 0.5x = 7.5
Or, x = 15
52. A milkman pays Tk. 6.40 per liter of milk. He adds water and sells the mixture at Tk. 8 per liter. By doing this, he makes 37.5%
profit. Find the proportion of water to milk received by the customer.
(A) 1:12
(B) 1:10
(C) 1:15
(D) 1:20
Solution:
Let the quantity of milk purchase be x and quantity of water added be y.
Then, ratio of water to milk be y : x.
CP = 6.4x
SP = 8(x+y)
Profit per cent = 37.5%
Therefore,
8(x+y)=6.4×1.375
Or, 8x + 8y = 8.8x
Or, 8y = 0.8x
Or, y/x= 0.8/8 = 1:10
53. A leading library charges c cents for the first week that a book is loaned and f cents for each day over one week. What is the cost
for taking out a book for d days, where d is greater than 7.
(A) c + fd
(B) cd
(C) c + f(d-7)
(D) cd + f
54. Find the compound interest at the rate of 10% per annum for four years on the principal which in four years at the rate of 4% per
annum gives Tk. 1600 as simple interest.
(A) Tk. 4641
(B) Tk. 4732
(C) Tk. 4321
(D) Tk. 4899
Solution:
P = I/nr = 1600/(4/100 x 4) = 10000
C= P(1+r)n = 10000(1+10/100)4 = 14641
Interest = 14641 10000 = 4641
55. A wheel rotates 10 times per minutes and movies 20 m during each rotation. How many feet does the wheel move in 1 hour?
(A) 10000
(B) 20000
(C) 18000
(D) 12000
Answer
Solution:
Number of times wheel moves in 1 hour = 10 X 60 = 600
:. Distance moves = (600 X 20) = 12000 m
1 m = 3.28 feet
12000 m = 3.28 X 12000 = 39,360 feet.
56. An amount of Tk. 366 is to be divided among P, Q and R in such a manner that Q gets two-third as much as P and R together, and
P may get half as much as Q and R together. What is the share of P?
(A) 130
(B) 122
(C) 173
(D) 146
Solution:
Given,
Q = 2/3(P+R)
Or, 3Q = 2P +2R
Again,
P = (Q+R)/2
Or, 2P = Q+R
Now,
P+Q+R = 366
Or, P+2P = 366
Or, 3P = 366
Or, P = 122

COMPUTER
57. Information on a computer is stored as what?
(A) analog data
(B) digital data
(C) modem data
(D) None of these
58. Graphical pictures that represent an object like file, folders etc. are:
(A) Icons
(B) Desktop
(C) Taskbar
(D) Windows
59. Computer Virus is a___________.
(A) Hardware
(B) Freeware
(C) Bacteria
(D) Software
60. Verification of a login name and password is known as
(A) authentication
(B) Configuration
(C) Accessibility
(D) Logging in
61. When cutting and pasting, cutting section is temporarily stored in
(A) Dashboard
(B) Hard drive
(C) Diskette
(D) Clipboard
62. The set of instructions that tells the computer what to do is
(A) Softcopy
(B) Software
(C) Hardware
(D) Hardcopy
63. Which one of this also known as read/write memory
(A) RAM
(B) ROM
(C) DVD
(D) Hard Disk
64. In a PowerPoint presentation animation can be repeated how many times?
(A) 2 to 5
(B) 2 to 10
(C) 1 to 5
(D) 1 to 20
65. Which key combination is used to insert a Page Break in MS Word?
(A) Shift+Enter
(B) Alt+Enter
(C) Space+ Enter
(D) Ctrl+Enter
66. A computer on internet is identified by:
(A) IP address
(B) e-mail address
(C) street address
(D) None of these
67. The basic unit or a worksheet into which you enter data in Excel is called a
(A) column
(B) box
(C) cell
(D) table
68. Operating system is the most common type of ______ software.
(A) Application
(B) System
(C) Communication
(D) None of these

GENERAL KNOWLEDGE
69. World Trade Organization came into existence in _______.
(A) 1992
(B) 1993
(C) 1994
(D) 1995
70. Mount Etna is a famous volcano located in __________.
(A) Argentina
(B) Italy
(C) Mexico
(D) Philippines
71. Which of the county is known as the Sugar Bowl of the World?
(A) Brazil
(B) Mexico
(C) Cuba
(D) Algeria
72.
(A) New York
(B) Sweden
(C) Canada
(D) Uganda
73. Head office of international court of Justice (II) is located in ______.
(A) New York, USA
(B) Tokyo, Japan
(c) Hague, Netherlands.
(D) None of these
74. Which of the following countries leads in the production of gold?
(A) China
(B) Bhutan
(C) Tibet
(D) None of these
75.
(A) 75 km
(B) 65 km
(C) 50 km
(D) 55 km
76. Which of the following country has recently topped the E-Government Development Survey 2018?
(A) Sweden
(B) Australia
(C) Denmark
(D) United States
77. Profits of a firm that are distributed or given out to its investors are called
(A) Dividends
(B) Equity
(C) Bands
(D) None of these
78. The device used for measuring altitudes is
(A) Audiometer
(B) Ammeter
(C) Altimeter
(D) Galvanometer
79. Which of the following is called Blue Planet?
(A) Saturn
(B) Mars
(C) Jupiter
(D) Earth
80. Point of sale (POS) machine is widely used by the
(A) Tellers
(B) Merchants
(C) Government
(D) Central bank

ENGLISH
Question (01-04) Fill in the blanks with right options
01. It is necessary standards are maintained
(A) insure
(B) influence
(C) ensure
(D) control
02. He is very ; he believes anything
(A) gullible
(B) fallible
(C) sensible
(D) credible
03. It is not for a man to be continued to the pursuit of wealth
(A) easy
(B) healthy
(C) possible
(D) common
04. I Hope that the rain will for our picnic tomorrow
(A) set back
(B) put off
(C) stay out
(D) keep off

Questions (05-07): Select the Word or phrase that best completes the sentence
05. The kid shad prepared for the quiz and they bore with many prizes in the finale
(A) bear away
(B) bear out
(C) bear down
(D) None of these
06. I have told him that there is many solutions to the problem and he need not worry so much.
(A) there are few
(B) there are many
(C) there are much
(D) None of these
07. You should not boasting of your achievements
(A) boast of
(B) boast at
(C) boast for
(D) None of these

Questions (08-10): Select the word gives under cach question which you think is the ANTONYM of the underlined word
08. The railway lines are EXTENDED and we are quite happy about it
(A) Protracted
(B) Widened
(C) Elongated
(D) Curtailed
09. We are ANXIOUS to avoid any problems with regard to this
(A) Worried
(B) Composed
(C) Cool
(D) Careless
10. He is extremely INTELLIGENT but proud
(A) arrogant
(B) weak
(C) dull
(D) ignorant

BANGLA
Question (11-20): Choose the correct answer
11. ?
(A)
(B)
(C)
(D)
12. ?
(A)
(B)
(C)
(D)
13. ?
(A)
(B) + =
(C) + =
(D) + =
14. ?
(A)
(B)
(C)
(D)
15. - ?
(A) +
(B) +
(C) +
(D)
16. ?
(A) -
(B) -
(C) -
(D) -
17. - ?
(A)
(B)
(C)
(D)
18. ?
(A)
(B)
(C)
(D)
19. ?
(A)
(B)
(C)
(D)
20. ?
(A)
(B)
(C)
(D)

MATHEMATICS
Questions (21-30) Read the following questions carefully and choose
21. The diameters of two circles are the side of a square and the diagonal of the square. The ratio of the areas of the smaller circle and
the larger circles is
(A) 1:2
(B) 1:4
(C) 1:√2
(D) √2:√3
Solution:
Diagonal of a square = √2×Side
Ratio of area of smaller circle to larger circle
=πr12/πr22=[π×(a/2)2] / [π×(√2a/2)2]
Here, a = Diameter of smaller circle
= 12=1:2
22. Two circles of equal radii touch externally at a point P From a point T on the tangent at P, tangents
TQ and TR are drawn to the circles with points of contact Q and R Respectively. the relation TQ and TR is
(A) TQ < TR
(B) TQ > TR
(C) TQ = TR
(D) TQ = 2TR
Solution:

23. A person rides a bicycle round a circular path of radius 50 m. The radius of the wheel of the bicycle is 30 cm. The cycle comes to
the starting point for the first time in 1 hour. what is the number of
revolutions of the wheel in 15 minutes?
(A) 20
(B) 25
(C) 30
(D) 35
Solution:
Circumference of the circular path = 2π50 = 100 π m
This means 100? m is the distance covered in 60 minutes
Therefore distance covered in 15 minutes = 1/4 x 100 π m = 25π m
Now distance covered in one revolution of the wheel = 2 π(1/2) m = π m
Therefore number of revolutions in 15 minutes = 25π /π = 25
24. In covering a certain distance, the speeds of A and B are in the ratio of 3:4. A takes 30 minutes more than B to reach the
destination. The time taken by A to reach the destination is
(A) 10 hour
(B) 1.5 hours
(C) 2.0 hours
(D) 25 hours
Solution:
Ratio of speed = 3:4
Ratio of time = 4:3
let A takes 4x hrs,B takes 3x hrs
then 4x-3x = 30/60 hr

x= ½ hr
Time taken by A to reach the destination is 4x = 4 * ½ = 2 hr
25. Two lots of onions with equal quantity, one costing TK.10 per kg and the other cost TK.15 per kg are mixed together and whole lot
is sold at Tk 15 per kg. What is the profit or loss ?
(A) 10% loss
(B) 10% profit
(C) 20% loss
(D) 20% profit
Solution:
Let each lot of onion-contains x k
onion, then total cost price of these two lots together
= 10x + 15x= 25x
Selling price of whole lot
= 15x(x+x)=15×2x=30x
Profit percentage
= (30x−25x)/25x × 100%
= 5x/25x × 100% = 20% profit
26. A and B are partners in a business. A contributes 1/4 of the capital for 15 months and B 2/3 of the profit. Find for how long B's
money was used.
(A) 6 months
(B) 8 months
(C) 10 months
(D) 12 month
Solution:
B received 2/3 of the profit
A:B=1:2
Let the total capital = x
Then A's capital = x/4
B's capital = x x/4 = 3x/4
Assume B's money was used for b months
Then A:B = (x/4)*15 : (3x/4)*b = 1 : 2
15/4 : 3b/4 = 1 : 2
15 : 3b = 1 : 2
5:b=1:2
5/b = 1/ 2
b = 5*2 = 10
27. If 6 men and 8 boys can do a piece of work in 10 days and 26 men and 48 boys can do the same in 2 days, the time taken by 15
men and 20 boys to do the same type of work will be
(A) 4 days
(B) 3 days
(C) 6 days
(D) 7 days
Solution:
Given that
6 men and 8 boys can do a piece of work in 10 days
26 men and 48 boys can do the same in 2 days
As the work done is equal,
10(6M + 8B) = 2(26M + 48B)
60M + 80B = 52M + 96B
M = 2B

Now Put (1) in 15M + 20B


15M + 10M = 25M
Now, 6M + 8B in 10 days
(6M + 4M) 10 = 100M
Then D(25M) = 100M
D = 4 days.
28. Pipe A alone can fill a tank in 8 hours. Pipe B can fill it in 6 hours. If both the pipes are opened and afler 2 hours pipe A is closed,
then the other pipe will fill the tank in
(A) 6 hours
(B) 3 hours
(C) 4 hours
(D) 2 hours
Solution:
Part of the tank filled by pipe A and B in 2 hours
2( 1/6 + 1/8)
= 7/12
Remaining part = 1 – 7/12 = 5/12
This part is filled by pipe B.
1
∴ Required Time = 5/12 × 8 = 10/3 = 3 hours
3
29. A train of length 150 m takes 10 seconds to cross another train 100 m long coming from the opposite direction. If the speed of first
train is 30 km/hr, what is the speed of second train?
(A) 48 km/hr
(B) 54 k km/hr
(C) 60 km/h
(D) 72 km/hr
Solution:
Let speed of the other train = S m /sec.
Speed of the first train = 30 kmph = (30 *5)/18 = 8.33 m/sec.
Relative speed = (8.33 + S) m/sec.
With this relative speed train needs to be covered (150 + 100) m in 10 seconds.
So,
ST = D
(8.33 + S) * 10 = 250
(8.33 + S) = 25
S = 16.67 m/sec
Speed = 16.67 m/sec = (16.67 * 18)/5 = 60 kmph.
30. In an acute angled triangle ABC, if sin2(A +B-C)=1 and tan(B+C-A)=√3 . then the value angle B is
(A) 52
(B) 60
(C) 30
(D) 67
Solution:
Given
Sin2(A+B-C)= 1
Sin2(A+B-C)= Sin900
A+B-C= 450
Also given,
tan(B+C-A)= √3
tan(B+C-A)= tan600
B+C-A=600
From (i) and(ii) we get, B= 52.5

GENERAL KNOWLEDGE
Questions (31-40): Read the following questions carefully and choose the right answer
31. The fraction of solar energy reflected from Earth into space is known as
(A) insolation
(B) albedo
(C) irradiation
(D) heat los
32. Which of the following metals is known to pollute the air of big cities with a large member vehicles ?
(A) Mercury
(B) Iron
(C) Lead
(D) Copper
33. Which from the following can cause a tsunami?
(A) Volcano
(B) Avalanche
(C) Tornado
(D) Earthquack
34. Who was awarded the Golden Glove award for the best goalkeeper at the World Cup 2014?
(A) Tim Krul
(B) Manuel Neuer
(C) Julio Cesar
(D) Sergio Romeo
35. The standard sea level in millibars is
(A) 1013 mb
(5) 1060 mb
(C) 960 mb
(D) 990 mb
36. The official working languages recognised by the UNO are
(A) Chinese and English
(B) French and Russian
(C) Spanish and Arabic
(D) All of the above
37. Which country won the first football World Cup
(A) Brazil
(B) Germany
(C) Uruguay
(D) Argentina
38. The branch of science that studies cells is called
(A) cytology
(B) entomology
(C) homoplasty
(D) hormonology
39. The scientist who first discovered that the earth revolves around the sun was
(A) Newton
(B) Dalton
(C) Copernicus
(D) Einstein
40. Which is the biggest literary award of Britain?
(A) David Colien Award
(B) Kalinya Award
(C) Owen Global Prize
(D) Golden Bear Award

SUBJECT RELATED QUESTIONS


Questions (41-80): Read the following questions carefully and choose the right answer
41. Suppose a program las floating constant 1.414, what's the best way to convert this as "float" data type?
(A) (float) 1.414
(B) float(1.414)
(C) 1.414f or 1.414F
(D) None of these
42. Consider the following variable declarations and definitions in C
i) int var_9=1;
ii) int 9_var=2 ;
i) int_=3
Choose the correct statement writ. above variables
(A) Both i) and ii) are valid
(B) Only i) is valid
(C) Both i) & iii) are valid
(D) All of these
43. For a given integer, which of the following operators can be used to "set" and reset" a particular bit respectively?
(A) I and &
(B) && and I
(C) & and I
(D) II and &&
44. Let x be an integer which can take a value of 0 or 1. The statement if (x==0)x=1: else x=0: is equivalent to which one of the
following?
(A) x=1+x
(B) x=1-x
(C) x=x-1
(D) x=1%x
45. Which of the following is not a stable sorting algorithm in its typical implementation
(A) Insertion Sort
(B) Merge Sort
(C) Quick Sort
(D) Bubble Sort
46. You have to sort 1 GB of data with only 100 MB of available main memory. Which sorting technique
will be most appropriate?
(A) Heap sort
(B) Quick sort
(C) Insertion sort
(D) Merge sort
47. Randomized quicksort is an extension of quicksort where the pivot is chosen randomly. What is the worst case complexity of sorting
n numbers using randomized quicksort
(A) O(n)
(B) O(n2)
(C) O(n logn)
(D) O(n!)
48. Consider the following recursive function fun(x, y). What is the value of fun(4,3)
int fun(int x, int y)
{
If(x==0)
retum y;
return fun(x-1, x + y);
}
(A) 9
(B) 10
(C) 12
(D) 13
49. The average number of key comparisons done in a successful sequential search in a list of length it is
(A) log n
(B) (n+1)/2
(C) (n-1)/2
(D) n/2
50. In UNIX, processes that have finished execution but have not yet had their status collected are known as
(A) Sleeping processes
(B) Stopped processes
(C) Zombie processes
(D) Orphan processes
51. In UNIX, the login prompt can be changed by changing the contents of the file
(A) gettydefs
(B) crontab
(C) init
(D) inittab
52. Which of the following UNIX command allows scheduling a program to be executed at the specifies time?
(A) nice
(B) cron
(C) date and time
(D) schedule
53. Which of the following is major part of time taken when accessing data on the disk?
(A) Settle time
(B) Rotational latency
(C) Waiting time
(D) Seek time
54. Which of the following programming helps you to learn Android programming
(A) C
(B) SOL
(C) Java
(D) Python
55. Which tool could be used for detecting vulnerability through SQL Injection?
(A) nmap
(B) metaslott
(C) bit defender
(D) UML
56. Which of the following data structure is non-linear type?
(A) Strings
(B) Lists
(C) Stacks
(D) None of these
57. Which of the following is not a standard synchronous communication protocol ?
(A) PAS
(B) SDLC
(C) SLIP
(D) SMTP
58. Which of the standard protocol for network management features?
(A) FTP
(B) SNA
(C) SNMP
(D) SMTP
59. How many pairs of stations can simultaneously communicate on Ethernet LAN ?
(A) 1
(B) 2
(C) 3
(D) Multiple
60. A path for carrying signals between a source and a destination is known as
(A) Router
(B) Channel
(C) Link
(D) Block
61. In SQL the command ----------- is used to recompile a view
(A) COMPILE VIEW
(B) DEFINE VIEW
(C) ALTER VIEW
(D) CREATE VIEW
62. Which of the following requires the most time in SDLC?
(A) Requirement Analysis
(B) Testing
(C) Deployment
(D) Design
63. Which of the declaration is correct?
(A) int length
(B) char int:
(C) int long:
(D) float double
64. POP3 is a protocol for
(A) Email Sending
(B) Email Composing
(C) Email Receiving
(D) Email Storing
65. Which server can you use to dynamically assign IP addresses to the PCs in a LAN ?
(A) DHCP server
(B) Web server
(C) DNS server
(D) FTP server
66. The operator that cannot be overloaded is
(A) ++
(B) ()
(C) ~
(D) ::
67. Which of the following is the destructor for the class Vehicle?
(A) *Vehicle()
(B) ~Vehicle()
(C) ~Vehicle(int value)
(D) *Vehicle(int value)
68. An n x n Array v is defined as follows v[i, j] = i-j for all i, j, 1 <= i <= n, 1 <= j <= n
The sum of the elements of the array is
(A) 0
(B) n-1
(C) n2 -3n +2
(D) n2 (n+1)/2
69. The maximum number of binary trees that can be formed with three unlabeled nodes is
(A) 16
(B) 3
(C) 5
(D) 4
70. What is the time complexity of Huffman Coding?
(A) O(N)
(B) O(NlogN)
(C) O(N(logN)2)
(D) O(N2)
71. Which of the following TCP/IP addresses constitute the loopback address?
(A) 1.1.1.1
(B) 255.255.255.255
(C) 127.0.0.0
(D) 127.0.0.1
72. Which of the following operator functions cannot be global, i.e.. must be a member function.
(A) Conversion Operator
(B) new
(C) delete
(D) All of these
73. Which of the following is not an operator in Java?
(A) instanceof
(B) sizeof
(C) new
(D) >>>=
74. Find the output of the following Java code line:
System.out println(math.floor(-7.4))
(A) -7
(B -7.4
(C) -8
(D) -7.2
75. Which function overloads the >> operator?
(A) gt()
(B) more
(C) ge()
(D) None of these
76. What is the output of the following code?
print 9//2
(A) 4
(B) 4.0
(C) 4.5
(D) Error
77. Which of these is not a core data type?
(A) Lists
(B) Dictionary
(C) Class
(D) Tuples
78. Which of the following function convert a string to a float in python?
(A) int(x [,base])
(B) float(x)
(C) long(x[,based])
(D) str(x)
79. Which of the following process scheduling algorithm may lead to starvation?
(A) FIFO
(B) Round Robin
(C) Shortest Job Next
(D) None of these
80. The maximum number of processes that can be in Ready state for a computer system with n CPUS is
(A) n
(B) n2
(C) 2n
(D) Independent of n
ENGLISH
(01-04) : Choose the word that is closest in meaning to the underlined word.
1. The teacher gave us explicit instruction.
(A) Detailed
(B) Definable
(C) Resolvable
(D) Demonstrative
2. When the leadership changed, his position in the organization became precarious.
(A) secure
(B) exalted
(C) important
(D) uncertain
3. The spectacle was most pleasing.
(A) Impression
(B) View
(C) Sight
(D) Scenery
4. He gains success by dint of hard work.
(A) means
(B) virtue
(C) way of
(D) force

Questions (05-08) Select the word that is most closely similar in meaning to the original word.
5. FRAGILE
(A) Trouble
(B) Discomfort
(C) Breakable
(D) Divert
6. INSURGENT
(A) Loyal
(B) Rebellious
(C) Sincere
(D) Confident
7. ABERRATION
(A) Deviation
(B) Deviate
(C) Drive
(D) Direction
8. ALLEGIANCE
(A) Sincere
(B) Obedience
(C) Faithful
(D) Loyalty

Questions (09-12) : Find out that word, which spelling is WRONG


9.
(A) Athletic
(B) Nephew
(C) Manual
(D) Amature
10.
(A) Irritate
(B) Ignorent
(C) Turmoil
(D) Terrible
11.
(A) Tution
(B) Passion
(C) Fashion
(D) Ration
12.
(A) Lenient
(B) Nationalism
(C) Transfered
(D) Overhaul

Questions (13-16) : Fill in the blank with right option


13. The papers are to be prepared ____ a month.
(A) within
(B) by
(C) on
(D) in
14. Nature has given us an abundant wealth ____ the form of forests.
(A) by
(B) in
(C) at
(D) with
15. I shall ring you up as soon as______
(A) I will arrive
(B) I shall arrive
(C) I arrive
(D) I shall be arriving
16. I just had to _____ the orders of my employer.
(A) carry on
(B) carry through
(C) carry away
(D) carry out

BANGLA
Question (17-32) : Choose the correct answer
17. ?
(A)
(B)
(C)
(D)
?
(A)
(B)
(C)
(D)
19. ?
(A)
(B)
(C)
(D)
20. ?
(A)
(B)
(C)
(D)
21. ?
(A)
(B)
(C)
(D)
22. ?
(A)
(B)
(C)
(D)
23. - ?
(A)
(B)
(C)
(D)
24. ?
(A)
(B)
(C)
(D)
25. ?
(A)
(B)
(C)
(D)
26. ?
(A)
(B)
(C)
(D)
27.
(A)
(B)
(C)
(D)
28. ?
(A)
(B)
(C)
(D)
29. ?
(A)
(B)
(C)
(D)
30. ?
(A)
(B)
(C)
(D)
31. ?
(A)
(B)
(C)
(D)
32. ?
(A)
(B)
(C)
(D)

MATHEMATICS
Questions (33-56): Read the following questions carefully and choose the right answer.
33. If m and n are whole numbers such that m n =121, the value of (m-1)n is-
(A) 1000
(B) 121
(C) 10
(D) 1
Solution:
We know that 112 = 121.
Putting m = 11 and n = 2,
we get:
(m 1)n + 1 = (11 1)(2 + 1) = 103 = 1000.
34. A pump can fall a tank with water in 2 hours. Because of a leak, it took 2 1/3 hours to fill the tank. The leak can drain all the water
of the tank in-
(A) 4 1/3 hours
(B) 14 hours
(C) 7 hours
(D) 8 hours
Solution:
Work done by the leak in 1 hr = (1/2 377) = 1/14
So, Leak will empty the tank in 14 hrs.
35. The difference between simple and compound interest on a sum of money at 20% per annum for 3 years is Tk. 48. What is the
sum?
(A) Tk. 550
(B) Tk. 500
(C) Tk. 375
(D) Tk. 400
Solution:
Let,
The sum = x.
SI = x×n×r = x×3×20/100=3x/5
CI = [x(1+20/100)3 x] = 216x/125 x = 91x/125
ATQ,
91x/125 3x/5 = 48
Or, x = 375
36. If A and B are in the ratio 3:4, and B and C in the ratio 12:13, then A and
C will be in the ratio-
(A) 3:13
(B) 36:13
(C) 13:9
(D) 9:13
Solution:
(A/B) × (B/C) = (3/4) × (12/13)
Or, A/C = 36/52 = 9:13
37. A bag contains an equal number of one rupee, 50 paisa and 25 paisa coins. If the total value is Tk. 35 how many coins of each
type are there?
(A) 20
(B) 15
(C) 18
(D) 22
Solution:
Let X coins of each type of there.
Total Value = Rs. 35.
Now,
X + X/2 + X/4 = 35.
4X + 2X + X = 140.
7X = 140.
X = 20
38. If 40% of a number is equal to two-third of another number, what is the ratio of first number to the second number?
(A) 2:5
(B) 5:3
(C) 3:5
(D) 7:3
Solution:
Let,
40% of A=(2/3)B
Then,
40A/100=2B/3
Or, 2A/5=2B/3
Or, A/B=(2/3 × 5/2)=5/3
So, A:B=5:3.
39. A man starts climbing a 11 m high wall at 5pm. In each minute he climbs up 1 m but slips down 50 cm. At what time will be climb
the wall?
(A) 5:30 pm
(B) 5:25 pm
(C) 5:21 pm
(D) 5:27 pm
Solution:
Man climbs 1 m and slips down 50 cm (0.5 m) in one minute
i.e. he climbs (1-0.5 = 0.5 m) in one minute.
But in the last minute he will be climbing 1 m as he gets on the top so no slip.
Time taken to climb 11 meter = [(10/0.5) +1] = 21 minutes.
He climbs the wall at 5:21 pm.
40. Two trains of equal length, running to opposite direction, pass a pole in 15 and 12 seconds. The trains will cross each other in-
(A) 15.5 seconds
(B) 18.5 seconds
(C) 20.2 seconds
(D) 14.4 seconds
Solution:
Let length of each train be x meter.
Then, speed of 1st train = x/18 m/sec;
Speed of 2nd train = x/12 m/sec;
Now,
When both trains cross each other, time taken;
=[2x/{(x18)+(x12)}]
=2x/(5x/36)
=(2x×36)/5x
=72/5
=14.4 seconds
41. A boat can travel with a speed of 13 km/hr in still water. If the speed of the stream is 4 km/hr, find the time taken by the boat to
go 68 km downstream.
(A) 4 hours
(B) 3 hours
(C) 5 hours
(D) 2 hours
Solution:
Speed downstream = (13 + 4) km/hr = 17 km/hr.
Time taken to travel 68 km downstream =68/17hrs = 4 hrs.
42. A bag contains 4 white 5 red and 6 blue balls. Three balls are drawn at random from the bag. The probability that all of them are
red is-
(A) 1/22
(B) 1/91
(C) 2/91
(D) 2/77
Solution:
Let S be the sample space.
15
Then, n(S) = number of ways of drawing 3 balls out of 15 = = C3 = 455
Let, E = event of getting all the 3 red balls.
n(E) = 5C3 = 10.
So, P(E) = n(E)/ n(S)= 10/455 = 2/91
43. The ratio between the perimeter and the breadth of a rectangle is: 5:1. the area of the rectangle is 216 cm 2, what is the length of
the rectangle?
(A) 16 cm
(B) 24 cm
(C) 18 cm
(D) none of these
Solution:
2(l + b)= 5/1
Or, 2l + 2b = 5b
Or, 3b = 2l
Or, b = 2l/3
Then, Area = 216 cm2
Or, l x b = 216
Or, l x 2l/3 = 216
Or, l2 = 324
Or, l = 18 cm.
44. The average age of A, B, C,D and E is 40 years. The average age of A and B is 35 years and the average of C and D is 42 years. Age
of E is
(A) 48
(B) 46
(C) 42
(D) 45
Solution:
A+B+C+D+E=40×5=200
A+B=35×2=70
C+D=42×2=84
Therefore,
E=(A+B+C+D+E)−(A+B+C+D)=200−70−84=46 years
45. The sum of the digits of two-digit numbers is 10 while when the digits are reversed, the number decrease by 54. Find the changed
number.
(A) 19
(B) 46
(C) 37
(D) 28
Solution:
Let number be (10x+y).
According to question,
(10x+y)-(10y+x) = 54;
10x-10y+y-x = 54;
Or, 9x-9y = 54;
Or, x-y = 6 (i)
Sum of digits,
(x+y) = 10; (ii)
(i)-(ii);
So, x-y-x-y = 6-10;
Or, -2y = -4;
Or, y = 2; and, x = 8.
Then, the required number is
(10y+x) = 10×2+8 = 28.
46. A shopkeeper earns a profit of 12% on selling a book at 10% discount on the printed price. The ratio of the cost price and the
printed price of the book is
(A) 45:56
(B) 45:51
(C) 47:36
(D) 47:51
Solution:
Let,
The CP be 100.
Hence,
SP=100+12% of 100=112
If the marked price be X,
then 90% of X=112
Or, x= (112×100)/90
Or, x = 1120/90
Hence,
Required ratio =100:1120/90=900:1120=45:56
47. The selling price of an article after giving two successive discounts of 10% and 5% on the marked price is TK 171. What is the
marked price?
(A) Tk. 240
(B) TK 220
(C) TK 200
(D) TL 250
Solution:
Equivalent Discount,
=(A + B) (AB/100)
= (10 + 5) (10*5/100)
= 14.5%.
Let MP = X.
Now,
X 14.5% of X = 171(Selling Price)
Or, 0.855X = 171
Or, X = 200.
Hence, MP = 200.
48. If A and B together can complete a work in 18 days, A and C together in 12 days, and B and C together in 9 days., then B alone
can do the work in
(A) 24 days
(B) 15 days
(C) 30 days
(D) 40 days
Solution:

(A+B) = 1/18 -(1)


(A+C) = 1/12 -(2)
(B+C) = 1/9 (3)
(1)+(2)+(3),
2(A+B+C) = (1/18 + 1/12 + 1/9) = ¼
Or, (A+B+C) = 1/8
B = (A+B+C) (A+C) = 1/8 1/12 = 1/24
B needs 24 days.
49. A tank can be filled with water by two pipes A and B together in 36 minutes. If the pipe B was stopped after 30 minutes, the tank
is filled in 40 minutes. The pipe B can alone fill the tank in-
(A) 45 minutes
(B) 60 minutes
(C) 75 minutes
(D) 90 minutes
Solution:
Let the pipes fill the tank in x minutes.
Part of tank filled by pipes A and B in one minute: 1/36;
Part of the tank filled by pipe A in 1 minute = 1/36 1/x;
ATQ,
30* 1/x + (1/36 1/x) = 1
Or, 30/x + 10/9 40/x = 1
Or, 10/x = 1/9
Hence, x = 90 minutes.
50. In a 729 litres mixture of milk and water, the ratio of milk to water is 7:2. To get a new mixture containing milk and water in the
ratio 7:3, the amount of water to be added is-
(A) 71 litres
(B) 81 litres
(C) 56 litres
(D) 50 litres
Solution:
Quantity of milk in 729 litre of mixture
= 7X729/9 = 567 litre
Quantity of water
= 729-567 = 162 litre.
Let x litre of water be added to become ratio 7:3.
Or, 7/3 = 567/(162+x)
Or, 162×7 +7x = 567×3
Or, 7x = 1701-1134 = 567
Or, x = 567/7 = 81 litre water is to be added.
51. In what ratio must a grocer mix two varieties of pulses costing Tk 15 and Tk 20 per kg respectively so as to get a mixture worth Tk
16.50 kg?
(A) 3:7
(B) 5:7
(C) 7:3
(D) 7:5
Solution:
By the rule of alligation:
Cost of 1 kg pulses of 1st kind Cost of 1 kg pulses of
nd
2 kind
Rs. 15 Rs. 20

Mean Price
Rs. 16.50

3.50 1.50
Required rate = 3.50 : 1.50 = 7 : 3.
52. Three partners shared the profit in a business in the ratio 5:7:8. They had partnered for 14 months,8 months and 7 months
respectively. What was the ratio of their investments?
(A) 5:7:8
(B) 20:49:67
(C) 38:28:21
(D) None of these
Solution:
Let their investments be x for 14 months, y for 8 months and z for 7 months respectively.
Then, 14x : 8y : 7z = 5 : 7 : 8.
Now, 14x/8y = 5/7
Or, 98x = 40y
Or, y = 49x/20
And, 14x/7z = 5/8
Or, 112x = 35z
Or, z = 112x/35 = 16x/5
So x : y : z = x : 49x/20 : 16x/5 - (Multiple by 20)
Or, x : y : z = 20 : 49 : 64
53. profit is-
(A) Tk 1425.00
(B) Tk. 1537.50
(C) TK. 1576.00
(D) TK. 1500.00
Soution:
Let the total profit be tk 100.
After paying 3/5) = tk. 57.

= 1500.
54. The sum of ages of 5 children born at the intervals of 3 years each, is 50 years. What is the age of the youngest child?
(A) 4 years
(B) 8 years
(C) 9 years
(D) None of these
Solution:
Let the ages of children be x, (x + 3), (x + 6), (x + 9) and (x + 12) years.
Then, x + (x + 3) + (x + 6) + (x + 9) + (x + 12) = 50
Or, 5x = 20
Or, x = 4.
Age of the youngest child = x = 4 years.
55. Six years ago, the ratio of the ages of Kunal and Sagar was 6: 5. Four years hence, the ratio of their ages will be 11:10. What is

(A) 18 years
(B) 20 years
(C) 16 years
(D) 22 years
Solution:
Let the ages of Kunal and Sagar 6 years ago be 6x and 5x years respectively.
Then, {(6x + 6) + 4}/{(5x + 6) + 4} =11/10
Or, 10(6x + 10) = 11(5x + 10)
Or, 5x = 10
Or, x = 2.

56. The price of 10 chairs is equal to that of 4 tables. The price of 15 chairs and 2 tables together is Tk. 4000. The total price of 12
chairs and 3 tables is:
(A) Tk. 3500
(B) Tk. 3750
(C) Tk. 3840
(D) None of These
Solution:
Let,
the cost of a chair and that of table be x and y respectively.
Then,
10x=4y or y=5x/2
So, 15x+2y=4000
Or, 15x+2×5x/2=4000
Or, 20x=4000
Or, x=200
So,y=(5/2)×200 = 500 Hence,
the cost of 12 chairs and 3 tables
=12x+3y=(2400+1500)=3900

COMPUTER
Questions (57-80) . Read the following questions carefully and choose the right answer.
57. The ____ contains commands associated with the My Computer window
(A) System menu
(B) Start menu
(C) Standard menu
(D) None of These
58. The modern keyboard typically has ____ Function keys.
(A) 10
(B) 12
(C) 14
(D) 16
59. Each excel file is a workbook that contains different sheets. Which of the following cannot be a sheet in workbook?
(A) work sheet
(B) chart sheet
(C) module sheet
(D) Data sheet
60. The ability to combine name and addresses with a standard document is called-
(A) formatting
(B) mail merge
(C) form letters
(D) All of These
61. Which of the following is word processing software?
(A) Wordperfect
(B) Wordpad
(C) MS Word
(D) All of These
62. Which of the following section does not exist in a slide layout?
(A) Animations
(B) Titles
(C) Lists
(D) Charts
63. In which menu can you find features like Slide Design, Slide Layout etc.?
(A) Insert Menu
(B) Tools Menu
(C) Format Menu
(D) Slide Show Menu
64. A light sensitive device that converts drawing, printed text or other images into digital form is
(A) Keyboard
(B) Plotter
(C) Scanner
(D) None of these
65. In order to tell Excel that we are entering a formula in cell, we must begin with an operator such as
(A) =
(B) `
(C) @
(D) #
66. Computer Virus is a _____.
(A) Hardware
(B) Bacteria
(C) Freeware
(D) Software
67. A Microsoft Windows is a(n)
(A) Graphic program
(B) Operating system
(C) Word Processing
(D) Database program
68. Which of the following is not an operating system?
(A) DOS
(B) Linux
(C) Oracle
(D) Windows

GENERAL KNOWLEDGE
69. Headquarters of UNO are situated at
(A) Geneva
(B) New York, USA.
(C) Paris
(D) None of these
70. Fathometer is used to measure
(A) Rainfall
(B) Sound intensity
(C) Ocean depth
(D) Earthquakes
71. Hygrometer is used to measure
(A) relative humidity
(B) purity of milk.
(C) specific gravity
(D) None of these
72. The first meeting of the UN General Assembly was held in which of the following city?
(A) New York
(B) Teheran
(C) San Francisco
(D) London
73. Wright brothers are regarded inventors of the
(A) Hot Air Balloon
(B) Aero plane
(C) Bicycle
(D) None of these
74. The energy of food is measured in
(A) Kelvin
(B) bushel
(C) calories
(D) None of these
75. The main object of which of the following UN agency is to help the underdeveloped countries in the task of raising their living
standards?
(A) IMF
(B) UNICEF
(C) UNDP
(D) IDA
76. BITAC is an institution for technical support/assistance to
(A) cottage industries
(B) industries
(C) RMG
(D) All of these
77. Which Football team stood first in the end-of-the-year FIFA global rankings 2018?
(A) Belgium
(B) France
(C) Switzerland
(D) Germany
78. The second major language (as per the number of speakers) in the world is:
(A) English
(B) Spanish
(C) Hindi
(D) Arabic
79. Epsom (England) is the place associated with
(A) Snooker
(C) Shooting
(B) Polo
(D) Horse racing
80. The ozone layer restricts
(A) Ultraviolet radiation
(B) Infrared radiation
(C) Visible light
(D) Gamma rays

ENGLISH
Questions (01-04): Read the following passage and answer the questions given below:
Raisins are dried grapes that have been eaten for thousands of years. Nearly 3,500 years ago, the first raisins were discovered as grapes
that were drying in the sun on a vine. In medieval Europe, raisins were used as sweeteners, medicine, and even as a form of money!
In America, raisins were first grown after an 1873 heat wave in California destroyed its valuable grape crop, leaving only dried, wrinkly,
but tasty grapes on the vines. Soon, farmers began developing seedless grapes in California that were thin-skinned and sweet. These
grapes would be purposely dried in the sun and became the popular dark raisin we eat and enjoy today. Later, a golden variety of raisin
was made by treating grapes with a chemical called sulfur dioxide and using special methods to dry them.

valleys, sunny climate, and hot temperatures provide the perfect conditions for grapes that are dried into raisins.
1. Which of the following were raisins not used as?
(A) medicine
(B) weapons
(C) sweetener
(D) money
2. Golden raisins were discovered.
(A) before dark raisins

(C) after dark raisins


(D) None of these
3. What would be the best title for the passage?
(A) Healthy snacks
(B) A history of raisins
(C) Discovery of raisins
(D) None of these
4. Which of the following has the same meaning as the word in the sentence below-
Later, a golden variety of raisin was made by treating grapes with a chemical called sulfur dioxide and using special methods to dry
them.
(A) Sale
(B) Fruit
(C) type of
(D) opposite of

Questions (05-08): Fill in the blank with right option


5. Children must be _____with love and care.
(A) brought up
(B) brought about
(C) brought for
(D) brought as
6. The family looked for a double bedroom ________for three days.
(A) soot
(B) suit
(C) suite
(D) sweet
7. It is natural in every man to wish _______distinction.
(A) on
(B) for
(C) of
(D) to
8. I am exactly like my mother. I think I have _______her.
(A) taken to
(B) taken off
(C) taken over
(D) taken after

Questions (09-12): Select the word or phrase that is most closely opposite in meaning to the original word
9. ACUTE
(A) Sharp
(B) Critical
(C) Dull
(D) Sensitive
10. ORNERY
(A) Straight-Forward
(B) Easy
(C) Complex
(D) Plain
11. VOLUNTARY
(A) Ordered
(B) Alternative
(C) Essential
(D) Compulsory
12. AFFLUENCE
(A) Poverty
(B) Continuance
(C) Diffidence
(D) Insurance

Questions (13-16): Find the correctly spelt word


13.
(A) Eflorescence
(B) Eflorescence
(C) Efflorescence
(D) Enorascence
14.
(A) Adventitous
(B) Adventitious
(C) Adventitus
(D) Adventituous
15.
(A) Aberrant
(В) Abbarant
(C) Aberant
(D) Abberant
16.
(A) Cappricious
(B) Caprisious
(C) Carisuous
(D) Capricious

BANGLA
17. ?
(A)
(B)
(C)
(D)
18. ?
(A)
(B)
(C)
(D)
19. ?
(A)
(B)
(C)
(D)
20. ?
(A)
(B)
(C)
(D)
21. ?
(A)
(B)
(C)
(D)
22. ?
(A)
(B )
(C)
(D)
23. -
(A)
(B)
(C)
(D)
24. - ?
(A)
(B)
(C)
(D)
25.
(A)
(B)
(C)
(D)
26. - ?
(A)
(B)
(C)
(D)
27. -
(A)
(B)
(C)
(D)
28. ?
(A)
(B)
(C)
(D)
29. ?
(A) +
(B) +
(C) +
(D) +
30. - ?
(A) √ +
(B) √ +
(C) +
(D) +
31. - ?
(A)
(B)
(C)
(D)
32. ?
(A)
(B)
(C)
(D)

MATHEMATICS
33. Which one of the following numbers can be removed from the set S = (02,4,5,9}without changing the average of set S?
(A) 0
(B) 2
(C) 4
(D) 5
Solution:
The average of the elements in the original set S is: (0+2+4+5+9) /5 =20 /5 =4.
If we remove an element that equals the average, then the average of the new set will remain unchanged. The new set after removing 4
is {0, 2, 5, 9}.
The average of the elements is,
(0+2+5+9) /4=16 /4 =4.
34. Find the least number of five digits which when divided by 40, 60, and 75, leave remainders 31,51 and 66 respectively.
(A) 10196
(B) 10199
(C) 197
(D) 10191
Solution:
Difference, 40-31 = 9;
60-51 = 9;
75-66 = 9;
Difference between numbers and remainder is same in each case.
Then,
The answer = {(LCM of 40, 60, 75)-9};
40 = 2×2×2×5;
60 = 2×2×3×5;
75 = 3×5×5;
LCM = 2×2×2×5×5×3 = 600;
But, the least number of 5 digits = 10000;
10000/600, we get remainder as 400.
Then, the answer = 10000+(600-400)-9; = 10191.
35. A trader sells his goods at a discount 20%. He still makes a profit of 25%. If he sells the goods at the marked price only, his profit
will be:
(A) 56,25%
(B) 25.36%
(C) 50,25%
(D) 54.25%
Solution:
Let the marked price = Rs. 100.
Then, SP = 100 - 20% of 100 = Rs. 80
Profit = 25%.
Let His CP = X.
SP = 80
X + 25% of X = 80
Hence, X = Rs. (100×80)/125 = Rs. 64.
CP = Rs. 64.
Profit after selling on marked price = 100-64 = Rs. 36
% gain = (36×100)/64 = 56.25%.
36. The cost price of 19 articles is same as the selling price of 29 articles. What is loss percentage?
(A) 52.30%
(B) 35.00%
(C) 34.48%
(D) 30.00%
Solution:
Let CP of each article be Rs. 29 and SP of each article be Rs. 19.
Loss Percentage
= [(29x−19x)×100] / 29x = 34.48%
37. While working 7 hour a day. Alone can complete a piece of work in 6 days and B alone in 8 days. In what time would they complete
it together 8 hour a day?
(A) 5 days
(B) 3 days
(C) 4 days
(D) 3.6 days
Solution:
A can complete the work in 7*6 = 42 hours;
1
B can complete the work in 7*8 = 56 hours;

(A+B) will take time to complete whole work,


working 7 hours a day = 42 * 56/98 = 24 hours;
Let X be the no. of days taken, working 8 hours a day to complete the work by A and B together.
Now, using work equivalence method ;
24 = 8 * X;
X = 24/8 = 3 days.
38. There is provision of food in fort for 1200 soldiers for 60 days. After 15 days 200 soldiers leave the fort. Remaining food will last for
how many days?
(A) 56 days
(B) 50 days
(C) 48 days
(D) 54 days
Solution:
Days left = 60- 15 = 45 days
Soldiers remaining = 1200 200 = 1000
Work equivalence method:
1200*45 = 1000*x;
Hence, x = 54 days.
39. 2 men or 3 women or 4 boys can do a piece of work in 52 days. Then the same piece of work will be done by I man, I woman and
I boy in -
(A) 48 days
(B) 36 days
(C) 45 days
(D) 50 days
Solution:
Work done by 2 men = 3 women = 4 boys
1 man = 2 boys.
1 woman = 4/3 boys
Thus,
Boys×days = 4×52 boys-days
Again,
1 man + 1 woman + 1 boys,
= 2 + (4/3) + 1
= 13/3 boys
Using work equivalent method,
boys×day = BOYS×DAYS
4×52 = (13/3) × x (let)
x = 48 days.
40. In a zoo, there are Rabbits and Pigeons. If heads are counted, there are 200, and if legs are counted there are 580. How many
pigeons are there?
(A) 90
(B) 100
(C) 110
(D) 120
Solution:
Let rabbits =x;
Pigeons =200-x;
now rabbit has 4 legs and pigeons has 2 legs
hence 4x+2*(200-x)=580;
x=90;
hence pigeons=200-90=110;;
41. P and Q invested in a business. The profit earned was divided in the ratio 2:3. If P invested tk. 40000, the amount invested by Q is-
(A) Tk. 40000
(B) Tk. 50000
(C) Tk. 70000
(D) Tk. 60000
Solution:
Let the amount invested by Q = q.
40000 : q = 2 : 3.
Or, 40000/q = 2/3.
Or, q = 40000 * (3/2) = Rs. 60000.
42. A, B, C subscribe Tk. 50,000 for a business. A subscribe Tk. 4000 more than B and B tk. 5000 more than C. Out of a total profit of
Tk. 35,000. A receives-
(A) Tk. 14,700
(B) Tk. 1,900
(C) Tk. 13,600
(D) Tk. 8,400
Solution:
Let C = x.
Then, B = x + 5000 and A = x + 5000 + 4000 = x + 9000.
So, x + x + 5000 + x + 9000 = 50000
Or, 3x = 36000
Or, x = 12000
A : B : C = 21000 : 17000 : 12000 = 21 : 17 : 12.
(21/50) = 14700
43. The angle of elevation of a ladder leaning against a wall is 60o and the foot of the ladder is 4.6 m away from the wall. The length of
the ladder is
(A) 2.3 m
(B) 9.2 m
(C) 4.6 m
(D) 7.8 m
Solution:
Cos60o= 4.6/length of ladder
Or, length of ladder = 4.6/ (1/2) = 4.6×2 = 9.2
44. n. After

(A) 12 years
(B) 14 years
(C) 18 years
(D) 20 years
Solution:
Let the present ages of son and father be x and (60 -x) years respectively.
Then, (60 - x) - 6 = 5(x - 6)
⇒ 54 - x = 5x - 30
⇒ 6x = 84
⇒ x = 14.

45. If the sum of two numbers is 22 and the sum of their squares is 404, then the product of two numbers is
(A) 40
(B) 44
(C) 80
(D) 88
Solution:
according to the given conditions x+y = 22
and x^2+y^2 = 404
now (x+y)^2 = x^2+y^2+2xy
so 22^2 = 404 + 2xy
so xy = 80/2 = 40
46. A tank is filled in 5 hours by three pipes A, B and C. The pipe C is twice as fast as B and B is twice as fast as A. How much time will
pipe A alone take to fill the tank?
(A) 20 hours
(B) 25 hours
(C) 35 hours
(D) None of these
Solution:
Let,
Pipe A alone takes x hours to fill the tank.
Then, pipes B and C will take x/2 and x/4 hours respectively to fill the tank.
So, 1/x+2/x+4/x=15
Or, 7/x=1/5
Or, x=35hrs
47. What is the rate of simple interest for the first 4 years if the sum of Tk. 360 becomes tk. 540 in 9 years and the rate of interest for
the last 5 years is 6%?
(A) 4%
(B) 5%
(C) 3%
(D) 6%
Solution:
Interest for the last 5 years
=PTR/100
=(360×5×6)/100=108 Tk
Interest for 9 year
=540−360
=180 Tk
So,interest for first four years=180−108=72 Tk;
Now, rate for first four years=(72×100)/(360×4)=5%
48. What is the amount of equal installment, if a sum of Tk. 1428 due 2 years hence has to be completely repaid in 2 equal annual
installments starting next year?
(A) Tk.700
(B) Tk.800
(C) Tk.650
(D) cannot be determined
Solution:
Asshort-cut of installment for 2 installments is given by,
P
Installment=[ ]
{(100/100+𝑅)+(100/100+𝑅)2}
There is the need of rate(R) which is unavailable in the question.
so,we cannot determine the answer
49. In a class, the number of girls is 20% more than that of the boys. The strength of the class is 66. If 4 more girls are admitted to the
class, the ratio of the number of boys to that of the girls is
(A) 1:2
(B) 1:4
(C) 3:5
(D) 3:4
Solution:
Girls:boys = 6:5;
Hence, girls = 6×(66/11) = 36;
Boys = 30;
New ratio, 30:(36 + 4) = 3:4.
50. If x : y = 5:2, then (8x +9y): (8x + 2y) is
(A) 29:22
(B) 22:29
(C) 26:61
(D) 61:26
Solution:
x/y = 5/2
Means x =5, y =2
Putting value of x and y in Expression.
8×5+9×2 = 58.
8×5+2×2 = 44.
58:44 =29:22.
51. Two number are in the ratio 3 : 5. If 9 is subtracted from each, the new numbers are in the ratio 12:23. The smaller number is
(A) 27
(B) 33
(C) 49
(D) 55
Solution:
Let
the number be 3x and 5x.
Then,(3x−9)/(5x−9)=12/23
oR, 23(3x−9)=12(5x−9)oR, 9x=99
oR, x=11.
sO,The smaller number =(3×11)=33
52. A monkey climbs a 60 m high pole. In first minute he climbs 6 m and slips down 3 m in the next minute. How much time is
required by it to reach the top?
(A) 35 minutes
(B) 33 minutes
(C) 37 minutes
(D) 40 minutes
Solution:
Monkey climbs 6 m in 1st minute and
slips down 3 m in next minute
i.e. Monkey climbs 3 meter in 2 minute
then he climbs in one minute,
= 3/2 m.
But in the last minute he climbs 6 m as he gets on the top so there is no slip.
Time required = 2 × (54/3) + (6/6) = 37 minutes.
53. Two trains 105 meters and 90 meters long, run at the speeds of 45 km/h and 72 km/h respectively, in opposite directions on
parallel tracks. The time which they take to cross each other, is:
(A) 8 seconds
(B) 7 seconds
(C) 5 seconds
(D) 6 seconds
Solution:
Length of the 1st train = 105 m;
Length of the 2nd train = 90 m.
Relative speed of the trains,
= 45+72 = 117 kmph
= 117×(5/18) = 32.5 m/sec;
Time taken to cross each other,
= (Length of 1st train + length of 2nd train) /relative speed of the trains.
Time taken = 195/32.5 = 6 secs.
54. The speed of a boat in still water in 15 km/hr and the rate of current is 3 km/hr. The distance travelled downstream in 12 minutes
is
(A) 1.2 km
(B) 3.6 km
(C) 1.8 km
(D) 2.4 km
Solution:
Speed downstream = (15 + 3) kmph = 18 kmph.
Distance travelled =18 × (12/60) km = 3.6 km.
55. A bag contains 2 red, 3 green and 2 blue balls. Two balls are drawn at random. What is the probability that none of the balls drawn
is blue?
(A) 5/7
(B) 2/7
(C) 10/21
(D) 11/21
Solution:
Total number of balls = (2 + 3 + 2) = 7.
Let, S be the sample space.
Then, n(S) = Number of ways of drawing 2 balls out of 7
= 7C2
= (7 x 6)/(2 x 1)
= 21.
Let,
E = Event of drawing 2 balls, none of which is blue.
n(E) = Number of ways of drawing 2 balls out of (2 + 3) balls.
= 5C2
=(5 x 4)/(2 x 1)
= 10.
P(E) =n(E)/ n(S)= 10/21
56. The length of a rectangular plot is 20 meters more than its breadth. If the cost of fencing the plot Tk.26.50 per meter is Tk. 5300,
what is the length of the plot in meters?
(A) 60
(B) 100
(C) 75
(D) 50
Solution:
Let length of plot = L meters, then breadth = L - 20 meters
and
perimeter = 2[L + L - 20] = [4L - 40] meters
ATQ,
[4L - 40] × 26.50 = 5300
Or, [4L - 40] = 5300 / 26.50 = 200
Or, 4L = 240
Or, L = 240/4= 60 meters.

GENERAL KNOWLEDGE
Study the following questions carefully the following questions fully and answer the questions given below:
57. The rocket launched from French carried how many satellites?
(A) 4
(B) 5
(C) 6
(D) 7
58. Professor Amartya Sen is associated with -
(A) Biochemistry
(B) Economics
(C) Electronics
(D) Geology
59. Each year World Red Cross and Red Crescent Day is celebrated on
(A) June 8
(B) June 18
(C) May 18
(D) May 8
60. For seeing objects at the surface of water from a submarine under water, the instrument
(A) Periscope
(B) Telescope
(C) Kaleidoscope
(D) Spectroscope
61. G-15 is an economic grouping of-
(A) 1st World Nations
(B) 2nd World Nations
(C) 3rd World Nations
(D) 4th World Nations
62. For galvanizing iron which of the following metal is used?
(A) Lead
(B) Zinc
(C) Aluminum
(D) Copper
63. Study of life in outer space is known as
(A) Neobiology
(B) Enterbiology
(C) Endbiology
(D) exobiology
64. The first man-made satellite, Sputnik I was launched by the former USSR in
(A) 1957
(B) 1955
(C) 1970
(D) 1967
65. The headquarter of the Organization of Petroleum Exporting Countries are
(A) Algiers
(B) Vienna
(C) Lagos
(D) Kuwait
66. The longest rail line of the world, Trans-Siberian line, is in
(A) China
(B) USA
(C) Russia
(D) India
67. The instrument used to measure electric current is
(A) electrometer
(B) galvanometer
(C) spectrometer
(D) ammeter
68. Brent index is associated with
(A) crude oil prices
(B) shipping rate index
(C) gold future prices
(D) All of these

COMPUTER
69. Graphical pictures that represent an object like file, folders etc. are:
(A) Task bar
(B) Icons
(C) Windows
(D) Desktop
70. The memory sizes in mainframe computers and advanced technology microcomputer are Expressed as
(A) Bytes
(B) Kilobytes
(C) Megabytes
(D) All of these
71. Which network protocol is used to send Email?
(A) SMTP
(B) FTP
(C) POP3
(D) None of these
72. The _____ program compresses large files into a smaller file
(A) WinShrink
(B) WinStyle
(C) Winzip
(D) All of these
73. Which components appear in the initial windows start up display?
(A) Dialog box
(B) Task bar
(C) Start menu
(D) All of these
74. Which of the following is an example of utility?
(A) Word
(B) Operating system
(C) Data recovery
(D) Antivirus
75. How can you update the values of formula cells if Auto Calculate mode of Excel is disabled?
(A) F8
(B) F10
(C) F9
(D) F 11
76. Which of the following Excel screen components can NOT be turned on or of
(A) Formula Bar
(B) Status Bar
(C) Tool Bar
(D) None of these
77. Which enables us to send the same letter to different persons?
(A) mail merge
(B) macros
(C) template
(D) None of these
78. Which of the following is not the Section Break Option?
(A) Odd Page
(B) Next Page
(C) Previous Page
(D) Even Page
79. What feature will you use to apply motion effects in between a slide exits and another enters?
(A) Animation Objects
(B) Slide Design
(C) Animation Scheme
(D) Slide Transition
80. Which of the following font effect is not available in Power Point Font dialog box?
(A) Strikethrough
(B) Underline
(C) Shadow
(D) Emboss

BANGLA
Question (01-15) : Choose the correct answer
?
(A)
(B)
(C)
(D)
2. ?
(A)
(B)
(C)
(D)
3. - ?
(A)
(B)
(C)
(D)
4. ?
(A)
(B)
(C)
(D)
?
(A)
(B)
(C)
(D)
?
(A)
(B)
(C)
(D)
?
(A) -
(B) -
(C) -
(D) -
?
(A) >
(B) >
(C) >
(D)
?
(A)
(B)
(C)
(D)
10. ?
(A)
(B)
(C)
(D)
11.
(A)
(B)
(C)
(D)
12. ?
(A) + =
(B) + =
(C) + =
(D) + =
13. ?
(A)
(B)
(C)
(D)
14. ?
(A) +
(B) +
(C) +
(D) +
15. ?
(A)
(B)
(C)
(D)
ENGLISH
Questions (16-20) : Fill in the blank with right option
16.
(A) under
(B) on
(C) in
(D) over
17. The company _____spouses of employees in the invitation to the banquet.
(A) are included
(B) have included
(C) is including
(D) has including
18. We cannot process the order ______ we get a copy of the purchase order.
(A) until
(B) that
(C) because
(D) when
19. The employees ______ about the closure before the announcement was made public.
(A) know
(B) knew
(C) known
(D) have known
20. ______ it was a holiday, the doctor performed the emergency surgery on the heart patient.
(A) During
(B) Even
(C) Although
(D) So

Questions(21-25): Select the pair or best expresses a relationship similar to that expressed in the pair.
21. MONEY: PECULATION
(A) Radiation : Bomb
(B) Bank: Cashier
(C) Comment: Insult
(D) Writing: Plagiarism
22. MEDICINE : ILLNESS
(A) hunger : thirst
(B) law: anarchy
(C) love: treason
(D) etiquette: discipline
23. EXTORT: OBTAIN
(A) plagiarize : borrow
(B) pilfer: steal
(C) explode: ignite
(D) consider: appeal
24. WAITRESS : RESTAURANT
(A) doctor: diagnosis
(B) actor role
(C) teacher : school
(D) driver: truck
25. EYES: TEARS
(A) Volcano : Lava
(B) Sea : Water
(C) Heart: Artery
(D) Hunger : Bred

Questions (26-30): Read the following passage and answer the questions given below:
Our M.S. Jensen EXPLORER is not an ordinary cruise ship. It was built for the purpose of exploration. Its shallow draft allows us to
explore remote islands not available to other cruise ships. Inflatable Zodiac rafts will land us on previously inaccessible beaches.
Even though the EXPLORER is relatively small, it still accommodates ninety-two passengers with all the comforts expected aboard a
modern cruising vessel. We have a Lido deck with a swimming pool, comfortable lounges, and spacious cabins. And although the air is
warm, soft, and inviting in these tranquil waters, the ship is fully air-conditioned. Each expedition carries an expert staff of naturalists,
anthropologists, and diving instructors.
26. What makes the Explorer an unusual cruise ship?
(A) Its safety equipment
(B) Its accommodations
(C) Its name
(D) Its structural design
27. What are the passengers most likely interested in?
(A) Competitive diving
(B) Astrology
(C) Nature study
(D) Sailing
28. What kind of weather is expected on the trip?
(A) Warm and sunny
(B) Thick tog
(C) Clear and chilly
(D) Frequent rainstorms
29. How will passengers get to unexplored beaches?
(A) By helicopter
(B) By raft
(C) By swimming
(D) By airplane
30. What feature of the cruise is NOT discussed?
(A) The employees
(B) The bedrooms
(C) The recreational
(D) The food

MATHEMATICS
Questions (31-60): Read the following questions carefully and choose the right answer.
31. Three houses are available in a locality. Three persons apply for the houses. Each applies for one house without consulting others.
The probability that all the three apply for the same house is
(A) 1/9
(B) 5/9
(C) 8/9
(D) 4/9
Solution:
One person can select one house out of 3= 3C1 ways =3.
Hence, three persons can select one house out of 3 in 3 × 3 × 3 =27
So, probability that all three apply for the same house is = 3/27 = 1/9
32. If the perimeter of a certain rectangle is 76 m and its area is 360 m 2, then what is the length of its shortest side?
(A) 13
(B) 15
(C) 18
(D) 10
Solution:
2(x+y)=76
Or, (x+y)= 38
Or, x = 38-y

From (2)
(38 y)y = 360
Or, 38y y2 = 360
2
Or, y 38y + 360 = 0
Or, (y 20)(y 18) = 0
So, y = 20 or y = 18
When y = 20, x = 18 and when y = 18, x = 20.
So, the shorter side is 18.
33. The simple interest received on a sum of money at the end of 10 years is two times of the principal. At the same rate of interest,
what would be the ratio of principal and compound interest received at the end of two years?
(A) 25: 11
(B) 20:11
(C) 20:9
(D) None of these
Solution:
Let,
Principal = x
So,
2x = x x 10 x r/100
Or, r = 20%
Again, Let,
Principal = 100
So,
C = 100(1 + 20/100)2
= 144
Interest = 144 100 = 44
Required ratio = 100:44 = 25:11
34. A student is to answer 10 out of 13 questions in an examination such that he must choose at least 4 from the first five questions.
The number of choices available to him is
(A) 140
(B) 196
(C) 280
(D) 346
Solution:
If the student answers 4 questions from the first five questions he can choose 6 questions from the remaining 8 questions. Therefore
number of combinations will be
=5C4×8C6=140
If the student answers 5 questions from the first five questions he can choose 5 questions from the remaining 8 questions; which will
be
=5C5×8C5=56
So, total number of choices= 140+56=196
35. For what value of x is 82x-4=16x?
(A) 2
(B) 3
(C) 4
(D) 6
Solution:
82x-4=16x
Or, 23(2x-4)=24x
Or, 6x-12 = 4x
Or, 2x = 12
Or, x = 6
36. Three dice are thrown together. Find the probability of getting a total of at least 6?
(A) 103/108
(B) 103/208
(C) 103/216
(D) 36/103
Solution
Total number of events=6 × 6 × 6=216
Let A be the event of getting a total of at least 6 and B denoted event of getting a total of less than 6 i.e.,3,4,5.
So,B={(1,1,1),(1,1,2),(1,2,1),(2,1,1),(1,1,3),(1,3,1),(3,1,1),(1,2,2),(2,1,2),(2,2,1)}
Favorable number of cases=10
So, P(B)=10/216
Or, P(A) = 1 P(B) = 1 (10/216) = 103/108
37. In a college Rahim scored 80 marks out of 150 in History and 95 marks out of 120 in English. If he wants to score 70% marks in 3
subjects, find the minimum marks he should score in Geography out of 100.
(A) 70
(B) 55
(C) 76
(D) None of these
Solution:
Total marks = 150+120+ 100 = 370
Marks obtained in 2 subjects = 80+95 = 175
Total marks to be obtained = 370 × 70% = 249
Marks needed in Geography = 249 175 = 74
38. If x=1+√2 and y = 1-√2, find the value of (x2+y2).
(A) 12
(B) 6
(C) 8
(D) 10
Solution:
x=1+√2
Or, x2 = (1+√2)2 = 1 + 2√2 + 2 = 3 + 2√2
y=1-√2
Or, y2 = (1-√2)2 = 1 2√2 + 2 = 3- 2√2
2 2
x +y = 3 + 2√2 + 3 2√2 = 6
39. X can do a piece of work in 20 days and Y can do the 1/7th of the same work in 5 days. In how days together they can complete
the 11/14th of the total work?
(A) 15 days
(B) 20 days
(C) 10 days
(D) 12 days
Solution:
Y can do in 1 day = 1/(7×5) = 1/35
X & Y together do in 1 day = 1/20 + 1/35 = 11/140
11/140 work is done in 1 day
So, 1 work is done in 140/11 day
So, 11/14 work is done in (11/14) × (140/11) = 10
40. The circumference of the circle and the perimeter of the square is equal and the ratio between the diameter of the circle and the
side of the square is 7:11. What is the area of the circle?
(A) 154 cm2
(B) 160 cm2
(C) 132 cm2

Solution:
Let, the side of the square = 11x and
diameter of the circle = 7x
ATQ,
2π(7x/2) = 4×11x
Or, 2 × 22/7 × 7x/2 = 44x

41. The average weight of 17 students is 90 kg. If the weight of teacher is also included, then the average weight is increased by 200
grams. Find the weight of the teacher?
(A) 93.6 kg
(B) 94 kg
(C) 93.4 kg
(D) 94.6 kg
Solution:
Total weight including teacher = 18×90.2 = 1623.6
Total weight of 17 students = 17 × 90 = 1530
So, weight of teacher = 1623.6 1530 = 93.6
42. A train can cross another train of equal length coming from the opposite direction with the speed of 108 km/h in 3 minutes. The
speed of the other train is 90 km/h. Find the length of the train.
(A) 5940 m
(B) 5490 m
(C) 4590 m
(D) 4950 m
Solution:
Relative speed = 108 + 90 = 198 km/h = 198 ×(5/18) m/s
Distance covered in 3 minutes = 198 ×(5/18) × 3×60 = 9900 m
So, length of a train = 9900/2 = 4950m
43. A can complete a piece of work in 12 days, A, B and C can complete the work in 6 days. Efficiency of B is 0.5 times the efficiency of
A. In how many days C can complete the work alone?
(A) 24 days
(B) 10 days
(C) 16 days
(D) 12 days
Solution:
A can complete 1/12 work a day.
So, B can complete (1/2)x(1/12) = 1/24 work a day.
So, A+B can complete (1/12 + 1/24) = 1/8 work a day
A+B+C can complete 1/6 work a day.
So, C can complete (1/6 1/8) = 1/24 work a day.
So, C needs 24 days to complete.
44. A shopkeeper sold an item at 20% profit and another item at 10% loss. If the cost price of both the items is same, find the overall
profit percent.
(A) 7.55%
(B) 6.00%
(C) 5.00%
(D) 6.50%
Solution:
Let, cost of each item = 100
At 20% profit, selling price = 120
At 10% loss, selling price = 90
Total selling price = 210
Total cost = 100+100 = 200
Profit = 210-200 = 10
Profit % = (10×100)/200 = 5%
45. 8 males and 6 females can do a piece of work in 5 days. 5 males and 6 females can complete the same work in 7 days. How many
days will a female take to do the job, if she works alone?
(A) 135 days
(B) 142 days
(C) 110 days
(D) 126 days
Solution:
8x + 6y = 1/5 -(1)
5x + 6y = 1/7 -(2)
(1)-(2),
3x = 2/35
Or, x = 2/105
From (2),
5 x 2/105 + 6y = 1/7
Or, y = 1/126
Days need = 126.
46. A is 30% more efficient than B. How much time will they, working together, take to complete a job which A alone could have done
in 23 days?
(A) 13 days
(B) 15 days
(C) 10 days
(D) None of these
Solution:
Ratio of times taken by A and B = 100 : 130 = 10 : 13.
Let, B takes x days to do the work.
Then, 10 : 13 :: 23 : x
Or, x = ( 23 × 13/10 )
Or, x = 299 /10

So, A and B together can complete the work in 13 days.


47. The area of lawn is 460 m2. If the length is 15 percent more than the breadth of the rectangular field. What is the length of the
field?
(A) 15 m
(B) 26 m
(C) 34.5 m
(D) None of these
Solution:
Let, Breadth = x
So, length = x + 15% of x = 23x/20
ATQ,
23x/20 × x = 460
Or, x2 = 400
Or, x = 20
So, length = 23×20/20 = 23
48. A boat can travel from point A to point B and return back to point A in 9 hours. Speed of the boat in still water is 8 km/h and the
speed of the stream is 4 kmh. Find the distance between A and B.
(A) 18 km
(B) 27 km
(C) 36 km
(D) 45 km
Solution:
ATQ,
d/(8+4) + d/(8-4) = 9
d= 27 km
49. A 10% monthly salary increase resulted in a tk. 9000 per year increase in salary for an employee. What was his monthly salary
before the increase?
(A) Tk.5000
(B) Tk.9000
(C) Tk.7500
(D) Tk. 12000
Solution:
Yearly increase = 9000
Monthly increase = 9000/12 = 750
10% of salary = 750
So, 100% of salary = (750×100)/10 = 7500
50. When a certain number is divided by 7, the remainder is 0, if the remainder is not 0 when the number is divided by 14, then the
remainder must be
(A) 7
(B) 5
(C) 3
(D) 8
Solution: option test
51. A mixture of 20 kg of sprit and water contains 10% water. How much water must be added to this mixture to raise the percentage
of water to 25%?
(A) 2
(B) 4
(C) 5
(D) 6
Solution:
In 1st mixture, water = 10/100 × 20 = 2
Spirit = 20-2 = 18
In 2nd mixture,
75 kg of spirit is contained in 100 kg mixture.
18 kg spirit is contained in 100/75 × 18 = 24 kg
So, water to be added = 24-20=4 kg
52. X and Y start a business by investing a certain amount in the ratio 9:16. Both of them invest for an equal period of time. At the end
of th

(A) 62 ½
(B) 60
(C) 55 2/3
(D) 43 3/4
Solution:
Required less %=
(16-9)/16 × 100
= 7/16 × 100
= 43 3/4
53. The average attendance of a college for the first three days of a week is 325 and first four days is 320. How many were present on
the fourth day?
(A) 300
(B) 315
(C) 305
(D) 350
Solution:
Total attendance for first 3 days = 325×3 = 975
Total attendance for first 4 days = 320×4 = 1280
Present on the 4th day = 1280-975 = 305
54. Tk. 355 has been divided among A, B, C in such a way that A had Tk. 20 more than B and C had Tk.15 more than A. How much was

(A) Tk. 135


(B) Tk. 100
(C) Tk. 120
(D) Tk. 150
Solution:
Let the share of A was x
A has 20 more than B.
So, Share of B was (x 20)
And C had 15 more than A So, Share of C was (x + 15)
Now,
Share of (A + B + C) = 355
Or, x + x 20 + x + 15 = 355
Or, 3x 5 = 355
Or, 3x = 360
Or, x = 120
Now, Share of C was x + 15 = 120 + 15 = 135
55. If 2 kg of metal, of which 1/3 is zinc and the rest is copper, be mixed with 3 kg of metal, of which ¼ is zinc and the rest is copper,
then what will be the ratio of zinc to copper in the mixture?
(A) 13:42
(B) 19:43
(C) 15:42
(D) 17:43
Solution:
In 2 kg of metal,
Zinc : copper = 2×1/3 : 2×2/3 = 2/3 : 4/3
In 3 kg of metal,
Zinc : copper = 3×1/4 : 3×3/4 = 3/4 : 9/4
Now, new ratio = (28/3 + ¾)/(4/3 + 9/4) = 17/43
56. By what percentage above the cost price, a fan should be sold if a shopkeeper wants to make a profit of Tk. 500 and the marked
price of the article is Tk 6000 which is 50% above the cost price?
(A) 25.0%
(B) 12.5%
(C) 20.0%
(D) None of these
Solution:
ATQ,
Cost price = 100/150 × 6000 = 4000
Selling price = 4000+ 500 = 4500
Required percentage above the cost price = 500/4000 × 100 = 12.5%
57. Out of two numbers, 4 times the smaller one is less than 3 times the larger one by 5, If the sum of the numbers is larger than 6
times their differences by 6, find the two numbers.
(A) 55 and 58
(B) 23 and 28
(C) 59 and 43
(D) 65 and 67
Solution:
Let the two numbers be x(smaller) and y(larger) .
ATQ,
4x = 3y 5
Or, 4x 3y = 5
Or, 3y
Again,
x + y = 6(y x) + 6
Or, x + y = 6y 6x + 6
Or, 7x
Multiply equation 1 by 5 and equation 2 by 3 we get :
15y 20x = 25
21x 15y = 18
Adding the two :
x = 43
From (1),
3y 4 × 43 = 5
Or, 3y 172 = 5
Or, 3y = 177
Or, y = 177/3
Or, y = 59
58. The sum of the squares of three numbers is 532 and the ratio of the first and the second as also of the second and the third is 3:2.
Then the first number is
(A) 18
(B) 6
(C) 12
(D) 20
Solution:
let they are x,y,z.
so x2+y2+z2=532.
Given,
x:y=3:2 and
y:z=3:2
then x:y:z=3×3:2×3:2×2=9:6:4
Now, numbers are 9a, 6a, 4a
So, (9a)2 + (6a)2 + (4a)2 = 532
Or, 81a2 + 36a2 + 16a2 = 532
Or, 133a2 = 533
Or, a2 = 4
Or, a = 2
So, the first number is = 9×2=18
59. Pipe P can fill an empty tank in 4 hours but pipe Q can completely empty the same tank in 8 hours. Both the pipes were opened
alternately after every two hours starting with pipe P then in how many hours, the tank was completely filled?
(A) 6 hours
(B) 5 hours
(C) 10 hours
(D) 12 hours
Solution:
Part filled by pipe P in 2 hour = 2/4= 1/2
Part empty by pipe Q in 2 hour = 2/8= 1/4

Pipe P and Q are opened alternatively after every 2 hours.


Part filled in every 4 hour = 1/2-1/4=1/4
Part filled in 8 hour = 2× 1/4= 1/2
remaining part =1−1/2= 1/2
Now it is pipe P's turn.
Time taken by pipe P to fill the remaining 1/2 part = 2 hour
Total time taken = 8 hour + 2 hour = 10 hour
60. The average age of A, B and C

(A) 130 years


(B) 60 years
(C) 75 years
(D) 100 years
Solution:
A+B+C = 30×3=90 (1)
ATQ,
B-A = C-B
Or, A+C = 2B
From (1),
B+2B = 90
Or, B = 30
D= B+40 = 30+40 = 70
So, B+D = 30+70 = 100

COMPUTER
61. ______ are introduced in Fourth Generation Computers:
(A) Microprocessors
(B) Vacuum Tubes
(C) Integrated Circuits
(D) None of these
62. Which one is called the brain of computer?
(A) RAM
(B) ALU
(C) CPU
(D) None of these
63. Which of the following is not an input device?
(A) Plotter
(B) Keyboard
(C) Scanner
(D) All of these
64. Which one is the largest space?
(A) Gigabyte
(B) Petabyte
(C) Terabyte
(D) None of these
65. Every computer connected to the Internet is identified by a unique four-part string, known as
(A) Host name
(B) Domain name
(C) IP address
(D) All of these
66. The rectangular area of the screen that displays a program, data and/or information is a
(A) Title bar
(B) Button
(C) Dialog box
(D) Window
67. What is a default file extension for all word documents?
(A) DOC
(B) FIL
(C) WRD
(D) TXT
68. Excel workbook is a collection of
(A) Chart
(B) Worksheet
(C) Word book
(D) None of these
69. In a spreadsheet program the _______contains related worksheets and documents.
(A) Column
(B) Formula
(C) Workbook
(D) All of these
70. =SUM(B1:B8) is an example of a
(A) Function
(B) Cell address
(C) Value
(D) Formula
GENERAL KNOWLEDGE
71. Which of the following companies has recently launched e-commerce platform for handloom weavers?
(A) Apple
(B) Google
(C) Microsoft
(D) Facebook
72. Which of the following cities has recently hosted the World Economic Forum (WEF) 2019 annual meet?
(A) Davos
(B) Paris
(C) London
(D) Madrid
73. Which one of the following has recently been named as 1st World Capital of Architecture for 2020?
(A) Tokyo
(B) Beijing
(C) England
(D) Rio de Janeiro
74. Which of the following cities has recently hosted Arab Economic and Social Development Summit?
(A) Dubai
(B) Beirut
(C) Muscat
(D) Doha
75. Which one of the following is an example of non-renewable resources?
(A) Wind
(B) Water
(C) Coal and minerals
(D) All of these
76. Where is Lake Superior, the largest fresh water lake in the world located?
(A) USA
(B) Brazil
(C) Canada
(D) Russia
77. The humidity of air measured in percentage is called
(A) Absolute humidity
(B) Relative humidity
(C) Specific humidity
(D) All of these
78. The part of Equatorial region which has well developed rubber plantations is
(A) Amazon basin
(B) Indonesia
(C) Zaire basin
(D) Malaysia
79. The study of population is known as
(A) Demography
(B) Chimatology
(C) Petrology
(D) Hydrology
80. The device used for measuring altitudes is
(A) Ammeter
(B) Altimeter
(C) Audiometer
(D) Audiometer

ENGLISH
Questions (01-03): Select the word or phrase that best completes the sentence
1. In comparison to the history of the world, human life is very transient.
(A) menninglessness
(B) short-lived
(C) poignant
(D) arbitrary
2. Writing a beautiful sonnet is as much as achievement as to finish a 400 pages novel.
(A) it is to finish
(B) if to finish
(C) to have finished
(D) finishing
3. Every individual is responsible for upholding the moral of society.
(A) keeping up
(B) developing
(C) maintaining
(D) judging

Questions (04-07) : Fill in the blank with right option


4. _____ pollution control measures are expensive, many industries hesitate to adopt them.
(A) Because
(B) Although
(C) However
(D) Despite
5. The _____ chosen for construction of the building is in the heart of the city.
(A) cite
(B) site
(C) slight
(D) sight
6.
(A) none

7. It is not _____ for a man to be confined to the pursuit of wealth.


(A) easy
(B) possible
(C) healthy
(D) common

Questions (08-10): Select the word that is most opposite in meaning to the original word
8. GAUDY
(A) Sumptuous
(B) Modest
(C) Ostentatious
(D) Blatant
9. HOSTILE
(A) negative
(B) positive
(C) inimical
(D) friendly
10. ILLUSTRIOUS
(A) Unknown
(B) Conspicuous
(C) Venerable
(D) Resplendent

BANGLA
Questions (11-20): Choose the correct answer
11. ?
(A) +
(B) +
(C) +
(D) +
12. ?
(A)
(B)
(C)
(D)
13. ?
(A)
(B)
(C)
(D)
14. ?
(A)
(B)
(C)
(D)
15. ?
(A) >
(B) >
(C) >
(D)
16. ?
(A)
(B)
(C)
(D)
17. ?
(A) + =
(B) + =
(C) + =
(D) + =
18. ?
(A)
(B)
(C)
(D)
19. ?
(A)
(B)
(C)
(D)
20. ?
(A)
(B)
(C)
(D)

MATHEMATICS
Questions (21-40): Read the following questions carefully and choose the correct answer.
21. If 3 men or 6 women can do a piece of work in 16 days, in how many days can 12 men and 8 women do the same piece of work?
(A) 4 days
(B) 5 days
(C) 3 days
(D) 2 days
Solution:
3m = 6w
So 12m + 8w = 24w + 8w = 32w
Since 6 women can complete work in 16 days, 32 women can complete it in :
16 × 6/32 = 3 days.
22. A milkman bought 15 kg of milk and mixed 3 kg of water in it. If the price per kg of the mixture becomes Tk. 22, what is cost price
of the milk per kg?
(A) Tk. 26.40
(B) TK, 28.00
(C) Tk. 22.60
(D) Tk. 24.00
Solution:
Let CP of milk be Rs. x per kg.
Therefore, 22:(x−22)=15:3
Or, 22/(x-22) = 15/3
Or, 5x = 132
Or, x = 26.40
23. If the rate of interest is 10% per annum and is compounded half yearly, the principal of Tk. 400 in 3/2 years will amount to
(A) Tk. 463.00
(B) Tk. 463.05
(C) Tk. 463.15
(D) Tk. 463.20
Solution:
C = P {1 + (r/n)}nt
= 400 {1 + (10/100×2)}(2x3)/2
= 463.05
24. A dealer offers a discount of 10% on the marked price of an article and still makes a profit of 20%. If its marked price is Tk. 800,
then the cost price of the article is
(A) Tk. 900
(B) Tk. 800
(C) Tk. 700
(D) Tk. 600
Solution:
M.P of an article = Rs. 800
After Discount = 90% of 800 = 720
Profit = 20%
CP = (720×100)/120 = 600
25. Pipe A alone can fill a tank in 8 hours. Pipe B can fill it in 6 hours. If both the pipes are opened and after 2 hours pipe A is closed,
then the other pipe will fill the tank in
(A) 6 hours
(B) 3 ½ hours
(C) 2 ½ hours
(D) 4 hours
Solution:
Part of cistern filled by A in 1 hr = 1/8
Part of cistern filled by B in 1 hr = 1/6
Therefore part of cistern filled by A and B in 1 hr = 1/8+1/6 = 7/24
In 2 hrs = 14/24 = 7/12
No of hrs taken by B to fill remaining part of cistern = remaining part of cistern/part of cistern filled per hr by B
= 5/12 ÷1/6
= 5/2
Time reqd by B to fill remaining part = 5/2 hrs = 2 ½ hours
26. One tap can fill a drum three times as fast as another tap. Both the taps together can fill the drum in 36 minutes. How much time
would the slower tap take to fill the drum?
(A) 144 minutes
(B) 108 minutes
(C) 124 minutes
(D) 160 minutes
Solution:
If the faster tap can fill the drum in M minutes, the slower tap fills it in 3M minutes
Both the pipes can fill (1/M + 1/3M) of the drum in 1 minute
1
/M + 1/3M = 1/36 or 4/3M = 1/36 or 3M = 144 or M = 48
Faster tap can fill the drum in 48 minutes.
Slower tap can fill the drum in 3 x 48 = 144 minutes.
27. 8 taps through which water flows at the same rate can fill a tank in 30 minutes. If two taps go out of order, how long will the
remaining taps take to the fill the tank?
(A) 35 minutes
(B) 40 minutes
(C) 37 minutes
(D) 38 minutes
Solution:
Given Time need by 8 taps to fill tank = 30 minutes

M = (8 x 30)/6 = 40 minutes
28. . What

(A) 8
(B) 10
(C) 12
(D) 16
Solution:

1 / 3x

and given one day work of both A and B = 1/12


4/3x = 1/12
Or, x = 16
29. In a business partnership among A, B, C and D, the profit is shared by A and B, B and C, C and D in 1: 3 ratio respectively. If the
total profit is Tk. 4,00,000 the share of C is:
(A) Tk. 112500
(B) Tk. 137500
(C) Tk. 90000
(D) Tk. 270000
Solution:
A/B = X/3X
B/C = 3X/9X
C/D = 9X/27X
So Share of C = 9/40 *400000 = 90000 Tk.
30. A tradesman marks his goods 10% above his cost price. If he allows his customers 10% discount on the marked price, how much
profit or loss does he make, if any?
(A) 1% loss
(B) 1% gain
(C) 5% gain
(D) No gain, no loss.
Solution:
Let C.P of goods = Tk. 100
M.P of goods = 110% of 100 = 110/100×100 = Tk. 110
After Discount S.P. of goods = 90% of 110
= 90/100 × 110= Tk. 99
Loss = 100 - 99 = Tk. 1
Loss % = 1/100 × 100=1
31. 2 men and I woman can complete a piece of work in 14 days, while 4 women and 2 men can do the same work in 8 days. If a man
gets Tk. 90 per day, what should be the wages per day of a woman?
(A) TK. 48
(B) Tk. 60
(C) Tk. 72
(D) Tk. 135
Solution:
ATQ,
(2M+1W)×14=(4W+2M)×8
Thus, M/W = 3/2
Therefore, if a man earns Tk. 90 a day, a woman earns Tk. 60 a day.
32. A, B and C subscribe together Tk. 50,000 for a business. A subscribes Tk. 4,000 more than B and B subscribes Tk. 5,000 more than
C. Out of a total profit of Tk. 35,000. A receives
(A) Tk. 8500
(B) Tk. 11998
(C) Tk. 12600
(D) Tk. 14700
Solution:
Let C = x.
Then, B = x + 5000 and A = x + 5000 + 4000 = x + 9000
So, x + x + 5000 + x + 9000 = 50000
⇒ 3x = 36000
⇒ x = 12000
A : B : C = 21000 : 17000 : 12000 = 21 : 17 : 12
So A's Share = Tk. 35000 × 21/50 = Tk. 14700
33. If the average of 10% of a number, 25% of that number, 50% of that number and 75% of that is 24. then the number will be
(A) 50
(B) 70
(C) 60
(D) 80
Solution:
Let the number be N.
so (N*10% + N*25% + N*50% + N*75%)/4 = 24
(0.1 * N + 0.25 * N + 0.5 * N + 0.75 * N)/4 = 24
(1.6*N)/4 = 24
1.6*N = 96
N = 60
34. A trader sells two cycles at Tk. 1,188 each and gains 10% on the first and loses 10% on the second. What is the profit or loss
percent on the whole?
(A) 1% loss
(B) 1% gain
(C) 2% loss
(D) No loss no gain
Solution:
1st case
if SP is Tk. 1188 , then CP = Tk. ( 100/110× 1188) = Tk. 1080
2nd case
if SP is Tk. 1188, then CP = Tk. (100/90×1188) = Tk. 1320
total CP = Tk. (1080+1320)= Tk. 2400
total SP = Tk. (1188+ 1188)= Tk. 2376
loss = Tk. (2400-2376) = Tk. 24
loss% = (24/2400×100) %
= 1%
35. A and B total
profit is
(A) Tk. 7500
(B) Tk. 15000
(C) Tk. 12000
(D) Tk. 10000
Solution:
Let the total profit be Tk.100
After paying to charity, A's share
=Tk. (90 × 5/9) = Tk. 50
If A's share Tk. 7500, Total profit=(100/50 × 7500) = Tk. 15000
36. 1/48 of a work is completed in half a day by 5 persons. Then, 1/40 of the work can be completed by 6 persons in how many days?
(A) 1
(B) 2
(C) 3
(D) 1/2
Solution:
1/48 of work is completed by 5 people in 1/2 day.
Therefore 2/48 of work is completed by 5 people in 1 day.
The work done by 1 person = 2/48 divided by 5.
=1/120.
Therefore work done by 6 people= 1/120 × 6=1/20.
1/20 is the work done by 6 people in 1 day.
We know that 1/40 is half of 1/20.
So, 6 people will take 1/2 day to complete 1/40 of the work.
37. A and B are partners in a business. A contributes 1/4 of the capital for 15 months and B receives 2/3 of the profit. Find for how
long B
(A) 6 months
(B) 8 months
(C) 10 months
(D) 12 months.
Solution:
B received 2/3 of the profit
A:B=1:2
Let the total capital = x
Then A's capital = x/4
B's capital = x x/4 = 3x/4
Assume B's money was used for b months
Then A:B = (x/4)*15 : (3x/4)*b = 1 : 2
15/4 : 3b/4 = 1 : 2
15 : 3b = 1 : 2
5:b=1:2
5/b = 1/ 2
b = 5*2 = 10
38. The ratio between the ages of A and B is 2:5. After 8 years, their ages will be in the ratio 1:2. What is the difference between their
present ages?
(A) 24 years
(B) 25 years
(C) 22 years
(D) 20 years
Solution:
Let the present age of A be 2x and B be 5x.
After 8 year, age will be (2x + 8) & (5x + 8).
The ratio of thei age after 8 year = 1/2
Now, (2x + 8) / (5x + 8) = 1 / 2
Cross multiplying both the sides, we get
4x + 16 = 5x + 8
x=8
Now, current age of A = 2*x = 2*8 = 16
Current age of B = 5*x = 5*8 = 40
Difference = 40 - 16 = 24 years.
39. A person invested part of Tk. 45000 at 4% and the rest at 6%. If his annual incomes from both are equal, then what is the average
cost of interest?
(A) 4.6%
(B) 4.8%
(C) 5.0%
(D) 5.2%
Solution:
Let the sum invested at 4% be P
⇒ 4 % of P = 6% of (45,000 P)
⇒ 4P = (6 × 45,000) 6P
⇒ 10P = 6 × 45,000 or P = 27,000
Therefore tk. 27,000 is invested at 4 % and tk. 18,000 is invested at 6 %.
Total interest = 4 % of tk. 27,000 + 6 % of tk. 18,000
tk. 1080 + tk. 1080 = tk. 2160
Average rate of interest = (2160 x 100)/45000 = 216/45 = 4.8 %
40. A person sold an article for Tk. 136 and made a loss of 15%. Had he sold it for Tk. x, he would have made a profit of 15%. Which
one of the following is correct?
(A) 190<x<200
(B) 170<x< 180
(C) 160<x<170
(D) 180<x< 190
Solution:
Cost price of the article = 136(1 + 0.15) = tk.160
To make a profit of 15% , selling price = 160(1 + 0.15) = tk.184

SUBJECTIVE QUESTIONS
Questions (41-80) : Read the following questions carefully and choose the correct answer.
41. Which of the following provides a programmer with the facility of using object of a class inside other classes?
(A) Inheritance
(B) Abstraction
(C) Composition
(D) Encapsulation
42. Type of software which is designed for users to customize programs is
(A) freeware
(B) shareware
(C) macros
(D) none of these
43. A major challenge in mixing SQL with a general-purpose language is mismatching in the
(A) Definition of data
(B) Manipulation of data
(C) Execution of data
(D) Output of data
44. In user facilities, copying of all records onto a main store from permanent store is considered as
(A) delete file
(B) rename file
(C) save file
(D) load file
45. To divide a class C network into a maximum of 14 subnets - each capable of having up to 14 hosts, the subnet mask used should
be:
(A) 255.255.255.0
(B) 255.255.255.192
(C) 255.255.255.240
(D) 255.255.255.78
46. If master and transaction file have keys in same order then it takes
(A) less time
(B) more time
(C) many hours
(D) many days
47. What is not the advantage of IPV6 over IPV4?
(A) longer address
(B) Classification of traffic
(C) More real IP addresses
(D) Jumbo Payload
48. What is the precedence of arithmetic operators (from highest to lowest)?
(A) %, +,/,*, -
(B) +, -, %,*,/
(C) %, +,-,*,/
(D) %, *, /,+, -
49. Once connection is set up, program can send SQL commands to database by using
(A) SQLExcelConn
(B) SQLDirect
(C) SQLExcelDirect
(D) SQIConnect
50.
(A) transaction file
(B) direct file
(C) order file
(D) sequence file
51. Testing of software with actual data and in actual environment is known as?
(A) Regression testing
(B) Beta testing
(C) Alpha testing
(D) None of these
52. What are the values of the following expressions:
2**(3**2)
(2**3)**2
2**3**2
(A) 64,512, 64
(B) 64,64,64
(C) 512,512,512
(D) 512, 64, 512
53. Special set of characters that computer associates with specific user identification is classified as
(A) password
(B) identity code
(C) inquiry code
(D) dump code
54. What will be the output of the following PHP code?
<?php

?>
(A) Error
(B) Hello World
(C) Nothing
(D) None of these
55. What is the output of the code shown?

(A) Error
(B) I more spam
(C) No output

56. Number of bits in BCD code used in computing are-


(A) seven bits
(B) twelve bits
(C) eighteen bits
(D) six bits
No answer
57. To check to see of the Web server you are trying to reach is available or is down, which command line utility should you use?
(A) PING
(B) ICS
(C) Telnet
(D) None of these
58. Device which converts digital signals into sound and sound from telephone handset into digital signals is classified as
(A) digital coupler
(B) analogue coupler
(C) acoustic coupler
(D) terminal coupler
59. What type of network provides access to the regional service providers and typically span distances greater than 100 miles?
(A) LAN
(B) MAN
(C) WAn
(D) WLAN
60. ____ provides a connection-oriented reliable service for sending message.
(A) TCP
(B) IP
(C) UDP
(D) None of these
61. Which of the following type is not supported for mobile application viewing for Google docs?
(A) Spreadsheets
(B) Word processor files
(C) Database files
(D) All of these
62. The term push and pop is related to the
(A) array
(B) stacks
(C) lists
(D) All of these
63. Type of error which occurs when data is transferred from one device to another is classified as
(A) transmission error
(B) interval error
(C) software error
(D) discontinued error
64. A Non-Functional Software testing done to check if the user interface is easy to use and understand
(A) Security Testing
(B) Unit testing
(C) Block Box Testing
(D) Usability Testing
65. A device that allows one of several analog or digital input signals which are to be selected and transmits input that is selected into a
single medium is called
(A) signal changer
(B) multiplexer
(C) demultiplexer
(D) digital transmitter
66. Which of the following medium is used for broadband local networks?
(A) Coaxial
(B) Optic fiber
(C) CATV
(D) UTP
67. Interleaving of records to form one file containing all records is classified as
(A) merging
(B) finding
(C) file learning
(D) searching
68. Program background, program functions and computing requirements are part of
(A) decision box
(B) statement box
(C) operations detail
(D) none of these
69. Variable which uses same name in whole program and in its all routines thus best classified as
(A) middle variable
(B) default variable
(C) local variable
(D) global variable
70. The name of the testing which is done to make sure the existing features are not affected by new changes
(A) Recursive testing
(B) Regression testing
(C) Whitebox testing
(D) Unit testing
71. Set of numbers used to check all groups record within limits of data is classified as
(A) variable check
(B) decimal check
(C) type check
(D) range check
72. Process of converting data or information into for which is readily available for processing is called
(A) encoding
(B) decoding
(C) translating
(D) data organization
73. All decimal values and integers are included in set of
(A) whole numbers
(B) natural numbers
(C) real numbers
(D) integers
74. Which of the following is not a Software Development Life Cycle Phase?
(A) Test Closure
(B) Coding
(C) Testing
(D) None of these
75. Any hardware or software which is used to connect two devices by enabling them to communicate is classified as
(A) analogue modem
(B) digital modem
(C) analogue interface
(D) interface
76. Waterfall model phase in which system design is prepared and this system design helps in specifying system requirements and
define overall system architecture is
(A) planning
(B) modeling
(C) construction
(D) communication
77. Language compilers and interpreters does not translate statements correctly when they have
(A) transcription error
(B) base errors
(C) syntax error
(D) logical error
78. Which of the following web service can be controlled by iAWSManager cloud app from an iPhone?
(A) EC2
(B) ELB
(C) SQS
(D) All of these
79. Issuance of cash through terminal outside bank is an example of
(A) Terminals
(B) interfaces
(C) hardware devices
(D) telecommunication
80. Five components that make up a data communications system are message, sender, receiver, medium and
(A) Code
(B) connecting device
(C) protocol
(D) All of these

ENGLISH
Questions (01-03) : Fill in the blank with right option
1. Lack of exercise and high fat diets have _____ to be factors in heart attack.
(A) long been known
(B) been long known
(C) known been long
(D) long known been
2. Spring is ________ it will be warmer soon.
(A) to be coming
(B) on the way
(C) coming
(D) prepared now
3. AIDS is not a dieses that can be ______ through the air or by insects.
(A) circulated
(B) transferred
(C) disseminated
(D) transmitted

Questions (04-06): In each of the following lists, one word is out of sequence from the other words. Find the word that is not similar to
the other words in each list.
4.
(A) Temporary
(B) Steady
(C) Short-lived
(D) Transitory
5.
(A) Objective
(B) Aim
(C) Goal
(D) Strategy
6.
(A) Inflation
(B) Investment
(C) Discourse
(D) Interests

Questions (07-10) : Select the word that is most closely similar in meaning to the original word.
7. UNBEARABLE
(A) Undefeated
(B) Tolerable
(C) Unpleasant
(D) Acceptable
8. ABERRATION
(A) Deviate
(B) Deviation
(C) Drive
(D) Direction
9. INTEMPERATE
(A) Sympathy
(B) Pathetic
(C) Talk In Self Control
(D) Anger
10. ALLEGIANCE
(A) Loyalty
(B) Sincere
(C) Obedience
(D) Faithful

BANGLA
Questions (11-20) : Choose the correct answer
11. ?
(A)
(B)
(C)
(D)
12. ?
(A)
(B)
(C)
(D)
13. ?
(A)
(B)
(C)
(D)
14. ?
(A) -
(B) -
(C)
(D) -
15. ?
(A)
(B)
(C)
(D)
16. ?
(A)
(B)
(C)
(D)
17. ?
(A)
(B)
(C)
(D)
18. ?
(A)
(B)
(C)
(D)
19. ?
(A)
(B)
(C)
(D)
20. ?
(A)
(B)
(C)
(D)

MATHEMATICS
Questions (21-40): Read the following questions carefully:
21. There are 15 boys and 10 girls in a class. If three students are selected at random, what is the probability that 1 girl and 2 boys are
selected?
(A) 1/40
(B) 1/2
(C) 21/46
(D) 7/41
Solution:
n(S) = Number ways of selecting 3 students out of 25 = 25C3 = 2300.
n(E)= (10C1 x 15C2) = 1050
P(E) = n(E)/n(s) = 1050/2300 = 21/46
22. A man takes 3 hours 45 minutes to row a boat 22.5 km downstream of a river and 2 hours 30 minutes to cover a distance of 10 km
up stream. Find the speed of the river current in km/hr.
(A) 1 km/hr
(B) 2 km/hr
(C) 3 km/hr
(D) 4 km/hr
Solution:
3 hours 45 minutes = 3.75 hr
2 hours 30 minutes = 2.5 hr
Speed upstream = 22.5 km / 3.75 h = 6 km/h
Speed downstream = 10 km / 2.5 h = 4 km/h
the speed of the river current = (6-4)/2 = 1 km/h
23. A square is inscribed in a circle of diameter 2a and another square is circumscribing circle. The difference between the areas of
outer and inner squares is
(A) a2
(B) 2a2
(C) 3a2
(D) 4a2
Solution:

Area or outer square = 2a x 2a = 4a2


In△BAD
(BD)2=(AD)2+(AB)2
(2a)2=x2+x2
4a2=2x2
2a2=x2
x=2 √a.
Area of inner square =√2a x √2a=2a2
Difference between the areas of the outer and inner square =4a2−2a2=2a2
24. The profit earned after selling an article for Tk.3,362 is the same as the loss incurred after selling the article for Tk 2,346. At what
selling price will a trader make a 20% profit on this article?
(A) 4639.4
(B) 4769.6
(C) 4830.8
(D) None of these
Solution:
Cost price = (3362+2346)/2 = 2854
Selling price = 2854 × 120/100 = 3424.8
25. A train 150 m long passes a pole in 15 seconds and crosses another train of the same length travelling in opposite direction in 8
seconds. The speed of the second train in (km/h) is
(A) 60 km/hr
(B) 66 km/hr
(C) 99 km/hr
(D) 72 km/hr
Solution:
Speed of the first train = 150/15 = 10 m/s
Relative speed of two trains = (150+150)/8 = 37.5
Speed of the second train = (37.5 10)×18/5 = 99km/hr
26. A jar contains white, red and green marbles in the ratios 2:3:5. Six more green marbles are added to the jars, and then the ratio
becomes 2:3:7. How many white marbles are there in the jar?
(A) 6
(B) 7
(C) 8
(D) 9
Solution:
Ratio of W : R : G= 2: 3:5
If 6 Green is added, Ratio becomes W: R:G = 2: 3:7
Difference of ratio for 6 marbles = 7 5=2
For 1 ratio = 3 marbles
White marbles = 2×3 = 6
27. An iron rod that weighs 24 kg is cut into two pieces so that one of these pieces weighs 16 kg and is 34 m long. If the weight of each
piece is proportional to its length, how long is the other piece?
(A) 11 m
(B) 17 m
(C) 34 m
(D) 68 m
Solution:
16 kg rod = 34m
24 kg rod = (34×24)/16 = 51m
Length of the other part is = 51-34 = 17m
28. A train starts from city Y at 2 PM and travels towards city X at 75 km/hr. Another train starts from city X at 1 PM and travels
towards Y at 60 km/hr. If the distance between these two cities is 330 km then at what time will they meet?
(A) 6:00 PM
(B) 5:30 PM
(C) 5:00 PM
(D) 4:00 PM
Solution:
Let,
They meet x hrs after 1PM.
So,
(Distance covered by first in x hrs) + [Distance covered by second in (x - 1) hrs] = 330
Or, 60x + 75(x-1) = 330
Or, 135x = 405
Or, x = 3
So, they will meet at (1+3)PM = 4 PM
29. 100 kg of solution A is mixed with 60 kg of solution B. If solution A has tín and copper in the ratio 1:4 and solution B has lead and
tin in the ratio 3:2, then what is the amount of tin in the new solution?
(A) 70 kg
(B) 36 kg
(C) 44 kg
(D) 56 kg
Solution:
In 100 kg of solution A,
Tin = 100×1/5 = 20
Copper = 100-20 = 80
In 60 kg of solution B,
Lead = 60 × 3/5 = 36
Tin = 60 36 = 24
In 160 kg of new solution,
Tin = 20+24 = 44
30. On reducing the entry fee by 35% in a park, the number of people coming to the park increased by 40%, then the percent increase
or decrease in the income from the entry fee is-
(A) 9% decrease
(B) 7% decrease
(C) 9% increase
(D) 5% decrease
Solution:
Let
Total income = a × b = ab
Now, reducing the entry fee by 35% in a park, the number of people coming to the park increased by 40%
∴ New entry fee = a - 35% of a = 0.65a
Number of people = b + 40% of b = 1.4 b
Total income = 0.65a × 1.4b = 0.91 ab
Decrease in income = ab 0.91ab = 0.09ab
% decrease in income = 0.09ab×100%/ab = 9%
31. One third of the faculty members of a department are female. Sixteen of the male teachers are unmarried, while 60% of them are
married. The total number of faculty members in the department is
(A) 72
(B) 60
(C) 30
(D) 90
Solution:
Let,
Total number = x
Number of female = 1/3 of x = x/3
Number of male = x-x/3 = 2x/3
Married male = 60% of 2x/3 = 2x/5
ATQ,
2x/3 2x/5 = 16
Or, 4x/15 = 16
Or, x = 60
32. Three numbers are added in pairs, the sums so obtained are 20, 27, and 23. What are those three numbers?
(A) 6,4, and 15
(B) 9, 11, and 14
(C) 10,8, and 17
(D) 8, 12, and 15
Solution:
Let, the number = a, b,c
So, a+b = 20
B+c = 27
C+a = 23
33. A hall, 20 m long and 15 m broad, is surrounded by a verandah of uniform width of 2.5 m. The cost of flooring the verandah at Tk.
3.50 per square meter is-
(A) Tk. 600
(B) Tk. 800
(C) Tk. 700
(D) Tk 500
Solution:
Area of verandah = [(25 x 20) -(20 x 15)] m2 = 200 m2
Cost of flooring = Tk. (200 x 3.50) = Tk. 700
34. If the length of each of the sides of three square garden plots is increased by 50 percent, by what percent is the sum of the areas
of the three plots increased?
(A) 125%
(B) 150%
(C) 200%
(D) 275%
Solution:
Let, the length of each side of the garden = 100
Area = 1002 = 10000
Area of 3 square = 3×10000 = 30000
50% increase of length = 100+100×50/100 = 150
Area = 1502 = 22500
Area of 3 square = 3×22500 = 67500
Increasing area = 67500-30000 =37500
Percentage = (37500×100)/30000=125%
35. A driver of an auto rickshaw sees a lorry 60 meters ahead of him. After 30 seconds the lorry is 90 meters behind. If the speed of
the auto rickshaw is 38 km/h, then what is the speed of the lorry?
(A) 23 km/h
(B) 20 km/h
(C) 25 km/h
(D) 18 km/h
Solution:
Relative speed = Total distance/ Total time = (60 + 90)/30 = 5 m/s
5 m/s = 5 × 18/5 = 18 kmph
Now Relative speed = speed of auto rickshaw speed of lorry
or, 18 = 38 speed of lorry
Speed of lorry = 38 18 = 20 km/h
36. There are 2 shirts. 3 jeans, 3 socks and 2 skirts. In how many ways a shopkeeper can arrange these things so that all the socks
come together and all the skirts come together?
(A) 4690
(C) 2520
(B) 3260
(D) 5040
Solution:
We will count 3 socks as 1 socks and 2 skirts as 1 skirt.
Total ways = 7!
= 7 × 6 × 5 × 4 × 3 × 2 × 1 = 5040
37. There is 90% increase in an amount in 9 years at simple interest. What will be the compound interest of Tk. 15000 after 4 years at
the same rate?
(A) 4533
(B) 5497
(C) 6962
(D) 4965
Solution:
Interest Rate is 90/9=10%
So,
C = 15000[(1+10/100)⁴-1]
=15000[(110/100)⁴-1]
=15000[1.4641-1]
=0.4641×15000
=6961.50
= 6962
38. Two numbers are in ratio of 21:26. If 8 is added in each, the new members are in ratio of 5:6. Find the ratio of numbers, if 6 is
subtracted from each number?
(A) 18:23
(B) 19:25
(C) 6:7
(D) 9:16
Solution:
(21x +8)/(26x+8) = 5/6
Or, x = 2
Numbers are 42, 52
New ratio = (42-6)/(52-6) = 36/46 =18/23
39. The ratio of milk to water in three containers of equal capacity is 3:2, 7: 3 and 11:4 respectively. The three containers are mixed
together. What is the ratio of milk to water after mixing?
(A) 21:9
(B) 61 : 29
(C) 41:18
(D) 38.8
Solution:
Let the capacity of each container be x.
Now, in first container there are 3 parts of milk and 2 parts of water. Mathematically it can be written as:
3/5x milk and 2/5x water.
Similarly, second container has 7/10x milk and 3/10x water.
And, third container has 11/15x milk and 4/15x water.
Now, after mixing:
Total milk = (3/5+7/10+11/15)x = 61/30x
Total water = (2/5+3/10+4/15)x = 29/30x
So, milk to water ratio is (61/30x)/(29/30x)
So, milk to water ratio is 61/29.
40. If AB and CD are two diameters of a circle of radius r and they are mutually perpendicular, then what is the ratio of the area of the
circle to the area of the ΔACD?
(A) π/2
(B) π/4
(C) 2π
(D) π
Solution:
Required ratio = πr2/{1/2(2r×r)} = π

SUBJECTIVE QUESTIONS
Questions (41-80): Read the following questions carefully
41. Link list can be implemented by using?
(Α) Array
(B) Pointers
(C) Both A and B
(D) None of these
42. Which of the following is a design pattern?
(A) Factory
(B) List
(C) Queue
(D) All of these
43. Method used in writing and designing of a program is termed as
(A) bottom-up method
(B) top-down method
(C) split method
(D) None of these
44. Place where large amounts of data are stored outside central processing unit is called
(A) peripherals
(B) control unit
(C) AL unit
(D) backing store
45. Which of the following programming helps you to learn Android programming?
(A) C
(B) SQL
(C) Java
(D) Python
46. Which one is modern light weight message exchange format?
(A) JSON
(B) XML
(C) MX
(D) HTML
47. In C++, the idea to hiding the details of how something is implemented is known as-
(A) inheritance
(B) encapsulation
(C) recursion
(D) polymorphism
48. The only language understood by a digital computer is called-
(A) Assembly language
(B) High level language
(C) Binary language
(D) None of these
49. Which of the following is a project scheduling method that can be applied to software development?
(A) PERT
(B) CPM
(C) Both A and B
(D) CMM
50. between two adjacent houses in
- In which case is the above true?
(A) Satellite
(B) Fiber optic
(C) Twisted pair
(D) Radio link
51. Which kind of software testing strategy starts with testing the fundamental components first?
(A) Top-down testing
(B) Bottom-up testing
(C) Stress testing
(D) Back to back testing
52. A path for carrying signals between a source and a destination is known as
(A) Router
(B) Channel
(C) Link
(D) Block
53. Which of the standard protocol for network management features?
(A) SNMP
(B) SNA
(C) FTP
(D) SMTP
54. In SQL, the _______ command is used to recompile a view
(A) COMPLIE VIEW
(B) DEFINE VIEW
(C) ALTER VIEW
(D) CREATE VIEW
55. Which of the following is not a standard synchronous communication protocol?
(A) SDLC
(B) PAS
(C) SLIP
(D) SMTP
56. In Java, which operator is used to create an object
(A) class
(B) scanf
(C) print
(D) None of these
57. Which of the following terms refers to the degree to which data in a database system are accurate and correct?
(A) Data integrity
(B) Data security
(C) Data validity
(D) None of these
58. The complexity of Bubble sort algorithm is
(A) O(n)
(B) O(log n)
(C) O(n2)
(D) O(n log n)

Job Information For All Students (Bcs & Bank)


PKB Executive Officer Exam Question Solution 2019
Post Name and Vacancy:

1. Executive Officer -General (Officer Equivalent)-30

MCQ Exam Date: 18 October 2019

MCQ Exam time: 3.00 PM to 4.00 PM

বাংলা অংশ সমাধানঃ


1. কানিট বাংলা ব াকরণ এর শাখা নয়? উ রঃ ভাষাত
2. ীয়মান এর িবপরীত শ শ িক? উ রঃ বধমান
3. ঐিহক এর িবপরীতাথক শ কানিট? উ রঃ পািরি ক
4. কানিট বানান? উ রঃ ি হায়ণ
5. িল া র হয় না এমন শ কানিট? উ রঃ কিবরাজ
6. ঝাঁেকর ক বাগধারার অথ? উ রঃ একই দেলর লাক
7. রাজা রামেমাহন রায় রিচত বাংলা ব াকরেণর নাম িক? উ রঃ গৗড়ীয় ব কারণ
8. মৗিলক শ কানিট? উ রঃ গালাপ
9. কান শ িট ফারিস? উ রঃ পেরশান
10. জ ে র বতমােন কিনে র িবেয় ক একশে বেল? উ রঃ পিরেবদন
11. য সমােস পূ বপদ সংখ াবাচক এবং সম পদ ারা সমাহার বুঝায় তােক বেল?
উ রঃ ি সমাস
12. আফতাব শে র সমাথক কানিট? উ রঃ অক ( মােন সূ য)
13. বৃ ি এর সি িবে দ িক হেব? উ রঃ বৃ +িত
14. দালনা শে র সিঠক কৃিত- ত য় কানিট? উ রঃ দু +না
15. ু িধত পাষাণ কান সমাস? উ রঃ কমধারয়

Edited and Solved by Jobstestbd.com


PKB Executive Officer Exam Question Solution 2019
Post Name and Vacancy:

1. Executive Officer -General (Officer Equivalent)-30

MCQ Exam Date: 18 October 2019

MCQ Exam time: 3.00 PM to 4.00 PM

ইংেরিজ অংশ সমাধানঃ


Select the alternative/synonym world from below Italic word.

16. Will you kindly open the knot?


Ans: unite
17. No one could explain how a calm and balanced Person like him could
penetrate such a mindless act on his friends.
Ans: Perpetrate
18.The greatest thing in style is to you have a use of metaphor.
Ans: Knowledge
19.if you are living near a market place you should be ready to bear
the disturbances caused by traffic.
Ans: to bear with
20. During his long discourse, he did not touch the Point.
Ans: Touch on

Fill in the blank:

21. Creative people are often ------- With their own uniqueness.
Ans: obsessed
22. Rainfall in the desert is not only low butt extremely-----.
Ans: erratic
23. If you smuggle goods into the country, they may be -----by the customs
authority.
Ans: confiscated
24. It is not -----for a man to be confined to the pursuit of wealth.
Ans: healthy
25. -----pollution control measures are expensive; many industries hesitate to adopt
them.
Ans: Because

Edited and Solved by Jobstestbd.com


PKB Executive Officer Exam Question Solution 2019
Analogy:
26. DELAY: EXPEDITE= Detain: Dispatch
27. SUBMISSIVE: DISOBEDIENT= Observe: Defy
28. ANARCHY: GOVERNMENT= Penury: Wealth
29. EXCITE: CLAM= Stimulate: cool down
30. LENGTHEN: PROLONG=Stretch: Extend

English question is copied from https://www.examveda.com/ and


http://www.leadthecompetition.in/index.html website

Edited and Solved by Jobstestbd.com


PKB Executive Officer Exam Question Solution 2019
Post Name and Vacancy:
1. Executive Officer -General (Officer Equivalent)-30
MCQ Exam Date: 18 October 2019
MCQ Exam time: 3.00 PM to 4.00 PM

গিণত অংশ সমাধানঃ


31. A man can walk uphill at the rate of 2.5 km/hr and downhill at the rate of 3.25 km/hr. If the
total time required walking a certain distance up the hill and return to the starting position is 4 hr
36 min, what is the distance he walked up the hill?
(A) 3.5 km (B) 4.5 km (C) 5.5 km (D) 6.5 km
Ans: (D) 6.5 km
Explanation:
Let x km is the distance travelled by the man walking up the hill.
Then,
x2.5+x3.5=43660...[As 4 hours 36 min = 4 36/60 hours]
⇒x2.5+x3.5=27660
⇒24x+18.40x=276
⇒42.4x=276
⇒x=6.50km
32 A, B and C enter into a partnership with a certain capital in which A’s contribution is Tk.
10.000. If out of a total profit of Tk. 1,000. A gets Tk. 500, B gets Tk. 300 then C’s capital is-
(A) Tk. 4000 (B) Tk. 3600 (C) Tk. 4800 (D) Tk. 4400
Ans: (A) Tk. 4000
Explanation:
A : B : C = 500 : 300 : 200 = 5 : 3 : 2.
Let their capitals be 5x, 3x and 2x respectively.
Then, 5x = 10000
=> x = 2000.
C's capital = 2x = Tk. 4000.
33. The marked price of an article is Tk.500. It is sold on two successive discounts of 20% and
10%. The selling price of that article is-
(A) Tk.350 (B) Tk. 375 (C) TK.360 (D) Tk.400
Ans: (C) TK.360
Explanation:
Equilant discount of successive discount of 20% and 10%
%=28%
∴ Selling price = (100 – 28)% of TK. 500 = 72% of 500

Hence option [C] is the right answer.


34. IF (p/q)n-1 =(q/p)n-3, then the value of n is-
(A) 2 (B) 1/2 (C) 7/2 (D) 1
Ans: (A) 2

Edited and Solved by Jobstestbd.com


PKB Executive Officer Exam Question Solution 2019
35. A vessel is filled with liquid. 3 parts of which are water and 5 parts syrup. How much of the
mixture must be drawn off and replaced with water so that the mixture may be half water and
half syrup?
(A) 1/3 (B) ¼ (C) 1/5 (D) 1/7
Ans: (C) 1/5
Explanation:
Let total be 80 so water : syrup = 30:50
This becomes 40:40 after transfer
So 10 of syrup moves from syrup to water, but it is a combination of syrup and water that is
removed...
So if in 50, we remove 10,
therefore in total (50+30=80), we remove 10∗805010∗8050=16
So as a fraction, 16 out of 80 is 16/80=1/5
36. One-fourth of the boys and three-eight of the girls in a school participated in the annual
sports. What proportional part of the total student population of the school participated in the
annual sports?
(A) 4/12 (B) 5/8 (C) 8/12 (D) None of these
Ans: (D) None of these
Explanation:
1/4 means, let total no. Of boys are 4 and out them 1 takes participate and
3/8 means, let total of girls are 8 and out of them 3 take participate
Now, total students in the school =(4+8)=12
And total students takes participate in annual sports =(1+3)= 4
It means, total 4 students take participate in sport out of 12 .
i.e 4/12 = 1/3
37. If the rate of simple interest is 12% per annum the amount that would fetch interest of Tk.
6000 per annum is?
(A) Tk. 50000 (B) Tk. 7200 (C) Tk. 72000 (D) None of these
Ans: (A) Tk. 50000
Explanation:
Rate of Interest = 12% per annum
Simple Interest =TK. 6000 per annum
Let 'P' is the principal
SI= (P x R x T)/100
where , SI= Simple Interest
T= Time
R= Rate

∴ 6000=(P x 1 x 12)/100
∴ P =(6000 x 100)/12
=TK. 50000
Hence the required amount is TK. 50000
38. The compound interest on a certain sum of money for 2 years at 10% per annum in Tk. 420.
The simple interest on the same sum at the same rate and for the same time will be?
(A) Tk. 350 (B) Tk. 375 (C) Tk. 380 (D) Tk. 400
Ans: (D) Tk. 400

Edited and Solved by Jobstestbd.com


PKB Executive Officer Exam Question Solution 2019
Explanation:
Let the principle is P.
As is given, compound interest= P*(1 +10/100)2 - P = 420
P*(11/10)2 - P = 420
P*(121/100) - P = 420
P*(121-100)/100 = 420
P = 2000
So, simple interest.= P*R*T / 100 = 2000* 2* 10 / 100 = 400
39. Tickets numbered 1 to 20 are mixed up and then a ticket is down at random. What is the
probability that the ticket down has a number which is a multiple of 3 or 5?
(A) 1/2 (B) 9/20 (C) 3/5 (D) 8/15
Ans: (B) 9/20
Explanation:
Here, S = {1, 2, 3, 4, ...., 19, 20}.
Let E = event of getting a multiple of 3 or 5 = {3, 6 , 9, 12, 15, 18, 5, 10, 20}.
P(E) = n(E)/n(S) = 9/20.
40. A bag contains 2 red Roses, 4 yellow Roses and 6 pink Roses. Two roses are drawn at
random. What is the probability that they are not of same color?
(A) 1/6 (B)14/33 (C) 2/3 (D) 5/6
Ans: (C) 2/3
Explanation:
Possible outcomes = (RY, YP, PR)
2C1 4C1 + 4C1 6C1 + 6C1 2C1
Required probability = (2C1 4C1 + 4C1 6C1 + 6C1 2C1)/(12C2)
= 8 + 24 + 12/66
= 44/66
= 2/3.
41. The sum of the present ages of a father and his son is 60 years. Five years ago, father’s age
was four times the age of the son. So now the son’s age will be-
(A) 15 years (B) 5 years (C) 20 years (D) 10 years
Ans: (A) 15 years
Explanation:
Let the present ages of son and father be x and (60 -x) years respectively.

Then, (60 - x) - 5= 4(x - 5)

55 - x = 4x - 20

5x = 75 => x = 15
42. A hall is 15 m long and 12 m broad. If the sum of the areas of the floor and the ceiling is
equal to the sum of the areas of four walls, the volume of the hall is:
(A) 720 m3 (B) 1200 m3 (C) 900 m3 (D) 1800 m3
Ans: (B) 1200 m3
Explanation:

Edited and Solved by Jobstestbd.com


PKB Executive Officer Exam Question Solution 2019

43. A cylindrical rod of iron, whose height is equal to its radius, is melted and cast into spherical
balls whose radius is half the radius of the rod. Find the number of balls.
(A) 2 (B) 3 (C) 4 (D) 6
Ans: 6
Explanation:
There are 6 spherical balls can be made .
volume of cylindrical rod = π*r^2*h.
Height = Radius (Given).
volume of cylindrical rod = π * r^2 *r = π*r^3.
volume of one spherical ball = 4/3 * π *( r/2)^3 .
No of spheical ball made by melting cylinder =
(Total volume of rod )/ (volume of single ball)
= (4/3 * π * r^3) / (4/3 * π *( r/2)^3)
=6
44. A person’s present age is two-fifths of the age of his mother. After 8 years, he will be one-
half of the age of his mother. How old is the mother at present?
(A) 36 years (B) 38 years (C) 40 years (D) 42 years
Ans: (C) 40 years
Explanation:

45. The length of a rectangle is halved, while its breadth is tripled. What is the percentage change
in area?
(A) 50% increase (B) 25% increase (C) 50% decrease (D) 75% decrease

Edited and Solved by Jobstestbd.com


PKB Executive Officer Exam Question Solution 2019
Ans: (A) 50% increase
Explanation:
Let original length = x and original breadth = y.
Original area = xy.
New length = x/2 and New breadth=3y
New area = 32xy
Therefore, Increase in area = New area-original area = 32xy−xy=12xy
Therefore, Increase % = increase in area original area*100=(12xyxy/)*100=50%
46. Fresh fruit contains 68% water and dry fruit contains 20% water. How much dry fruit can be
obtained from 100 kg of fresh fruits?
(A) 20 kg. (B) 30 kg (C) 35 kg (D) 40 kg
Ans: (D) 40 kg
Explanation:
The fruit content in both the fresh fruit and dry fruit is the same.
Given, fresh fruit has 68% water.so remaining 32% is fruit content. weight of fresh fruits is
100kg
Dry fruit has 20% water.so remaining 80% is fruit content. Let weight if dry fruit be y kg.
Fruit % in fresh fruit = Fruit% in dry fruit
Therefore, (32/100) x 100 = (80/100 ) x y
we get, y = 40 kg.
47. A tap can fill a tank in 6 hours. After half the tank is filled, three more similar tanks are
opened. What is the total time taken to fill the tank completely?
(A) 3 hrs 15 min (B) 3 hrs 45 min (C) 4 hrs. 15 min (D) None of these
Ans: (B) 3 hrs 45
Explanation:
Time taken by one tap to fill half of the tank = 3 hrs.
1 2
Part filled by the four taps in 1 hour = 4 x = .
6 3
1 1
Remaining part = 1 - = .
2 2
21
: :: 1 : x
32
1 3 3
x = x 1 x = hours i.e., 45 mins.
2 2 4
So, total time taken = 3 hrs. 45 mins.
48. The salaries of A, B, and Care in the ratio of 1:2:3. The salary of B and C together is Tk.
6000. By what percent is the salary of more than that of A?
(A) 100% (B) 150% (C) 200% (D) 250%
Ans: (C) 200%
Explanation:
Let the salaries of A, B, C be x, 2x and 3x respectively.
Then,2x + 3x = 6000 => x = 1200.
A's salary = Tk. 1200, B's salary = Tk. 2400, and Cs salary Tk. 3600.
Excess of C's salary over A's=[ (2400 /1200) x 100] = 200%.
49. Two numbers are in the ratio 3:7. If 6 be added to each of them, then they are in the ratio 5:9.
Find the numbers?

Edited and Solved by Jobstestbd.com


PKB Executive Officer Exam Question Solution 2019
(A) 9 and 21 (B) 11 and 17 (C) 7 and 17 (D) 13 and 23
Ans: (A) 9 and 21
Explanation:
Let the two numbers be x and y
Given x : y = 3 : 7 .....(1)
Now, x+6 : y+6 = 5 : 9 .....(2)
From (1), x = 3y/7
From (2), 5y - 9x = 24
=> 5y - 9(3y/7) = 24
=> y = 21
=> From(1), x = 9
Hence, the two numbers be 9 and 21
50. 28root? +1426 = ¾ of 2872
(A) 576 (B) 1296 (C) 676 (D) 1444
Ans: (C) 676
51. A bag contains 50 P. 25 P and 10 P coins in the ratio 5: 9:4. amounting to Tk. 206. Find the
number of coins of each type respectively.
(A) 360, 160, 200 (B) 160, 360, 200 (C) 200.160.300 (D) 200,360,160
Ans: (D) 200,360,160
Explanation:
let ratio be x.
Hence no. of coins be 5x ,9x , 4x respectively
Now given total amount = Rs.206
=> (.50)(5x) + (.25)(9x) + (.10)(4x) = 206
we get x = 40
=> No. of 50p coins = 200
=> No. of 25p coins = 360
=> No. of 10p coins = 160
52. Painter A can paint a house in 16 days, and painter B can do the same work in 20 days. With
the help of painter C, they paint the house in 8 days only. Then, Painter C alone can do this task
in-
(A) 80 days (B) 85 days (C) 79 days (D) 76 days
Ans: (A) 80 days
Explanation-
A's 1 day's work =
1

16

B's 1 day's work =


1

20

(A + B + C)'s 1 day's work =

Edited and Solved by Jobstestbd.com


PKB Executive Officer Exam Question Solution 2019
1

10

=> C's 1 day's work =


1 1 1
- +
10 16 20
=
1

80

So, C can paint the house in 80 days.


53. A and B start at the same time with speeds of 40 km/hr and 50 km/hr respectively. If in
covering the journey A takes 15 minutes longer than B, the total distance of the journey is-
(A) 46 kın (B) 48 km (C) 50 km (D) 52 km
Ans: (C) 50 km
Explanation:
Suppose distance of journey = x km
Time taken by A to Cover x km = x/40 hours.
Time taken by B to Cover x km = x/50 hours.
By Hypothesis,
x
/40 - x/50 = 15/60 or x/200= 15/60
x = (15 x 20)/6
= 50 km
54. A person borrows Tk. 5000 for 2 years at 4% p.a. simple interest. He immediately lends it to
another person at 6-¼ % per annum for 2 years. Find his gain in the transaction per year.
(A) 112.50 (B) Tk. 167.50 (C) Tk. 150 (D) Tk.225
Ans: (A) 112.50
Explanation:
gain in 2 years =Tk.[(5000*254*2100)−(5000*4*2100)]
= Tk. (625 - 400)
= Tk. 225
gain in 1 year =Tk.(2252)=112.50

55. Two trains running in opposite directions cross a man standing on the platform in 27 seconds
and 17 Seconds respectively and they cross each other in 23 seconds. The ratio of their speeds is:
(A) 1:3 (B) 3:2 (C) 3:4 (D) None of these
Ans: (B) 3:2
Explanation:
Let the speeds of the two trains be x m/sec and y m/sec respectively.
Then, length of the first train = 27x metres,
and length of the second train = 17y metres.
27x + 17y= 23

Edited and Solved by Jobstestbd.com


PKB Executive Officer Exam Question Solution 2019
x+ y
27x + 17y = 23x + 23y
4x = 6y
x 3
= .
y 2

56. A tank has two leakages. The first leakage alone can empty the tank in 9 min and the second
alone would have done it in 6 min. If water leaks out at a constant rate, how long does it take
both the leakage together to empty the tank?
(A)5.1 min (B) 3.5 min (C) 4 min (D) 3.6 min
Ans: (D) 3.6 min
57. The time taken by a man to travel 36 miles downstream is 90 min less than to go the same
distance upstream. The speed of the man in still water is 10 mph. Find the speed of the stream?
(A) 2 mph (B) 2.5 mph(C) 3.5 mph (D) 5 mph
Ans: (A) 2 mph
Explanation:
Let the speed of the stream x mph. Then,
Speed downstream = (10 + x) mph,
Speed upstream = (10 - x) mph.
36 36 90
- =
(10 - x) (10 + x) 60
72x x 60 = 90 (100 - x2)
x2 + 48x - 100 = 0
(x+ 50)(x - 2) = 0
x = 2 mph.
58. A boat runs at 22 km per hour along the stream and 10 km per hour against the stream speed
of the boat in still water to that of the speed of the stream
(A) 2:3 (B) 5:3 (C) 7:3 (D) 8:3
Ans: (D) 8:3
59. In certain store, the profit is 320% of the cost. If the cost increases by 25% but the selle price
remains constant, approximately what percentage of the selling price is the profit
(A) 30% (B) 70% (C) 100% (D) 250%
Ans: (B) 70%
Explanation:
Let C.P.= Rs. 100. Then, Profit = Rs. 320, S.P. = Rs. 420.
New C.P. = 125% of Rs. 100 = Rs. 125
New S.P. = Rs. 420.
Profit = Rs. (420 - 125) = Rs. 295.
295 1475
Required percentage = x 100 = % = 70%
%

60. (52.02 squire – 34.01squire) / 17.99 x root? = 1720


(A) 25 (B) 20 (C) 400 (D) 625
Ans: (C) 400

Edited and Solved by Jobstestbd.com


PKB Executive Officer Exam Question Solution 2019
Post Name and Vacancy:

1. Executive Officer -General (Officer Equivalent)-30

MCQ Exam Date: 18 October 2019

MCQ Exam time: 3.00 PM to 4.00 PM

সাধারণ ান অংশ সমাধানঃ


61. The market condition when goods and services are not freely available
and thus the prices are relatively high is called?
(A) seller’s market (B) rights issue (C) sinking fund (D) recession
Ans: (C) sinking fund
62. The longest rail line of the world, Trans-Siberian line, is in
(A) China (B) Russia (C) USA (D) Saudi Arabia
Ans: (B) Russia
63. The lower limit of perpetual stow in mountains such as the Himulayas is
termed as the
(A) Ice line (B) timber line (C) boundary line (D) snow line
Ans: (D) snow line
64. The significance of peace is denoted by which of the following symbol?
(A) Olive branch (B) Green light (C) Lotus (D) Red flag
Ans: (A) Olive branch
65. Radio link is a tool of banking communication in
(A) WAN area (B) Remote area (C) LAN area (D) VAN area
Ans: (B) Remote area
66. A commonly used graphic format for the Web is
(A) GIF (B) TXT (C) BMP (D) TIF
Ans: (A) GIF

Edited and Solved by Jobstestbd.com


PKB Executive Officer Exam Question Solution 2019

67. What is a portion of a document in which you set certain page


formatting options?
(A) Page Setup (B) Page (C) Section (D) Document
Ans: (C) Section
68. Which key will open an Open dialogue box?
(A) Ctrl+F12 (B) FIZ (C) Alt+F12 (D) Shift + F12
Ans: (A) Ctrl+F12
69. Which of the following keyboard shortcut can be used for creating a
chart from the selected cells?
(A) F10 (B) FII (C) F4 (D) F2
Ans: (B) FII
70. Which of these toolbars allows changing of Fonts and their sizes?
(A) Standard (B) Print Preview (C) Formatting (D) None of these
Ans: (C) Formatting
71. Identify the volatile storage device amongst the following devices?
(A) ROM (B) Hard disk (C) Magnetic tape (D) RAM
Ans: (D) RAM
72. Web site’s main page is called its
(A) Homepage (B) Browser page (C) Search Page (D) Bookmark
Ans: (A) Homepage
73. What is a portion of a document in which you set certain page
formatting options?
(A) Page (B) Section (C) Document (D) Page Setup
Ans: (B) Section
74. To open an existing workbook, click the Open button on the —– toolbar?
(A) Form (B) Drawing (C) Standard (D) Formatting

Edited and Solved by Jobstestbd.com


PKB Executive Officer Exam Question Solution 2019
Ans: (D) Formatting
75. Which of the following requires computer memory in large amounts?
(A) Imaging (B) Graphics (C) Voice (D) All of above
Ans: D) All of above
76. In which year a resolution Uniting for Peace’ was adopted by UN General
Assembly?
(A) 1960 (B) 1965 (C) 1950 (D) 1980
Ans: (C) 1950
77. Name the instrument used to measure relative humidity
(A) Hygrometer (B) Hydrometer (C) Barometer (D) None of these
Ans: (A) Hygrometer
78. Queensland and Northern Territory Aerial Service is an International
Airline of?
(A) Afghanistan (B) Belgium (C) East Africa (D) Australia
Ans: (D) Australia
79. The branch of science that studies cells is called
(A) entomology (B) cytology (C) homoplasy (D) hormonology
Ans: (B) cytology
80. The acid name of Vitamin B is
(A) nicotinamide (B) riboflavin (C) thiamine (D) ascorbic acid
Ans: (C) thiamine

Edited and Solved by Jobstestbd.com


Rupali Bank Senior Officer MCQ Question Solution 2019

Bank Name: Rupali Bank Limited


Post name And Vacancy was:
1. Senior Officer-423
Senior Officer MCQ Exam Date: 15 November 2019

ইংেরিজ অংশ সমাধানঃ

Read the following sentences and then select the answer option that correct the underlined
sections:
1. There are not many men who are so famous that they are frequently referred to by their short
names only? Ans: initials
2. We demonstrated to them how we were prepared the artistic patterns. Ans: had prepared
3. By such time you finish that chapter, I will write a letter. Ans: By the time
4. She cooks, washes dishes, does her homework and then relaxing. Ans: then relaxes
5. One of the most significant phenomenons of our time has been the development of cinema.
Ans: phenomena
Select the word or pharse which express the best meaning:
6. Voracious- Ans: hungry
7. Admonish--------Ans: reprimand
8. Reckless-------Ans: Rash
9. Wretched ----Ans: poor
10. Restraint ----Ans: obstacle
Fill in the blank with right option:
11. -----Google and Facebook, as the largest platform for content distribution, are said to be -----
Systems that will filter fake news.
Ans: both, creating
12. Laughter has a strong from social-------In in boarding groups of people together and also
helps make the Message more-----. Ans: function, relatable
13. The----- of fake news is not new all recent, only its potential to reach people has -----Due to
online Platforms and applications that are free. Ans: Advent, amplified
14. The traditional practices of yoga lay great---- on the importance of------ in the Form of
Pranayma. Ans: emphasis, breathing
15. Special----- are made for their meals, but officials---- the long working hours Without any
what time. Ans: arrangements, rue
16. Fate Smiles---- those who untiringly grapple with stark relatives of life. Ans: on

Solved by Jobstestbd.com
Rupali Bank Senior Officer MCQ Question Solution 2019

Bank Name: Rupali Bank Limited


Post name And Vacancy was:
1. Senior Officer-423
Senior Officer MCQ Exam Date: 15 November 2019

বাংলা অংশ সমাধানঃ


১৭. কান েলা দশী শ ? উ রঃ গ) খাকা, চাঁপা
১৮. িবিধ শ িটর িবপরীত শ কানিট? উ রঃ ক) িনেষধ
১৯. সূ য এর িবপরীত শ িক? উ রঃ ঘ) কানিটই না
২০. Referendum এর বাংলা পািরভািষক শ ? উ রঃ খ) গণেভাট
২১. কান সি িট িনপাতেন িস ? উ রঃ ক) পর + পর = পর র
২২. বানান কানিট? উ রঃ ঘ) মূ ধন
২৩. দািমনী শে র অথ? উ রঃ খ) িবদু ৎ
২৪. ভূ ষি র কাক অথ িক? উ রঃ গ) দীঘায়ু ব ি
২৫. কান কিব িনেজেক বাঙািল বেল পিরচয় িদেয়েছন? উ রঃ ক) ভুসু কুপা
২৬. কানিট ঈষৎ অেথ অব য়ীভাব সমাস? উ রঃ গ) আরি ম
২৭. িনেচর কানিট পক কমধারয়? উ রঃ মনমািঝ
২৮. ব াকরেণর ধান কাজ িক? উ রঃ ঘ) ভাষার িবে ষণ
২৯. কানিট তৎসম শ ? উ রঃ খ) চ ন
৩০. পয়ারা কান ভাষার থেক আগত শ ? উ রঃ গ) পতুগীজ
৩১. ভাষা মানু েষর মুখ থেক কলেমর মুেখ আেস, উে াটা করেত গেল মুেখ ধু কািল পেড়” ক বেলেছন?
উ রঃ ক) মথ চৗধু রী
৩২. িনেচর কান বচনিট হতভাগ অেথ ব ব ত? উ রঃ গ) আটকপােল

Solved by Jobstestbd.com
Rupali Bank Senior Officer MCQ Question Solution 2019

Bank Name: Rupali Bank Limited


Post name And Vacancy was:
1. Senior Officer-423
Senior Officer MCQ Exam Date: 15 November 2019

গিণত অংশ সমাধানঃ


33. In steel water, a boat can travel at 5 km/hr . it takes 1 hour to row to a place and come back.
if the velocity of the stream is 1 km/hr, how far is the place?
Ans: A (2.4 K.M)
34. two men P and Q start from a place working at 5 Ei km/hr and 6.5 Eid km/hr, respectively.
what is the time they will take to be 92 km apart, if they work in oxide directions? Ans: D (8 hr)
35. Two men A and B run a 4 km race on a circular course of ¼ km, if there is freed are in the
ratio of 5:4How much time does the winner pass the other? Ans: A (thrice)
36. A sum of money amounts to Tk. 6690 after 3 years and to TK.10035 after 6 years on
compound interest, find the sum. Ans: D (Tk. 4460)
37. One card is drawn at random from a pack of 52 cards. what is the probability that the card
drawn is a face card? Ans: A (1/13)
38. If two times of the daughters is in years in included to the Mother’s age the total is 70 and if
two times of the mother's age is included to the daughters age, the total age is 95. So, the
mother’s age is? Ans: C (40)
39.A student multiplied a number by 5/3, What is the percentage error in the calculation?
Ans: A (64%)
40. 10 years ago, A was half of B in age, if the ratio of their present ages is 3:4, then what will be
the total of their present ages? Ans: B (35 years)
41. Tthe size of the wooden block is 5*10* 20. How many such blocks will be required to
construct a solid wooden cube of minimum size? Ans: D (8)
42. A tank is 25 m long, 12 m wide and 6 m Deep. the cost of plastering its walls and bottom at
75 Paisa per m2 is : Ans: C (558Tk)
43. The difference of two numbers is 20% of the larger number, if the smaller number is 20, e
than the larger number is? Ans: A (25)
44. 12 buckets of water fill a tank when the capacity of his tank is 13.5. How many buckets will
be needed to fill the same tank, if the capacity of bucket is 9 litres? Ans: A (8)
45. The sum of three numbers is 98. if the ratio of the first to second is 2:3 and that of the second
to the third is 5:8, then the second number is: Ans: D (30)
46. If k:l =4:3And l:m=5:3 then find k:l:m? Ans: A (20:15:9)
47. The value of………...is. Ans: C (4)
48. A 270 M long train running at the speed of 120 km/hr crosses another train running in the
opposite direction at the speed of 80 km/hr in 9 seconds. what is the length of the other train?
Ans: A (230 m)
49. Three partners shared the profit in a business in the ratio 5:7:8. They have partnered for 14
months, 8 months and 7 months respectively. what was the ratio of their Investments?
Ans: D (20:49:64)

Solved by Jobstestbd.com
Rupali Bank Senior Officer MCQ Question Solution 2019
50. In a class there are 15 boys and 10 girls. Three students are selected at random. the
probability that 1 girl and 2 boys are selected is? Ans: B (21/46)
51. An error plan size along the four sides of a square at the speed of 200, 400, 600 601 800
km/hr . find the average speed of the plane around the field. Ans: B (375 km/hr)
52. X can complete a certain work in the same time in which Y and Z together can do it. If X and
Y together can finish it in 10 days and Z along in 15 days, then Y alone can do it in?
Ans: B (25 days)
53. 98.98/11.03+7.014*15.99=? Ans: D (121)
54. A and B Start a business with initial investment in the radio 12:11 Dayal Annual fees wire in
the rate 4:1 if A invested the money for 11 months. B invested the money for?
Ans: A (3 months)
55. If m = 7 -4√3 then (√m + 1) 1/√m) =? Ans: C (4)
56. A container contains 40 litres of milk. from this container for litres of male was taken out and
replaced by water. this process was repeated for two times, how much milk is now contained in
the container? Ans: A (29.16 litres)

Solved by Jobstestbd.com
Rupali Bank Senior Officer MCQ Question Solution 2019

Bank Name: Rupali Bank Limited


Post name And Vacancy was:
1. Senior Officer-423
Senior Officer MCQ Exam Date: 15 November 2019

কি উটার ও সাধারণ ান অংশ সমাধানঃ


৫৭. Which among the following is associated with excel? Ans: Spreadsheet
৫৮. The basic unit of a worksheet into which you enter data in Excel is called? Ans: Cell
৫৯. Wi-fi uses? Ans: Radio waves
৬০. What should be used if a company wants to include its company name and logo at
the bottom of every page of a brochure? Ans: Footer
৬১. Protection and protect sheet option can be selected from? Ans: Tools
৬২. In order to tell Excel that we are entering a formula in cell, we must begin with an
operator such as-Ans: =
৬৩. The bar at the top of a window that bears the name of the window is known as?
Ans: Title bar
৬৪. Different icons of application software can be found in which bar in latest version
Microsoft Windows? Ans: Start Menu
৬৫. What is the shortcut key for line break? Ans: Shift + Enter
৬৬. In which window we display data in the form of tables in a row or column format in
MS-Access? Ans: Sub-form
৬৭. Verification of a login name and password is known as? Ans: Authentication
৬৮. If you press ___, the cell accepts your typing as its contents? Ans: Enter
৬৯. The frequency of which of the following is the highest? Ans: Gamma rays
৭০. The first attempt in printing was made in England by? Ans: James Arkwright
৭১. Which company nickname is big blue? Ans: IBM
৭২. The headquarter of united nations fund for programmes population activities (UNFPA)
are at? Ans: New York
৭৩. The law of natural selection is associated with? Ans: Darwin
৭৪. Guarantee to an exporter that the importer of his goods will pay immediately for the
goods ordered by him, is known as? Ans: Letter of Credit (L/C)

Solved by Jobstestbd.com
Rupali Bank Senior Officer MCQ Question Solution 2019

৭৫. Fathometer is used to measure? Ans: Ocean depth


৭৬. Lance Armstrong, a sportsperson of international repute, belongs to which of the
following countries? Ans: USA
৭৭. Study of life in outer space is known as? Ans: exobiology
৭৮. Stress test is related to assess health of which institution? Ans: Bank
৭৯. Which of the following is not the official language of united nations organization?
Ans: Portuguese
৮০. The normal threshold of hearing is around? Ans: 25-45 db

Solved by Jobstestbd.com
InDEx
1. Combined 5 Banks Officer -2018 01-07
2. Sonali, Janata- Senior Officer (ICT) -2018 07-12
3. House Building Finance Corporation Senior Officer -2017 12-18
4. Bangladesh Bank Assistant Director (General) -2018 18-24
5. Sonali Bank Ltd Senior Officer -2018 24-29
6. Agrani Bank Ltd Senior Officer (Auditor) -2017 29-34
7. Rupali Bank Ltd Officer (Cash) -2018 35-42
8. Agrani Bank Ltd Officer (Cash) -2017 43-49
9. Bangladesh Krishi Bank Ltd Officer (Cash) -2017 49-55
10. Bangladesh Development Bank Ltd Senior Officer -2017 55-61
11. Bangladesh Bank Officer (General) -2018 61-67
12. Rupali Bank Officer – 2019 67-73

Collected By: Md. Raihan Ahmed


Job Information For All Students (Bcs & Bank)
JOB INFORMATION FOR ALL STUDENTS (BCS & BANK)

D. 01 B
10.
BANGLA -
A. 02 B
1. -
A. , , , B.
B. , , , C. 03 B
C. , , , D.
D. , , , 11. , ,
- 04 C
2. -
A. A.
B. B. 05 D
C. C.
D. D.
12. - - 06 B
3. - -
A. A.
B. B.
07 D
C. C.
D. D.
4. - 13. - 08 C

A. A.
B. B. 09 B
C. C.
D. D.
10 B
5. - - - - 14. -
A.
- B. 11 D
A. C.
B. D.
12 C
C. 15. - -
D. A.
6. - B. 13 C
A. C.
B. D.
14 A
C. 16. -
D. A.
B. 15 A
7. -
A. C.
B. D. 16 C
C.
D.
17 A
8. - 17. Unfortunately, their house _____while they were at the
A. restaurant celebrating the anniversary.
B. A. got burgled 18 C
C. B. went burled
D. C. burgled
9. - D. had buried
A. 18. Use the right form of verb. I had no difficulty (to find)
B. their house although they had said that people often did.
C. A. to find

1
Collected By: Md. Raihan Ahmed
JOB INFORMATION FOR ALL STUDENTS (BCS & BANK)

B. to finding 27. The best conjunction to link: 19 C


C. in finding
D. for finding understood the problem correctly.
19. - A. when 20 B

A. to set fire B. as
B. to burn C. if 21 C
C. kindle D. but
D. to lit 28. Change the form of voice: I was surprised to see that
20. If Elucidate: Clarity then he had been beaten black and blue. 22 C

A. Mystify : Enlightenment A. I was surprised to see he had been beaten black and
B. Illuminate : Light blue. 23 D
C. Aggravate : Problem B. It surprised me to see that he had been beaten black
D. Conceal : Oblivion and blue
21. Select the correct linking word. He should have spent all C. It was surprising that he had been beaten black and 24 C

the weekend preparing for his test; he in fact just lay in bed blue
watching videos. D. That he had been beaten black and blue was
25 B
A. Nevertheless before He surprising
B. However before He 29. The word skedaddle is not related to-
C. Whereas before He A. toddle 26 B

D. Despite before He B. dash


22. Translate into English: , C. scamper 27 D
D. shroud
A. A pet lamb makes a cross ram 30. When the evening spread out against the sky like a
28 B
B. A square peg in a round hole patient etherized upon a table an example of
C. A scolded dog fears cold water. A. metaphor
D. A still tongue makes a wise head B. chiasmus 29 D
23. C. oxymoron
my argument with an anecdote whose significance will soon D. personification
30 D
become apparent. 31. cannot live
A. predict
B. announce A. They said that human cannot live without air. 31 B
C. herald B. They said that we cannot live without air.
D. preface C. They said that it is not possible to live without air.
32 A
24. The correctly spelt word is D. They realized that people could not live without air.
A. Scizophrenic 32.
B. Sciophrynic A. latin Gramines 33 C

C. Schizophrenic B. Romanian Granicu


D. Selzophrink C. old English Gramen
25. After the football match the crowd _________ out of the D. German Gramineen
stadium into the nearest cafes and restaurants.
A. dripped MATHEMATICS
B. poured 33. The values of p for equation 2×2-4x + p = 0 to have
C. trickled real roots is
D. leaked A. P≤ 2
26. B. P≥2
A. fresh C. P≤2
B. cure-all D. P ≥ -7
C. disease
D. problem

2
Collected By: Md. Raihan Ahmed
JOB INFORMATION FOR ALL STUDENTS (BCS & BANK)

Solution: =(a+1/a)3 3.a.1/a(a+1/a) 34 B


Here, =23 -3.2
b2 4ac ≥ 0
2 a=2 =8-6
Or, (-4) 4×2×p≥ 0 35 B
b= -4 =2
Or, -8p≥ -16
c=p 38. If 10% of x is equal to 25% of y, and y=16, what is
Or, p≤2
the value of x? 36 D
34. How many integers from 1 to 100 are divisible by 3 but not A. 4
by 8 ? B. 6.4
A. 30 C. 24 37 C

B. 29 D. 40
C. 31 Solution: 38 D
D. 32 10% of x = 25% of 16
Solution: Or, 10x/100 = 16× 25/100
39 A
Integers divisible by 3 = 100/3 = 33 Or, x/10=4
LCM of 3 & 8 = 24 Or, x=40
Integers divisible by 24 = 100/24 = 4 39. If sinA + sin2A = 1, then the value of the expression 40 A
Integers divisible by 3 and not by 8 = 33-4 = 29 (cos2A + cos4A) is
35. If x is an integer and y = 2x 8, what is the least value A. 1
41 A
of x for which y is less than 9? B. ½
A. -9 C. 2
B. -8 D. 3
C. -7 Solution:
D. -6
sinA + sin2A = 1
Solution:
Or, sinA=1- sin2A
Let, Or, sinA=cox2A
y=8
Or, -2x-8=8 Now,
Or, -2x=16 cos2A + cos4A
Or, x= 8 = cos2A + (cos2A)2

36. If x:y = 5:3, then (8x+5y) : (8x-5y) =? =sinA + sin2A

A. 5:11 =1

B. 6:5 40. A pole casts a√3m long shadow on the ground at an


C. 5:6 elevation 600, the height of the pole is-
D. 11:5 A. 3m
Solution: B. √3m
x:y=5:3 C. 3√3m
Now, D. 2m
(8x+5y) : (8x-5y) Solution:
=(8×5 + 5×3) : (8×5 5×3) tanθ = height/ground
=55 : 25 Or, √3 = height/√3
=11 : 5 Or, height = √3.√3=3
37. If a+1/a = 2, what is a3 + 1/a3? 41. If the difference between the circumference and
A. ½ diameter of a circle is 90 cm, then the radius
B. 7 approximately is-
C. 2 A. 21 cm
D. 3/2 B. 19 cm
Solution: C. 20 cm
a3 + 1/a3 D. 22 cm

3
Collected By: Md. Raihan Ahmed
JOB INFORMATION FOR ALL STUDENTS (BCS & BANK)

Solution: 2nd × r = 3rd 42 B


ATQ, Or, 9 × r = 3
2πr 2r = 90 Or, r = 1/3
Or, r(π-1) = 45 Now, 43 A

Or, r(22/7 1) =45 4th term = 3 × 1/3 = 1


Or, 15r/7 = 45 5th term = 1× 1/3= 1/3 44 C
Or, r = 45×7/15=21 46. If two fair coins are flipped, what is the probability
42. If the length of a side of a regular pentagon is 4 cm, the that one will come up heads and the other tails?
area of the pentagon is approximately- A. ¼ 45 C

A. 25cm2 B. 1/3
B. 27cm2 C. ½ 46 C
C. 29cm2 D. ¾
D. 32cm2 Solution:
Solution: 47 C
Total events, (HH, HT, TT, TH) = 4
Area of the pentagon, One Head or Tail, HT, TH = 2
2
=1/4(√{5(5+2√5)} × 4 So, probability = 2/4=1/2 48 B
= 4(√(25+10√5) 47. All possible three digit numbers are formed by 1 3.5.
=4√(25+10×2.23) If one number is chosen randomly, the probability that it
49 C
=4√47 would be divisible by 5 is
=4×6.86 A. 1/9
=27.44 =27 B. 2/9
43. The height of an equilateral triangle with a side 2 cm is C. 1/3
A. √3cm D. ¼
B. 2√3cm Solution:
C. 3√2cm Total ways of forming 3-digit number, 3! = 6
D. √5cm Number divisible by 5 will have 5 in the unit
Solution: digit.
height of the equilateral triangle So, 5 at the unit position, 3-digit number is
=√3×a/2 = 2! = 2
=√3×2/2 =√3 Probability = 2/6=1/3
44. 48. The line perpendicular to y =x-2 is.
A. 54 A. Y=2x+1
B. 56 B. 2y=-2x-5
C. 58 C. 2y = x +7
D. 60 D. Y=3x+1
Solution: Solution:
2
3+1 = 4 Straight line equation,
4+22 = 8 y=mx + b
2
8+3 = 17 In the equation, m=1
17+42 = 33 m=slope of the
so, the slop of the line
33+52 = 58 line
perpendicular to
45. The second and third terms of a geometric series are 9 and y= 1/m = 1/1 = -1
3 respectively. The fifth term of the series is Among the options, only B
A. 1 has the slope -1.
B. 1/9 49. if logx2 9/16 = ½ the value of the base is
C. 1/3 A. 16/ 9
D. 1/27 B. 9/16
Solution: C. 256/81

4
Collected By: Md. Raihan Ahmed
JOB INFORMATION FOR ALL STUDENTS (BCS & BANK)

D. 81/256 50 B
3x 7y = 0
Solution:
Or, 3x = 7y
logx2 9/16 = ½
Or, x= 7y/3
Or, x2(-1/2) = 9/16 51 B
Now,
Or, x-1 = 9/16
X + 2y = 13
Or, 1/x= 9/16 52 A
Or, 7y/3 + 2y = 13
Or, x=16/9
Or, 13y/3 = 13
Now,
Or, y = 13×3/13=3
the value of the base x2 = (16/9)2=256/81 53 B
54. Which of the following can be rearranged to form
50. nC1 + nC2 + nC3 + .. .. nCn = ?
Bangla word(s)
A. 2n 54 C
a. b. c. d.
B. 2^n-1
A. c
C. n(n-1)(n2+1)/2
B. b 55 A
D. 2^n-1
C. a&d
51. The solution of the inequality I7 3xI < 2 is-
D. a&b
A. -3<x<5/3
56 A
55.
B. 3>x>5/3
comet.
C. -3<x<5/2
A. July 2061 57 B
D. -3<x<-5/3
B. June 2051
Solution:
C. May 2050
If (7-3x) is positive, 58 A
D. March 2060
7-3x < 2
56. The sum of squares of 3 consecutive integers is less
Or, -3x < -5
than 97. What is the greatest possible value of the
Or, 3x > 5
smallest one?
Or, x > 3/5
A. 4
Again if, (7-3x) is negative,
B. 5
7-3x > 2
C. 6
Or, 3x > -9
D. 7
Or, x < 3
Solution:
So, 3 > x > 5/3
From the options,
52. The difference in taka between simple and compound
42 + 52 + 62 = 77
Interest at 5% annually on a sum of Tk 2000 after 2 year is
52 + 62 + 72 = 110
A. 5
So, the smallest is 4
B. 50
C. 20
GENERAL KNOWLEDGE AND COMPUTER
D. 200
57. The Federal Reserve System of the USA is framed by
Solution:
_____ Federal Reserve Banks.
Diff between CI & SI A. 13
= 2000(1+5/100) 2
2000 2000×2×5/100 B. 12
=2000(1+1/20) 2
2000 200 C. 11
= 2205 2000 200 D. 10
=5 58. Bangladesh defeated Ireland in the final match of the

53. If 3x-7y = 0 and x + 2y = 13 then y is


A. 25 runs
A. 2
B. 45 runs
B. 3
C. 75 runs
C. 4
D. 97 runs
D. 7
Solution:

5
Collected By: Md. Raihan Ahmed
JOB INFORMATION FOR ALL STUDENTS (BCS & BANK)

59. The slogan of World Population Day 2018 is D. 59 B


A. Investing in Teenage Girl 68. The best film title awarded in the 41″ Bangladesh
B. Family Planning is a Human Right National Film Award 2016 is
C. Empower People, Develop Nations A. 60 A

D. Investing in Young People B.


60. During the Liberation War of Bangladesh the Secretary C. 61 A
General of UN was D.
A. U Thant 69. In the present Fiscal Year, the following sector has
B. Kurt Waldheim earned the highest export growth. 62 D

C. Dag Hammarskjold A. RMG sector


D. Boutros Boutros-Ghali B. Agricultural sector 63 A
61. The former name of Switzerland was C. Industry sector
A. Helvetia D. Fisheries sector
B. Rhodesia 70. The autobiography titled is written by 64 C

C. Dockland A. Panna Kalsar


D. Salisbury B. Sara Zaker
65 D
62. The percentage of Russian Investment in Rooppur Nuclear C. Ferdausi Mazumder
Power Plant is D. Enamul Huq
A. 75% 71. The word Twibill is related to 66 A

B. 80% A. sharpened weapon


C. 85% B. old fashioned gun 67 C
D. 90% C. lethal ammunitions
63. Bangladesh Shilpa Bank and Shilpa Rin Sangstha have D. poisonous swordAnswer: sharpened weapon
68 B
merged into 72. The gold medal winner from Bangladesh in the 59th
A. BDBL International Mathematical Olympiad is
B. DBBL A. Tamjid Morshed 69 A
C. DDBL B. Tahnik Noor
D. VDBL C. Ahmed Jawad
70 C
64. The cave where the 12 young Thal football players got D. Joydeep Saha
trapped and rescued later recently, is 73. A computer program that translates one program
A. Tham Phirman Instruction at a time into machine language is called a/an 71 A
B. Tham Lạt A. Interpreter
C. Tham Luang B. CPU
72 C
D. Emerald C. Compiler
65. The FIFA World Cup 2018 Champion France received an D. Simulator
amount of USD_ million as prize money. 74. The accuracy of the floating point numbers 73 A

A. 30 represented in two 16-bit words of a computer is


B. 35 approximately 74 B
C. 36 A. 16 digits
D. 38 B. 6 digits
75 B
66. Export growth rate achieved in FY 2017-18 is C. 9 digits
A. 5.81% D. All of above
B. 6.61% 75. Which of the following memories must be refreshed
C. 8.87% many times per second?
D. 5.52% A. Static RAM
67. Tumbru Rohinga Camp of Bangladesh is located in B. Dynamic RAM
A. Khagrachhori C. EPROM
B. Rangamati D. ROM
C. Bandarban

6
Collected By: Md. Raihan Ahmed
JOB INFORMATION FOR ALL STUDENTS (BCS & BANK)

76. A name or number used to identify a storage location is B. 76 C


called C.
A. a byte D.
B. a record 5. ? 77 B

C. an address A.
D. a bit B. 78 D
77. The octal equivalence of 111010 is C.
A. 81 D.
B. 72 6. - 79 A

C. 71 A.
D. 74 B. 80 C
78. A__ is a collection of predefined design elements and color C.
schemes. D.
A. Feature 7. ? 01 D

B. Hyperlink A.
C. Palette B.
02 A
D. Theme C.
79. A Proxy server is used for D.
A. providing security against unauthorized users 8. - 03 C

B. processing client requests for web pages A.


C. processing client requests for database access B. 04 A
D. providing TCP/IP C.
80. The short cut key to replace a data with another in an excel D.
05 C
sheet is 9. ,
A. Ctrl+R -
B. Shift+R A. 06 A
C. Ctrl+H B.
D. Ctrl+F C.
07 C
D.
10. ,
- 08 D
BANGLA A.
1. - B.
09 B
A. C.
B. D.
C. 11. - 10 D
D. A.
2. - B. 11 D
A. C.
B. D.
C. 12 D
12. -
D. A.
3. - B. 13 B
A. - C.
B. D.
C. 13. -
D. A.
4. B.
A. C.

7
Collected By: Md. Raihan Ahmed
JOB INFORMATION FOR ALL STUDENTS (BCS & BANK)

D. B. 14 A
14. ?- C.
A. D.
B. 23. The past form of the verb Tear is 15 C

C. : A. Teared
D. B. Torn 16 D
15. C. Tore
A. + D. Tears
B. + 24. Choose the right meaning of: Take off. 17 C

C. A. Stop Sleeping
D. + B. Leave the world 18 D
16. : C. Leave the ground
A. D. Stop driving
B. 25. Which of the following sentences is in past indefinite 19 B

C. tense?
D. A. Raihan was sleeping.
20 C
B. It was raining.
C. It has been raining.
17. D. Shohail had a car. 21 A

A. Heighten 26. Choose the correct tag question.


B. Intensity A. 22 C
C. Diminish B. People can fly, can they?
D. Morbid C.
23 B
18. Adding narratives and details to the basic content serves to D.
- the book by producing a fuller account 27. Choose the correct spelling.
A. invalidate A. Corridor 24 C
B. objectify B. Coridour
C. celebrate C. Corridour
25 D
D. enrich D. Cooridor
19. 28. Which of the following sentences is correct?
frowned on by most of his elders, who found them too ______ A. He started at six of the doming 26 A
for their conservative values. B. He started on six at the morning
A. heretical C. He started at six in the morning
27 A
B. Meticulous D. He started on six in the morning
C. precise 29. Which of the following statements is correct?
D. incoherent A. If I am a bird. 28 C

20. B. If I was a bird.


A. expanse C. If I a bird. 29 D
B. determination D. If I were a bird.
C. Virtue 30. - Translation:
30 B
D. proclivity A. Two thieves are cousins.
21. He was killed______ the robber ________ hatchet. B. Birds of a feather flock together
A. C. Two birds fly together 31 D
B. D. Bird of the same group fly together
C. 31. Correct Plural form of Half is:
D. A. Halfs
22. He has no good cause __________ complaint. Darkness was B. Halffs
the cause ____ his losing his way. C. Halve
A. D. Halves

8
Collected By: Md. Raihan Ahmed
JOB INFORMATION FOR ALL STUDENTS (BCS & BANK)

32. 32 D
n(CUB) = n(C) + n(B) n(C∩B)
Transform the sentence from affirmative to negative.
Or, 60 = 27 + 42 n(C∩B)
A. Science students should not apply for the post.
Or, n(C∩B) = 9
B. Other than science students no one should apply 33 A
37. A person buys a TV worth BDT 3,90,000 with a down
C. Science students can only apply for the post.
D. None but the science students can apply for the post. 34 A
installment. How many more installments does he have to
pay if his installments had to double after each successive
MATHEMATICS
payment? 35 B
33. A stock is currently valued at TK 40 per share. If the price
A. 6
increases by 20 percent and then decreases by 25 percent what
B. 7
will be the new value of the share? 36 A
C. 8
A. Tk. 36
D. 10
B. tk. 40
Solution: 37 A
C. Tk.48
D. Tk. 30 After down payment and first
Solution: installment = 3,90,000 40,000
38 D
5,000 = 3,45,000
First increase by 20%
ATQ,
So, per share value = 40 +40×20% = 48
39 A
Then, decrease by 25%, n
Sn = a(r 1) / (r-1)
So new value will become = 48 48×25% = 36
Or, 3,45,000 = 5000(2n 1) /
34. After dividing a positive integer, Y, by 3, the remainder is (2-1) r = 10000/5000 = 2
2; but when Y is divided by 7 the remainder is 4. What is the Or, 5000(2n 1) = 3,45,000
least possible value of Y? Or, 2n 1 = 69
A. 11 Or, 2n = 70
B. 22 26 = 64
C. 18 27 = 128
D. 32 more installments he has to
35. In a graph there are two curves, y1=2x-5 and y2= -x+10. pay = 7-1=6
y2 will be greater than y1, when- 38. Painting a wall costs BDT. 50 per square meter. If the
A. x>5 20m long wall has a width of 10m, how much would it
B. x<5 cost to paint 7/10 of the wall?
C. -1<x A. 5000
D. X<9 B. 10000
Solution: C. 8000
Given, D. 7000
y1=2x-5 and y2= -x+10 Solution:
y2 > y1 20m × 10m × 50 × 7/10 = 7000
Or, x + 10 > 2x 5 39. For a software development project, a given group can
Or, 15 > 3x be divided into 8 groups of 3 coders each. How many
Or, x<5 groups can be formed if the manager decides to have 6

36. In a group of 60 people, 27 like coke and 42 like borhani coders in each group?

and each person likes at least one of the two drinks. How A. 4

many people like both coke and borhani? B. 5

A. 9 C. 6

B. 129 D. 8

C. 69 Solution:

D. 15
Solution:

9
Collected By: Md. Raihan Ahmed
JOB INFORMATION FOR ALL STUDENTS (BCS & BANK)

45. There are total 40 dishes of rice and meat. If the ratio 40 A
Total coders = 8 × 3 = 24
of number of rice dish to the number of meat dish is 3 to
Numbers of groups each having 6 coders = 24/6 = 4
2. how many meat dishes are there?
40. A man walks 10m east but runs 5m west every day. How 41 D
A. 16
many days will it take him to cross a building 380m east?
B. 24
A. 75 days
C. 8 42 A
B. 76 days
D. 32
C. 78 days
Solution:
D. 80 days 43 A
Let,
41. Bangladeshi supporters in a stadium double every match.
Rice 3x, meat 2x
In the eighth match, there were 48000 supporters which was
ATQ, 44 B
the full capacity of the stadium. In which match did the
3x + 2x = 40
Bangladeshi supporters fill up half the capacity of the stadium?
Or, x = 8
A. 2nd match 45 A
Meat = 2x = 2X8 = 16
B. 4th match
46. You bought 11 pencil and erasers worth BDT 80. If
C. 6th match
erasers cost half that of a pencil and you bought one 46 C
D. 7th match
extra cruiser how much is the eraser worth?
42. 6, 7, 9,13,__, ___. What are the two missing numbers in the
A. 3
series? 47 B
B. 4
A. 21,37
C. 5
B. 17.21
D. 6 48 C
C. 21, 39
Solution:
D. 17,19
43. 3/4th of a square were taken to form Shape A and the rest Let,

was made to form shape B. Shape A was divided into four Pencil x

equal squares (Shape C), what will be the ratio of the one And eraser = x/2

Shape C to one Shape B? 5x + 6x/2 = 80

A. 3:4 Or, x = 10

B. 4:5 eraser = x/2 = 10/2=5

C. 3:5 47. Jashim buys 10 CDs for BDT 200. If DVDs cost BDT 20
D. 5:6 more, how many DVDs can he buy for the same amount?
Solution: A. 4

A= 3x/4 B. 5

B= x/4 C. 6

C= (3x/4) / 4 = 3x/16 D. 10

C:B = 3x/16:x/4 = 3:4 Solution:

44. A bag and a book costs BDT 1100. If the bag costs 1000 Cost of a CD = 200/10 = 200

more than the book, how much does the book cost? Cost of a DVD = 20+20 = 40

A. 100 Then number of DVD = 200/40=5

B. 50 48. In a room there are six Bengali, twelve engineers and


C. 30 fifteen football players. Only one of them was a Bengali
D. 200 Engineer who played Football. Two were Bengali Engineers
Solution: but did not play football and two were Bengali football

Book costs x players and were not engineers. If there were 24 people

Bag cost x+1000 in the room, and at least one of them were Bengali,

ATQ, engineer or a football player, how many were engineers

X+1000+x = 1100 and played football but not Bengali?

Or, x=50 A. 1
B. 9

10
Collected By: Md. Raihan Ahmed
JOB INFORMATION FOR ALL STUDENTS (BCS & BANK)

C. 6 D. Norway 49 B
D. 3 57. The first Bangladeshi to earn Grand Master title is:
Solution: A. Niaz Morshed
n(BUEUF) = n(B)+n(E)+n(F)+n(B∩E∩F)-n(B∩E)-n(B∩F)- B. Ziaur Rahman 50 C

n(E∩F) C. Rani Hamid


Or, 24 = 6+12+15+1-2-2- n(E∩F) D. Quazi Motahar Hossain 51 D
Or, n(E∩F) = 6 58.
Protik for their contributions in the Liberation War of
GENERAL KNOWLEDGE AND COMPUTER Bangladesh? 52 C

49. Which of the following is known as the silent city? A. 2


A. Sydney B. 3 53 A
B. Rome C. 4
C. Venice D. 5
D. Bangkok 59. 54 B

50. Bangabandhu Satellite-I was launched from which place? A. Sylhet and Rangamati
A. Zurich Space Center B. Rangpur and Dinajpurx
55 A
B. London Space Center C. Mymensingh and Netrokona
C. Kennedy Space Center D.
D. New York Space Center 60. Who was the first editor of literary magazine 56 B

51. The first parliament election in Bangladesh was held in


A. 1970 A. Pari Chad Mitra 57 A
B. 1971 B. Bankim Chandra Chattopadhy
C. 1972 C. Akshay Kumar Datta
58 A
D. 1973 D. Promoth Chowdhury
52. The opening match of 2018 football world cup will take 61. Which one of the following is not a Central Bank?
place between A. Bangladesh bank 59 C
A. Russia and Morocco B. Bank of England
B. Russia and Egypt C. Federal Reserve System
60 B
C. Russia and Saudi Arabia D. State Bank of India
D. Egypt and Uruguay 62. Which of the following country is the highest emitter
53. of CO2? 61 D
A. Alan Ginsberg A. Bangladesh
B. Monica Ali B. India
62 D
C. Robert Itosi C. Paldistan
D. Anisul Hoque D. China
54. - 63. The deepest lake in the world is 63 B

A. 1st World War A. Lake Adelman


B. 2nd World War B. Lake Baikal 64 D
C. Gulf War C. Lake Malawi
D. Iraq War D. Lake Huron
65 B
55. Which country of the world does not have its capital? 64. In which country did the Mau Mau uprising (1952-60)
A. Monaco occur?
B. Nauru A. Uganda
C. Tue Vallue B. Zambia
D. The Marshal Island C. Nigeria
56. Which one of the following is not Scandinavian Country? D. Kenya
A. Sweden 65. Who invented QWERTY keyboard?
B. Spain A. Steve Jobs
C. Denmark B. Christopher Latham Sholes

11
Collected By: Md. Raihan Ahmed
JOB INFORMATION FOR ALL STUDENTS (BCS & BANK)

C. Brian Sams C. Fax Pro 66 C


D. Anderson Palimar D. Power Point
66. Which one of the following is the reason for 75. Why do we need to normalize a database?
A. To remove redundancy 67 D

A. excessive typing B. To make data meaningful


B. watching for long hours C. To make database secure 68 D
C. excessive anxiety due to online search D. To make database consistence
D. radiation 76. Physical connection between Microprocessor Memory
67. Which one is not an example of disruptive technology? and other parts is called 69 C

A. 3D printing A. Address bus


B. Internet of things B. Data bus 70 D
C. Robotics C. Path
D. Executive dashboard D. Hub
68. Cloud computing is __________ option for small firms. 77. DFD stands for 71 A

A. poor A. data file disk


B. rigid B. data flow diagram
72 A
C. rental C. disk flat database
D. expensive D. disk file database
69. Which is not a performance characteristic of hard disk 78. Which of the following is not an antivirus software? 73 D

drive? A. Win-pro
A. data transfer time B. AVG 74 B
B. response time C. MeAfee
C. power consumption D. Symantec
75 A
D. shelf life 79. 1(one) nibble equal to
70. Extranet allows A. I bit
A. insiders only B. 2 bit 76 A
B. authorized outsiders only C. 4 bit
C. all outsiders D. 8 bit
77 B
D. both insiders and authentic outsiders 80. int number[] ={10, 20, 30, 40, 50} number[3]=?
71. Which part of a class is invoked when an object is A. 10
initialized in java? B. 20 78 A
A. constructor C. 30
B. fields D. 40
79 C
C. methods
D. class
72. What is the binary of 68? 80 D

A. 01000100 BANGLA
B. 10000100 01. 01 B
C. 00100100 A.
D. 00010100 B.
02 B
73. Which one of the following is not required in a gaming C.
engine? D.
A. rendering 02. 03 D
B. audio A.
C. physics B.
D. closure C.
74. Which of the following is a spreadsheet software? D. -
A. Adobe Acrobat 03.
B. MS Excel A. , ,

12
Collected By: Md. Raihan Ahmed
JOB INFORMATION FOR ALL STUDENTS (BCS & BANK)

B. , , A. 04 D
C. , , B.
D. , , C.
04. D. 05 A

A. 14.
B. A. 06 C
C. B.
D. C.
05. D. 07 D

A. 15.
B. 08 B
C. B.
D. C.
06. 09 C

A. 16. -
B. A.
10 B
C. B.
D. C.
07. D. 11 B

A.
ENGLISH
B. 12 B
17. -
C.
A. Jam-packed
D.
B. Jammed 13 B
08.
C. Vacant
A. , ,
D. Forlorn
B. , , 14 A
18.
C. , ,
D. , , A. Plentiful
B. Copious 15 A
09.
C. Scanty
A.
D. Lavish 16 C
B.
19. Birds of a same feather means _______ .
C.
A. Same kind of bird
D.
17 A
B. Persons of similar nature
10. ?
C. Birds of prey
A.
D. Persons of week nature. 18 C
B.
20.
C.
The bold -faced word means -
D. 19 B
11. A. Eulogized
B. Appreciated
A.
C. Criticized 20 C
B.
D. Extolled
C.
21.
D. 21 B
Substitute in one word
12.
A. Man-eater
A.
B. Carnivorous
B.
C. Beast
C.
D. Cannibal
D.
13.

13
Collected By: Md. Raihan Ahmed
JOB INFORMATION FOR ALL STUDENTS (BCS & BANK)

22. The bold-faced phrase in the sentence 31. One who compiles dictionary is called a___________ . 22 D
A. Demographer
A. Verb B. Dictionary-maker
B. Adjective C. Lexicographer 23 C

C. Adverb D. Monographer
D. Noun 32. 24 B
23. Bangla ______ .
A. Man is not mortal A. ,
B. Man cannot be mortal B. , 25 A

C. Man is not immortal C. ,


D. Man will not be mortal D. 26 A
24.
MATHEMATICS
A. Be it done as It is said
33. In ▲ABC, AB = BC and AC is the hypotenuse. The 27 B
B. Let it be done as I say
value of C is
C. It should be done as I say
A. 35°
D. Let it be done as is said by me
28 A
25. Which one is correct indirect narration? B. 45°
C. 60°
A. You told me that you would be careful
D. 90° 29 B
B. You said to me that you are careful
Solution:
C. You told me that you will be careful
D. You have been careful .:. C angle = 180° 90° = 45°-degree
30 A
26. 34. The distance from the centre of a circle to the
-faced section is an circumference is(
31 C
error ? ?)
A. are A. arc
B. common B. diameter 32 D
C. which C. radius
D. usually D. secant
33 B
27. 35. If a man was r years old s years ago, how many years
The right form of the verb in bracket is - old will he be t years from now?
A. changing A. s +r+t 34 C
B. changed B. r + S +t
C. has changed C. s-r + t 35 B
D. has been changing D. r- s +t
28. Solution:
A. How long is it 36 A
S years ago, age = r years
B. How long time there is
After s years, age will be = r+ s
C. How long it is
.:. And after t years age will be = r + s + t
D. How long she has
36. A and B are complementary to each other. If A =115°
29. -
then B is
A. Vague
A. 65°
B. Irregular
B. 110°
C. Adore
C. 20°
D. Amity
D. 290°
30.
Solution:
A. The art of tragedy
complementary angle 90O । But here, A =115°.
B. The art of history
So, it must be supplementary angle where the sum of two
C. Tribal art
angles will be 180O.
D. The art of story-telling

14
Collected By: Md. Raihan Ahmed
JOB INFORMATION FOR ALL STUDENTS (BCS & BANK)

B = (180O -115O) = 65O C. 9 times 37 C


37. If 5% more is gained by selling an article for Tk.350 than D. 12 times
selling it for Tk. 340, the cost of the article is: Solution:
A. Tk.180 38 B
Let, diameter = 4.
B. Tk.150
So area = π(4/2)2 = 4π.
C. Tk. 200 39 B
Now diameter = (4 × 3) = 12
D. Tk.250
so area = π(12/2)2 = 36π
Solution:
So Times of area = 36π / 4π = 9 times
Selling price difference = (350 340) = 10tk 40 C

5% = 10tk. 41. If x = ya, y = zb and z = xc then the value of abc is


100% = 10*100/5 = 200tk. A. 1
41 A
38. In first 1000 natural numbers, how many integers exists B. 0
such that they have a remainder 4 when divided by 7 and a C. 0.5
remainder 9 when divided by 11? D. Infinity 42 B

A. 11 Solution:
B. 13 x = ya
43 B
C. 15 Or, x= zab
D. 17 Or, x= x abc
Solution: Or, abc = 1 44 B

When Divided by 7, 42. Which of the following is the average of first five

A = 7x + 4 prime numbers?

So, numbers can be: 4, 11, 18, 25, 32, 39, 46, A. 4.5

Again, B. 5.6

when divided by 11, C. 7.5

A = 11y + 9 D. 8.6
Solution:

Here, 53 is common. First five prime numbers = 2,3,5,7, 11


Now, LCM of 7 & 11 is 77. Average = (2+3 +5+7+11)/5 = 28/5 = 5.6
So, the numbers pattern is : 77x + 53 43. Samir is twice as old as Babul was two years ago. If
Then, the difference between their ages is 2 years, how old is
77x ≤ 1000 Samir now?
Or, x≤12.2 A. 8 years
x can be 0, 1, 2, - 12 total 13 B. 6 years
39. If n 5 is an even integer, what is the next large C. 10 years
consecutive even integer ? D. 12 years
A. n 7 Solution:
B. n 3
C. n 4
-2
D. n-2
ATQ
Solution:
S-2 = 2(S-2-2)
n-5 is even,
Or, S-2 = 2S-8
So let n = 9 Option a) 9 7 = 2 Option b) 9- 3 =
Or, S= 6
6 Option c) 9-4 = 5 Option d) 9- 2 = 7 .: large
even is option b. 44. 300 grams of sugar solution has 40% sugar in it. How
much sugar should be added to make it 50% in the
40. When the diameter of a circle is tripled, the area of the
solution?
circle will be increased by
A. 40 grams
A. 3 times
B. 60 grams
B. 6 times
C. 80 grams

15
Collected By: Md. Raihan Ahmed
JOB INFORMATION FOR ALL STUDENTS (BCS & BANK)

D. 90 grams a2 + b2= 41 45 C
Solution: Or, (a + b)2 2ab = 41
300 × 40% + x = 50% (x + 300) Or, (a + b)2 2×20 = 41
Or, 120 + x = 0.5x + 150 Or, (a + b)2 = 81 46 B

Or, x 0.5x =150 120 Or, a + b = 9


Or, 0.5x = 30 Perimeter = 2 (a + b) = 2×9 = 18 47 A
Or, x = 30/0.5 = 60 gm. 49. Which of the following equations does not represents
45. A circular garden with diameter of 20 meters is surrounded a straight line?
by a walkway of width 1 meter. What is the area of the A. y = 2x + 3 48 D

walkway? B. y = 2x2 + 3
A. 41πm C. y(2 + x) = 3 49 B
B. 41πm D. y + x-3 = 7
C. 21πm2 Solution:
D. 21πm 50 B
x power
Solution:
Area of the garden without road = π(20m/2)2 = 100 πm2 50. If x-1/x = √3 then x+1/x = ? 51 D
So with road radius of the garden = (20/2 + 1)m = 11m A. 3√3
Area with road = π (11m)2 = 121πm2 B. √7
52 C
Area of the road = (121 πm2 100 πm2) = 21πm2 C. 2√3
46. A 240 m long train passed a pole in 24 seconds. How long D. 7
will it take to pass a 650 m long platform? Solution:
A. 65 sec Given,
B. 89 sec x-1/x = √3
C. 100 sec Or, (x-1/x)2 = (√3)2
D. 130 sec Or, (x + 1/x)2 -4.x.(1/x) = 3
Solution: Or, (x + 1/x)2 = 3 + 4 = 7
Or, x + 1/x = √7
The train has to go = (240 + 650) = 890 m. 51. The difference between two numbers is 5 and the
Time = 890/10 = 89 seconds difference between their squares is 65. What is the larger

47. A card is randomly drawn from a deck of 52 cards. What is number?

the probability of getting an Ace of King or Queen ? A. 13

A. 3/13 B. 11

B. 2/13 C.

C. 1/13 D. 9

D. 4/13 Solution:

Solution: Let the larger number is = a


Then, the other number is = a-5
There are 4 Ace, 4 kings, 4 queens. Total 52 cards So
ATQ,
probability = (4+ 4 + 4)/52 = 3/13
a2 (a-5)2 = 65
48. The diagonal of rectangle is √41 cm and its area is 20cm2,
or, a2 a2 + 10a 25 = 65
What is the perimeter of the rectangle?
or, a = 90/10 = 9
A. 16cm
52. Find the least number of six digits which is exactly
B. 17cm
divisible by 15, 21 and 28.
C. 20cm
A. 100480
D. 18cm
B. 100270
Solution:
C. 100380
Length = a, width = b
D. 100340
So, area ab = 20
Solution:
diagonal = √(a2 + b2) = √41
Least six-digit number is = 100000.

16
Collected By: Md. Raihan Ahmed
JOB INFORMATION FOR ALL STUDENTS (BCS & BANK)

Lcm of 15, 21, 28 is = 420 A. Thursday 53 C


Then, 100000/420 = quotient 238 and remainder = 40 B. Sunday
So, the number is = 100000 + 420 40 = 1000380 C. Monday
53. The sum of fourth and twelfth term of an Arithmetic D. Saturday 54 C

progression is 20. What is the sum of the first fifteen terms of Solution:
that arithmetic progression? After (7 X 9) = 63 days it is Monday. After 61 days is 55 A
A. 300 Saturday.
B. 120
57. The position of Bangladesh in the UN peace activities
C. 150 56 D
is -
D. 130
A. first
Solution: 57 B
B. second
ATQ, C. fifth
Nth term of AP = a + (n-1)d D. seventh
58 B
Sum of an AP = n/2(2a + (n-1)d) 58. The cultural Capital of SAARC in 2017 is ________ .
Sum of 4th and 12th term = a +(4-1)d + a + 11d = 2a +14d A. Paharpur
= 20 (given) B. Mahasthangarh 59 C
Sum of 1st 15 terms = 15/2( 2a + (15 1)d ) = 15/2 (2a + C. Nalanda
14d) = (15/2) *20 = 150 D. Taxila
60 B
54. The average of ten numbers is 7. What will be the new 59. The island Vashanchor is situated in _________ .
average if each of the numbers is multiplied by ? A. Bhola
A. 45 B. Patuakhali 61 B
B. 52 C. Noakhali
C. 56 D. Chittagong 62 C
D. 55 60. The main source of income of Brunei is
Solution: A. mines of gold
63 D
Average = 7 B. petroleum
Numbers are 4 times. C. Iron
So the new average = (7 x ) = 56 D. tourism 64 B
55. A bag contains 2 white balls, 3 black balls and 4 red balls. 61. The Bangladesh Cricketer who scored century for the

In how many ways can 3 balls be drawn from the bag; if at first time in ICC World Cup

least one ball is to be included in the draw? A. Mushfiqur Rahim

A. 64 B. Mahmudullah

B. 32 C. Shakib Al Hasan

C. 128 D. Tamim Iqbal

D. 256 62. One of the most powerful women in the world ranked

Solution: ost Powerful


Women in 2016 is
we have 3 choices
A. Tsai Ing-wan
All three black =3C3; [ 3C3
B. Aung San Suu Kyi
]
C. Sheikh Hasina
Two are black and one is non black = 3C3 X 6C1[
D. Angela Markel
]
63. Aurangabad Fort is situated at __________ .
One is black and two are non black = (3C1 x 6C2)
A. Chawkbazar of Dhaka
Total number of ways
B. Lahore of Pakistan
= 3C3 + (3C2 x 6C1) + (3C1 X 6C2) = 1 + 18 + 45 = 64
C. Delhi of India
D. Lalbagh of Dhaka
GENERAL KNOWLEDGE AND COMPUTER
64. Permanent settlement was abolished in Bengal in -
56. If it is Monday today, which day will come after 61 day of
A. 1950
it?
B. 1793

17
Collected By: Md. Raihan Ahmed
JOB INFORMATION FOR ALL STUDENTS (BCS & BANK)

C. 1946 B. Illustrator 65 A
D. 1796 C. Access
65. Bangladesh touched 7 percent GDP growth rate for the first D. Excel
time in fiscal year 74. Which is not a type of sorting algorithm? 66 D

A. 2015-2016 A. Bubble
B. 2016-2017 B. Selection 67 D
C. 2014-2015 C. Split
D. 2017-2018 D. Merge
66. First Lt. Governor of newly formed province of Eastern 75. 68 C

Bengal Assam in 1905 was __ stands for


A. Lord Curzon A. Natural 69 C
B. Lord Minto B. Neutral
C. Lord Hardinge C. Normal
D. Bamphylde Fuller D. Nano 70 D

67. Current position of Bangladesh in Human Development 76. Which is not an Object Oriented Programming
Index is __________ . Language?
71 C
A. 142nd A. C#
B. 141th B. C
C. 140th C. Java 72 B

D. 139th D. CH
68. The country ranks first in 2017 Global Climate Risk Index is 77. Which file extension indicates only graphics files? 73 D
A. TXT
A. China B. GIF
74 C
B. Bangladesh C. STK
C. Honduras D. DOC
D. India 78. Which protocol provides e-mail facility among 75 A
69. The position of Bangladesh in 2017 Index of Economic different hosts?
Freedom is A. TELNET
76 B
A. 120th B. SNTP
B. 128th C. FTP
C. 137th D. SMTP 77 B
D. 142nd 79. How many bits are used by Unicode to represent one
70. Bangladesh has become a member of IMF in character?
78 D
A. 1972 A. 8
B. 1991 B. 16
C. 1977 C. 32 79 B

D. 1974 D. 48
71. ans. b 80 B
A. Lives of Girls and Women 80. Which of the following is a language translator?
B. Dangling men A. Pearl
01 D
C. The Remains of the Day B. Assembler
D. The Golden Notebook C. Java Script
72. Not an artifact D. BCD
A. Coin
B. Copperplate
C. Manuscript
D. Inscription BANGLA
73. Which one is the spreadsheet analysis software? 1.
A. Latex A. , ,

18
Collected By: Md. Raihan Ahmed
JOB INFORMATION FOR ALL STUDENTS (BCS & BANK)

B. , , D. - 02 B
C. , , 11.
D. , , -
2. __________ 03 A

A.
A. B. 04 B
B. C.
C. D.
D. 12. 05 A

3. + =? A.
A. B. 06 D
B. + C.
C. + D.
D. + 07 C

4.
A. 13. If you can win his attention _________for you.
08 B
B. A. the so much better
C. B. the better so much
D. C. so much the better 09 C

5. D. so much for better


- - - 14. 10 A
A. A. optimistic
B. B. restless
11 B
C. C. hopeless
D. D. bloody
6. 15. 12 B
A. A. brave
B. B. exciting
13 C
C. C. cold
D. D. dull
7. 16. If Asylum: Shelter then 14 A
A. - A. Hospice: Exile
B. - B. Harbour; Concealment
15 B
C. C. stronghold; Defense
D. - D. Patisate : Display
. 17. Translate into English: 16 C

A.
B. A. He has no political axe to grind. 17 A
C. B. He has no political will to move on.
D. C. He has no political idol to follow.
18 B
9. - - D. He has no strong political stand to speak up.
A. 18.
B. - consensus. This is an example of 19 B
C. A. metaphor
D. B. metonym
10. C. euphemism
A. D. dysphemism
B. - 19. Choose the correct spelling
C. - A. Enterprener

19
Collected By: Md. Raihan Ahmed
JOB INFORMATION FOR ALL STUDENTS (BCS & BANK)

B. Entrepreneur 27. 20 B
C. Enterpreneur
D. Entrepreneure A. I was surprised to see he had been beaten black and
20. We were at a disadvantage _____ that we did not have a blue. 21 C

very good knowledge of the language the others were using B. It surprised me to see that he had been beaten black
A. by and blue. 22 A
B. In C. It was surprising that he had been beaten black and
C. with blue.
D. for D. That he had been beaten black and blue was 23 C

21. He escaped by surprising.


A. 28. Cha 24 A
B. the breadth of a hair asked a soft voice close behind me.
C. A. What I was doing there alone was asked by a soft
D. a breadth of a hair voice. 25 A

B. A soft voice told me as his son and asked whether I


Match words to their dictionary definitions (22-23): was alone. 26 A
22. The state of having contradictory or conflicting emotional C. A soft voice from my behind asked me if I was alone.
D. Addressing me as his son a soft voice asked if I was
27 B
A. ambivalence alone.
B. Exigency
C. reprisals MATHEMATICS 28 D
D. constraint 29. Shonghoti and Shouhardo Clubs consist of 200 and
23. 270 members respectively. If the total member of the two
29 C
A. disparate clubs is 420 then how many members belong to both
B. magnanimous clubs?
C. presentiment A. 30 30 D
D. incongruent B. 40
24. Think of one word only which can be used appropriately in C. 50
31 B
all three sentences. D. 60
i) The employment crisis is _______that it is affecting 25% Solution:
people. (200 + 270) 420 = 50
ii) ________torrential rain is rare in this part of the world. 30. The one-third of the complementary angle to 60ºis
A. 150º
_______but I disliked some of his mannerisms. B. 100º
A. such C. 40º
B. so D. 10º
C. high Solution:
D. much Complementary angle of 600 is 300 and one third of 300
25. He was a king who ruled his subject with a high hand. is ⅓ × 300 = 100
A. oppressively 31. The area of a rhombus is 96 sq. cm and the length of
B. kindly a one of the diagonals is 16 cm. The length of the other
C. conveniently diagonal is
D. sympathetically A. 18
26. B. 12
A. perplexed C. 9
B. clear up D. 6
C. explain Solution:
D. Enlighten Area of rhombus is 1/2×e×f [here, e,f= Diagonal]
1/2×16×f=96

20
Collected By: Md. Raihan Ahmed
JOB INFORMATION FOR ALL STUDENTS (BCS & BANK)

8×f=96 D. Infinity 32 B
f=96/8=12 Solution:
32. The ratio of two numbers is 3:4 and their sum is 630. The 〖x=y〗^a=z^ab=x^abc
smaller one of the two numbers is x^1=x^abc 33 C

A. 360 abc=1
B. 270 37. 1+ sinθ = x cosθ, then tanθ is 34 D
C. 180 A. (x2 + 1)/x
D. 120 B. (x2 1)/x
Solution: C. (x2 + 1)/2x 35 C

Let one number is =3X D. (x2 1)/2x


Another number is =4X Solution: 36 A
3x+4x=630 (1+ sinθ)/ cosθ = x
x=630/7=90 Or, 1/ cosθ + sinθ/ cosθ = x
smaller number is=3x=3×90=270 Or, secθ + tanθ = x -(i) 37 D

33. If 4^(2x+1) 32, then x=?


Sec2θ tan2θ = 1
A. 2
38 D
B. 3 Or, (Secθ + tanθ)( Secθ tanθ) = 1
Or, x(Secθ tanθ) = 1
C. 3/4
Or, Secθ tanθ = 1/x -(ii) 39 B
D. 4/3
Solution:
(i)-(ii)
4^(2x+1)=32 40 C
2 tanθ = x 1/x
2^(2(2x+1))=2^5
Or, tanθ = (x2 1)/2x
4x+2=5
38. The difference between two numbers is 5 and the
x=3/4
difference between their squares is 65. What is the larger
34. What will be the difference in taka between simple and
number?
compound Interest at 10% on a sum of Tk. 1000 after 4 years?
A. 13
A. 31.90
B. 11
B. 32.10
C. 8
C. 44.90
D. 9
D. 64.10
Solution:
Solution:
〖(x+5)〗^2-x^2=65
Simple interest = 1000×4×10/100=400
x^2+10x+25-x^2=65
Compound interest = 〖p(1+r)〗^n-p
10x=40
〖=1000×(1+10/100)〗^4-1000
x=4
=1464.1-1000
∴the larger number =4+5=9
=464.1
39. A train 240 m long passed a pole in 24 seconds. How
difference=464.1-400=64.1
long will it take to pass a platform 650 m long?
35. In a series of 6 consecutive odd numbers if 15 is the 6th
A. 65 s
number, what is the 4th number in the series?
B. 89 s
A. 7
C. 100 s
B. 9
D. 130 s
C. 11
Solution:
D. 13
240 m passes 24 s
Solution:
1 m passes 24/240 s
5/1^st 7/2^nd 9/3^rd 11/4^th 13/5^th 15/6^th
(240+650) = 890 m passes 24X890/240 s = 89 s
36. If x=ya;y=zb and z=xc y then abc is
40. What is the slope of the line perpendicular to the
A. 1
line Y = 5x +97
B. O
A. 5
C. 1/7
B. -5

21
Collected By: Md. Raihan Ahmed
JOB INFORMATION FOR ALL STUDENTS (BCS & BANK)

C. 1/5 A. 31 41 B
D. -1/5 B. 32
Solution: C. 39
In case of perpendicular line, D. 49 42 D

m1 X m2 = -1 46. A football team is to be consisted out of 14 boys. In


-5 X m2 = -1 how many ways the team can be chosen so that the 43 A
m2 = 1/5 owner of the ball is always in the team?
41. If y/ x = 3/7 and x + 2y = 13 then y is A. 135
A. 2 B. 143 44 B

B. 3 C. 169
C. 4 D. 129 45 C
D. 7 No Answer
Solution: Solution:
y/ x = 3/7 14-1c11-1 46

or, x = 7y/3 = 13 c 10 = 286


Again, 47. Which of the following can be arranged into an English
47 C
x + 2y = 13 word?
or, 7y/3 + 2y = 13 A. ANSLAT B.LSNIT C. OTATM D. WQRGS
or, y = 3 A. C 48 A

42. If 1-3x ≠ 4, then B. A


A x ≠ -2 C. A &d 49 C
B x ≤ -7 D. C&d
C x ≠ -1 48. October 1985 corresponds to Bangla year
50 C
D x ≤ -1 A. 1392
Solution: B. 1391
1-3x ≠ 4 C. 1394 51 A
Or, -3x ≠ 3 D. 1390
Or, 3x ≤ -3 49.
52 D
Or, x ≤ -1 A. voice
43. If a pole 6 m high casts a shadow 2√3 m long on the B. bald
ground, then the elevation of the sun is C. flame
A. 60º D. castle
B. 45º 50. What is the probability that an integer selected at
C. 30º random from those between 10 and 100 inclusive is a
D. 90º multiple of 5 or 9?
Solution: A. 27/89
tanθ = 6/2√3 B. 20/91
or, tanθ = tan60º C. 27/91
Or, θ = 60º D. 23/89
44. if a, b and c are the lengths of the three sides of a triangle, 51.
then which of the following is true? A. Adding 2 velars
A. a + b < c B. Drinking energy beverage
B. a b<c C. Eating sweet fruits
C. a + b = c D. Changing outfits
D. a +b = ≤ c 52. The sum of 3 consecutive Integers is less than 75.
Solution: What is the greatest possible value of the smallest one?
A. 16
45. The next number of the sequence is B. 19
4 3 9 3 19 C. 22

22
Collected By: Md. Raihan Ahmed
JOB INFORMATION FOR ALL STUDENTS (BCS & BANK)

D. 23 61. Awami Muslim League was founded in 53 B


A. Rose Garden
GENERAL KNOWLEDGE AND COMPUTER B. Rup Mchal Cinema Hall
53. Which of the following organization of World Bank is known C. Ahsan Manji 54 B

D. Madhur Canteen
A. IBRD 62. In 2018-19 Budget of Bangladesh, forecast of private 55 C
B. IDA Investment is
C. IFC A. 10.35% of GDP
D. MIGA B. 15 25% of GDP 56 A

54. During the Liberation War of Bangladesh, the President of C. 20.45% of GDP
USSR was D. 25.15% of GDP 57 A
A. Nikita Khrushchev 63.
B. leonid Brezhnev awarded the title Player of the Match?
C. Mikhail Gorbachev A. Rumana 58 C

D. Nikolai Podgorny B. Salma


55. Which of the following is the name of the bank established C. Jahan Ara
59 A
by BRICS? D. Panna
A. BRICS Development Bank. 64. Black Monday related to
B. Developing Bank A. Stock Market 60 B

C. New Development Bank B. Terrorism


D. Newly Developed Bank C. Environment 61 A
56. Which of the followings is the regulator of Capital Market in D.
Bangladesh? 65. Panmunjom Declaration is a peace treaty signed
62 D
A. BSEC between
B. DSE A. North Korea & South Korea
C. Bangladesh Bank B. USA and North Korea 63 A
D. ICB C. USA and Vietnam
57. Special Drawings Rights (SDR) is related to D. China and Japan
64 A
A. IMF 66. The official match ball used by FIFA in World Cup
B. IDB 2018
C. ADB A. Tiento 18 65 A
D. HSBC B. Telstar 18
58. How many global goals have been included in Sustainable C. Tricolore 18
66 B
Development Goals (SDGs)? D. Tango 18
A. 15 67. The slogan adopted by CEDAWE
B. 16 A. Women rights are human rights 67 B

C. 17 B. Equal rights for all


D. 18 C. let women enjoy equal rights 68 D
59. Which country does not along to G-7? D. Preserve women rights
A. Russia 68. The Asian team defeated a Latin American team for
69 C
B. Japan the first time in the history of FIFA World Cup
C. Germany A. Saudi Arabia
D. Italy B. South Korea
60. Budget deficit (% of GDP) stands at FY 2018-19 of C. UAE
Bangladesh is D. Japan
A. 3.9 69. Which device may be used for primary input of OCR?
B. 4.9 A. Keyboard
C. 5.9 B. Plotter
D. 6.3 C. Scanner

23
Collected By: Md. Raihan Ahmed
JOB INFORMATION FOR ALL STUDENTS (BCS & BANK)

D. Printer D. Bookmark 70 A
70. The protocol that provides infotainment is 79. URL stands for
A. FTP A. Universal Resource Locator
B. SMTP B. Uniform Resource Location 71 B

C. TELNET C. Uniform Router Locator


D. SNMP D. Unified Resource Locator 72 C
71. Fill in the gap: ____ make easier to find visited web pages 80. OTG cable is not related to
later A. Smart Phone
A. Placeholders B. Camcorder 73 D

B. Bookmarks C. DSL
C. URLS D. Processor 74 C
D. Add-ons
72. DVD stands for
A. Digital Video Disk 75 B

B. Digital Value Disk BANGLA


C. Digital Versatile Disk 1. Legal statement- 76 C
D. Data Video Disk A. -
73. In MS Word, which shortcut key is used for reviewing a B.
text? C. - 77 C

A. Ctrl+N D.
B. Ctrl+E 2 78 A
C. Ctrl+ S A.
D. Ctrl+Q B.
79 B
74. The device used for both input and output purposes C.
A. Stylus D.
B. Printer 3. 80 D
C. Touch screen A.
D. Mouse B.
01 B
75. Which of the Following is not a virus C.
A. worms D.
B. adware 4 - 02 A
C. Trojan horse A.
D. malware B. 03 D
76. You can include names and addresses from a source in a C.
MS-Word document automatically by using D.
A. slides 04 B
5.
B. table A.
C. mail merge B. 05 A
D. hyperlink C
77. Office LANs which are scattered geographically on large D
06 B
scale can be connected by the use of corporate 6.
A. CAN A.
B. WLAN B. 07 C
C. WAN C.
D. CLWN D.
78. In the address bar, first page of a Website is termed as 7.
A. Homepage
B. Index A.
C. Title page B.

24
Collected By: Md. Raihan Ahmed
JOB INFORMATION FOR ALL STUDENTS (BCS & BANK)

C. C. 08 D
D. D.
.
A. 09 B

B, 17. Choose the word closest to the meaning of the


C. , underlined word: 10 C
D. The Patient needed an anodyne for his strained nerves.
9. A. Alcohol
A. B. Opium 11 C

B. C. Medicine
C. D. Pain-killing balm 12 D
D. 18. Select the pair from the following options which is set
10. in opposition:
A. . A. Scholar Pedagogic 13 B

B. B. Humid Arid
C. C. Erroneous Faulty
14 B
D. D. Horrible Atrocious.
11. , ,
Choose the correct options to fill in the blanks (19-24): 15 B

? 19.
A. A. prominent 16 C
B. B. aggressive
C. C. weak
17 D
D. D. aware
12 20. This evidence _ with what one already knows.
A. - A. ties in 18 B
B. - B. sets in
C. C. gives in
19 B
D. D. goes down
13. 21.
. means that he had to _. 20 A
A. A. apologize
B. B. surrender
21 A
C. C. leave
D. D. eat his food
14. - ? 22. We should__for a rainy day. 22 C

A. A. put down
B. B. put into 23 B
C. C. put something by
D. D. put things off
24 C
15. 23.
A. A. in
B. B. at
C. C. of
D. D. on
16. He can hardly keep the wolf from the door. 24. The man who follows his nose actually follows his -.
A. passion
A. B. rationality
B, C. instinct

25
Collected By: Md. Raihan Ahmed
JOB INFORMATION FOR ALL STUDENTS (BCS & BANK)

D. memory D. 40% 25 C
25. ľ may be expressed by the following Interjection: 34. If x is an integer and y= 2x-8, what is the least
A. Ha! value of x for which y is less than 9?
B. Hush! A. -9 26 C

C. Bravo! B . -8
D. Hurrah ! C.-7 27 D
26. D. -6
A. Lethargic 35.
B. Prone to anger is 28 D

C. Prone to tear A. 34
D. Energetic B.36 29 D
27. A lame duck organization is one which needs C. 38
A. authority D. 40
B. staff 36. If x: y = 5:3, then (8x 5y) : (8x + 5y) = ? 30 A

C. power A. 5:11
D. support B. 6:5
31 C
28. The concluding part of a literary work is called C. 5:6
A. Epigraph D. 3:8
B. Epitaph 37. 32 C

C. Conclusion A. 529
D. Epilogue B. 462 33 C
29. People who believe anything told to them are called C. 629
A. Faithful D. 523
34 B
B. Credible 38. If A = {1, 2, 3, 4, 5}, then the number of proper
C. All-believing subsets of A is
D. Credulous A. 120 35 C
30. Choose the opposite of the following idiomatic expression B. 30
C. 31
36 A
A. To turn over a new leaf D. 32
39. How many terms of Arithmetic Progression (A.P.) 21,
37 C
D. To A. 10
31. B . 15
38 C
A. an administratoress C. 22
B . an administress D. 27
C. an administratrix 40. Which of the numbers below is not equivalent to 4%? 39 B

D. an administrer A. 1/25
32. He is such a Good Samaritan. It means . B. 4/100 40 C
A. He is very clever C. 0.40
B. He is a fool D. 0.04
41 D
C. He is a helpful person 41. After being dropped a certain ball always bounces
D. He is a bad man back to 2/5 of the height of its previous bounce. After the
first bounce it reaches a height of 125 inches. How high
MATHEMATICS (in inches) will it reach after its fourth bounce?
33. If w is 10% less than x, and y is 30% less than z, they wy is A. 20
what percent less than xz? B.8
A. 10% C .5
B. 20% . D. 3.2
C.37%

26
Collected By: Md. Raihan Ahmed
JOB INFORMATION FOR ALL STUDENTS (BCS & BANK)

42. How many integers from 1 to 1000 are divisible by 30 but A. increase by 1 42 A
not by 16? B. the same
A. 29 C. double
B .31 D. triple 43 B

C. 32 51. If a+2b = 6 and ab= 4 what is 2/a + 1/b?


D.38 A. 1/2 A
44
43. The slope of the line perpendicular to the line y = -5x+9 is B.1 C

C. 3/2
A. 5 D. 2 45 A

B. 5 52. The number of parallelograms that can be formed


C. 1/5 from a set of four parallel lines intersecting another set of 46 A
D. -1/5 three parallel lines is
44. If m and p are positive integers and (m + p) m is even, A. 6
which of the following must be true? B. 9 47 D

A. If m is odd, then p is odd C. 12


B. If m is odd, then p is even D. 18
48 B
C. If m is even, then p is even 53. There are 5 red and 3 black balls in a bag. Probability
D. If m is even, then p is odd of drawing a black ball is
45. A pole 6m high casts a shadow 2√3m long on the ground, A. 5/8 49 C

B. 1/2
A. 60° C. 3/8 50 B
B. 45° D. 1/4
C . 30° 54. Find the largest fraction from the following:
51 C
D. 90° A. -5/11
46. All possible three digit numbers are formed by 1, 2, 3. If B. -8/13
one number is chosen randomly, the probability that it would C.-7/19 52 D
be divisible by 111 is D.-15/97
A. 0 55. If x + 1/x=3, then x-1/x = ?
53 C
B. 2/9 A. √5
C. 1/3 B. √13
D. ¼ C . √7 54 D
47. If secθ + tanθ = x, then tan θ is D. 0
A. (x2 + 1)/x 56. The factors of x2 5x 6 are:
55 A
B. (x2 1)/x A. (x-6)(x + 1)
C. (x + 1)/2x B. (x+6)(x 1)
D. (x2 1)/2x C. (x 3)(x + 2) 56 A

48. The area of a triangle with sides 3 cm, 5 cm and 6 cm is D. (x 3)(x 2),
A. 2√3cm2 57 C
B. 2√14cm2 GENERAL KNOWLEDGE AND COMPUTER
C. 5√12cm2 57. Which of the following is not a programming
58 B
D. 4√14cm2 language?
49. The value of k, if (x 1) is a factor of 4×3 + 3×2 4x + k, A. Java
is B. Pascal
A. 1 C. MS-Excel
B. 2 D. C++
C. -3 58. Which disk is used to cold boot a PC?
D. 3 A. Setup disk
50. If the radius of cylinder is halved and height is doubled, B. System disk
then what will be the curved surface area? C. Diagnostic disk

27
Collected By: Md. Raihan Ahmed
JOB INFORMATION FOR ALL STUDENTS (BCS & BANK)

D. Program disk A. 1 year 59 B


59. A collection of unprocessed items is . B. 9 months
A. information C. 2 years
B. data D. 3 years 60 D

C. memory 68. Brand name YKK is related to


D. reports A. Zipper 61 A
60. Which of the following types of menu shows the further B. Cars
sub-choices? C. Cell phone
A. Reverse D. Motorcycle 62 D

B. Template 69. In which country Aztec civilization was originated?


C. Scrolled A. USA 63 A
D. Pull-down B. Greece
61. Which of the following is not a logical function used in Ms- C. Mexico
Excel? D. Egypt 64 C

A. ELSE 70. The Bangabandhu Satellite -1 was launched by


B. ROUND A. Falcon 9 Block
65 C
C. MIN B. Spacebar
D. RAND C. NASA
62. Which of the following error occurs when software tries to D. Ariane 5 66 C

access protected memory? 71. Total matches of the FIFA 2018 World Cup will be
A. Segmentation Fault A. 128 67 C
B. Display-time Error B. 64
C. I0 Error C. 32
68 A
D. Run-time Error D. 60
63. If you wish to extend the length of the network without 72. Scientific discovery is a kind of
having the signal degrade, you would use a A. National property 69 C
A. Repeater B. Personal property
B. Router C. Collective property
70 A
C. Gateway D. International property
D. Switch 73.
64. Ctrl, Shift and Alt are called __________ keys. mentioned in our constitution on the article 71 B
A. function A. 4
B. adjustment B. 7
72 D
C. modifier C. 15
D. application D. 17
65. Which institution in a federal system of government is 74. 73 B

? A. Baseball
A. Executive B. Tennis 74 C
B. Legislature C. Chess
C. Judiciary D. Football
75 B
D. None of above 75. Among the Bara Bhuiyans, ________was buried in the
66. According to the SIPRI fact sheet, the third most military premises of the University of Dhaka.
spender country in 2017 was- A. Isa Khan 76 B
A. USA B. Musa Khan
B. China C. Mahmud Khan
C. Saudi Arabia D. Alawol Khan
D. Russia 76. The highest exporter of natural gas of the Middle East
67. The term of a non-permanent member of the UN Security is
Council is A. Saudi Arabia

28
Collected By: Md. Raihan Ahmed
JOB INFORMATION FOR ALL STUDENTS (BCS & BANK)

B. Qatar 5. ) , ) 77 A
C. Kuwait , ) , ) ,
D. United Arab Emirates )
77. In July 2018, the so-called 78 D

A. Shanghai A.
B. Stockholm B. 79 A
C. Saare C.
D. Sofia D.
78. The winner of the recent WTA tennis open tournament is 6. 80 D

A. Elina Svitolina A.
B. Maria Sharapova B. 01
C. Serena Williams C.
D. Caroline Wozniacki D.
79. The official motto of the 2018 Commonwealth Games was 7. Alimony 02 B

A.
B.
03 D
C.
D.
80. At Cannes Film Festival 2018, the Best Actor honour went . - 04 D

to A.
A. Hirokazu Kore-eda B. 05 B
B. Alice Rohrwacher C.
C. Lucas Dhont D.
06 B
D. Marcello Fonte 9.
A. , ,
B. , , 07 C
C. , ,
BANGLA D. , ,
08 C
1. : A, C, D
A. 10. -
B. A. - 09

C. B.
D. C. -
10 D
: D.
2. 11. -
A. , , 11 A
A. , ,
B. , , B. , ,
C. , , C. , , 12 D
D. , , D. , ,
3. 12.
13 C
A. A.
B. B.
C. C.
D. D.
4. 13.
A. A.
B. B.
C. C.
D. D.

29
Collected By: Md. Raihan Ahmed
JOB INFORMATION FOR ALL STUDENTS (BCS & BANK)

14. B. Magician 14 A
| C. Imposter
D. Rogue
A. 23. Arrange the words given below in a meaningful 15 C

B. sequence:
C. 1. Presentation 2.Recommendation 3. Arrival 4. Discussion 16 C
D. 5. Introduction
15. , A. 5,3,4,1,2
B. 3,5.4.2.1 17 A

A. C. 3,5,1,4,2
B. D. 5,3,1,2,4 18 C
C. 24. One who travels from place to place is called -
D. A. Itinerant
16. B. Mendicant 19 D

A. C. Journeyman
B. D. Tramp
20 B
C. 25.
D.
17. In the new place he had to earn by the sweat of his brow, A. Example is more than precept. 21 D

The bold-faced part stands for ________ . B. Example is preferable than precept.
A. Very hard work C. Example is better than precept. 22 C
B. Frown of his brow D. Example is acceptable than precept.
C. Seriously 26. An example of a plural form is __________
23 C
D. Idly A. News
18. Fill in the gap: ______ by nature, Jones spoke very little even B. Axis
to his own family members. C. Cattle 24 A
A. Garrulous D. None
B. Equivocal 27. The cor
25 C
C. Taciturn A. Neither you nor he is to blame.
D. Arrogant B. Neither you nor he are to blame.
19. - C. Neither you nor he has to blame. 26 C
A. Prefer to take juice D. None of the above.
B. Boil 28.
27 A
C. Make a stew A. Communication
D. Suffer for his own act B. Calculation
20. Antonym of PRECARIOUS is ~ C. Company Act 28 A

A. Dangerous D. Conjuration
B. Safe 29. 29 C
C. Cautious A. Diamond so.
D. Easy B. Diamond is so.
30 B
21. Complete the sentence: Despite his best efforts to conceal C. So is diamond.
his anger people came to know that ______________ . D. So diamond is.
A. He failed to give us an impression of his agony 30. 31 C
B. He succeeded in obscuring his emotions. A. Because
C. He was pleased. B. Despite
D. He was annoyed. C. Instead
22. D. Account
31.
A. Liar A. That day

30
Collected By: Md. Raihan Ahmed
JOB INFORMATION FOR ALL STUDENTS (BCS & BANK)

B. The previous date 40. 32 A


C. The day before
D. The past day heritage?
32. Correct the tag part: He suggested that we should be A. UNO 33 A

B. UNESCO
A. C. SAARC 34 B
B. Is it D. UNICEF
C. 41. What will be the difference between simple and
D. None of above compound interest at 10% on a sum of Tk. 1000 after 4 35 B

years?
MATHEMATICS , GENERAL KNOWLEDGE AND COMPUTER A. Tk. 31.90 36 D
33. The book Padma, Meghna, Jamuna is B. Tk. 32.10
A. Political novel C. Tk. 44.90
B. Poetical work D. Tk. 64.10 37 B

C. Autobiographical novel Solution:


D. Romantic novel Diff = 1000(1+10/100)4 1000 1000×4×10/100 38 A
34. Recently Bangladesh has given recognition to - = 1410.10 -1000 -400
A. Albania = 64.10
B. Kosovo 39 A
42. x=ya, y=zb, z=xc then abc is __________
C. Catalonia
A. 1
D. Palestine 40 B
B. O
35. The first Chinese traveler who came in Bangladesh is -
C. 0.5
A. It-Sing
D. Infinity 41 D
B. Fa Hien
Solution:
C. Huen Tsang
Given,
D. Dzon Dong 42 A
x=ya
36. The length of the irregular deltaic coastline of Bangladesh is
Or, x=( zb)a
about -
Or, x=(xc)ab 43 B
A. 750 Km
Or, x1 = xabc
B. 700 Km
Or, abc = 1
C. 650 Km 44 D
43. If a side of a equilateral triangle is 4 cm, its height is _
D. 600 Km
A. √3
37. The first international organization wherein Bangladesh
B. 2√3
received membership is_
C. 16√3
A. FAO
D. 32√3
B. Commonwealth
Solution:
C. OIC
D. NAM ½ of a side = 4/2 = 2
38. Then,
A. Carles Puigdemont Height = √(42 22)
B. Paul Bekaert = √12
C. Charles Michel =2√3
D. Theo Francken 81
44. log√3 =?
39. A. 9
to ________ B. 7
A. Wheat C. 6
B. Paddy D. 8
C. Cotton Solution:
D. Jute log√381 = log√33^4 = log√3√3^8 = 8 log√3√3 = 8

31
Collected By: Md. Raihan Ahmed
JOB INFORMATION FOR ALL STUDENTS (BCS & BANK)

45. If x 1/x=-√3, then x4+1/x4 =? Or, 2x + 24 = 40 + x 45 A


A. 23 Or, x= 16
B. 27 49. A machine bought for Tk. 3200 is depreciated at 25%
C. 3 using the reducing balance method. After two years its 46 D

D. 9 remaining book value is _________


Solution: A. Tk. 1600 47 A
x4+1/x4 = (x2)2 + (1/(x2)2 B. Tk. 1800
= (x2 + 1/x2)2 2. x2.1/x2 C. Tk. 2400
={(x 1/x)2 + 2.x.1/x}2 2 D. Tk. 2850 48 D

={(-√3)2 + 2}2 2 Solution:


=(3+2)2 2 49 B
Depreciation after 1st year = 3200×25/100 = 800
=52 2
Remaining = 3200 800 = 2400
=23 nd
Depreciation after 2 year = 2400×25/100 = 600
46. If x is 30% greater than y, what percent of y is x? 50 C
Remaining = 2400 600 = 1800
A. 70
B. 77 50.
51 B
C. 120 A. Nominal Ledger
D. 130 B. General Ledger
Solution: C. Purchase Ledger 52 C

x = y + 30% of y D. Sales Ledger


x = 130y/100 51. A provision for doubtful debts is created _________
53 C
Let A. When debtors become bankrupt
a% of y = x B. To provide for possible bad debts
C. When debtors cease to be in business 54 B
ay/100 = 130y/100
Or, a = 130 D. To write off bad debts
47. If sin X = 1, then tan X/2=? 52. What should happen if the balance on a suspense
A. 1 account is of a material amount?
B. 0 A. Should be written off to the balance sheet period
C. √2 B. Carry forward the balance to the next
D. Infinity C. Find errors before publishing the final accounts
Solution: D. Write it off to profit and loss account
sin X = 1 53. Given opening capital of Tk. 16500, closing capital Tk.
sin X = sin 900 11350, drawings were Tk. 3300, and further investments
X = 900 are Tk. 40000, then ________
Now, A. Profit Tk. 41850
tan X/2= tan 900/2 = tan450 = 1 B. Loss Tk. 41000
48. C. Loss Tk. 41850
D. Profit Tk. 41000
Solution:
A. 12 Let,
B. 24 Profit/loss = x
C. 18 ATQ,
D. 16 16500-3300+40000+x = 11350
Solution: Or, x= -41850(Loss)
Let, 54. In a limited company which of the following is shown
x years from today Aziz will be 12 + x in the appropriation account?
His father will be 40 + x A. Debenture interest
ATQ, B. Proposed dividend
2(12 + x) = 40 + x C.

32
Collected By: Md. Raihan Ahmed
JOB INFORMATION FOR ALL STUDENTS (BCS & BANK)

D. A. Controller and Auditor 55 B


55. What type of asset is trademark? B. Comptroller and Auditor
A. Fixed C. Central Auditor
B. Intangible D. Chief Auditor 56 A

C. Fictitious 62. Repairs to machinery have been debited to machinery


D. Current account. The correcting entry is _________ 57
56. If stock Tk. 30000, debtors Tk. 45000, cash Tk. 15000, and A. Machinery debit, Repairs credit
creditors Tk. 30000, then current ratio is _______ B. Repairs debit, Machinery credit
A. 3:1 C. Repairs debit, Cash credit 58 B

B. 2:1 D. Machinery debit, Cash credit


C. 3.5:1 63. Financial statement comprises of ______ components. 59 B
D. 2.5:1 A. Three
Solution: B. Four
Current Ratio = CA/CL C. Five 60 D

(Stock + debtors + cash) / creditors D. Six


=(30000+45000+15000)/30000 64. Tax GDP ratio in Bangladesh is
61 B
=90000/30000 A. 19%
=3:1 B. 30%
57 Annual depreciation of a long term asset is charged for C. 10% 62 B

which accounting principle? D. 5%


A. Full disclosure 65. The highest personal income tax in Bangladesh is 63 C
B. Materiality A. 20%
C. Going concern B. 25%
64
D. Money measurement C. 30%
No Answer. Matching Principle D. 40%
58. If beginning stock Tk. 60000, closing stock Tk. 70000, cost 66. Who are the users of external audit? 65 C
of goods sold Tk. 660000, and sales Tk. 990000, then number A. Managers
B. Employees
66 D
A. 50 days C. Internal auditors
B. 39 days D. Share holders
C. 64 days 67. Independence of an auditor means Not influenced 67 B
D. 10 days
Solution: A. ICAB
68 A
Number of days = (closing stock/ goods sold) × 365 B. Management
days C. Government
D. The leader 69 A
= (70000/660000) × 365 days
=38.71 days 68. Our value added tax is regulated by the VAT Act of
= 39 days A. 1991 70 D
B. 1989
59. Unsecured revenue is ________
C. 1993
A. Asset
D. 1984
B. Liability
69. Audit cost of a business dependent on .
C. Equity
A. The size of the business
D. Tax
B. ICAB regulations
60. Bangladesh follows accounting standard issued by _________
C. The size of transactions audited
A. FASB
D. The time when audit takes place
B. РСАОВ
70. Nominal interest rate is 15% and corporate income
C. INTOSAI
tax rate is 30% then the effective interest rate is
D. IASB
A. 11.5%
61. In the abbreviation OCAG, CA stands for __________

33
Collected By: Md. Raihan Ahmed
JOB INFORMATION FOR ALL STUDENTS (BCS & BANK)

B. 11% D. It is capitalized 71 D
C. 10.5% 77. If the profit margin is 25%, what is mark up?
D. 4.5% A. 20%
Solution: B. 25% 72 B

15×30% = 4.5% C. 30%


71. D. 33.33% 73 D
Solution:
A. 25 days
Profit = 25
B. 20 days 74 D
Cost = 100-25=75
C. 8 days
Mark up % = (25/75)% = 33.33%
D. 10 days 75 C
78. Direct material is Tk. 80000, direct labour is Tk. 20000
72.
and overhead cost is Tk. 30000. what is conversion cost in
A. Bureau of Revenue
Tk.? 76 C
B. Board of Revenue
A. 160000
C. Bank Revenue
B. 150000
D. Business Record
77 D
C. 140000
73. Which one is deduced from net income in the calculation of
D. 130000
operating cash flow
Solution: 78 D
A. Depreciation
B. Decrease in current asset 80000+20000+30000= 130000
C. Increase in current asset 79 B
79. Mr. X has Sales Tk. 500000 on which VAT is included.
D. Gain on sale of investment
74.
A. 75000 80 B
A. Writing error
B. 65217
B. Supplementary error
C. 61218
C. Clerical error
D. 52354
D. Error of principle
Solution:
75.
500000- 500000/1.15= 65217
equity.
A. Profit 80. There is a case against a company. If the case is
B. Increase successful, it will result in claim Tk. 500000. There is a
C. Decrease high chance of happening the result. The financial
D. No change statement should record
76. Revenue is recognized when A. Contingent liability
A. Cash is received B. Liability
B. Profit is ensured C. Contingent asset
C. It is earned D. Expense

34
Collected By: Md. Raihan Ahmed
JOB INFORMATION FOR ALL STUDENTS (BCS & BANK)

35
Collected By: Md. Raihan Ahmed
JOB INFORMATION FOR ALL STUDENTS (BCS & BANK)

36
Collected By: Md. Raihan Ahmed
JOB INFORMATION FOR ALL STUDENTS (BCS & BANK)

37
Collected By: Md. Raihan Ahmed
JOB INFORMATION FOR ALL STUDENTS (BCS & BANK)

38
Collected By: Md. Raihan Ahmed
JOB INFORMATION FOR ALL STUDENTS (BCS & BANK)

39
Collected By: Md. Raihan Ahmed
JOB INFORMATION FOR ALL STUDENTS (BCS & BANK)

40
Collected By: Md. Raihan Ahmed
JOB INFORMATION FOR ALL STUDENTS (BCS & BANK)

41
Collected By: Md. Raihan Ahmed
JOB INFORMATION FOR ALL STUDENTS (BCS & BANK)

42
Collected By: Md. Raihan Ahmed
C. 01 A
D.
BANGLA 10.
A. 02 D
1.
A. B.
B. C. 03 A
C. D.
D. 11. Insomnia
A. 04 B
2.
A. B.
B. C. 05 A
C. D. ,
D. 12. .
06 C
3.
A. A.
B. B.
07 A
C. C.
D. D.
4. 13. , 08 D

A.
B. A. 09 A
C. B.
D. C.
10 A
5. D.
A. 14.
B. A. 11 C
C. B.
D. C.
12 C
Ans: D.
6. 15. A golden key can open any door
A. A. 13 C
B. B.
C. C.
14 B
D. D.
7. 16.
A. 15 A

A. B.
B. C. 16 B
C. D.
D.
17 C
8. ENGLISH
A. 17. Select the pair in the following options which is set in
B. opposition:
C. A. Doggish-Canine
D. B. Manly-Virille
9. C. Divine-Infernal
A. D. Earthly-Terrestrial
B. |

43
Collected By: Md. Raihan Ahmed
JOB INFORMATION FOR ALL STUDENTS (BCS & BANK)

18. In the time of danger, we should try to take the bull by the 26. This method is not wrong; but there may be a more 18 B
horns means _____ method than this.
A. Surmount it A. suitable
B. Deal the situation decisively B. effective 19 A

C. Overcome it C. right
D. Control ourselves D. working 20 B
19. The art of cultivating and managing gardens. is called 27.
A. Horticulture sentence:
B. Gardening bizarre that they created 21 A

C. Trimming
D. Beautification A. beautiful 22 A
20. A gentleman should be true _____his words. B. ugly
A. from C. dull
B. to D. peculiar 23 B

C. in 28. He succeeded by dint _____ his hard labour.


D. on A. of
24 D
21. A fourteen-lined poem is called a_____ B. by
A. sonnet C. for
B. epilogue D. because 25 A

C. ballad 29.
D. lyric A. He was said to be a good candidate 26 B
22. A disease affecting many persons at the same time and B. He was considered to be a good candidate
place is called _____ C. It was said by them that he is to be a good candidate
27 D
A. epidemic D. He was believed by them as a good candidate
B. infection 30.
C. sickness 28 A
D. injury A. The Captain commands the company to move
forwards
29 A
Choose the word/ phrase from the list which is nearest in B. The Captain Commanded to move the company
meaning to the boldfaced parts of the sentences (23-24) forward.
23. His letter fueled my doubts C. The Captain instructed the company for a forward 30 A
A. Ended move.
B. Increased D. The Captain requests the company to move
31 C
C. Caused forwards.
D. Reaffirmed ans. The Captain commands the company to move
24. The 19th century was the hey-day of the Renaissance in forwards 32 C

Bengal. 31.
A. days of decline
B. peak-point A.
C. period of stagnation B.
D. time of up-rise C.
25. Choose the correct alternative from the options to fill in the D.
blanks: 32.
is to ______ A. completed
A. make progress B. maintained
B. make little progress C. celebrated
C. progress towards collision D. enjoyed
D. proceed to travel
ans. make progress

44
Collected By: Md. Raihan Ahmed
JOB INFORMATION FOR ALL STUDENTS (BCS & BANK)

MATHEMATICS ( × ) + = 33 A
2 2
33. If sinA + sin A = 1, then the value of the expression cos A
+ cos4A is =? 37. A median of a triangle divides it into two
A. 1 A. A congruent triangles 34 A

B. ½ B. triangles of equal area


C. 2 C. isosceles triangles. 35 B
D. 3 D. right triangles
Solution: 38. Which of the following angle can be constructed with
Given that, the help of a ruler and a pair of compasses? 36 A

sinA + sin2 A=1 A.35°


Or, sin A = 1 cos2A B. 40° 37 B
2
Or, sin A = cos A C. 37.5°
Or, sin2 A = cos4 A D. 47.5°
Or, 1 2
cos A = cos A 4
39. If a, b and c are the lengths of the three sides of a 38 C
4 2
Or, cos A + cos A = 1 triangle, then which of the following is true?
34. A pole 6 m high casts a shadow 2√3m long on the ground, A. a + b < c
39 B
elevation is? B. a b<c
A.60° C. a + b =
B. 45° D. a + b ≥ c 40 B

C. 30° Solution:
D. 90° 41 C
Solution:
tanϴ = / ,
Or, tanϴ = 6/2√3
Or, tanϴ = 2×√3×√3/2√3 = √3 ,
Or, tanϴ = tan 60° b a b<c
Or ϴ = 60°
35. If a + 1, 2a+1, 4a l are in Arithmetic Progression, then 40. Which line is parallel to y = x 2?
value of a? A. y = 2x+1
A. 1 B. 2y = 2x 6
B. 2 C. 2y = x+7
C. 3 D. y= 3x+1
D. 4 Solution:
Solution: y = x-2
Since, y = mx + c
(2a + 1) (a + 1) = (4a 1) (2a+1) Here slope = 1
Or, 2a + 1-a-1 = 4a -1- 2a 1
Or, a = 2a-2
Or, a = 2. 2x = 2y 6
36. Suppose today is Friday. what day of the week will it be 65 Or, x = y -3
days? Or, y= x+ 3
A. Saturday So, Here slope = 1
B. Monday 41. The area of a triangle with sides 3 cm, 5 cm and 6 cm
C. Tuesday is?
D. Friday A. 2√3 cm2
Solution: B. 4√14 cm2
C. 2√14 cm2
D. 2√5 cm2
Solution:

45
Collected By: Md. Raihan Ahmed
JOB INFORMATION FOR ALL STUDENTS (BCS & BANK)

Half perimeter, b2 = 4ac 42 A


s=(3+5+6)/2 = 7 Or, b2 = 4×9×81
Area = √{7(7-3)(7-5)(7-6)} = √(7×4×2×1) = 2√14 Or b = ± 54
42. The pair of co-prime numbers is ___? 47. If secθ + tanθ = x, then tanθ is ____ 43 D

A. 2, 3 A. (x2+1)/x
B. 2, 4 B. (x2-1)/x 44 C
C. 2, 6 C. (x2 +1)/2x
D. 2, 110 D. (x2-1)/2x
43. The value of k, if (x 1) is a factor of 4x3 + 3x2 4x + k, Solution: 45 D

is____ We know that,


A. 1 sec2θ tan2θ = 1 46 D
B. 2 Or, (secθ + tanθ)(secθ tanθ) =1
C. 3 Or, x (secθ tanθ) = 1
D. -3 Or, secθ tanθ = 1/x (i) 47 D

Solution: again, seco + tan0 = x (ii)


x-1 , x=1 0 From (ii)-(i)
48 D
2tanθ = (x2 1)/x
4x3 + 3x2 4x + k = 0 Or, 2tanθ= (x2 1)/x
Or, 4 (1) + 3 (1)3 2
4.1 +k = 0 Or, tanθ = (x 2
1)/2x 49 C

Or, 4 + 3- 4+k = 0 48. Consider that w + x = 4, x + y = 25 and y + w =


Or, k = -3 15, Then the average of w, x, y is ____ 50 B
44. There are 5 red and 3 black balls in a bag, probability of A. 3
drawing a black ball is ___ B. 4
A. 5/8 C. 5
B. ½ D. 6
C. 3/8 Solution:
D. ¼ w+ x + x + y + y +w = -4 + 25 + 15
Solution: Or, 2w+ 2x + 2y = 36
Total balls = 5 + 3 = 8 Or, 2 (w + x + y) = 36
and black ball = 3 Or, w + x + y =18
So, probability of getting black ball is = 3/8 Or (w + x + y)/3 = 18/3 = 6
45. The total surface area of a hemisphere of radius r is ___ So, average of w, x, y = 6
2
A. 4π 49. What is the original price of a Y-shirt, if the sale price
B. πr2 after 15% discount is 272 ?
2
C. 2πr A. 300
D. 3πr2 B. 280
Solution: C. 320
Total surface area of sphere = 4πr2 D. 314
2
but hemisphere surface is half of sphere surface = 2πr + Solution:
2
surface area of a circle = πr ATQ,
So, total surface area of hemisphere = 2πr2+πr2= 3πr2 85% = 272
2
46. The roots of the equation 9x bx + 81 = 0 will be equal, So, 1% = 272/85
if the value of b is 100% = (272 × 100)/85 = 320
A. ±9 50. Tk. 500 is deposited in a savings account which pays
B. ±18 7% annual interest compounded semi-annually. To the
C. ±27 nearest Taka, how much is in the account at the end of
D. ±54 the year ?
Solution: A. 542
When roots be equal, B. 536

46
Collected By: Md. Raihan Ahmed
JOB INFORMATION FOR ALL STUDENTS (BCS & BANK)

C. 512 if, y = 100 then x = 100 + 30 = 130 51 C


D. 524 then, x is 130% of y
Solution: 55. The lengths of two sides of a right-angled triangle are
I = P(1+r)n 13 cm and 5 cm respectively. The length of the third side 52 C

= 500{1 + 7/(2×100)}2 is _____


= 500× (1 + 0.035)2 A. 13 53 A
2
= 500×(1.035) B. 17
= 535.6125 C. 11
= 536(near about) D. 12 54 D

51. If logx ¼ = -2, the x = ? Solution:


A. -1/2 c2 = a 2 + b 2 55 D
B. ½ Or, 132 = 52 + b2
C. 2 Or, 169 = 25 + b2
Or, b2 = 144 56 B
D. 3
Solution: Or, b = 12
logx ¼ = -2 56. The present age of Habib and Shikha are in the ratio
57 B
Or, x -2
=¼ of 6 : 4. Five years ago their ages were in the ratio of 5:
Or, x-2 = 2-2 3, How old is Habib now?
A. 24 58 B
Or, x = 2
52. If 5% is 30% gained by selling an article for BDT 350 than B. 30
selling it for BDT 340, the cost of the article is C. 36 59 B
A. BDT 180 D. 42
B. BDT 150 Solution:
C. BDT 200 Let,
D. BDT 250
Solution:
Selling price difference = 350 340 = 10 tk. ARQ,
And % difference is = 5% (6x-5)/(4x-5) = 5/3
5% of cost = 10tk Or, 18x 15 = 20x 25
1% of cost = 10/5 tk. Or, 2x = 10
100% of cost = (10×100)/5 = 200tk. Or, x = 5
53. If x = ya, y = zb and z = xc then the value of abc is
A. 1
B. 0 GENERAL KNOWLEDGE AND COMPUTER
C. 5 57. Which of the following is a programming language?
D. infinity A. Lotus
Solution: B. Pascal
Given, C. Ms-Excel
x = ya D. Netscape
Or, x = (z ) b a 58.
Or, x = (x ) c ab A. Setup disk
Or, (x) abc
=x 1 B. System disk
Or, abc = 1 C. Diagnostic disk
54. If x is 30% greater than y, what percent of y is x? D. Program disk
A. 70 59. A collection of unprocessed items is -?
B. 77 A. information
C. 120 B. data
D. 130 C. memory
Solution: D. reports

47
Collected By: Md. Raihan Ahmed
JOB INFORMATION FOR ALL STUDENTS (BCS & BANK)

60. The amount of vertical space between lines of text in a A. Iran 60 C


document is called B. Nigeria
A. double-space C. Brazil
B. indentation D. Norway 61 D

C. single-space 69. The first player to score 10,000 runs in T20 cricket is
D. crop 62 D
61. In excel which of the following symbols is used before a A. AB de Villiers
numeric value so that it can be treated as labeled value? B. Chris Gayle
A. C. Virat Kohli 63 B

B. = (equal) D. Tamim Iqbal


C. _ (underscore) 70. The author of the book Unstoppable: My Life So Far 64 A
D. is
62. Which of the following types of menu shows the further A. Mary Kom
sub-choices B. Maria Sharapova 65 A

A. Reverse C. Malala Yousufzai


B. Template D. Martina Navratilova
66 A
C. Scrolled 71.
D. Pull down A. Cristiano Ronaldo
63. Trackball is an example of a/an: B. Gareth Bale 67 A

A. Programming device C. Lionel Messid


B. Pointing device D. Neymar 68 B
C. Output device 72. The name of the first Robot that had given citizenship
D. Software device by Saudi Arabia is
69 B
64. If you wish to extend the length of the network without A. Rebeka
having the signal degrade, you would use a B. Anita
A. Repeater C. Sophia 70 B
B. Router D. Mariah
C. Gateway 73. The war strategy of Muktibahini is known as -
71 A
D. Switch A. Mujibnagar strategy
65. The main function of the Financial Action Task Force (FATF) B. Rajarbag strategy
is to combat - C. Teliapara strategy 72 C
A. money laundering D. Muktifouz strategy
B. corruption 74. The number of lines of our National Anthem played at
73 C
C. double taxation any national ceremony is first
D. black money A. 10 lines
66. The agenda 2063 is a strategic framework for the socio- B. 4 lines 74 B

economic transformation of ~ C. 6 lines


A. Africa D. 5 lines 75 B
B. Asia 75.
C. South America 2017-18 is
76 D
D. North America A. 3 lac 80 crore
67. Recently the country withdrew from Un refugee programme B. 4 lac 266 crore
is C. 4 lac 40 crore
A. Tanzania D. 5 lac 600 crore
B. Myanmar 76. The number of common rivers between Bangladesh
C. Syria and India is -
D. USA A. 50
68. The country, pioneer in launching the FM radio B. 52
broadcasting is - C. 53

48
Collected By: Md. Raihan Ahmed
JOB INFORMATION FOR ALL STUDENTS (BCS & BANK)

D. 54 A. 77 B
77. The national Jute day is on B.
A. 5th March C.
B. 6th March 78 C
D.
C. 5th April
6. -
D. 6th April 79 A
A.
78. Teletalk recently launched a promotional offer for woman is
B.
A. Bohnnishika
C. 80 A
B. Anannya
D. ।
C. Aporajita
D. Shampurna 7. - -
01 D
79. The name of the ancient locality of Bengai is A.

A. Gaur B.
B. Taralipi C. 02 C

C. Anga D. ।
D. Koshala 8. - 03 B
80. The name of the organization who recently invited the A. ।
three Nobel (peace) laureates in Bangladesh is - B.
A. Naripaksha 04 A
C.
B. BRAC
D. ।
C. ASA 05 B
9. - -
D. Nije Kori
A.
B. 06 D
C.
D. 07 D
BANGLA
1. - 10.

A. A. A
08
B. D
B.
C. C.
09 C
D. D.

2. - 11. -
A. 10 B
A.
B. B.

C. C. 11 C
D. D.

3. - - 12. - - ! -
12 D
A. ?

B. A.
13 B
C. B.

D. C.

4. - D.

A. 13. -

B. A.

C. B.

D. C.

5. - D.

49
Collected By: Md. Raihan Ahmed
JOB INFORMATION FOR ALL STUDENTS (BCS & BANK)

14. - D. species 14 C
A. 23. Which is not an English Prefix
B. A. er-
15 B
C. B. im-
D. C. un-
15. - D. in 16 C

A. 24.
B. sentence is 17 C
C. A. A child likes none but sweets.
D. B. A child likes nothing but sweets.
18 A
16. - - C. A child likes not more sweets.
A. - D. A child likes but sweets.
B. - 25. 19 D

C. A. No one can do it.


D. - B. We cannot do it. 20 A
C. It is impossible to do it.
ENGLISH D. Most people cannot do it.
21 C
17. Correctly Spelt word is 26. The indirect narration
A. Awetism is
B. Auotism A. He said that he can do the work. 22 A

C. Autism B. He said that I could do the work.


D. Auetism C. He said that I will do the work. 23 A
18. D. He said that he could do the work.
A. Nadir 27. I can give you no assurance____help 24 B
B. Peak A. for
C. Apex B. of
25 A
D. Zenith C. about
19. D. to
A. Express 26 D
28. The principal made an inquiry ____ the case.
B. Live A. at
C. Wealthy B. in 27 B
D. Consider C. of
20. D. into 28 D
A. I may not go out this evening. 29. ____do you spell your surname?
B. I should not go out this evening. A. when
29 B
C. I might go out this evening. B. how
D. I shall not go out this evening. C. where
21. 30 C
D. who
A. Politicial 30.
B. Politically A. Fotunately 31 A
C. Politicize B. Fortunate
D. Policy C. Fortune
22. The plural of D. Fortunating
A. spectra 31.
B. spikes A. optative sentence
C. spectrums

50
Collected By: Md. Raihan Ahmed
JOB INFORMATION FOR ALL STUDENTS (BCS & BANK)

B. exclamatory sentence 37. 32 A


C. assertive sentence
D. None of these
33 A
32. The Correct Translation - A. 12
A. A bad workman quarrels with his tools. B. 24
B. A bad dancere coplains about the stage. C. 18 34 D

C. Pride goes before destruction D. 16


D. The wearer knows best where the shoe pinches. Solution: 35 A

MATHEMATICS So,
36 C
33. The lengths of two sides of a triangle are 7 cm and 4 cm 2(12+x) = 40+x
respectively. The length of the third side is? Or, 24+2x = 40+x So, x = 16
A. greater than 3cm 38. If 61% of Bangladeshi people like cafe, and 74% like 37 D

B. less than 3cm tea, how many like both?


C. equal to 3cm A. 13% 38 D
D. all are true B. 16%
34. 34. Consider that w + x = 4, x + y = 25 and y + w = 15. C. 26%
39 C
Then the average of w, x, y is D. 35%
A. 3 Solution:
B. 4 (61-74)%-100% = 35% 40 D

C. 5 39. A grocer buys some eggs at Tk. 3 each. He finds that


D. 6 12 of them are broken, but he sells the others at Tk. 4 41 C
Solution: each and makes profit of Tk. 96. How many eggs did he
W+x+x+y+y+w=-4+25+15 buy?
Or 2w+2x+2y=36 A. 140
Or 2(w+x+y)=36 B. 142
Or w+x+y=36/2=18 C. 144
So average of w,x,y=18/3=6 D. 150
35. How long will it take for an amount Tk. 450 to yield Tk. 81 Solution:
as interest at 4.5% per annum of simple interest Let, The grocer buys = x eggs
A. 4years ATQ,(x-12)-3x = 96
B. 4.5years Then, x = 144
C. 5years 40. What is the original price of a T-shirt, if the sale price
D. 5.5years after 16% discount Is 264?
36. Club A has 20 members and Club B has 27. If a total of 42 A. 300
people belong to the two clubs, how many people belong to B. 214
both clubs? C. 320
A. 3 D. 314
B. 4 Solution:
C. 5 84% = 264
D. 6 So, 100% = 314.29 or 314
Solution: 41. The present ages of John and Mary are In the ratio of
Here Total =42 6 : 4 . Five years ago their ages were in the ratio of 5 : 3.
But 20+27 = 47 How old is John now?
So, Both is = 47-42 = 5 A. 42

51
Collected By: Md. Raihan Ahmed
JOB INFORMATION FOR ALL STUDENTS (BCS & BANK)

B. 36 46. If the area of a rhombus is 54 sq. cm and the length 42 B


C. 30 of one of the diagonals is 6 cm then the length of the
D. 24 other diagonal is
43 A
Solution: A. 18
6x-5:4x-5 = 5:3 B. 12
Or, 20x-25 = 18x-15 So, x = 5. John now is 65 = 30 C. 9 44 B

42. A man buys doughnuts at the rate of Tk. 35 per 100 pieces D. 6
and sells them at Tk. 7.20 per dozen. If the profit is Tk. 30, Solution: 45 C
how many doughnuts did he buy? Area of rhombus is ½(x*6) = 54 or 3x = 54 So, x = 18
A. 60 47. If one number exceeds another number by 14 and the
46 A
B. 120 larger number is times the smaller number, then the
C. 180 smaller number is
D. 210 A. 23 47 C

Solution: B. 26
Cost price of one doughnuts = 35/100 = 0.35 C. 28 48 B
Selling price of D. 31
Profit in one doughnuts = 0.6-0.35 = 0.25tk Solution:
49 D
So. Total doughnuts = 30 / 0.25 = 120 3/2 = 1.5
43. If y% of x = 29, then x = ? Let small number is x, so large number is 1.5x
A. 2900/y ATQ, 50 D

B. 29x/y 1.5x-x = 14 or. 0.5x = 14 so, x = 28


C. 29y/x 48. The ratio between two numbers is 3:4 and their sum
D. 29xy is 420. The greater one of the two numbers is
Solution: A. 320
y% of x=29 B. 240
So x=29*100/y=2900/y C. 180
44. How many tangents to a circle can be drawn from an D. 120
external point? Solution:
A. 1 If the number is 3x and 4x then 3x+4x =420 or 7x =420
B. 2 so x = 60 so the greater number is 460 = 240
C. 3 49. If 4^x+1=32, then x =?
D. 4 A. 2
Solution: B. 3
: C. 2/3
D. 3/2
45. The one-third of the complementary (Supplementary) angle Solution:
to 30° is 4^x+1=32
A. 150° Or, 22(x+1) =25 = or, 2x+2 = 5 or 2x = 3 So, x = 3/2
B. 100° 50. log√381
C. 50° A. 9
D. 10° B. 7
Solution: C. 6
Supplementary angle to 30 is 180-30 = 150 whose one third is D. 8
50 degree Solution:
81
log√3 = log√33^4 = log√3√3^8 = 8 log√3√3 = 8

52
Collected By: Md. Raihan Ahmed
JOB INFORMATION FOR ALL STUDENTS (BCS & BANK)

51. What will be the difference between simple and compound C. 11 51 D


interest at 10% on a sum of Tk. 1000 after 4 years? D. 13
A. Tk. 31.90 Solution:
52 A
B. Tk. 32.10 The series is 5, 7, 9,11,13 and 15
C. Tk. 44.90 So 4th number is 11
D. Tk. 64.10 56. How many permutations of seven different letters may 53 A

Solution: be made?
simple interest=1000*10*4/100=400 tk A. 1 54 B
10%=1/10 B. 7
So compound=1000*(1+1/10)^4-1000 C. 7!
55 C
=1000*(11/10)^4 -1000 D. 6!
=1000*11*11*11*11/10*10*10*10 -1000 Solution:
=1464.1-1000 =464.1 number of permutations=7 factorial or 7! 56 C

So difference=464.1-400=64.10
52. If GENERAL KNOWLEDGE AND COMPUTER 57 C
57. The name of the player included in the guineas book
A. 33.33 of the world is
58 B
B. 68.67 A. Shakib Al Hasan
C. 11.67 B. Tamim Iqbal
D. 31.50 C. Zobera Rahman Lenu 59 A

Solution: D. Rani Hamid


let B income 100 58. The tenure of the Governor of Bangladesh Bank is 60 D
A. 3 years
B. 4 years
61 B
53. After a 20% price decrease, a computer monitor is on sale C. 5 years
for Tk. 7200. What is its original price? D. 6 years
62 B
A. Tk. 9000 59. The duration of National ID cardd is
B. Tk. 10000 A. 10 years
C. Tk. 11000 B. 12 years
D. Tk. 12000 C. 15 years
Solution: D. 18 years
80% = 7200 S, 100% = 9000 60. The Ministry which controls Bank and Financial
54. The numbers 2, 3, 5 and x have an average equal to 4. institutions In Bangladesh is
What is the value of x? A. Ministry of industry
A. 4 B. Ministry of commerce
B. 6 C. Ministry of planning
C. 8 D. Ministry of finance
D. 10 61. The length of the Padma Bridge will be
Solution: A. 5.5km
Total = 4*4 = 16 B. 6.15km
Other number is 16- (2+3+5) = 6 C. 6.00km
55. If 15 is the 6th number in a series of 6 consecutive odd D. 5.75km
numbers what is 4th number in the series? 62. The name of the first Bangladeshi search engine is
A. 7 A. Butterfly
B. 9 B. Pipilika
C. Doel

53
Collected By: Md. Raihan Ahmed
JOB INFORMATION FOR ALL STUDENTS (BCS & BANK)

D. Kingfisher B. Unity party 63 C


63. Secondary Market is related to C. Tudeh party
A. Labour market D. None of these
64 A
B. Job market 72. The First Nawab of Bengal is
C. Stock market A. Alivardi khan
D. Agricultural market B. Murshid Quli khan 65 C

64. ATM stands for C. Sirajuddaula


A. Automated Teller Machine D. Shujauddin khan 66 B
B. Auto Teller Machine 73. The cell reference for a range of cells that Stars in cell
C. Automatic Teller Machine C1 and goes over to column H and down to row 10 is -
67 B
D. Automatic Toll Machine A. C1:10H
65. B. C1:H10
A. Borrowing For central bank C. C1:H-10 68 D

B. Borrowing from finance ministry D. C1:H:10


C. Inter Bank short term lending and borrowing. 74. Which topology does not allow transferring data 69 D
D. Withdrawal of money from commercial bank by deposit directly from computer to computer?
holders A. Ring
70 A
66. Bangladesh got the membership of UN in B. Bus
A. 1972 C. Tree
B. 1974 D. Star 71 A

C. 1975 75. In MS word, Which of the following line spacing is


D. 1973 invalid? 72 B
67. Bangladesh is not the member of one of the forum A. Single
A. BIMSTEC B. Double 73 B
B. G-8 C. Triple
C. D-8 D. Multiple
74 D
D. WTO 76. Which one is not pointing device?
68. The first ever Bangladeshi origin woman selected as the A. Touchscreen
75 C
Speaker of Tower Hamlets Council in England is B. OMR
A. Rushanara Ali C. Mouse
B. Rupa HuQ D. Light pen 76 B
C. Tulip Siddiqi 77. Trojan horse is a kind of-
D. Sabina Akhtar A. virus 77 B
69. The oldest news agency of the workd is B. malware
A. CNN C. spyware
78 C
B. AFP D. worm
C. BBC 78. The Universal gate is-
79 A
D. Reuters A. Ex-OR
70. The organization received the Novel Peace Prize three times B. AND
A. International committee of the Red Cross C. NOR
B. Amnesty International D. NOT
C. United Natios Organization 79. The Protocol used for resolving IP address from a
D. Transparency international domain name is
71. The Rulling Party of Syria is A. DNS
A. Ba ath party B. HTTP

54
Collected By: Md. Raihan Ahmed
JOB INFORMATION FOR ALL STUDENTS (BCS & BANK)

C. SMTP A. 80 C
D. TCP B.
80. WWW stands for- C.
01
A. World Whole Web D.
B. Wide World Web 8. -
C. World Wide Web A. 02 A

D. Web World Wide B.


C. 03 D
D.
9. -
04 D
A.
BANGLA B.
1. - C. 05 D

A. D.
B. 10. - 06 B
C. A.
D. B.
07 C
C.
2. - D.
A. 11. - 08 A

B. A. √ +
C. B. √ 09 C
D. C. √ +
3. - D. √ + 10 B
A. 12. -
B. A.
11 B
C. B.
D. C.
4. - - 12 C
D.
A. 13. -
B. A. - 13 C
C. B. -
D. C. 14 C
5. - - D. -
A. 14. - -
15 B
B. A.
C. B.
D. C.
6. - D.
A. 15. -
B. A.
C. B.
D. C.
7. - D.

55
Collected By: Md. Raihan Ahmed
JOB INFORMATION FOR ALL STUDENTS (BCS & BANK)

16. - 24. This is my 16 C


A. added to the bracketed part.
B. A. er
17 A
C. B. ial
D. C. ially
D. No suffix 18 C

ENGLISH 25.
17. Change into Passive: Who is creating this mess? A. Raising eyebrows 19 A
A. By whom is this mess being created? B. Showing indifference
B. Who has been created this mess? C. Showing Surprise
20 D
C. By whom this mess is being created? D. Frightening somebody
D. By whom has this mess been created? 26.
18. Use appropriate preposition: Finding himself short - A. a soldier 21 C

money, he wrote - his uncle - help. B. a thirsty man


A. in, to, to C. a valuable employee 22 D
B. with, to, for D. a highly paid worker
C. of, to, for 27. We waited until the plane
23 B
D. to, to, about A. did not take off
19. B. took off
- C. had not taken off 24 B

A. I will certainly come whether it rains or not. D. had taken off


B. Whenever there is rain I shall come. 28. week 25 D
C. If it rains I shall not come. A. an
D. If I come it will not rain. B. a 26 C
20. Generally speaking, ____ peacocks are more beautiful than C. for
peahens. The gap require D. the
27 B
A. the 29.
B. a A. most able
C. an 28 B
B. ablest
D. no article C. best able
21. The bold him a piece of D. abler 29 B
my mind 30.
A. Expressed Love A. canon 30 A
B. Fell in love B. mouse
C. Expressed anguish C. Physics
31 A
D. Trusted him D. Medium
22. A den for small animals is called 31. The correct spelling is
A. Dyke 32 A
A. Surveillance
B. Shed B. Survellance
C. Cave C. survailance
D. Hutch D. surveilance
23. 32.
A. Apostrophe A. Short drive
B. Dash B. Irresistible force
C. Hyphen
D. Comma

56
Collected By: Md. Raihan Ahmed
JOB INFORMATION FOR ALL STUDENTS (BCS & BANK)

C. Refuse C. 8 33 B
D. Solitary D. 9
Solution:
34 A
MATHEMATICS let two number a and b
33. A card is randomly drawn from a deck of 52 cards. What is a-
the probability of getting of a king or queen? and. 35 B

A. 3/13
or, (a+b)(a-b)=65 or a+b
B. 2/13 36 D
by adding (i) and (ii) we get 2a = 18 Or a = 9 So, the
C. 1/13
larger number is 9.
D. 4/13
37. The lengths of two sides of a right angle triangle are 37 D
Solution:
13cm and 5cm respectively. The length of the third side is
In a deck of 52 cards, there are:
A. Greater than 15 cm
4 Kings 38 D
B. Less than 10 cm
4 Queens
C. Equal to 3 cm.
That would be a total of 4+4= = 8 cards 39 A
D. Equal to 12 cm.
Hence, probability of getting a King or a Queen = 8/52 = 2/13
Solution:
34. The diagonal of a rectangle is √41 cm and its area is 20
The third side is 12 cm. 40 D
cm2. What is the perimeter of the rectangle?
The hypotenuese is the 13, and 3rd side is 5
A. 18 cm
Check with Pythagorean theorem
B. 17 cm
13 ^2 = 169
C. 20 cm
12^2 = 144 and 5^2 =25
D. 18 cm
144 + 25 = 169
Solution:
38. Consider that w+x= 4, x+y=25 and y+w=15. Then
L2+b2 = (diagonal)2=40
the average of w,x,y is
Also, lb=20
A. 3
(l+b)2=l2+b2+2lb
B. 4
= 41 + 40 =81
C. 5
(l + b) = 9.
D. 6
Perimeter = 2(l + b) = 18 cm
Solution:
35. IF x−1/x= √3 then x+1/x=?
W+x+x+y+y+w=-4+25+15
A. 3√3
Or 2w+2x+2y=36
B. √7
Or 2(w+x+y)=36
C. 2√3
Or w+x+y=36/2=18
D. 7
So average of w,x,y=18/3=6
Solution:
39. How long will it take for an amount TK 450 to yield TK
x−1/x= √3
81 as interest at 4.5% per annum of simple interest?
(x−1/x)2= 3
A. 4 years
(x+1/x)2 - 4= 3
B. 4.5 years
x+1/x= √7
C. 5 years
36. The difference between two numbers is 5 and the
D. 5.5 years
difference between their squares is 65.What is the larger
Solution:
number?
Time = (100 x 81) / (450 x 4.5) = 4 years.
A. 13
40.
B. 11

57
Collected By: Md. Raihan Ahmed
JOB INFORMATION FOR ALL STUDENTS (BCS & BANK)

A. 12 By solving we get X=30 41 C


B. 24 44. Tk. 5000 is deposited in a savings account which pays
C. 18 7% annual interest compounded semi-annually. To the
42 C
D. 16 nearest Taka, how much is in the account at the end of
Solution: the year?
A. 5423 43 C

So, 2(12+x) = 40+x B. 5356


Or, 24+2x = 40+x C. 5122 44 B
2x-x = 40 16 D. 5247
x = 16 Solution:
45 B
41. A grocer buys some eggs at TK 3 each. He finds that 12 of 5000(1+ 0.07/2)2 = 5356
them are broken, but he sells the others at TK 4 each and 45. A cricket team has won 40 games out of 60 played. It
makes profit of TK 96. How many eggs did he buy? has 32 more games to play. How many of these must the 46 D

A. 140 team win to make it record 70% win for the season?
B. 142 A. 20 47 A
C. 144 B. 25
D. 150 C. 23
Solution: D. 32
Let, The grocer buys = x eggs Solution:
ATQ, Let the no of games the team will win be W
4(x-12) 3x = 96 Total no of games = ( 60+32)=92
Or, 4x-48-3x=96, Then x = 144 Total no of game won = (40+W)
42. What is the original price of a T-shirt, if the sale price after Record= 70%
15% discount is 272? Therefore
A. 300 {(40+W)/92}*100=70
B. 280 Solve for W
C. 320 W= 25
D. 314 46. Three workers, X, Y and Z, are paid a total of Tk.
Solution: 5,500 for a particular job. X Is paid 133.33% of the
Let P be the original price amount paid to Y and Y Is paid 75% of amount paid to Z.
.85*P = 272 How much is paid to Z?
P = 320 A. Tk.1780
43. The present ages of John and Mary are in the ratio of 6:4. B. Tk.1890
Five years ago their ages were in the ratio of 5:3. How old is C. Tk.1975
John now? D. Tk. 2000
A. 42 Solution:
B. 36 Ratio of payment of X and Y = 133.33 : 100 or 4:3
C. 30 Again ratio of Y and Z is Y:Z = 75:100 or 3:4
D. 24 So, ratio of X, Y and Z = 4:3:4
Solution: Sum of the ratio = 4+3+4 = 11
Let John present age X So, Z got = 5500 X 4/11 = 2000
47. IF the interest of Tk H at H% in 4 years is Tk H then
H =?
ATQ, A. 25
X-5/Y- B. 20

58
Collected By: Md. Raihan Ahmed
JOB INFORMATION FOR ALL STUDENTS (BCS & BANK)

C. 30 53 7 11 77 48 3
D. 32
Solution:
49 C
Total interest = (H% of H) 1000 53, 130, 207 977
=H/100 x H x 4= H2/25 ( ( 77
ATQ, (977-53)/77 +1 =13 50 B

H2/25=H ( H 51. A train 240 m long train passed a pole in 24 second.


H2 = 25H How long will it take to pass a platform 650 m long? 51 B
H = 25. A. 65 sec
48. If logx1/9 =-2 then x =? B. 89 sec
52 B
A. -1/3 C. 100 sec
B. 1/3 D. 130 sec
C. -3 Solution: 53 D

D. 3 train speed in 1 second is = 240/24 =10 m/s


Solution: The train has to go = 240+650 = 890m 54
logx 1/9
=-2 So Total time taken =890/10 = 89 seconds

x -2
= 1/9 52. The average of the smallest and largest primes
55 B
x=3 between 60 and 80 is

49. If 5% is gained by selling an article for BDT 350 than selling A. 60

it for BDT 340, the cost of the article is ? B. 70

A. BDT 180 C. 60

B. BDT 150 D. 77

C. BDT 200 Solution:

D. BDT 250 60 80 61

Solution: 79

Selling price difference = 350-340 = 10 tk. 61 79 (61+79)/2 = 70

And % difference is = 5% 53. In the triangle ABC if AB > AC then which of the

5% of cost = 10tk, following is true?

Or, 1% of cost 10/5 Tk A. ∠ABC > ∠ACB

So 100% of cost = (10 x 100)/5 = 200Tk B. ∠ABC < ∠BAC

50. In first 1000 natural numbers, how many Integers exist C. ∠ACB > ∠BAC

such that they leave a remainder 4 when divided by 7 and a D. ∠ACB > ∠ABC

remainder 9 when divided by 11? 54. If x2-7xy+y2 is divided by x-2y, the result is

A. 11 A. 3x+2y

B. 13 B. 3x-2y

C. 15 C. 2x-3y

D. 17 D. 2x+3y

Solution: No Answer

7 , 4, 11, 18, 55. If the first and sixth term of a geometric series are

25, 32, 39, 46, 53, 60 . . . . . respectively 1/2 and 1/64, then the common ratio is-

11 9 9, A. ¼
20, 31, 42, 53, 64 B. ½
53 C. 1
D. 2

59
Collected By: Md. Raihan Ahmed
JOB INFORMATION FOR ALL STUDENTS (BCS & BANK)

Solution: 62. Which topology does not allow transferring data 56 A


1/2 1/64 directly from computer to computer?
A. Ring
57 D
B. Bus
1/64 C. Tree
1/2 D. Star 58 C

1/2 63. Trojan horse is a kind of-


56. If the length of a side of an equilateral triangle is 4 cm its A. Virus 59 A
height is B. Malware
A. 2√3 C. Spyware
60 B
B. 4√3 D. Worm
C. 16√3 64. The primary Additive colors of a color monitor are
D. 32√3 A. Blue, Magenta, Green 61 B

Solution: B. Red, Blue, Yellow


The altitude of any equilateral triangle is the product of the C. Red, Green, Yellow 62 D
SIDE LENGTH and (√3)/2 D. Red, Green, Blue
therefore, the answer to this problem is (4)·(√3)/2 = 2√3 cm 65. Bangladesh receives highest Foreign Direct Investment
63 B
from
GENERAL KNOWLEDGE AND COMPUTER A. Japan
57. Which of the following is spreadsheet analysis software? B. USA 64 D

A. Latex C. UK
B. Illustrator D. UAE 65
C. Access Singapur [World Investment Report-2017]
D. Excel 66. The goodwill ambassador of UNDP from Bangladesh 66 A
58. Which is not an Object Oriented Programming Language is-
A. C# A. Masheafe Bin Mortaza
67
B. C B. Jewel Aich
C. Java C. Fazle Hasan Abed
D. C++ 68 B
D. Shakib Al Hasan
59. How many bits are used by Unicode to represent one 67. The percentage of industry in the GDP of Bangladesh
character? is- 69 A
A. 8 A. 17.31%
B. 16 B. 24.73%
C. 32 C. 28.4%
D. 48 D. 43.6%
60. Which one is a language translator? No Answer
A. Pearl 68. The number of goals to achieve SDG is-
B. Assembler A. 8
C. JavaScript B. 17
D. BCD C. 18
61. The cell reference for a range of cells that starts in cell C1 D. 27
and goes over to column H and down to row 10 is- 69. Summit of G-20 hold in 2017.
A. C1:10H A. Hamburg
B. C1:H10 B. New York
C. C1:H-10
D. C1:H:10

60
Collected By: Md. Raihan Ahmed
JOB INFORMATION FOR ALL STUDENTS (BCS & BANK)

C. Paris A. US $2425 70 B
D. London B. US $2275
70. The latest member of Commonwealth C. US $2580
71 C
A. Somalia D. US $2379
B. Rwanda 78. The Paradise Papers is a-
C. Mali A. A daily newspaper 72 B

D. Nigeria B. A confidential e-document


71. The country is going to introduce 5G internet service- C. A secret agency 73 D
A. Japan D. An intelligence branch
B. China 79. The Coalition year: 1996 to 2012 is written by-
74 A
C. South Korea A. Amartya Sen
D. USA B. Jyoti Basu
72. The direction of rotation of earth on its axis is- C. Rehman Sobhan 75 A

A. East to West D. Pranab Mukharjee


B. West to East 80. The total number of documents and collection 76 C
C. North to South recogn
D. South to North -
77 D
73. ISBN is used to identify A. 317
A. Software B. 327
B. Hardware C. 417 78 B

C. Periodicals D. 427
D. Books 79 D
View Answer
74. Expected gas reserve in the recent discovered Bhola gas 80 D
field is ___ Billion cft. BANGLA
A. 700 1.
01 D
B. 600 A.
C. 850 B.
C. 02 B
D. 650
75. The Capital of Catalonia, the autonomous region of Spain D.
is- 2. 03 B
A. Barcelona A.
B. Girona B. 04 C
C. Madrid C.
D. Tarragona D.
76. New Chairman of Commonwealth Parliamentary Association 3.
(CPA) elected in 2017 is- A.
A. Shirin Sharmin Chowdhury B.
B. Sir Alan Haselhurst C.
C. Emilia Monjowa Lifaka D.
D. Hon. Vicki Dunne 4.
77. As per the human development report of UN development A.
Programme of 2017, the average per capita income of women B.
is- C.
D.

61
Collected By: Md. Raihan Ahmed
JOB INFORMATION FOR ALL STUDENTS (BCS & BANK)

5. D. 05 B
A. 14. Congizable
B. A.
06 B
C. B.
D. C.
6. D. 07 A

A. 15.
B. A. 08 C
C. B.
D. - C.
09 A
7. D.
16.
A. A. 10 B

B. B.
C. C. : 11 B
D. D.
8.
12 D
A. ENGLISH
B. 17. Deciduous trees are trees that
C. A. lose the leaves annually 13 B

D. B. are extremely big


9. , C. have small leaves 14 C
A. D. have fleshy leaves
B. 18. A makes no difference between man and man in
15 C
C. spreading his love.
D. A. philander
16 D
10. B. philistirie
A. C. philosopher
B. D. philanthropist 17 A

C. 19. His friend ___ his word much to his despair.


D. A. went back on 18 D
11. B. went away from
A. C. Went with
19 A
B. D. went in
C. 20. Change the voice: Can you help me to scoop out
20 A
D. seeds from melon?
12. ____to scoop out the seeds from the melon?
A. A. Can I be helped by you 21 A

B. B. Can I helped by you


C. C. Could I helped by you
D. D. Can I been helped by you
13. 21. Savior is always better than the ___
A. A. destroyer
B. B. rodeemer
C. C. executioner .
D. toilurer

62
Collected By: Md. Raihan Ahmed
JOB INFORMATION FOR ALL STUDENTS (BCS & BANK)

22. Choose the appropriate reporting speech: Sara promised A. Questionaire 22 B


Sana, _______ B. Questionoir
A. C. Questionnaire
23 A
B. D. Questionafr
C. 31. Technology has developed ___ great speed.
D. A. on 24 B

23. Complete shutdown (be) observed today against new law. B. at


A. is being B. in 25 B
B. can be C. by
C. is been 32. Choose the best translation of
26 B
D. was been
24. A sedentary worker requires more calories than a one. A. The rain fell in torrents.
A. trained B. It was drizzling with gusty wind. 27 C

B. mobile C. It was raining cats and dogs.


C. Stationary D. The rain came down in torrent. 28 C
D. Laborious
25. The Judge decided to exonerate the accused. The bold MATHEMATICS
29 C
faced word means 33. How many integers from 1 to 1000 are divisible by 30
A. purnsh but not by 16?
B. acquit A. 29 30 C

C. cross-examine B. 31
D. C. 32 31 B
26. Choose the appropriate D. 38
34. If * is defined for all positive real numbers a and b by
32 B
A. informed a*b= ab/(a+b), then 10*2=?
B. advised A. 5/3
33 A
C. told B. 5/2
D. requested C. 5
27. D. 20/3 34 A

Solution:
A. Psychology a*b= ab/(a+b) = (10×2)/(10+2) = 20/12 = 5/3 35 B
B. Psycholinguistics 35. If the length of a rectangle is increased by 20% and
C. Psychiatry width is decreased by 20%. what is the change in area of
36 B
D. Psychosis the rectangle?
28. A. unchanged
A. a rate chart B. decreases by 4%
B. young C. increases by 4%
C. right on properties D. increases by 5%
D. compassionate Solution:
29. L = length

A. to act like a great man W = width

B. to act like a her Area = (1)LW

C. to act like a coward Length increases by 20%, width decreases by 20%:

D. to act like a devil 1.2L × 0.8W = (.96) LW

30. Identify the correct spelling 0.96 is 4 % less than 1


The area decreases by 4 %

63
Collected By: Md. Raihan Ahmed
JOB INFORMATION FOR ALL STUDENTS (BCS & BANK)

36. The dimensions of a box are 2, 3 and 4 meters. The cost of D. 2 36 A


painting the outer sides of the box at the rate of Tk. 3 per Solution:
square meter is log 36/ log 6 = log 62 / log 6 = 2log6/log6 = 2
37 D
A. Tk. 156 40. If 1 2x ≤ 3, then
B. Tk. 120 A. x ≤ -2
C. Tk. 136 B. x ≥ -2 38 A

D. Tk. 160 C. x ≤ -1
Solution: D. x ≥ -1 39 D
Area of the box, Solution:
= 2 (ab + bc + ca) 1 2x ≤ 3
40 D
= 2 (2×3 + 3×4 + 4×2) Or, -2x ≤ 2
= 2 (6 + 12 + 8) Or, x≤1
= 2×26 Or, x ≥ -1 41 A
2
= 52 41. If sinA + sin A = 1, then the value of the expression
The cost of painting the outer sides of the box = 52×3 = 156 cos2 A + cos4 A is
42 B
37. In each expression below, N represents a negative integer. A. 1
Which expression could have a negative value? B. 1/2
43 C
A. N 2
C. 2
B. 6-N D. 3
C. - N Solution: 44 A
D. 6 + N Given,

Solution: sinA + sin2 A = 1


If N = -2, 6+ N = 6 + (-2) = 4 positive Or, sinA = 1 sin2 A
If N = -7, 6 + N = 6 +(-7) = -1 negative Or, sinA = cos2A
But in case of other options, result becomes always positive. Or, sin2A = cos4A
38. If the length of rectangle A is one-half the length of Or, 1 cos2A = cos4A
rectangle B, and the width of rectangle A is one-half the width Or, cos2 A + cos4 A = 1
of rectangle B, what is the ratio of the area of rectangle A to 42. A median of a triangle divides it into two
the area of rectangle B? A. congruent triangles
A. 1/4 B. triangles of equal area
B. 1/2 C. isosceles triangles
C. 1/1 D. right triangles
D. 2/1 43. Which number logically follows the sequence?
Solution:
Let, A. 6
Length of B = x B. 17
Length of A = x/2 C. 19
Again, D. 21
Width of B = y 44. How many cases do you need if you have to pack 112
Width of A = y/2 pairs of shoes into cases that each hold 28 shoes?
So, A. 8
A:B = (x/2 × y/2) : xy = xy/4 : xy = 1/4:1 B. 10
39. Log 36/log 6 = C. 12
A. 5 D. 14
B. 8 Solution:
C. 3 The twist is in the presence of the word pair.

64
Collected By: Md. Raihan Ahmed
JOB INFORMATION FOR ALL STUDENTS (BCS & BANK)

112 pairs of shoes need to be packed in cases, which is 50. The day after the day after tomorrow is 4 days before 45 D
equivalent to 224 shoes. (1 pair is equivalent to 2 units. ) Each Monday. What day is it today?
case can hold a maximum of 28 shoes (notice the absence of A. Monday
46 A
the word pair ). B. Tuesday
Using unitary method, C. Wednesday
To pack 28 shoes, we need 1 case. D. Thursday 47 B

To pack 1 shoe, we need 1/28 case. 51.


To pack 224 shoes, we need (1/28)x224 cases, which is equal represents 48 B
to 8. A. voice
45. person who wants to increase the level of good cholesterol B. bald
49 C
in his/her body, should C. bloat
A. take vitamins D. castle
B. eat green leafy vegetables 52. 50 A

C. sleep sufficiently have any brothers or sons. Who is my successor?


D. take omega-3 fatty acids A. Nephew 51 C
46. Materials for rain-proof coats and tents owe their water- B. Niece
proof properties to -. C. Daughter
52 C
A. surface tension D. Myself
B. specific gravity 53. At the end of a banquet 10 people shake hands with
C. viscosity each other. How many handshakes will there be in total? 53 C

D.elasticity A. 100
47. In a row in the theatre the seats are numbered B. 20 54 C
consecutively from T1 to T50. Sumon is sitting in seat T17 and C. 45
Shajib is sitting in seat T39. How many seats are there between D. 90. 55 A
them? Solution:
A. 23
B. 21
C. 22 to shake again. The next person shakes the remaining 8
D. 20
Solution:
39-17 = 22 so on until the last two people shake hands.
Seats between them are = 22-1 = 21 9+8+7+6+5+4+3+2+1=45 shakes
48. Which of the following can be arranged into a 5-letter 54. In a crime, three suspects X, Y, and Z were caught
English word?
(a)HRGST (b)RILSA (c)TOOMT (d)WQRGS stole it. I did n
A. a & c that Z was lying and there was only one thief. Who was
B. b & c the thief?
C. c & d A. X
D. a & d. B. Y
49. Siddik has a new set of golf clubs. Using a club 8, 7 and 6 C. Z
the ball travels an average distance of 100m, 108m, 114m D. Someone else
respectively. How far will the ball go if he uses a club 5? 55. How many cases do you need if you have to pack 112
A. 122m pairs of shoes into cases that each hold 28 shoes?
B. 120m A. 8
C. 118m B. 10
D. 116m C. 12

65
Collected By: Md. Raihan Ahmed
JOB INFORMATION FOR ALL STUDENTS (BCS & BANK)

D. 14 C. 3 56 C
D. 8
GENERAL KNOWLEDGE AND COMPUTER 64. Russian President Vladimir Putin recently elected
57 B
56. President for the
have the same birthdays. Because they might be born in A. 2nd time
different B. 3rd time 58 A
A. centuries C. 4th time
B. culture D. 5th time 59 D
C. countries 65. The eighth division of Bangladesh is
D. casts A. Rangpur
60 C
57. The latest member-state of NATO is B. Comilla
A. Albania C. Barisal
B. Montenegro D. Mymensingh. 61 C

C. Croatia 66. The international body of UN which gives the


D. Romania recognition of the transformation from LDC to developing 62 B
58. One of the official languages of the Organization of Islamic countries
Cooperation (OIC) is A. Committee for Development Policy
63 A
A. French B. Committee for Developing Policy
B. Spanish C. Committee for National Development Policy
C. Russian D. Committee for International Development Policy 64 C

D. Chinese 67. The host city of the 21st Commonwealth Games -2018
59. The Atrai flows through district of Bangladesh is 65 D
A. Dhaka A. Brisbane
B. Sylhet B. Gold Coast 66 A
C. Bogra C. Gladstone
D. Dinajpur D. Darwin
67 B
60. The first post-liberation war sculpture in Bangladesh is 68. Asma Jahangir is a renowned _____ activist.
A. Shongshoptok A. human rights
B. Aparajeyo Ban gki 68 A
B. animal rights
C. Jagroto Chowrongi. C. political rights
D. Sho part to Shadhinota D. legal rights 69 C
61. The motion 69. The year 2019 will mark the 100 years since the peace
on the life of treaty of 70 B
A. Albert Einstein A. Vienná
B. Isaac Newton B. Yalta
71 A
C. Stephen Hawking C. Versailles
D. Steve Jobs D. Helsinki
62. National Voters Day of Bangladesh is observed on 70. The best picture winning in the 90 Academy Award is
A. April 4 A. Darkest Hour
B. March 1 B. The Shape of Water
C. October 10 C. Dunkirk
D. September 10 D. Lady Bird
63. According to Global Firepower Index 2017 the place of 71. Bangladesh Bank recently announced a new CRR to
India is in. increase the flow of money by in the money market.
A. 4 A. 10000 crore
B. 7

66
Collected By: Md. Raihan Ahmed
JOB INFORMATION FOR ALL STUDENTS (BCS & BANK)

B. 12000 crore B. adware 72 B


C. 15000 crore C. spam
D. 16000 crore D. malware
73 A
72. The name of the passenger train runs between Kolkata and 80. ___ are used to identify a user who returns to a
Khulna is website.
A. Shouhardo A. Cookies 74 C

B. Bandhon B. Plugins
C. Mukti C. Scripts 75 C
D. Progoti D. ASPs
73. A set of step-by-step procedures for accomplishing a task
76 D
is known as a(n)
A. algorithm
B. hardware BANGLA 77 B

C. firmware 1.

D. application A.
78 C
B.
74. An electronic path, that sends signals from one part of
C.
computer to another is a
D. 79 D
A. Logic Gate
2.
B. Modem
A.
C. Bus 80 A
B.
D. Serial Port C.
75. QWERTY is used with reference to D. 01 B
A. Monitor 3.
B. Printer A. 02 C
C. Keyboard B.
D. Mouse C.
03 B
76. a set of rules. D.

A. network 4.
04 A
B. domain
A.
C. hypertext
B.
D. protocol 05 C
C.
77. How many full adders are needed to add two 4bit numbers
D.
with a parallel adder? 06 C
5.
A. 8 A.
B. 4 B.
07 A
C. 2 C.
D. 16 D.
78. Chip is a common nickname for a (n) 6.
A. transistor A.

B. resistor B.
C.
C. Integrated circuit
D.
D. semiconductor
7. Translate into Bangla: He has no political axe to grind.
79. Software, such as viruses, worms and Trojan horses, that
A.
has a malicious intent is known as
B.
A. spyware

67
Collected By: Md. Raihan Ahmed
JOB INFORMATION FOR ALL STUDENTS (BCS & BANK)

C. ENGLISH 08 B
D. Fill in the gaps (17-19) with appropriate words:
8. 17. Universities should never be made _____ mechanical
A. organizations. 09 D

B. A. into
C. B. of 10 B
D. C. from
9. ? D. by
A. 18. of 11 C

B. dreams suggests that dreams are a representation of


C. unconscious desires. 12 C
D. A. In spite of
10. B. in order to
A. C. in terms of 13 A

B. D. Consistent with
C. 19. Unlike most other diseases, leprosy ________ in
14 A
D. isolation.
11. A. avoids
A. B. results 15 D

B. C. locuses
C. D. creates 16 C
D. 20. Use the right form of verb: She (to form) quite a
12. picture of Bangladesh before she read this travel book.
17 A
A. A. forms
B. B. is forming
C. C. had formed 18 D
D. D. was formed
13. ,/ 21.
19 B
- A. stain
A. B. benefit
B. C. esteem 20 C
C. D. exaltation
D. 22.
21 A
14. bold faced phrase means
A. , , A. to connect
B. , , B. to remove 22 B

C. , , C. to throw
D. , , D. to take 23 B
15. , 23. If Augment: Increase then
A. A. Cohesion : Coinside
24 A
B. B. Coalesce: Combine
C. C. Collapse: Collide
D. D. Conceal: Concur
16. - 24. Choose the best translation:
A.
B. A. He is an erudite professor of English
C. B. He is a professor with outstanding knowledge in
D. English.

68
Collected By: Md. Raihan Ahmed
JOB INFORMATION FOR ALL STUDENTS (BCS & BANK)

C. He is at home in English. B. Interjection 25 D


D. He is a professor of English with vast understanding. C. Adverb
25. Cha D. Adjective
26 B

A. The all arrangements have been made by the city planner MATHEMATICS
B. All the arrangements are making by the city planner. 33. If both x and y are prime numbers, which of the 27 D
C. The all arrangements had been making by the city planner. following CANNOT be the product of x and y?
D. All the arrangements are being made by the city planner A. 6
26. please B. 10 28 D

C. 35
A. They asked him to visit their office today. D. 27 29 B
B. They requested him respectfully to visit their office that day. Solution:
C. They addressed him Sir and requested him to visit the office 3 × 2 = 6 , 5 × 2 = 10 , 5 × 7 = 35 , 3 × 9 = 27
today 9 is not a prime number. So answer is 27 30 B

D. They told him in humble manner to visit their office that day 34. How many integers from 1 to 1000 are divisible by 30
27. Choose the correct form: I had spoken to them about my but not by 16?
31 D
childhood. A. 29
A. l had been speaking to them about my childhood. B. 31
B. I had been speaking about my childhood to them C. 32 32 B

C. I spoken to them about my childhood. D. 38


D. I spoke to them about my childhood. Solution: 33 D
28. The misspelled word is Divisible by 30 = 1000/30 ≈ 33
A. Idiosyncrasy LCM 30 & 16 = 240
34 A
B. Personnel Divisible by 240 = 1000/240 ≈ 4
C. Exhilarate So divisible by 30 but not 16 = 33 4 = 29
D. Chauffuer 35. P and Q are brothers. R and S are sisters. The son of 35 C
29. lateral damage P is brother of S. Q is related to R as
during clashes between rioters and the police. The bold faced A. son
36 C
phrase is an example of B. brother
A. Simile C. uncle
B. Euphemism D. father 37 C
C. Metonymy 36. If w is 10% less than x, and y is 30% less than z, then
D. Hyperbole wy is what percent less than xz?
30. which Is A. 10%
a/an _____ word. B. 20%
A. Turkish C. 37%
B. Arabic D. 40%
C. Persian Solution:
D. French Let x : w = 100 : 90 = 10 : 9
31. The best conjunction to link is _____for joining sentences: And z : y = 100 : 70 = 10 : 7
I. will keep continuing this job. So wy = 9 × 7 = 63 and xz = 10 × 10 =100
II.You like it or not. So wy is less by = (100-63) × 100/100 = 37%
A. when 37. Every 3 minutes, 4 litres of water are poured into a
B. since 2,000 litre tank. After 2 hours, what percent of the tank is
C. otherwise full?
D. whether A. 0.4%
32. Choose the POS category of the bold faced word: Well, I B. 4%
will be able to finish this today. C. 8%
A. Preposition D. 12%

69
Collected By: Md. Raihan Ahmed
JOB INFORMATION FOR ALL STUDENTS (BCS & BANK)

Solution: 43. If 1 2x ≤ 3, then 38 C


2 hours = 120 minutes A. x ≤ -2
In 120 minutes water are poured into tank = 160 litres B. x ≥ 2
Required percentage = (160 × 100)/2000 = 8% C. x ≤ -1 39 D

38. The next number in the sequence 3, 6, 11, 18, 27, -is D. x ≥ -1
A. 34 Solution: 40 B
B. 36 1 - 2x ≤ 3
C. 38 Or, - x ≤ 1
D. 40 Or, x ≥ -1 41 C

39. The HC.F. of two numbers is 24. The number which can be 44. If y/x =1/3 and x + 2y = 10 then x is
their LCM.is A. 2 42 D
A. 84 B. 3
B. 128 C. 4
C. 148 D. 6 43 D

D. 120 Solution:
Solution: y/x=1/3 then
44 D
120 is multiple of 24 x/y=3 and x=3y
40. The average of eight numbers is 14. The average of six of 3y+2y=10
these numbers is 16. The average of the remaining two 5y=10 45 B

numbers is y=2
A. 4 x=6 46 C
B. 8 45. How many real roots does the polynomial 2x 3 +8x-7
C. 16 have?
47 C
D. Data inadequate A. None
Solution: B. One
Total of 8 numbers = 14 × 8 = 112 C. Two
Total of 6 numbers = 16 × 6 = 96 D. Three
Average of remaining 2 numbers = (112 96)/2 = 8 46. If xy = 2 and xy^2 = 8, what is the value of x?
41. If A = (1,2,3,4,5), then the number of proper subsets of A A. 4
is- B. 2
A. 120 C. 1/2
B. 30 D. 8
C. 31 Solution:
D. 32 Given that,
Solution: xy = 2
2^5-1= 31 therefore, x = 2/y
42. Two numbers are in the ratio 2:5. If 16 is added to both and,
the numbers, their ratio becomes 1:2. The numbers are xy^2 = 8
A. 16,40 putting the value of x = 2/y
B. 20,50 2/y × y^2 = 8
C. 28,70 2y = 8
D. 32,80 y=4
Solution: now as we know
Let the numbers 2x, 5x xy = 2
AtQ, x×4=2
(2x + 16) / (5x + 16) = ½ x = 1/2
x= 16 47. If a -1/a=2 what is a 3 1/a 3 ?
The numbers 32 and 80 A. 16
B. 10

70
Collected By: Md. Raihan Ahmed
JOB INFORMATION FOR ALL STUDENTS (BCS & BANK)

C. 14 52. if logx 9/16 =- ½ the value of the base is 48


D. 12 A. 16/9
Solution: B. 9/16 C
a3 - 1/ a3 C. 256/81 49
D
= (a -1/a)3 + 3 a. 1/a (a -1/a) D. 81/256
= 8 + 3(2) Solution: 50 C
= 14 log_x (9/16)
48. The three sides of a triangle are: x+1,2x-1 and 2x+1 - 1/2 ⇔ x^- 1/2
respectively and the perimeter is 26 cm. The length of the 9/16 = 1/√x 51 D

smallest side is 9/16 ⇔ √x


A. 5 cm :. X = 256/81 52 C
B. 3 cm 53. If cos A +cos^2 A = 1, then the value of the
C. 4 cm expression (cos^2 A + cosA) is
D. 7 cm A. 1 53 A

No Answer B. 1/2
Solution: C. 2
54 A
x+1+2x-1+2x+1= 26 D. 3
=> 5x= 25 54. The difference in taka between simple and compound
=> x= 5 interest at 5% annually on a sum of Tk. 5000 after 2 years 55 B

Smallest side = 5+1= 6 is


49. There are 10 true false questions in an examination. These A. 12.5 56
questions can be answered in B. 25
A. 20 ways C. 50
B. 100 ways D. 500
C. 2^10 ways Solution:
D. 1024 ways Compound interest = 5000(1 + 5/100)2 - 5000 = 512.5 tk
50. If two fair coins are flipped, what is the probability that Simple interest = 5000 × 2 × 5/100 = 500
one will come up heads and the other tails? Difference = 512.5 -500 = 12.5
A. 1/4 55. For y=-2x-8, what is the least value of x for which is
B. 1/3 less than 9?
C. 1/2 A. -9
D. ¾ B. -8
Solution: C. -7
There are four equally likely outcomes: HH, HT, TH and TT. D. -6
Solution:
9= -2x-8
So the probability is 2/4 = 1/2 => -2x= 17
51. If the diagonal of a square measures 16√2 cm, what is the => x= -8.5[ so less than -8.5 means -8]
area of the square in sq. cm? 56. A group of 7 members having a majority of boys is to
A. 32√2 be formed out of 7 boys and 4 girls. The number of ways
B. 64√2 the group can be formed is
C. 128 A. 80
D. 256 B. 100
Solution: C. 90
diagnol of a square =side • √2 D. 110
No Answer
X•√2=16•√2 Solution:
X=16 No. Of way= (7C6*4C1)+(7C5*4C2)+(7C4*4C3) = 294
2
So the area of the square is = 16 = 256 sq. cm

71
Collected By: Md. Raihan Ahmed
JOB INFORMATION FOR ALL STUDENTS (BCS & BANK)

GENERAL KNOWLEDGE AND COMPUTER 66. The former name of Bangabhaban was 57 A
57. The film Let There Be Light is directed by A. Chameli House
A. Zahir Raihan B. Governor House
B. Tanvir Mokammel C. Burdwan House 58 D

C. Alamgir Kabir D. Chancery House


D. Tareque Masud 67. Which of the following is not a mobile banking service 59 B
58. Cash Flow Statement is prepared from operating in Bangladesh?
A. profit and loss account A. Rocket
B. additional information B. U Cash 60 C

C. balance sheet C. Q-cash


D. all of the above D. Bkash 61 B
59. Community Bank Bangladesh Ltd. is a concern of 68. Along with Abhijit V. Banerjee and Esther Duflo ___ has
A. RAB been honoured with the Nobel Prize in Economics 2019.
B. Police A. Oliver Hart 62 A

C. Ansar-VDP B. William Nordhaus


D. BGB C. Michael Kremer
63 C
60. The Permanent Representative of Bangladesh to the UN is D. Paul Romer
A. Mashree Binte Shams 69. E-KYC is a/an
B. Saida Muna Tasneem A. Banking Service 64 A

C. Rabab Fatima B. Health Service


D. Ismat Jahan C. Legal Service 65 C
61. D. Insurance Service
has been received by 70. _______ is a monetary policy instrument.
66 B
A. Narendra Modi A. Tax
B. Sheikh Hasina B. Open market operation
C. Amartya Sen C. savings 67 C
D. Shirin Sharmin Chaudhury D. fiscal deficit
62. The national helpline for emergency health services- 71. This year, 22 Sraban 1426 was the ____ death
68 C
A. 16263 anniversary of Rabindranath Tagore.
B. 16123 A. 75th
C. 999 B. 80th 69 A
D. 333 C. 78th
63. at the Best FIFA D. 81th
70 B
- 72. _____ is a popular database management software.
A. Cristiano Ronaldo A. MS-Access
B. Virgil van Dijk B. MS-Outlook 71 C

C. Lionel Messi C. MS-InfoPath


D. Neymar D. MS-Groove 72 A
64. The first online news agency of Bangladesh is 73. Find the odd one out.
A. BDNews24.com A. ORACLE
73 B
B. Focus Bangla B. C++
C. Bangladesh news service C. SYBASE
D. News Network D. INFORMIX 74 D
65. In a career development focus, information about 74. The primary requisite of a good computer
Individual interests and preferences is a part of programmer is
A. training and development A. mathematical mind
B. performance appraisal B. scientific knowledge
C. human resource planning C. hardware skill
D. recruiting and placement D. logical mind

72
Collected By: Md. Raihan Ahmed
JOB INFORMATION FOR ALL STUDENTS (BCS & BANK)

75. The ROM is composed of 78. In MS-Word, the shortcut key Shift+Ctrl+E is used for 75 B
A. magnetic cores A. track changing
B. microprocessors B. mail merge
C. photoelectric cells C. table drawing 76 B

D. liquid disks D. hyper linking


76. _____ is considered to be The Father of Artificial Intelligence, 79. _____ is a dedicated site for professionals. 77 C
A. George Boole A. Wikipedia
B. John McCarthy B. Face book
C. Alan Turing C. Instagram 78 A

D. Allen Newell D. Linkedin


77. ___ is the study of recording, storing and analyzing 80. - 79 D
computer data. A. intellectual property
A. Database B. online banking
B. Big data C. social security 80 A

C. Data Science D. child safety


D. Biostatistics

Job Information For All Students (Bcs & Bank)

73
Collected By: Md. Raihan Ahmed
fuad

InDEx
TOPIC PAGE
1. Investment Corporation Officer - 2011 01 - 08
2. Investment Corporation Senior Officer – 2011 08 - 15
3. Rupali Bank Officer – 2013 15 - 23
4. Rupali Bank Senior Officer – 2013 23 - 32
5. Probashi Kallyan Bank Executive Officer(Cash) - 2014 32 - 40
6. Probashi Kallyan Bank Senior Officer – 2014 40 - 49
7. Bangladesh Development Bank Officer – 2014 49 - 53
8. Investment Corporation Senior Officer – 2014 53 - 58
9. Bangladesh Bank Assistant Director – 2014 58 - 67
Tanisha Tabassum
10. Pubali Bank Junior Officer/Junior Officer(Cash) – 2014 67 - 76
Ummeh Rabeya
11. Pubali Bank Officer/Senior Officer – 2014 76 - 85
12. Pubali Bank Junior Officer – 2016 85 - 91 Raihana Pervin Rontu
13. Pubali Bank Senior Officer - 2016 91 - 95 Mehadi Hassan Fuad
14. Pubali Bank Trainee Assistant Teller – 2017 95 - 103 Abdullah Ashique
15. Bangladesh Bank Assistant Director – 2016 103 - 112
16. Bangladesh Bank Officer(Cash) – 2016 112 - 120
17. Bangladesh Bank Officer(IT) – 2016 120 - 123
18. Janata Bank Assistant Executive Officer – 2015 123 - 131
19. Janata Bank Assistant Executive Officer(Teller) – 2015 131 - 139
20. Janata Bank Executive Officer - 2017 139 - 148
21. Pubali Bank Junior Officer Cash – 2019 148 - 156
22. Pubali Bank Trainee Assistant Officer Cash – 2019 156 - 164

Collected By : Md. Raihan Ahmed


Job Information For All Students (Bcs & Bank)
Job Information For All Students (Bcs & Bank)

(D)
(E)
BANGLA 8 ?
1 ? (A)
(A) 
(C)
(C) (D) 
(D) (E)
(E) 9 ?
2. : (A) 
(A) + 
+ (C)
(C) + (D)
(D) + (E)
(E) + 10 ?
3. ? (A)
(A) 
(C) 
(C) (D)
(D) (E)
(E) 11 ?
4 ? (A)
(A)
(C)
(C)  (D)
(D) (E) 
(E) 12 ?
5 ? (A)
(A) 
(C)
(C) (D)
(D) (E)
(E)  13. ?
6 ? (A)
(A)  
(C)
(C) (D)
(D) (E)
(E) 14
7 ? ?
(A)  (A)

(C) (C) 

1
Collected By: Md. Raihan Ahmed
Job Information For All Students (Bcs & Bank)

(D) (B) Permit


(E) (C) grant
15. ? (D) accede
(A) (E) refuse
 22. The young executive was not chosen for the project
(C) ______ his lack of qualifications.
(D) (A) despite
(B) because
(C) as
(D) although
16. What kind of noun is "cattle"? (E) for
(A) Proper 23. Which of the following is not an adjective?
(B) Common (A) humble
(C) Material (B) humor
(D) Collective (C) humane
(E) Possessive (D) humid
17. The closest meaning of 'fertile' is- (E) honest
(A) useful 24. What is the appropriate meaning of the word
(B) barren 'redundant'?
(C) gloomy (A) few
(D) harmony (B) many
(E) ominous (C) mostly
18. Those customers are ______ the most demanding we've (D) abundant
ever had. (E) unnecessary
(A) so far 25. The word 'ecological' is related to
(B) by far (A) atmospheric
(C) since far (B) biological
(D) to far (C) environmental
(E) too far (D) demographi
19. what is the meaning of the word 'sovereign'? (E) geographical
(A) imperial 26. We need both ornament and implement in our society;
(B) royal we need the artist and the __
(C) loyal (A) beautician
(D) supreme ruler (B) artisan
(E) independent (C) architect
20. What is the noun of the verb 'depart'? (D) politician
(A) departed (E) musician
(B) department 27. Even if you don't ______ what I have to say, I would
(C) departing appreciate your listening to me with an open mind.
(D) departure (A) like
(E) deported (B) anticipate
21. Which of the following is not a synonym of 'consent'? (C) concur with
(A) concede (D) deviate from

2
Collected By: Md. Raihan Ahmed
Job Information For All Students (Bcs & Bank)

(E) hear Solution:


28. I couldn't ______ with him in the race. Let, initial average = x
(A) keep in So, total runs=9x
(B) keep up And total runs scored in 10 innings = 9x +100
(C) keep with So, new average =(9x+100)/10
(D) deep against Since average increase by 8
(E) keep on ATQ,
29. Entrepreneur : Profit x+ 8=(9x+100)/10
(A) Philanthropist : Charity 10x+80= 9x+100
(B) Organizer : Union x = 20
(C) Hermit : Lonely New average 20+8=28
(D) Charlatan : Converts 33. Masud has twice as much money as Selim has and
(E) Scholar : Knowledge Selim has 50% more money than what Bashir has. If the
30. Fish : Aquarium average money with them is Tk. 110, then Masud has :
(A) Tree : Forest (A) 55
(B) Plant : Greenhouse (B) 60
(C) Keeper : Zoo (C) 90
(D) hedge : Garden (D) 180
(E) Star : Planetarium (E) 70
Solution :
MATHAMETICS (Solved By Tanisha tabassum) Let Bashir has = x tk
31. The average of 50 numbers is 38. If two numbers Then selim has= 3x/2 tk
namely 45 and 55 are discarded, the average of the Then Masud has = 3x tk
remaining number is : Now ratio of Masud: Selim: Bashir = 6:3:2
(A) 36.50 They have total = 110 × 3 = 330 Taka
(B) 37 Then Masud × 330 = 180 tk
(C) 37.50 34. A man spends 75% of his income. his income is
(D) 38 increased by 20% and he increased his expenditure by 10%.
(E) 40 His savings are increased by :
Solution: (A) 10%
Average for 50 number is 38. (B) 25%
The total is 38×50=1900 (C) 37.50%
The total of discarded number is 45+55=100 (D) 50%
Hence the total for 48 number is 1900-100= 1800 (E) 45%
So, The new average is 1800÷48=37.5 Solution :
32. A cricketer has a certain average for 9 innings. In the Let income =100
10th innings he scores 100 runs, thus increasing his average Expenditure : 75
by 8 runs. His new average is : So, saving (100 - 75) = 25
(A) 20 New income = 100 + 20% of 100= 120
(B) 24 New expenditure = 75 + 10% of 75 = 82.5
(C) 28 New saving = 120 82.5 = 37.5
(D) 32 Increase of saving = 37.5 25 = 12.5
(E) 35 % increase in saving = (12.5/25) × 100 =50%

3
Collected By: Md. Raihan Ahmed
Job Information For All Students (Bcs & Bank)

35. If a commission of 10% is given on the marked price of 37. An ambulance travels 10 miles at a speed of 50 miles
a book, the publisher gains 20%. If the commission is per hour. how fast must the ambulance travel on the return
increased to 15% the gain is : trip if the round trip travel time is 20 minutes?
(A) 16.67% (A) 75 mph
(B) 13.33% (B) 55 mph
(C) 15.17% (C) 60 mph
(D) 17% (D) 65 mph
(E) 10.5% (E) 30 mph
Solution : Solution :
Let CP= 100 Given, Distance = 10 miles, speed = 50 miles per hour
And Marked price= x Time taken = distance ÷ speed
SP= x- 10% of x = 10÷ 50 = 1/5 hour = 1/5×60 =12 mins
x- 10% of x= 120 Total time = 20 mins
x-10x/100=120 Time for returned trip = 20 - 12 =8 min
9x/100=120 8 mins = 8/60 = 2/15 hours
9x=1200 Speed = distance ÷ time
X=133.33 = 10 ÷ 2/15
If 15% Commission was given, then =10 × 15/2
SP= 133.33- 15% of 133.33 = 150/2
SP= 133.33- 20 =75 mph
SP= 113.33 38. The sum of principal and simple interest of a certain
Gain= 113.33-100=13.33% amount of money would be Tk. 460 after three years from
36. A man donated 5% of his income to a charitable now and Tk. 500 after five years from now. What is the
organization and deposited 20% of the remainder in a related interest rate?
bank. If he now has Tk. 1919 left, then his income is : (A) 18%
(A) Tk. 2300 (B) 6%
(B) Tk. 2500 (C) 5%
(C) Tk 2525 (D) 7.5%
(D) Tk. 2558 (E) 20%
(E) Tk. 2250 Solution:
Solution : (5-3) = 2 years interest = (500 - 460) = 40 tk
Let total income is x = 20 tk
5% for Charitable organisation is 5% of x = 5x/100 = 0.05 x 3 year's interest 20 × 3 = 60 tk
Balance available ( x-0.05x) = 0.95x Principle amount = (460-60) = 400 tk
20% deposited in bank = 20% of 0.095x Interest rate = (60×100)/(400×3) = 5%
= (20 × 0.95x)/100 39. Humayra borrowed Tk. 240, free of interest, to pay for
= .19 x her new hair band. If she pays back 20% of the amount
Now left ( 0.95x - 0.19x) = 0.76x each month and has already paid back Tk. 120. how many
ATQ, months have passed since she took the loan?
0.76x= 1919 (A) 3 months
x= 1919/ 0.76 (B) 3.5 months
x = 2525 (C) 2.5 months

4
Collected By: Md. Raihan Ahmed
Job Information For All Students (Bcs & Bank)

(D) 2 months nor motor bikes?


(E) 4 months (A) 110
Solution: (B) 90
As She pays 20% of the amount each month (C) 50
So, 20% of 240 (D) 30
= 20/100 × 240 (E) 10
= 48 tk Solution:
She paid back 120 tk Given total = 120, n(C)= 55, n( B)=35 and n (C U B) =
So, remaining (240-120)= 120 20, none=?
48 tk pays back in 1 month Total =n(c)+n(B) n(CUB) +none
So, 120 tk pays back in 120/48 = 2.5 months 120= 55+35-20+none
40. The owner of Aarong starts to calculate the percentage None= 50
of customers who purchase Tagas. if 40% of the total 43. the average age of 12 children is 20 years. If the age of
customers who visit the store actually purchase goods and one more child is added, the average decreased by 1. What
15% of them purchase tagas, then what percentage of is the age of child added later?
Aarong's customers are Taga buyers? (A) 5
(A) 4% (B) 7
(B) 6% (C) 19
(C) 8% (D) 20
(D) 12% (E) 15
(E) 13% Solution:
Solution: Average of 12 is 20
Let the total Customers number = 100 Total = 12 × 20 = 240
Then Customers who actually purchase goods = 40 Let the age of one more is x
And Customers who purchase tagas = 40 × 15/100 = 6 So, total 240 + x
Required percentage = 6% Average of 13 is (240 + x)/13
41. The price of desktop computer is discounted by 20%, ATQ
and reduced price is then discounted by 20%. The series of (240 + x)/13 = 19
successive discount is equivalent to a single discount of : 240 + x = 13 × 19
(A) 50% 240 + x = 247
(B) 40% x= 7
(C) 36% 44. The length of a rectangle is 3 times of its width. If the
(D) 28% width of the rectangle is 5 inches, what is the rectangle's
(E) 20% area, in square inches?
Solution: (A) 15
Let the price of desktop computer be 100 (B) 20
Then after first discount price becomes 80 (C) 30
Then again it is discounted by 20% (D) 75
So the Price Becomes = 80 (80 × 20/100) = 64 (E) 40
So a single discount will be = (100 64) = 36%. Solution:
42. Of the 120 households in an area, 55 have cars, 35 Given Width of the rectangle = 5 inches
have motor bikes, and 20 have both cars and motor bikes. So length = 3 × 5 = 15 inches
How many of the households in the area have neither cares

5
Collected By: Md. Raihan Ahmed
Job Information For All Students (Bcs & Bank)

Area of a rectangle is = 5 × 15 = 75 sq. Inch (A) Cache


2
45. For all x >2, (2x +2x-12)/(x-2) simplifies to : (B) PS
(A) 2(x-2) (C) DSP
(B) x+3 (D) Flops
(C) 2(x+3) (x-2) (E) DVD
(D) x-2
(E) 2(x+3)  GENERAL KNOWLEDGE
Solution: 51. Were is the headquarters of the European Union (EU)
2𝑥 2 +2𝑥−12) situated?
=
𝑥−2
(A) London
2(𝑥 2 +𝑥−6)
= (B) The hague
𝑥−2
2(𝑥+3)(𝑥−2)
= (C) Brussels
𝑥−2
= 2(x + 3) (D) Rome
(E) Geneva

COMPUTER 52. Which one is the largest district of Bangladesh area

46. Which one of the following is the first program of wise?

Microsoft? (A) Dhaka

(A) Windows XP (B) Rajshahi

(B) Windows 98 (C) Rangamati

(C) MS DOS (D) Mymansingh

(D) Windows 7 (E) Chittagong

(E) Windows 97 53. When was the first gas field discovered in our country?

47. DOT MATRIX is a kind of : (A) 1950

(A) Scanner (B) 1962

(B) Operating System (C) 1975

(C) Software (D) 1955

(D) Printer (E) 1960

(E) Monitor 54. 'The odyssey' was written by :

48. What is the elaboration of VOIP? (A) Milton

(A) Voice Over Internet Program (B) Tennyson

(B) Voice Over Internet Protocol (C) Shakespeare

(C) Voice of Internet Program (D) Homer

(D) Voice on Internet protocol (E) Shelly

(E) Voice of internet Program 55. The smallest continent of the earth is :

49. The Buying and selling of goods over the internet is (A) Europe

known as : (B) Austrealia

(A) E-buying (C) Antarictica

(B) E-commerce (D) South America

(C) E-selling (E) Africa

(D) E-business 56. When is the International Woman's Day celebrated

(E) E-Banking every year?

50. Which of the following is related to memory of a (A) 8th March

computer? (B) 8th April

6
Collected By: Md. Raihan Ahmed
Job Information For All Students (Bcs & Bank)

(C) 18 th March (A) Master Isha Khan


(D) 18th April (B) Louis Kahn
(E) 7th March (C) Marinal Huq
57. Which of the following is a foreign bank operating in (D) Stephn Johnson
Bangladesh? (E) Leo Nardo Da Vinci
(A) Standard Bank 64. Head office of ILO is located in :
(B) American Express Bank (A) Dhaka
(C) Commercial Bank of Ceylon (B) New York
(D) royal Bank of Scotland (C) Geneva
(E) Premiere Bank (D) Paris
58. The word 'anthrax' is a/an _____ word : (E) Manila
(A) Latin 65. the Banking Companies Act was enacted in Bangladesh :
(B) Greek (A) 1993
(C) Spanish (B) 1991
(D) English (C) 1994
(E) Japanese (D) 1881
59. The committee of the Norwegian Parliament awards the (E) 1996
Nobel Prize for : 66. The law that 'Bad money drives good money out of
(A) Economics circulation' was given by :
(B) Peace (A) Fisher
(C) Medicine (B) Samuelson
(D) Literature (C) Pigou
(E) Physics (D) Marshall
60. The newly elected president of Brazil is : (E) Gresham
(A) Dilma Rouseff 67. Which of the following is not the measure for reducing
(B) Angela Merkel inflation?
(C) Hillary Clinton (A) increase in production
(D) Gloria Sebastian (B) reduction of money supply
(E) Maria Gonzalez (C) increase in bank rate
61. Who was the first chief Justice of Bangladesh? (D) reduction of reserve requirement
(A) A S M Sayem (E) selling government securities
(B) A N Hamidullah 68. The main source of government revenue of Bangladesh
(C) Kamrul Hasan is-
(D) Abu Hena Mostafa Kamal (A) Income Tax
(E) Barrister Abul Hasnat (B) Corporate Tax
62. Which country has adopted Bangla as one of their state (C) Value Added Tax (VAT) 
languages? (D) Capital Gain Tax
(A) Rwanda (E) Excise Duty
(B) Eritrea 69. Investment Corporation of Bangladesh (ICB) was
(C) Sierra Leon established in :
(D) Liberia (A) 1972
(E) Sudan (B) 1980
63. Who was the architect of Tajmahal? (C) 1982

7
Collected By: Md. Raihan Ahmed
Job Information For All Students (Bcs & Bank)

(D) 1990
(E) 1976
70. BDBL stands for : BANGLA
(A) Bureau for Development of Bank Ltd. 1 ?
(B) Basic Development Bank Ltd. (A)
(C) Bangladesh Development Bank Ltd.
(D) Bangladesh Development Bureau Ltd. (C) 
(E) Bagura Development Bank Ltd. (D)
71. SLR stands for: (E)
(A) Self Leading Restriction 2. ?
(B) Statutory Liquidity Reserve (A)
(C) Strict Liquid Ratio
(D) Statutory Level of reserve (C)
(E) State for Liberty and Rights (D) 
72. What could be the minimum possible value of a share (E)
of a joint stock company? 3. ?
(A) Face Value (A)
(B) Net Asset Value (NAV)
(C) Liquidating Value (C) 
(D) Zero Value (D)
(E) Replacement Value (E)
4. ?
73. The free market involves : (A)
(A) the free provision of products
(B) Subsidizing products by the government (C)
(C) Market forces of supply and demand (D)
(D) All trade without licenses (E) 
(E) Free import and export of products 5 ?
(A)
74. The Dhaka stock exchange (DSE) was established in :
(A) 1972 (C) 
(B) 1952 (D)
(C) 1954 (E)
(D) 1956 6 ?
(E) 1993 (A)

75. What does 'ATM' stand for? (C)


(A) Automatic Teller Machine (D) 
(B) Automated Teller machine (E)
(C) Augmented Teller Machine 7. ?
(D) Automatic Telling Machine (A)
(E) Automated Transfer Machine
(C)

8
Collected By: Md. Raihan Ahmed
Job Information For All Students (Bcs & Bank)

(D)  (E)
(E) 15 ?
8 - ? (A) +
(A) +
 (C) + 
(C) (D) +
(D) (E) +
(E)
9 ?
(A) 16. Look far away _____ five minutes _____ every thirty
minutes of reading or using the computer.
(C) (A) in, in
(D) (B) from, to
(E)  (C) for, after
10. . ? (D) like, in
(A) (E) between, after
17. Lily hardly helps her mother with the house work
(C) nowadays, _____ ?
(D)  (A) is it
(E) (B) does she
11. , , , ? (C) isn't it
(A) (D) doesn't she
(E) isn't she
(C)  18. The government has stepped ______ measures to curb
(D) wine flu.
(E) (A) in
12. ? (B) out
(A) (C) on
(D) up
(C)  (E) over
(D) 19. They learned all their lines for the play ______.
(E) (A) for heart
13 ? (B) by heart
(A) (C) from heart
(D) to heart
(C)  (E) in heart
(D) 20. He should arrive by six o'clock ______.
(E) (A) at the latest
14 ? (B) by the latest
(A) (C) top the latest
(D) in the latest
(C)  (E) for the latest
(D)

9
Collected By: Md. Raihan Ahmed
Job Information For All Students (Bcs & Bank)

Question(21-23): Find the correct option for the bracketed (B) happy
part. (C) messy
21. Most students like to read (these kind of books) during (D) compulsory
their spare time. (E) reckless
(A) these kind of books 28. Linguistics is the science of :
(B) these kind of book (A) Music
(C) this kind of book (B) Language
(D) this kinds of books (C) Behavior
(E) those kind of books (D) Visuals
22. I have never heard (that kind of an excuse). (E) Handwriting
(A) that kind of an excuse 29. A 'Philanthropist' is one who :
(B) that sort of an excuse (A) Loves mankind
(C) that kinds of excuse (B) hates mankind
(D) these kinds of excuse (C) studies plants
(E) that kind of ecuse (D) kills his kin
23. (If I would have known about) the traffic jam at (E) loves him/herself
Nilkhet, I would have taken an alternative route. 30. Where was Ernest Hemingway born?
(A) If I would have known (A) England
(B) If I could have kinown (B) New Zealand
(C) If I would of known (C) Australia
(D) If I was aware (D) Canada
(E) Had I known (E) USA
24. Choose the most nearly opposite to the word : Eligible
(A) idealistic MATHAMETICS
(B) reasonable 31. The sum of five consecutive integers is 105. The sum of
(C) turbulent the first two is :
(D) unsuitable (A) 19
(E) pacific (B) 21
25. Choose the most nearly opposite to the word : Enclose (C) 39
(A) prevent (D) 41
(B) begin (E) 45
(C) take-out Solution:
(D) give m + (m+1 ) + (m+2) + (m+3) + (m+4) = 105
(E) endanger 5m + 10 = 105
26. Choose the synonym of the following words : Resort 5m = 95
(A) efficiency m = 19
(B) defects First 2 number is 19 and 20
(C) remedy Sum of the first two is = 19 + 20 = 39
(D) disadvantages 32. If 5x + 2y = 10 and x + 8y = 46, then 3x + 5y equals :
(E) paradise (A) 14
27. Choose the synonym of the following words : (B) 28
Mandatory (C) 56
(A) massive

10
Collected By: Md. Raihan Ahmed
Job Information For All Students (Bcs & Bank)

(D) 112 of the square decreases by :


(E) 45 (A) 14%
Solution: (B) 21%
Given 5x + 2y = 10 and x + 8y = 46 (C) 49%
By solving we get x = -6/19 and y = 110/19 (D) 51%
Now 3x + 5y (E) 70%
= 3(-6/19) + 5(110/19) Solution:
= -18/19 + 550/19 Suppose the diagonals of the square = 100 cm.
= 532/19 The area of the square = 100*100/2 = 5,000 sq cm.
= 28 Now the diagonals are reduced to 70 cm. So its area =
33. An overnight express firm charges Tk. 2 per kg for the 70*70/2 = 2450 sq cm.
first 10 kg of packet weight and Tk. 1 for each kg of or So by reducing the diagonals of the square by 30%, the
fraction of kg over 10 kg of packet weight. How much will it area becomes (5000 2450)*100/5000 = 51% of the
charge to ship a package which weighs 17.5 kg? original.
(A) 8 Conclusion: By reducing the diagonals of a square by 30%,
(B) 20 its area will reduce by 49%.
(C) 28 37. How many of the integers between 150 and 160 are
(D) 36 prime numbers?
(E) 48 (A) 1
Solution: (B) 2
10 × 2 = 20 tk (C) 3
7.5 × 1 = 7. 5 = 8 tk [ Because fraction kgs will also charge (D) 4
1 tk] (E) 5
Total cost = 20 + 8 = 28 tk Solution:
34. Of the following fractions, which one is less than 2/3? 151 and 157
(A) 7/8 38. City B is 4 miles east of City A. City C is 3 miles north
(B) 5/6 of City B. City D is 8 miles east of City C and City E is 6
(C) 3/4 miles north of City D. What is the distance from City A to
(D) 3/5 City E?
(E) 5/7 (A) 5 miles
Solution: (B) 10 miles
2/3 = 0.6666 and 3/5 = 0.6 (C) 15 miles
35. At a selling price of Tk. 275 a book yield 25% profit. (D) 20 miles
What selling price will yield the profit of 10%? (E) 25 miles
(A) 210 Solution: E
(B) 242
(C) 221
(D) 235
C D
(E) 240
Solution: A
Cost price = 275 × 100/125 = 220 B R
Selling price at 10% profit = 220 × 110/100 = 242 AR = AB + BR = 4 + 8 = 12
36. If the side of a square decreases by 30%, then the area RE = RD + DE = 3 + 6 = 9

11
Collected By: Md. Raihan Ahmed
Job Information For All Students (Bcs & Bank)

AE2 = AR2 + RE2 At least One paperback= {total} - {none} = 70 - 15 = 55.


AE = √144 + 81 = 15 42. When tossed, a certain coin has equal probability of
landing on either side. If the coin is tossed 3 times, what is
39. Mr. Rahim purchased one lot of shares (50 shares per the probability that it will land on the same side each time?
lot) of company X for Tk. 5000 and sold 3/5th of it after its (A) 1/8
value doubled. He sold the remaining shares at a price 3 (B) 1/4
times higher than the original purchase price. What was his (C) 1/3
total profit? (D) 3/8
(A) Tk. 5000 (E) ½
(B) Tk. 5500
Solution:
(C) Tk. 6000
We need to determine the probability that a coin lands on
(D) Tk. 7000
the same side for all 3 tosses of the coin. This means that
(E) Tk. 7500
the 3-toss outcome will be either H-H-H or T-T-T.
Solution:
P(T-T-T) = 1/2 x 1/2 x 1/2 = 1/8
Cost price per share = 5000/50 = 100
P(H-H-H) = 1/2 x 1/2 x 1/2 = 1/8
3/5 of the share = 30 share
Thus, the probability that the coin lands on the same side
Sold 30 share at = 30 × (100 × 2) = 6000 tk
each time is 1/8 + 1/8 = 2/8 = 1/4.
Sold 20 share at = 20 × (100 × 3) = 6000 tk
43. Of the fruits that arrive at the cannery, 20 percent by
Profit = (6000 + 6000) 5000 = 7000 tk
weight is rejected before processing. Of the fruit that is
40. (-5)(4)(2)(-1/2) (3/4) = ?
processed, 15 percent by weight is rejected before canning.
(A) -20
Of the fruit that arrives at the cannery. what percent by
(B) -40
weight is canned?
(C) 20
(A) 32%
(D) 15
(B) 35%
(E) 18
(C) 65%
Solution:
(D) 68%
(-5)(4)(2)(-1/2) (3/4)
(E) 70%
(−5) ×4 ×2 ×(−1) ×3
= Solution:
2 ×4
= 15 Let Fruits arrived at cannery = 100
41. There are 8 books on a shelf, of which 2 are Selected for processing = 100 20 = 80
paperbacks and 6 are hardbacks. how many possible Percentage of canned fruits = 80 (80 × 15/100) = 68%
selections of 4 books form this shelf include at least 1 44. If x percent of 40 is y, Then 10x equals :
paperback book? (A) 4y
(A) 40 (B) 10y
(B) 45 (C) 25y
(C) 50 (D) 100y
(D) 55 (E) 400y
(E) 60 Solution:
Solution: x% of 40 = y
x = 100y/40
Selections of 4 books out of 8: 8C4 = 70;
10x = 25y
Selections of 4 hardback books out of 6 (so none
45. If x/y = 5/z, then 25y2 equals
paperback): 6C4 =15;
(A) x2 /25

12
Collected By: Md. Raihan Ahmed
Job Information For All Students (Bcs & Bank)

(B) x3 z2 GENERAL KNOWLEDGE


2 2
(C) x z  51. Which of the following banks promoted the programme
2
(D) 5x of 'Green Banking' in Bangladesh?
2 2
(E) (1/5) x z (A) HSBC Bank Ltd.
Solution: (B) Bangladesh Bank
x/y = 5/z (C) Janata Bank
xz = 5y (D) Grameen Bank
2 2 2
25y = x z (E) Dutch Bangla Bank Ltd.
52. Which of the following institutions do not perform
COMPUTER commercial banking function?
46. What does MICR mean? (A) Bangladesh krishi Bank
(A) Magnetic Idle Cassette Recorder (B) Rajshahi Krishi Unnayan Bank
(B) Magnetic Ink Code Reader (C) IDLC Finance Ltd.
(C) Magnetic Ink Cases Reader (D) Social Islami Bank
(D) Magnetic Ink Character Reader (E) IFIC Bank
(E) magnetic Ink Code Reader 53. Which of the following is not a function of central bank?
47. VGA stands for: (A) Open market operation
(A) Video Graphics Array (B) Lender of the last resort
(B) Visual Graphics Array (C) Issue notes
(C) Volatile Graphics Array (D) Provide long-term credit
(D) Video Graphics Adapter (E) Devise monetary policy
(E) Visual Gallery Adapter 54. What is the current name of the dissolved 'The Oriental
48. Which of the following is not an output device? bank Limited'?
(A) Monitor (A) Al-Arafah Islami Bank Limited
(B) Printer (B) ICB Islamic Bank Limited
(C) Mouse (C) Jamuna Bank
(D) Speaker (D) ICB Bank Limited
(E) Projector (E) Shahjalal Islami Bank Limited
49. The 'add or remove programs' utility can be found in : 55. Which of the following is the main regulator of the
(A) Control panel securities market of Bangladesh?
(B) CPU (A) Bangladesh Bank
(C) Desktop (B) ICB
(D) Search Engine (C) Dhaka Stock Exchange
(E) My Computer (D) SEC
(E) Controller of Capital Issues
50. Which of the following is a spreadsheet software? 56. According to the new law adopted in 2010 for the
(A) MS Excel insurance sector, which institution will be responsible for
(B) MS Word governing the insurance sector?
(C) PowerPoint (A) Controller of Insurance
(D) Adobe Acrobat (B) Insurance Regulatory commission
(E) Fox Pro (C) Insurance commission
(D) Insurance Controlling Authority
(E) Insurance Regulatory Authority

13
Collected By: Md. Raihan Ahmed
Job Information For All Students (Bcs & Bank)

57. The interest rate charged for inter-bank borrowing is (E) to decrease export
called : 64. Greenland belongs to which country?
(A) Call money rate (A) USA
(B) Bank rate (B) UK
(C) Prime rate (C) Denmark
(D) Floating rate (D) Canada
(E) Financing rate (E) Norway
58. An actuary is a professional working in the : 65. The Gurkhas are original inhabitants of :
(A) Banking sector (A) India
(B) Insurance sector (B) China
(C) Leasing sector (C) Nepal
(D) Chemical sector (D) Pakistan
(E) Pharmaceutical sector (E) Bhutan
59. CDBL, stands for : 66. Which chemical element is represented by symbol Zn?
(A) Central Deposityory bangladesh Limited (A) Zinc
(B) Common Depository Bank Limited (B) Boron
(C) Central Development Bank Limited (C) Chloride
(D) Common Development Bank Limited (D) Folic acid
(E) Central Depository Bank Limited (E) Lutien
60. Which of the following is not an investment bank? 67. International Environment Day is on :
(A) Salomaon Brothers (A) March 7
(B) Goldman Sacks (B) June 5
(C) ICB (C) February 16
(D) IFIC (D) November 16
(E) Merrill Lynch (E) January 31
61. The author of Harry Potter book is : 68. IMF is the result of :
(A) Alva Fisher (A) Havana Conference
(B) JK Rowling (B) Bretton Woods Conference
(C) James King (C) Geneva Conference
(D) John hanke (D) Rome Conference
(E) Shakespeare (E) Asia Conference
62. How many teams are playing in ICC world Cup Cricket 69. 'City of Golden Gate' is :
2011? (A) London
(A) 10 (B) Paris
(B) 12 (C) San Francisco
(C) 16 (D) Havana
(D) 14 (E) New York
(E) 11 70. The normal temperature of human body in Celsius Scale
63. What is the main objective of currency devaluation? is :
(A) to increase import (A) 35.7
(B) to increase export (B) 36.5
(C) to increase purchasing capacity (C) 36.9
(D) to increase money supply (D) 37.8

14
Collected By: Md. Raihan Ahmed
Job Information For All Students (Bcs & Bank)

(E) 39.4 (C)


71. Nobel Laureate in economics in 2010 was : (D)
(A) Paul Krugaman 2 /
(B) Oliver Williamson - :
(C) Peter Diamond, Dale Mortensen, and Christopher (A)
Pissarides
(D) Eric Maskin (C) 
(E) None of these (D)
72. Bangladesh secured her first One Day International 3 -
series win against : (A)
(A) Indai
(B) Sri Lanka (C) - 
(C) West Indies (D)
(D) New Zealand 4.
(E) Zimbabwe ?
73. Peace Bridge is situated between the borders of : (A) , 
(A) Israel and Palestine ,
(B) North Korea and South Korea (C) ,
(C) India and Bangaldesh (D) ,
(D) India and Pakistan 5. ?
(E) USA and Canada (A) ,
74. Which of the country is known as 'The country of , 
copper'? (C) ,
(A) India (D) ,
(B) Thailand 6. ?
(C) Zamibia (A) -
(D) Ireland
(E) USA (C)
75. What is the name of organization working worldwide (D) 
against corruption? 7. - ?
(A) Green peace (A)
(B) Transparency International 
(C) Amnesty International (C)
(D) Interpol (D)
(E) UNDP 8 -
-
(A)

BANGLA (C) 
1 - (D)
: 9. ?
(A) (A) 


15
Collected By: Md. Raihan Ahmed
Job Information For All Students (Bcs & Bank)

(C) (A)
(D) 
10. ? (C)
(A) (D)
19 -
(C) (A)
(D) 
11. ? (C) 
(A) (D)
20. ?
(C)  (A) - 
(D) -
12. ? (C) -
(A) (D) -
21. ?
(C) (A)
(D) 
13 - (C) - 
(A) (D)
22 -
(C)  (A)
(D)
14 - (C) 
(A)
 23 -
(C) (A)
(D)
15 - (C) 
(A) (D)
24. 'Payer' -
(C)  (A)
(D)
16. ? (C)
(A) ,  (D) 
, 25. 'Scroll' -
(C) , (A) 
(D) ,
17. ? (C)
(A) (D)

(C)
(D)
18 -

16
Collected By: Md. Raihan Ahmed
Job Information For All Students (Bcs & Bank)

(A) Espy
Question(26-30): Select correct spelling of the word in (B) Defame
capital. (C) Acquire
26. EMBARASSMENT (D) Cogitate
(A) Embrarassment 34. INDICTMENT
(B) Embarrasment (A) Eulogy
(C) Embarrassment (B) Tirade
(D) Embarasment (C) Crime
27. CALENDER (D) None of them
(A) Calender 35. FASTIDIOUS
(B) Calander (A) Monotonous
(C) Callendar (B) Aloof
(D) Calendar (C) Easygoing
28. RHYTHEM (D) Interminable
(A) Rhythem
(B) Rythim Question(36-40): Select from the options, the pair having a
(C) Rhythm similar relationship to the given pair.
(D) Rythem 36. BIOLOGY : LIFE SYSTEM
29. KINDERGARDEN (A) science : facts
(A) Kindergarden (B) petrology : transportation
(B) Kindegarten (C) anthropology : human kind's development
(C) Kindergarten (D) graphology : law
(D) Kindargarten 37. DEGREE : TEMPARATURE
30. MESCHIEVOUS (A) Ounce : Weight
(A) Mischievous (B) Fathom : Volume
(B) Mischievious (C) Mass : Energy
(C) Mischivous (D) Light : Heat
(D) Mischievious 38. GUN : HUNTING
(A) Driver : Springboard
Question(31-35): Choose the word which is most nearly (B) Horse : Polo
opposite in meaning to word in capital letters. (C) Bat : Cricket
31. TACITURN (D) Recquet : Tennis
(A) Spoken 39. FOREST : TRESS
(B) Talkative (A) Lawn : GRASS
(C) Leisurely (B) Shore : Lake
(D) Placatory (C) Valley : Stream
32. CASTIGATE (D) Ocean : Rocks
(A) Flatten 40. DEPTH : SEA
(B) Change (A) Ray : Sun
(C) Extol (B) Length : Width
(D) None of them (C) Peak : Valley
33. SALUTE (D) Height : Mountain

17
Collected By: Md. Raihan Ahmed
Job Information For All Students (Bcs & Bank)

MATHAMETICS (Solved By Ummeh Rabeya) (A) 12, 5


41. The least number of 4 digits which is a perfect square, (B) 13, 4
is : (C) 14, 3
(A) 1000 (D) 24, 10
(B) 1016
(C) 1024 Let, bigger number is x, smaller is y
(D) 1036 A/q,
(x+y)x = 204------(1)
e know, least 4 digit numbers is 1000 (x-y)y= 35------(2)
We get ,√1000= 31.62 ----------------------
So , least 4 digit numbers that perfect square is 32^2= x + y = 169------(3)
1024 From 1 equation,
42. The average of 7 consecutive number is 20. The largest x = 35/y + y
of these numbers is : From 3 equation we get,
(A) 20 x + y = 169
(B) 22 (35/y + y) + y = 169
(C) 23 1225/y + 70 + 2y =169 [( a+b ) ]
(D) 24 1225/y + 2 y = 99
1225 + 2 y = 99y
Let the consecutive numbers are, x x+1 x+2 x+3 2 y - 99y + 1225= 0
x+4 x+5 x+6 2y - 50y - 49y + 1225 = 0
2y (y - 25 ) - 49(y - 25)=0
+ x +1 + x + 2 + x + 3 + x + 4 + x + 5 + x + 6 /7=20 y =25
7x + 21 = 20 * 7 y =5
7x = 140 -21 x= 35/5+ 5 = 12
7x = 119 x=12, y= 5
= 17 45. A is two years older than B who is twice as old as C. If
43. If one-third of one-fourth of a number is 15, then the total of the ages of A, B and C be 27, Then how old is
three-tenth of that number is : B?
(A) 35 (A) 7
(B) 36 (B) 8
(C) 45 (C) 9
(D) 54 (D) 10

Let the number is x Let , C = x


B = 2x
(x/4) * (1/3) = 15 A = 2x + 2
x/12 = 15
x = 180 X + 2x + 2x + 2 = 27
180 * 3/10 = 54 5x = 25
44. The sum of two positive integers multiplied by the X=5
bigger number is 204, and their difference multiplied by the So B is 5 * 2 = 10 years old
smaller number is 35. The numbers are :

18
Collected By: Md. Raihan Ahmed
Job Information For All Students (Bcs & Bank)

46. If 35% of a number is 12 less than 50% of that number, (A) Tk. 45,000
then the number is : (B) Tk. 50,000
(A) 40 (C) Tk. 60,000
(B) 50 (D) Tk. 80,000
(C) 60
(D) 80 Let invest tk
40000 : = 2/3
Let the number is x B = 40000 *3/2
35x/100 = (50x/100) -12 B = 60,000
12 = (50x/100) - (35x/100) 50. If 5 men and 9 women can do a piece of work in 19
x = 20 * 12/3 days, then in how many days will 3 men and 6 women do
x = 80 the same work?
47. If selling price is doubled, The profit triples. Find the (A) 12
profit percent. (B) 15
(A) 66 (C) 18
(B) 100 (D) 21
(C) 105
(D) 120 5 men do 19 days work 1 part
1 man does 1 day work 1/ 5 * 19
Let cost price = x 3 men do 1 day work 3/95
Selling price = y Again, 9 women do 19 days work 1 part
A q, 1woman does 1 day work 1/ 19 * 9
3(y - x) = 2y - x 6 women do 1 day work 6/ 9 * 19 = 2/57
y = 2x ( 3m + 6w ) do in 1 days 3/95 + 2/57 = 1/15 part
So , profit = 2x- x = x 1/15 part do in 1 day
Profit percentage = x * 100/x = 100% 1 part do in = 15 days
48. The ratio of three numbers is 3 : 4 : 5 and the sum of 51. A person travels form P to Q at a speed of 40 kmph
their squares is 1250. The sum of the numbers is : and returns by increasing his speed by 50%. What is the
(A) 30 average speed for both the trips?
(B) 50 (A) 36 kmph
(C) 60 (B) 45 kmph
(D) 90 (C) 48 kmph
(D) 50 kmph
Let the numbers are 3x, 4x, 5x
A q, 50% increase of 40km = 60km
(3x) + (4x) + (5x) = 1250 We know ,
50x = 1250 verage speed = 2xy/x+y
x = 25 2 * 40 * 60 / 40+60 = 48 km/h
x=5 52. A train 360 m long is running at a speed of 45
Sum of the numbers (15 + 20 + 25) = 60 km/hour. In what time will it pass a bridge 140 m long?
49. X and Y invested in a business. They earned some profit (A) 40 seconds
which they divided in the ratio of 2 : 3. If X invested Tk. (B) 42 seconds
40,000 the amount invested by Y is :

19
Collected By: Md. Raihan Ahmed
Job Information For All Students (Bcs & Bank)

(C) 45 seconds We know , c= p(1+r/100)^n


(D) 48 seconds
1348.32 = 1200 (1 + r/100)^2
45 k /h = 45 * 5/18 = 25/2 m/s 1.1236 = {(100 +r )/100}^2
t = 140 + 360 * 2/25 11236 = (100+r)^2
= 500 * 2/25 = 40 s √11236 = 100 + r
53. The speed of a boat in still water is 10 km/hr. If it can 106 - 100 = r
travel 26 km downstream and 14 km upstream in the same = 6%
time, the speed of the stream is : 56. The area of a rectangle is 460 square meters. If the
(A) 2 km/hr length is 15% more than the breadth, what is the breadth
(B) 2.5 km/hr of the rectangular field?
(C) 3 km/hr (A) 15 meters
(D) 4 km/hr (B) 26 meters
(C) 34.5 meters
Let current speed = x (D) None of these
Downstream = 10 + x Solution:
Upstream = 10 - x Let , breadth x,
length = 15% more than breadth = 115x/100
26/ 10+x = 14/ 10-x ATQ,
140 + 14x = 260 - 26x x * 115x/100 = 460
x = 3 km x2= 400
54. A sum of money amounts to Tk. 9800 after 5 years and x = 20
Tk. 12005 after 8 years at the same rate of simple interest 57. In a 500m race, the ratio of the speeds of two
per annual is : contestants A and B is 3 : 4. A has s start of 140 m. Then,
(A) 5% A wins by :
(B) 8% (A) 60m
(C) 12% (B) 40m
(D) 15% (C) 20m
(D) 10m
8 years amount 12005 Solution:
5 years amount 9800 A has to cover (500-140) = 360
3 years amount =3205 A goes 3m when B goes 4m
1 year amount 3205/3 So A goes 360 m when B goes (360*4)/3= 480m
& 5 years amount 3205 * 5 /3 = 3675 tk A wins by (500-480)= 20m
So, principal = 9800 - 3675 = 6125tk 58. The angle between the minute hand and the hour hand
r = (3675*100)/ 6125*5 = 12% of a clock when the times 4.20?
55. At what rate of compound interest rate per annual will (A) 00
a sum of Tk. 1200 become Tk. 1348.32 in 2 years? (B) 10
(A) 6% (C) 5
(B) 6.5% (D) 20
(C) 7% Solution:
(D) 7.5% We know hour's hand turn per minute 1/2°
Here, 20 minutes go 10 °

20
Collected By: Md. Raihan Ahmed
Job Information For All Students (Bcs & Bank)

So, angle between the hour hand & the minute hand of a (C) 54%
clock is 10° (D) 64%
59. The market value of a 10.5% stock, in which an income Solution:
of Tk. 756 is dervided by investing Tk.900, brokerage being Let a number is x
1/4%, is: Error,
(A) Tk. 108.25 = 5x/3 - 3x/5
(B) Tk. 112.20 = (25x-9x)/15
(C) Tk. 124.75 = 16x/15
(D) Tk. 125.25 So, the percentage of error
Solution: = (16x * 100 *3)/(15 * 5x)
If income 756 then investment 900 = 64%
If income 10.5 then investment (900 * 10.5)/756 = 125 63. A certain amount is distributed among A, B and C. A
Market value = 125 -1/4 = 124.75 gets 3/10 and B 1/4 of the whole amount . If C gets Tk. 81
60. In how many ways can the letters of the word 'LEADER' then B gets:
be arranged? (A) 45 TK. 
(A) 72 (B) 32 TK.
(B) 144 (C) 36 TK.
(C) 360 (D) 40 TK.
(D) 720 Solution:
Solution: Let a certain amount is x
Here LEADER = 6! A = 3x/10, B = x/4
& Repeat word = 2! ATQ,
The word can be arraged in 6!/2! = 360 ways (3x/10) + (x/4) = x - 81
61. When the numerator of a frction increases by 4, the 11x = 20x - 1620
fraction increases by 2/3. The denominator of the fraction 9x = 1620
is: x = 180
(A) 2 So B gets, 180/4 = 45 Tk
(B) 3 64. A, B and C can complete a piece of work in 24, 6 and
(C) 4 12 days respectively . Working together , They will complete
(D) 6 the same work in:
Solution: (A) 4 * 1/3 hrs. 1/14 day
Let the numerator is x & denominator is y (B) 7/24 day
So the fraction is x/y (C) 3 * 3/7day
ATQ, (D) 4 days
(x + 4)/y= x/y + 2/3 Solution:
(x + 4)/y = (3x + 2y)/3y 1 day A does 1/24 part
2
3xy + 12y = 3xy + 2y 1 day B does 1/6 part
2
12y = 2y 1 day C does 1/12 part
y= 6 They do together in 1 day ( 1/24+1/6+1/12) part
62. A student multiplied a number by 3/5 instead of 5/3. = 21/72 = 7/24
What is the percentage of error in the calculation? 7/24 part do in 1 day
(A) 34% So 1 part do 24/7 day = 3 * 3/7 day
(B) 44%

21
Collected By: Md. Raihan Ahmed
Job Information For All Students (Bcs & Bank)

65. A pump can fill with water in 2 hours. Because of a (B) Bank of England
leak, it took 2*1/3hours to fill the bank. The leak can drain (C) Federal Reserve System
all water of the tank in: (D) State Bank of India
(A) 4 *1/3 hrs. 71. According to Forbes Which was the most Valuable
(B) 7 hrs. football club in the world for 2012?
(C) 8 hrs (A) Barcelona
(D) 14 hrs. (B) Real Madrid
Solution: (C) Manchester United
Because of leak fill the tank in 7/3 h (D) None of them
In 1 hour fill the tank 3/7 part 72. Morisi was ousted form power in a military coup in
Fill the tank in 2 hour Egypt on
So in 1 hour fill the tank 1/2 part (A) 12 June 2013
ATQ, (B) 23 August 2013
1/2 - 3/7 = 1/14 part in 1 hour (C) 3 July 2013
So, 1 part fill the tank 14 hour (D) 29 May 2013
73. ECNEC stands for
GENERAL KNOWLEDGE (A) Executive Committee of National Economic Councilo
66. Ban Ki-moon is the _____ General Secretary of the (B) Executive Council of National Economic Committee
United Nations. (C) Economic Council of National Executive Committee
(A) 7th (D) Economic Committee on National Executive Council
(B) 8th 74. What does BRAC stand for?
(C) 9th (A) Bangladesh Rural Advancement Commission
None of these (B) Bangladesh Rural Advancement Company
67. Russia allowed whistle blower Edward Snowden (C) Bangladesh Rural Advancement Corporation
temporary asylum on (D) Bangladesh Rural Advancement Committee
(A) 15 August 2013 75. Who is Harry Potter?
(B) 1 August 2013 (A) A name of a historical drama
(C) 31 December 2012 (B) A name of a character in a novel
(D) 18 July 2013 (C) A name of writer of a novel
68. The largest stock exchanges in the world by market (D) A name of an actor
capitalization in 2013 is 76. The current Venezuelan president is
(A) London Stock Exchange (A) Nicolas Maduro
(B) Tokyo Stock Exchange (B) Hugo Chavez
(C) New York Stock Exchange (C) Victor Chavez
(D) None of them (D) Jose Maria Vargas
69. The world's most heavily fortified border is located 77. Where is the headquarters of International Labour
between Office?
(A) North and South Korea (A) London
(B) Israel and Lebanon (B) Geneva
(C) India and Pakistan (C) Paris
(D) North and South Sudan (D) New York
70. Which is not a Central Bank? 78. Who regulates the Indian Stock Market?
(A) Bangladesh Bank (A) State Bank of India

22
Collected By: Md. Raihan Ahmed
Job Information For All Students (Bcs & Bank)

(B) Security and Exchange Board of India (D) Ministry of Land


(C) Security Exchange Commission of India 86. The 'Swatch of No Ground' hole is located in
(D) India Security Exchange Commission (A) The Meghna
79. What is the terminal year of the Bangladesh Perspective (B) Goalanda
Plan? (C) Aricha
(A) 2015 (D) The Bay of Bengal
(B) 2020 87. What index is used in Bangladesh measure Inflation?
(C) 2016 (A) GDP Deflator
(D) 2021 (B) Wholesale Price Index
80. Which European country first recognized Bangladesh as (C) Retail Price Index
a sovereign state? (D) Consumer Price Index
(A) Albania 88. Which countries belong to MINT?
(B) East Germany (A) Mexico, Ireland, Nigeria and Turkey
(C) Poland (B) Mexico, Indonesia, Nigeria and Turkey
(D) Romania (C) Malaysia, Indonesia, Nigeria and Turkey
81. Bangladesh is not member of which of the following (D) Malaysia, Iran, Nigeria and Turkey
organization? 89. The first female Director General of the Bangla Academy
(A) IMF was
(B) ILO (A) Nilima Ibraham
(C) OIC (B) Begum Sufia Kamal
(D) OPEC (C) Lily Islam
82. When was the World Trade Organization (WTO) (D) Sanjida Khatun
established? 90. Which is the largest Cricket Stadium by capacity?
(A) 1st Junary 1992 (A) The Lords
(B) 1st January 1993 (B) Melbourne Cricket Ground
(C) 1st July 1994 (C) Eden Gardens
(D) 1st January 1995 (D) None of them
83. The headquarter of World intellectual Property
Organization (WIPO) is in
(A) New Yourk
(B) Geneva BANGLA
(C) Washington D.C. 1. ?
(D) Vienna (A)
84. The number of Juvenile Correction Center in Bangladesh
is (C) 
(A) 5 (D)
(B) 10 2. - ?
(C) 1 (A) - 
(D) 3 -
85. The survey of Bangladesh is under which Ministry? (C) -
(A) Ministry of Planning (D) -
(B) Ministry of Defence 3. + =
(C) Ministry of Home (A)

23
Collected By: Md. Raihan Ahmed
Job Information For All Students (Bcs & Bank)

(C) 
(C)  (D) -
(D) 12. , , , -
4. ? (A) 
(A)
(C)
(C) 
(D) 13. ?
5. ? (A)
(A)
(C)
(C)  (D) 
(D) 14. ?
6. Currect account ' (A)
(A) 
(C)
(C)  (D)
(D) 15. -
7. ? (A)
(A)
(C)
(C)  (D) 
(D) 16. -
8. - (A)
-
(A) (C) 
(D)
(C) 17. - ?
(D)  (A) ,
9 -
(A) (C) ,
 (D) , 
(C) 18. -
(D) (A)
10. -
(A) (C) 
(D)
(C) 19.
(D)  ?
11. ? (A) -
(A) - -
- (C) - 

24
Collected By: Md. Raihan Ahmed
Job Information For All Students (Bcs & Bank)

(D) - (D) Accompliesh


20. - 28. Choose the correctly spelt word.
(A) (A) Vulantary
(B) volantery
(C) (C) Voluntary
(D)  (D) None to the above
21. - 29. Choose the correctly spelt word.
(A) (A) Houmourous
(B) Humorous
(C) (C) Humourus
(D)  (D) Humoros
22. 30. Choose the correctly spelt word.
? (A) Superceed
(A) (B) Supersede
(C) Superseed
(C) (D) Supercede
(D) 
23. ? Question(31-35): Choose the word that conveys the same
(A) meaning as the word given in capital letters
 31. IMPUTE
(C) (A) blame
(D) (B) attribute
24. - (C) apply
(A) (D) expect
32. ANXIETY
(C)  (A) worry
(D) (B) comfort
25. ? (C) faith
(A) (D) reassurance
33. EXIGENCY
(C) (A) dilemma
(D)  (B) constraint
(C) demand
(D) occasion
26. Choose the correctly spelt word. 34. SERENITY
(A) Pietty (A) agitation
(B) Piety (B) protest
(C) Pitie (C) stir
(D) None of the above (D) calmness
27. Choose the correctly spelt word. 35. REFUTE
(A) Accomplish (A) void
(B) Accomplesh (B) frustrate
(C) Accomphish (C) disprove

25
Collected By: Md. Raihan Ahmed
Job Information For All Students (Bcs & Bank)

(D) hinder 'The setting of a story (effects the story's plot)'


36. My uncle arrived while I ______ the dinner. (A) effect the story's plot
(A) would cook (B) effect the stories plot
(B) cooked (C) affects the story's plot
(C) had cooked (D) None of the above
(D) was cooking 45. Choose the one that best expresses the meaning of the
37. The candles have been blown ______ by the wind. given word : AMICABLE
(A) out (A) Interesting
(B) away (B) Loving
(C) up (C) Affectionate
(D) None of the above (D) Friendly
38. The team is _______ eleven players. 46. When one is 'pragmatic', he is being-
(A) made of (A) wasteful
(B) made up of (B) productive
(C) made up (C) practical
(D) made (D) None of the above
39. Dhaka is becoming one of the ------- cities in Asia. 47. Which sentence is correct?
(A) more busy (A) This is an unique case
(B) more busier (B) The is a unique case
(C) busiest (C) This is a very unique case
(D) None of the above (D) None of the above
40. I finally killed the fly ______ a rolled up newspaper. 48. The error in sentence, 'One of the recommendation
(A) by made by the committee was accepted by the authorities' is-
(B) with (A) recommendation
(C) through (B) was
(D) None of the above (C) accepted by
41. Which sentence is correct? (D) None of the above
(A) The Nile is longest river in Africa 49. Animals that can live on land and water
(B) The Nile is longest river in the Africa (A) invectivorous
(C) The Nile is the longest river in Africa (B) dinosaurs
(D) None of the above (C) amphibians
42. Chose the one that best expresses the opposite (D) reptile
meaning of the given word : Optimist 50. The word 'electorate' means :
(A) Romantic (A) election office
(B) Idealistic (B) a body of voters
(C) Pessimist (C) candidates
(D) None of the above (D) None of the above
43. 'Get along' means :
(A) Adjust MATHAMETICS (Solved By Raihana Pervin Rontu)
(B) Have smooth relations 51. How many times in a day, the hands of a clock are
(C) Walk straight?
(D) None of the above (A) 22
44. Select the option that corrects the bracketed section - (B) 24

26
Collected By: Md. Raihan Ahmed
Job Information For All Students (Bcs & Bank)

(C) 44 Selling price will be 1200+240 = 1440


(D) 48 55. The sum of two numbers is 25 and their difference is
Solution: 13. find their product.
In 12 hours, the hands coincide or are in opposite direction (A) 104
22 times. (B) 114
In 24 hours, the hands coincide or are in opposite (C) 315
direction 44 times a day. (D) 325
52. The number of girls in a class is 5 times the number of Solution:
boys. Which of the following can not be the total number of
children in the class? Then the other is x+13
(A) 24 So, x+x+13= 25
(B) 30 Or, 2x= 12
(C) 35 → x= 6
(D) 126 And the other number = 6+13= 19
Solution: Multiplication of these two , 6*19 = 114
56. In what ratio must tea at Tk. 62 per kg be mixed with
tea at Tk. 72 per kg so that the mixture must be worth Tk.
The number of total children will be (x+5x), that means the 64.50 per kg?
number will be multiplication of 6. (A) 3 : 1
So, 24, 30, 126 (B) 3 : 2
53. A number is doubled and 9 is added. If the resultant is (C) 4 : 3
trebled, it becomes 75. What is the number? (D) 3 : 5
(A) 3.5 Solution:
(B) 6 Let the quantity of Rs 62 per kg tea be x kgs and that of Rs
(C) 8 72 per kg be y kgs.
(D) None of the Above Therefore, we can say that
Solution: 62*x + 72*y = (x+y) * 64.5
Let, the n 72y - 64.5y = 64.5x - 62x
Then, 2x+9= 75/3 7.5y = 2.5x
Or, 2x= 25-9 or
Or, 2x= 16 x:y = 3:1
X=8 Thus three parts of Rs 62 per kg tea is mixed with 1 part of
54. The difference between the cost price and sale price of Rs 72 per kg tea in order to make the mixture worth Rs
an article is Tk. 240. If the profit is 20%, the selling price is: 64.5 per kg.
(A) Tk. 1440 Shortcut:
(B) Tk. 1400
(C) Tk. 1600 62 72
(D) None of the Above
Solution: 64.50
When 20 tk profit can be gained from cost 100 tk
(100*240)/20 = 1200 7.50 2.50
tk 75 25
3 1

27
Collected By: Md. Raihan Ahmed
Job Information For All Students (Bcs & Bank)

57. The average of five numbers is 27. If one number is After 2 hours or 120 minutes the tank will fill = (120/3)*4
excluded, the average becomes 25. The excluded number = 160 liters
is: So, in percentage that will be (160/2000)*100 = 8%
(A) 25 60. if y/x = 1/3 and x + 2y =10, then x is-
(B) 27 (A) 2
(C) 30 (B) 3
(D) 35 (C) 4
Solution: (D) 6
Total of the 5 numbers = 27*5 = 135 Solution:
Total of the 4 numbers = 25*4= 100 y/x = 1/3
So, the excluded number is 135-100 = 35 or, y= x/3
58. If a train runs at 40 kmph, it reaches its destination late Here, x+ 2*x/3 =10
by 11 minutes but if it runs at 50 kmph, it is late by 5 Or, 5x/3 = 10
minutes only. The correct time for the train to complete its Or, 5x = 30
journey is : Or, x = 6
(A) 13 minutes 61. Ten years ago, A was half of B in age. If the ratio of
(B) 15 minutes their present age is 3 : 4, what will be the total of their
(C) 19 minutes present ages?
(D) 21 minutes (A) 20 years
Solution: (B) 30 years
(C) 45 years
So, s/(40/60) = t+11 (D) None of these
Or, (s*60)/40 -11 = t Solution:
Or, 3s/2 -11 = t
Another,
s/(50/60) = t+5 Now,( x/2+10) : (x+10) = 3:4
or, (s*60)/50 -5 = t Or, 2x+ 40 = 3x+30
or, 6s/5 5 =t Or, x =10
now, 3s/2 -11 = 6s/5 -5
or, 3s/2 6s/5 = 11-5 Total of their age = 20+15 = 35
or, 3s = 60 62. The least number by which 294 must be multiplied to
or, s = 20 make it a perfect square, is :
then, 3*20/2 -11 = t (A) 2
or, t = 30 11 = 19 (B) 3
59. Every 3 minutes, 4 liters of water are poured into a (C) 6
2,000, liter tank. After 2 hours, what percent of the tank is (D) 24
full? Solution:
(A) 0.4% Factor of 294 = 2 × 3 × 7 × 7
(B) 4% If we multiply 2 and 3 then it will be a perfect square
(C) 8% 63. A sum of Tk. 12,500 amounts to Tk. 15,500 in 4 years
(D) 12% at the rate of simple interest. What is the rate of interest?
Solution: (A) 3%
(B) 4%

28
Collected By: Md. Raihan Ahmed
Job Information For All Students (Bcs & Bank)

(C) 5% (C) 6%
(D) 6% (D) 8%
Solution: Solution:
Interest = 15,500 12,500 = 3000 𝐀×𝐁
Formula: A + B +
𝟏𝟎𝟎
3000 = 12500 × 4 × r/100 20 ×(−20)
Here 20 -20 + = -4%
Or, r = 100 × 3000/(12500 × 4) 100
68. A motorist travels to a place 150 km away at an average
Or, r = 6%
speed of 50 km and returns at 30 km per hour. His average
64. Some persons can do a piece of work in 12 days. Two
speed for the whole jurney in km per hour is :
times the number of such persons will do half of that work
(A) 35
in :
(B) 37.5
(A) 6 days
(C) 40
(B) 4 days
(D) None
(C) 3 days
Solution:
(D) 12 days
Method 1:
Solution:
Time = Distance/Speed
X persons do 1 of work in 12 days
Time taken in going = 150/50 km/h = 3 km/h
X/2 persons do ½ of work in 12/(2 × 2) = 3 days
Time taken in returning = 150/30km/h = 5km/h
65. Find the least number exactly divisible by 12, 15, 20
Total time = ( 3+5 ) h
and 27.
Total distance = ( 150+150 ) km = 300 km
(A) 540
Average speed = Total Distance/Total time
(B) 430
= 300 km/ 8 h
(C) 320
= 37.5 km/h
(D) 300
Method 2:
Solution:
For average speed we know that average speed = 2xy/(x+y)
LCM of the 12, 15, 20, 27 = 2*2*3*5*9 = 540
So, average speed = 2*50*30/(50+30) = 3000/80 =
66. In a 100m race, A beats B by 10m and C by 13m. In a
37.5km/hr
race of 180m, B will beat C by
69. Four years ago, the father's age was three times the age
(A) 5.4m
of his son. The total of the ages of the father and the son
(B) 4.5m
after four years, will be 64 years. What is the father's age at
(C) 5m
present?
(D) 6m
(A) 32 years
Solution:
(B) 36 years
Here, A/B = 100/(100-10) = 100/90
(C) 44 years
A/C = 100/(100-13) = 100/87
(D) None of these
B/C = 90/87 = 30/29
Solution:
Then B runs 180 and C runs (29/30 × 180) = 174m
u
B will beat C by 180-174 = 6m
3x
67. The price of tea being increased by 20%, a man reduces
Then, x+4+4+3x+4 +4= 64
his consumption by 20%. By how much percent will his
Or, x = 12
expenses for tea be decreased?
(A) 2%
70. If the length of rectangle A is one-half the length of
(B) 4%
rectangle B, and the width of rectangle A is one- half the
width of rectangle B, what is the ratio of the area of

29
Collected By: Md. Raihan Ahmed
Job Information For All Students (Bcs & Bank)

rectangle A to the are of rectangle B? Then, 25*.5 = 12.5


(A) 1/4 74. If the arithmetic mean of seventy five numbers is
(B) 1/2 calculated, it is 35. If each number is increased by 5, then
(C) 1/1 mean of new numbers is:
(D) 2/1 (A) 30
Solution: (B) 40
Length of b = x, so length of a = x/2 (C) 70
Width of b = y, so width of a = y/2 (D) 90
Area of b = xy and area of a = xy/4 Solution:
Ratio of area of rectangle A to B = xy/4 : xy = 1/4 Arithmetic mean of 75 members = 35
71. A starts business with Tk. 3500 and after 5 months, B Sum of 75 numbers = (75 × 35) = 2625
joins with A as his partner. After a year, the profit is Total increase = (75 × 5) = 375
divided in the ratio 2 : 3. What is B's contribution in the Increased sum = (2625 + 375) = 3000
capital? Increased average = 3000/75 = 40
(A) Tk. 7500 75. 0.01 is what percent of 0.1?
(B) Tk. 8000 (A) 1/100
(C) Tk. 8500 (B) 1/10
(D) Tk. 9000 (C) 10
Solution: (D) 100
2x profit gained in 12 months from 3500 Solution:
3x profit gained in 12 months from (3500 × 3x)/2x 0.1* x/100 = .001
3x profit gained in 7 months from 5250 × 12/7 = 9000 Or, 0.1*x = 1
72. A sum of money is to be distributed among A, B, C, D Or, x = 1/.1
in the proportion of 5 : 2 : 4 :3 . If C gets Tk. 1000 more Or, x = 10
than D, what is B's share?"
(A) Tk. 500 GENERAL KNOWLEDGE
(B) Tk. 1500 76. The headquarters of World Intellectual property
(C) Tk. 2000 Organization (WIPO) is located in _______.
(D) None of these (A) Paris
Solution: (B) New York
Let A, B, C, D gets 5x, 2x, 4x, 3x respectively. (C) Geneva
Then difference between C and D = 4x 3x = x (D) None of the Above
According to qus, x = 1000 77. What does GMT stand for?
× 1000 = 2000 (A) Greenwich Mean Time
73. What number should be divided by √0. 25 to give the (B) General Mean Time
result as 25? (C) Geographical Mean Time
(A) 12.5 (D) None of the Above
(B) 25 78. Which of the following institutions won the Nobel Peace
(C) 50 Prize in 2012?
(D) 125 (A) International committee of the Red Cross
Solution: (B) United Nations
√ 0. 25 = .5 (C) European Union
(D) None of the Above

30
Collected By: Md. Raihan Ahmed
Job Information For All Students (Bcs & Bank)

79. Which one of these planets is nearest to the earth? 87. In computer terms, what does 'encryption of data'
(A) Mercury mean?
(B) Venus (A) Data cannot be sent over the internet
(C) Neptune (B) Data is encoded so it cannot be read without decoding
(D) None of the Above software
80. What is the best way to protect your hard drive date? (C) Data is kept locked in special room
(A) Regular backups (D) None of the Above
(B) Periodically defragment it 88. Where is the working capital of the European Union?
(C) Run scandisk at least once a weak (A) London
(D) None of the Above (B) Austria
81. Which of the following is the longest river in the world? (C) Brussels
(A) Amazon (D) None of the Above
(B) Mississippi 89. What is the capital of Turkey?
(C) Nile (A) Istanbul
(D) None of the Above (B) Turashka
82. Acquired Immune Deficiency Syndrome (AIDS) is caused (C) Ankara
by (D) None of the Above
(A) Bacteria 90. Who is the first non-European to win the Nobel Prize in
(B) Fungus Literature?
(C) Virus (A) Pearl S. Buck
(D) None of the Above (B) Rabindranath Tagore
83. Number of Officials of the UNESCO (C) Sinclair Lewis
(A) 2 (D) Dugene O'Neil
(B) 4 91. The sun is a ____
(C) 6 (A) star
(D) None of the Above (B) planet
84. Which country from the following is NOT the member (C) asteriod
of the United Nations? (D) None of the Above
(A) Vactican City 92. Who is the first woman to climb Mount Everest?
(B) Afghanistan (A) Raha Moharrak
(C) North Korea (B) Nishat Majumder
(D) None of the Above (C) Junko Tabei
85. Which of the following countries is not a member of the (D) None of the Above
SAARC? 93. The volatile memory of the computer is known as :
(A) Sri Lanka (A) ROM
(B) Afghanistan (B) BIOS
(C) Myanmar (C) RAM
(D) Maldives (D) None
86. Physical components of a computer are called _______. 94. What is the highest rank of an officer of the Bangladesh
(A) Software Army?
(B) Hardware (A) Brigadier General
(C) Malware (B) General
(D) Terminals (C) Major General

31
Collected By: Md. Raihan Ahmed
Job Information For All Students (Bcs & Bank)

(D) None of the Above 2. - ?


95. The 5 rings in Olympic Symbol represent _____. (A)
(A) 5 continents 
(B) 5 planets (C)
(C) 5 oceans (D)
(D) None of the Above 3. ?
96. What part of the computer interprets and executes (A) 
instructions that are posed to it?
(A) RAM (C)
(B) CPU (D)
(C) ROM 4. ?
(D) Cache (A) , ,
97. The largest democratic country in the world is _____. , , 
(A) United States (C) , ,
(B) United Kingdom (D) , ,
(C) India 5. ' ' ?
(D) None of the Above (A)
98. At present, how many countries have the member
status of the EU? (C)
(A) 27 (D) 
(B) 30 6. ' ' -
(C) 28 (A) 
(D) None of the Above
99. Who wrote 'September on Jessore road'? (C)
(A) Monica Ali (D)
(B) Robert Frost 7. ' ' -
(C) Allen Ginsberg (A) 
(D) None of the Above
100. Entomology is the science that studies ______. (C)
(A) Human behavior (D)
(B) Insects 8. - -
(C) The formation fo rocks (A)
(D) None of the Above
(C) 
(D)
9. ' '--
?
BANGLA (A)
1 - : 
(A) + (C)
+ (D)
(C) + 10. ' ' -- ?
(D) +  (A)

32
Collected By: Md. Raihan Ahmed
Job Information For All Students (Bcs & Bank)

 (D) + =
(C) 19. Cease fire ?
(D) (A)
11. > -
? (C) - 
(A) (D)
20. ' ' ?
(C)  (A)
(D)
12. ' , ___' - (C) 
(A) (D)
 21. ---
(C) (A) 
(D) -
13. ? (C)
(A) (D)
 22. ' ' -- ?
(C) (A)
(D) 
14. (C)
(A) (D)
23. ---
(C) (A)
(D) 
15. ' - ' ? (C) 
(A) (D)
24. ?
(C) (A) 
(D) 
16. ? (C)
(A)  (D)
25. ' - ' ?
(C) (A)
(D) 
17. ' ' -- ? (C)
(A) (D)

(C) 
(D) Question(31-35): Select from the options, the pair having a
18. ? similar relationship to the given pair.
(A) + =  26. NUMERATOR : DENOMINATOR : :
+ = (A) Fraction : Decimal
(C) + = (B) Divisor : Quotient

33
Collected By: Md. Raihan Ahmed
Job Information For All Students (Bcs & Bank)

(C) Ratio : Proportion (A) cementary


(D) Dividend : Divisor (B) pursuit
27. SCHOOL : TUITION : : (C) dyeing
(A) Game : Loss (D) dying
(B) Lawyer : Client
(C) Hospital : Insurance Question(36-40): Choose the one that can be substituted
(D) Church : Tithe for the given word/phrase.
28. SQUINT : EYES : : 36. Official misconduct
(A) Grapple : Iron (A) malefactor
(B) Grope : Hands (B) malfeasance
(C) Lips : speech (C) maltreatment
(D) Limp : Limbs (D) maladministration
29. NOISOME : GARBAGE : : 37. A person who eats too much
(A) Liquid : Perfume (A) glutton
(B) Heavy : Metal (B) reveller
(C) Loud : Music (C) sensualist
(D) Fragrant : Incense (D) omnivore
30. CYNOSURE : BRILLIANT : : 38. Animals that eat much
(A) Student : Attentive (A) herbivorous
(B) Map : Legible (B) omnivorous
(C) Word : Common (C) carnivorous
(D) Magnet : Attractive (D) vegetarian
31. Select the misspelt word : 39. A religious discourse
(A) mortgage (A) preach
(B) bouquet (B) stanza
(C) pursue (C) sanctorum
(D) privelege (D) sermon
32. Select the misspelt word : 40. A place for sick people who need long periods for
(A) parralet recovery.
(B) chasis (A) clinic
(C) ballet (B) hospital
(D) gauge (C) sanatorium
33. Select the misspelt word : (D) asylum
(A) neighbour
(B) mischief Question(41-45): Select the word or phrase that is most
(C) mischievous closely opposite in meaning to the capitalised word.
(D) deciple 41. CAPRICIOUS
34. Select the misspelt word : (A) redoubtable
(A) diarrhoea (B) constant
(B) chauffeur (C) phlegmatic
(C) typhoide (D) solitary
(D) dysentery 42. COURTEOUS
35. Select the misspelt word : (A) flaccid

34
Collected By: Md. Raihan Ahmed
Job Information For All Students (Bcs & Bank)

(B) emollient (A) rebuke


(C) insolent (B) slam
(D) scrupulous (C) praise
43. ANOMALOUS (D) condemn
(A) capacious
(B) vicious MATHAMETICS (Solved By Mehadi Hassan Fuad )
(C) connected 51. On selling 17 balls at Tk. 720, there is a loss equal to
(D) usual the cost price of 5 balls. The cost price of a ball is ----
44. SOVORY (A) Tk. 45
(A) sad (B) Tk. 50
(B) hidden (C) Tk. 55
(C) lost (D) Tk. 60
(D) unpalatable Solution:
45. AMELIORATE If loss is 5 CP is the loss then adding 5 CP to 720 should be
(A) increase equal to 17 CP
(B) worsen 720 + 5 CP = 17 CP
(C) clasp CP = 720/12 = 60
(D) dissemble 52. Find a number such that when 15 is subtracted from 7
times the number, the result is 10 more than twice the
Question(46-50): Select from the alternative, the word that number.
conveys the same meaning as the word given in capital (A) 5
leeters. (B) 7
46. TORTUOUS (C) 9
(A) extreme (D) 15
(B) indirect Solution:
(C) uneven Let the number is x
(D) incidental According to the question,
47. LIMPID 7x-15=2x+10
(A) moist 7x-2x=15+10
(B) dear 5x=25
(C) transparent X=5
(D) dark 53. The difference of two numbers is 20% of the larger
48. PERPETUAL number. If the smaller number is 20, then the larger
(A) everlasting number is :
(B) firm (A) 25
(C) equable (B) 45
(D) steady (C) 50
49. MELEF (D) 80
(A) kindness Solution:
(B) brawl Let the larger number is x
(C) simple song According to the question
(D) primitive dance x-20=20x/100
50. APPLAUD

35
Collected By: Md. Raihan Ahmed
Job Information For All Students (Bcs & Bank)

x=25 (C) Tk. 600


54. Two-fifth of one-fourth of three-seventh of a number is (D) Tk. 800
15. When is half of that number? Solution:
(A) 94 Let the sum be x.
(B) 96 Then,[x(1+10/100)2-x]=525
(C) 188 Sum =2500
(D) None of these S.I.= (2500*5*4)/100
Solution: = Rs. 500
Let the number is =x 58. The greatest number that exactly divides 105, 1001 and
According to the question, 2436 is :
2/5*1/4*3/7* x= 15 (A) 3
X=350 (B) 7
So half of the number is = 350/2 or 175 (C) 11
55. A truck covers a distance of 550 metres in 1 minute (D) 21
where as a bus covers a distance of 33 kms in 45 minutes. Solution:
The ratio of their speeds is : H.C.M of 105,1001,2436 is 7.
(A) 3 : 4 59. The angle between the minute hand and the hour hand
(B) 4 : 3 of a clock when the time is 8.30, is :
(C) 3 : 5 (A) 80 degree
(D) 50 : 3 (B) 75 degree
Solution: (C) 60 degree
Ratio of speeds: (D) 105 degree
= (550/60) : (33*1000)/(45*60) Solution:
=3:4 |(11M-60H)/2|= |(11*30-60*8)/2|=75
56. A sum of Tk. 312 was divided among 100 boys and girls 60. The sum of the ages of 5 children born at the intervals
in such a way that each boy gets Tk. 3.60 and each girl Tk. of 3 years each is 50 years. What is the age of the youngest
2.40. The number of girls is : child?
(A) 35 (A) 4 years
(B) 40 (B) 8 years
(C) 60 (C) 10 years
(D) 65 (D) None of these
Solution: Solution:
Let, the number of girls is X Let the ages of children be x, (x + 3), (x + 6), (x + 9) and
So, the number of boys is (100-x) (x + 12) years.
According to the question Then, x + (x + 3) + (x + 6) + (x + 9) + (x + 12) = 50
(100-x)*3.60+2.40X=312 5x = 20
X=40 x=4
57. The compound interest on a certain sum for 2 years at 61. Two pipes A and B can fill a tank in 15 minutes and 20
10% per annum is Tk. 525. The simple interest on the same minutes respectively. Both the pipes are opended together
sum for double the time at half the rate percent per annum but after 4 minutes, pipe A is turned off. What is the total
is : time required to fill the tank?
(A) Tk. 400 (A) 10 min 20 sec
(B) Tk. 500 (B) 11 min 45 sec

36
Collected By: Md. Raihan Ahmed
Job Information For All Students (Bcs & Bank)

(C) 12 min 30 sec SO, B beats A by 100m


(D) 14 min 40 sec 65. 54 is to be divided into two parts such that the sum of
Solution: 10 times the first and 22 times the second is 780. The
Let, B work individual X minutes. bigger part is :
According to the question, (A) 24
4/15+ x+4/20=1 (B) 34
X=44/3 (C) 30
X=14 min 40s. (D) 32
62. The simple interest on a sum of money will be Tk. 600 Solution:
after 10 years. If the principal is trebled after 5 years, what Let the bigger part is x and smaller part is (54-x)
will be the total interest at the end of the tenth year? According to the question
(A) Tk. 600 10x+22(54-x0=780
(B) Tk. 900 X=34.
(C) Tk. 1200 66. A train of length 150 metres takes 40.5 seconds to cross
(D) Tk. 1500 a tunnel of length 300 metres. What is the speed of the
Solution: train in km/hr?
Simple interest after 10 years = 600 (A) 13.33
Simple after 1 yr = 60 (B) 26.67
For first 5 years = 60*5 = 300 (C) 40
For next 5 years as the principle has been trebled, interest (D) 66.67
also would be trebled. Solution:
Hence, V= (150+300)/40.5 *18/5= 40
Simple interest for next 5 years = (60*5*3) 67. A and B can work in 8 days, B and C can do the same
= 900 work in 12 days. A, B and C together can finish it in 6 days.
Hence Total simple interest = 300 + 900 = 1200 A and C together will do it in :
63. If a and b are odd numbers, then which of the (A) 4 days
following is even? (B) 6 days
(A) a+b (C) 8 days
(B) a+b+1 (D) 12 days
(C) ab Solution:
(D) ab+2 A+ b = 8days
Solution: B +C =12 days
Let a=3 and b=5 A+ b+ C =6days
So, 3+5=8, 3+5+1=9, 3*5=15, 3*5+2=17. A = 1/6 - 1/12 = 12-6/12*6 = 6/12*6= 1/12
64. A can run 22.5 m while B runs 25 m. In a kilometer C = 1/6 - 1/8 = 8-6/8*6 = 2/48 = 1/24
race B beats A by : A+C = 1/12 +1/24 = 24+12/ 24*12 =36/24*12= 1/8=
(A) 100 m 8days.
1
(B) 111 m 68. A 6% stock yields 8%. The market value of the stock is :
9
(C) 25 m (A) Tk. 48
(E) 50 m (B) Tk. 75
Solution: (C) Tk. 96
When B goes 25m A goes 22.5m (D) Tk. 133.33
When B goes 1000m A goes 22.5*1000/25 = 900 m

37
Collected By: Md. Raihan Ahmed
Job Information For All Students (Bcs & Bank)

Solution: (C) 7.2 cm


For an income of 8, investment = Tk. 100. (D) 7.3 cm
For an income of 6, investment = (100/8*6) = Tk. 75 Solution:
69. 0.213 ÷ 0.00213=? In first condition area is =18*10=180 cm2
(A) 1 In Second case, let the breath is x
(B) 10 So, area is 25x=180 or x=7.2 cm.
(C) 100 74. Aunik and Anam started a business investing Tk. 22,500
(D) None of these and Tk. 35,000 respectively. Out of a total profit of Tk.
Solution: 13,800, Anam's share is :
.213 ÷ .00213 = 100 (A) Tk. 5400
70. Two whole numbers whose sum is 72 cannot be in the (B) Tk. 7200
ratio : (C) Tk. 8400
(A) 5 : 7 (D) Tk. 9600
(B) 3 : 5 Solution:
(C) 3 : 4 Investment Ratio
(D) 4 : 5 Aunik: Anam= 22500:35000= 9:14
Solution: Profit share of Anam= 14*13800/23= Tk. 8400
75. The least perfect square number divisible by 3, 4, 5, 6
71. The average of six numbers is 3.95. The average of two and 8 is :
of them is 3.4, while average of the other two is 3.85. What (A) 900
is the average of the remaining two numbers? (B) 1200
(A) 4.5 (C) 2500
(B) 4.6 (D) 3600
(C) 4.7 Solution:
(D) 4.8 L.C.M of 3, 4, 5, 6 and 8 is 120
Solution: And the number is 3600 which is a perfect square.
Sum of the remaining two numbers
= (3.95 * 6) - [(3.4 * 2) + (3.85 * 2)] GENERAL KNOWLEDGE
= 23.70 - (6.8 + 7.7) = 23.70 - 14.5 = 9.20. 76. How much is the Per Capita Income of Bangladesh
Required average = (9.2 / 2) = 4.6. presently in US$?
72. A man can row upstream at 8 kmph and downstream at (A) 3341
13 kmph. The speed of the stream is : (B) 1861
(A) 2.5 km/hr (C) 1044
(B) 4.2 km/hr (D) 2001
(C) 5 km/hr 77. Which organization won the Nobel Peace Prize for
(D) 10.5 km/hr climate change related activities?
Solution: (A) UNDP
Speed of the stream= (13-8)/2=2.5 km/h (B) IPCC
73. The length of a rectangle is 18 cm and its breadth is 10 (C) UNEP
cm, When the length is increased to 25 cm, what will be (D) UNESCO
the breadth of the rectangle if the area remains the same? 78. Cannes International Film Festival is held in ---
(A) 7 cm (A) Germany
(B) 7.1 cm (B) France

38
Collected By: Md. Raihan Ahmed
Job Information For All Students (Bcs & Bank)

(C) Spain (B) U. Thant


(D) England (C) Ban Ki-moon
79. Who was the first Chief Minister of the undivided (D) Boutros Boutros-Ghali
Bangal? 87. Who upheld Bangladesh to the world in 1971 during the
(A) Syed Mahmud liberation war?
(B) Syed Amir Ali (A) George Harrison
(C) Nawab Abdul Latif (B) Simon Dring
(D) A. K. Fazlul Huq (C) Pandit Ravi Sarkar
80. The international Mother Language Day was declared in (D) All of them
---- 88. What is the name of the Capital of East Timur?
(A) 1997 (A) Longvek
(B) 1999 (B) Dili
(C) 1998 (C) Nein Feto
(D) 2000 (D) Vera Cruz
81. Which is the second country that recognized the 89. Who was the Commander-in-the of the 1971 liberation
independent Bangladesh? war of Bangladesh?
(A) India (A) Shamsul Huq
(B) Srilanka (B) Sheikh Mujibur Rahman
(C) Bhutan (C) Syed Nazrul Islam
(D) Malaysia (D) None of these
82. What country is Usain Bolt originally from? 90. Who is the newly elected Prime Minister of Australia?
(A) England (A) Edmund Barton
(B) Sudan (B) Tony Abbott
(C) Jamaica (C) Kevin Rudd
(D) Barbados (D) Julia Gillard
83. 'Burkina Faso' is the name of a --- 91. Which one is the largest gas-field of Bangladesh?
(A) Sea (A) Haripur
(B) Country (B) Titas
(C) Mountain (C) Bakhrabaad
(D) River (D) None of these
84. Which one is not a state owned commercial bank? 92. 'Cape of Good Hope' is situated in ----
(A) Sonali (A) Mozambique
(B) Pubali (B) Egypt
(C) Rupali (C) Mexico
(D) Janata (D) South Africa
85. 'The Concert for Bangladesh' was held in --- 93. What is the name of the Parliament of Bangladesh in
(A) London English?
(B) New York (A) Parliament of Bangladesh
(C) Washington D.C (B) National Parliament
(D) Liverpool (C) House of the Nation
86. The First Secretary General of the United Nations (D) None of these
elected from Asia was ---- 94. What is a name of the sculpture situated in Bangla
(A) Trygve Lie Academy that has been built in the memory of the martyrs

39
Collected By: Md. Raihan Ahmed
Job Information For All Students (Bcs & Bank)

of the language movement? (D) + 


(A) Vashar Kotha 2. ?
(B) Vashar Shadhinota
(C) Moder Asha 
(D) Moder Gorob (C)
95. World Population Day is observed on ---- (D)
(A) 1 June 3. ?
(B) 11 July , ,
(C) 7 July , ,
(D) 1 July (C) , , 
96. Which one is situated to the north of Bangladesh? (D) , ,
(A) Mijoram 4. ' ' ?
(B) Meghalaya
(C) Tripura
(D) Myanmar (C) 
97. Which one is the fifth EPZ of Bangladesh? (D)
(A) Comilla 5. ?
(B) Ishwardi 
(C) Mongla
(D) Uttara (C)
98. When was Sikkim merged with India? (D)
(A) 1972 6. ?
(B) 1975 ,
(C) 1973
(D) 1974 (C)
99. Which is the tributary of the river Padma? (D) 
(A) Tangon 7. ' ' ?
(B) Punarvaba
(C) Korotoa
(D) Kopotaksha (C) 
100. The inventor of the World Wide Web (WWW) is - (D)
(A) Bill Gates 8. -- ' ' -
(B) Andy Grove 
(C) Steve Jobs
(D) Tim Berners-Lee (C)
(D)
9. ' ' ?

BANGLA 
1. ' ' ? (C)
+ (D)
+ 10. ' ' --- ?
(C) +

40
Collected By: Md. Raihan Ahmed
Job Information For All Students (Bcs & Bank)

(D)
(C)  19. ' '-
(D) - ?
11. ? 

(C)
(C) (D)
(D)  20. ' ' ----
12. ' ' --- ?

(C)
(C)  (D)
(D) 21. ' ' ' ' ?
13. - ?

 (C)
(C) (D) 
(D) 22. ' '-- ' ' ---
14. - ?

 (C)
(C) (D)
(D) 23. ' '
15. ' ' -- ' '
?
(C) 
(D)
(C) 24. ' ' -- ' ' ---
(D) 
16. ?
 (C) 
(D)
(C) 25. ' ' ---
(D)
17. ' ' ?
(C)
(D) 
(C) 
(D)
18. ' ' --- Question(26-30): Select from the alternative, the word that
conveys the same meaning as the word given in capital
letter.
(C) 

41
Collected By: Md. Raihan Ahmed
Job Information For All Students (Bcs & Bank)

26. BIFURCATION 34. METICULOUS


(A) discontinue (A) careless
(B) bend (B) particular
(C) division (C) calculated
(D) radiate (D) planned
27. COPIOUS 35. PLETHORA
(A) liberal (A) profusion
(B) abundant (B) current
(C) heavy (C) balance
(D) broad (D) shortage
28. ERRANT
(A) sinful Question(36-40): Select from the options, the pair having a
(B) unreliable similar relationship to the first pair.
(C) shifting 36. SAMPLE : UNIVERSE ::
(D) confusing (A) Plan : Research
29. DESICCATED (B) Individual : Population
(A) dry (C) Mathematics : Statistics
(B) drain (D) Element : Electron
(C) clear 37. ADDICTED : DEDICATED : :
(D) fade (A) Slavish : Kindly
30. WRECK (B) Fanatical : Enthusiastic
(A) afraid (C) Acute : chronic
(B) destroy (D) Temporary : permanent
(C) expire 38. NOTABLE : NOTORIOUS : :
(D) nervous (A) Philanthropic : Benevolent
(B) Philandering : Pleasant
Question(31-35): Select the word or phrase that is most (C) Heinous : Atrocious
closely opposite in meaning to the capitalised word. (D) Philanthropic : Miserly
31. DISSEMINATED 39. EARNEST : IMMORAL::
(A) network (A) Land : Evil
(B) interact (B) Dissolute : Lascivious
(C) concentrated (C) Restrained : Wanton
(D) acquaint (D) Shore : Reef
32. AMBIVALENT 40. APOSTATE : RELIGION : :
(A) submissive (A) Loyal : Faith
(B) misleading (B) Traitor : Country
(C) suspect (C) Vital : Church
(D) decisive (D) Disloyal : Colonies
33. REVEALED
(A) bare Question(41`-45): Choose the one that can be substituted
(B) open for the given words/phrase.
(C) concealed 41. A person who looks at the dark side of everything.
(D) naked (A) sadist

42
Collected By: Md. Raihan Ahmed
Job Information For All Students (Bcs & Bank)

(B) blind 49. To feather one's nest


(C) pessimist (A) to gain popularity
(D) optimist (B) to cheat others
42. Animals who eat flesh of another animal. (C) to be selfish
(A) maneate (D) to provide for future
(B) beast 50. At first blush
(C) carnivorous (A) at first sight
(D) cannibal (B) in the beginning
43. Elimination of a racial group by killing. (C) to be the first
(A) homicide (D) immediately
(B) regicide
(C) genocide MATHAMETICS (Solved By Tanisha tabassum)
(D) patricide 51. The difference between the place value and the face
44. A disease that spreads by means of germs carried in value of 6 in the numeral 856973 is --
atmosphere. (A) 973
(A) infectious (B) 6973
(B) epidemic (C) 5994
(C) contagious (D) None of these
(D) endemic Solution:
45. Killing of one's own brother Face value of 6 in the numeral 856973 is 6
(A) murder Place value of 6 in the numeral 856973 is 6000
(B) matricide Required difference: 6000 6 = 5994
(C) fratricide 52. The L.C.M. of two numbers is 48. The numbers are in
(D) genocide the ratio 2 : 3. The sum of the numbers is :
(A) 28
Question(46-50): Select from the alternatives, the words (B) 32
that best describe the meaning of the idiom or phrase given (C) 40
in the questions. (D) 64
46. To break the ice Solution:
(A) to accomplish a difficult task Let the number are 2x and 3x
(B) to be the first to begin as per question,
(C) to bring out good news 2x × 3x = 48
(D) to get the truth 6x = 48
47. To catch one's eye x = 38/6
(A) to pay attention x=8
(B) to attract attention So the numbers are 16 and 24
(C) to admire someone And their sum 16 + 24 = 40
(D) to be found guilty 53. The value of (1+.1+.01+.001) is :
48. To sit on the fence (A) 1.001
(A) to keep away (B) 1.011
(B) to be in danger (C) 1.003
(C) to remain neutral (D) 1.111
(D) to guard someone Solution :

43
Collected By: Md. Raihan Ahmed
Job Information For All Students (Bcs & Bank)

1+.1+.01+.001 = 1.111 Solution:


54. What mathematical operation come at the place of '?' in In case 1, The average of 2,7,6 and x is 5
the equation : 2?6-12/4+2=11. (2 + 7 + 6 + x)/4 = 5
(A) + (15 + x)/4 = 5
(B) - 15 + x = 20
(C) * x= 5
(D) / In case 2, the average of 18, 1, 6,x and y is 10
Solution: So, (18 + 1 + 6 + x + y)/5 = 10
2 ? 6 - 12/4 + 2 = 11 25 + x + y = 10
By Setting (×) well get, 2 × 6 -12/4 + 2 = 11 25 + 5 + y = 10 [putting the value of x=5]
L.H.S=R.H.S y = 20
55. A boy was asked to multiply a number by 25. He 58. A number is as much greater than 36 as the less than
instead multiplied the number by 52 and got the answer 86. Find the number.
324 more than the correct answer. The number to be (A) 51
multiplied was : (B) 61
(A) 12 (C) 71
(B) 15 (D) 81
(C) 25 Solution:
(D) 32 Let the number be x
Solution: ATQ,
Let the number be a x - 36 = 86 - x
52a = 25a + 324 x + x = 86 + 36
52a -25a = 324 2x = 122
27a = 324 x = 61
a = 12 59. A two-digit number becomes five-sixth of itself when its
56. Three-fifth the square of a certain number is 126.15. digits are reversed. Two digits differ by one. The number is:
What is the number? (A) 45
(A) 14.5 (B) 54
(B) 75.69 (C) 56
(C) 145 (D) 65
(D) 210.25 Solution:
Solution: Let the unit place digit be x and tens place digit be y
Let the number be x Therefore the original number is 10y + x
3/5√x = 126.15 And the new number is 10x + y
√x = 126.15 × 5/3 Therefore :
√x = 210.25 50y + 5x = 60x + 6y
x = 14.5 44y = 55x
57. The average of 2, 7, 6 and x is 5 and the average of 18, 4y - 5x = 0__________(i)
1, 6, x and y is 10. What is the value of y? y x = 1____________( ii)
(A) 5 (ii) × 4
(B) 10 4y-4x=4
(C) 20 (i) (ii)
(D) 30 (4y - 5x) - (4y - 4x) = 0-4

44
Collected By: Md. Raihan Ahmed
Job Information For All Students (Bcs & Bank)

- 5x - 4x = -4 Let CP = x
-x = -4 ATQ,
x=4 832 x = x - 448
y= x+1= 5 2x = 1280
Therefore the original number is 54. x = 640
60. The ratio between the present ages of P and Q is 6 : 7. Profit = 50% of 640 = 320
If Q is 4 years older than P, what will be the ratio of the SP = CP + profit
ages of P and Q after 4 years ? SP = 640 + 320 = 960 tk
(A) 3 : 4 63. The ratio of three numbers is 3 : 4 : 7 and their
(B) 3 : 5 product is 18144. The numbers are :
(C) 4 : 3 (A) 9, 12, 21
(D) None of these (B) 15, 20, 25
Solution : (C) 18, 24, 42
Let P and Q ages are 6x years and 7x years (D) None of these
Then Solution:
7x - 6x = 4 Let, the number are 3x, 4x, 7x
x=4 And 3x × 4x × 7x = 18144
So required ratio will be 84x3 = 18144
(6x + 4) : ( 7x + 4) x3 = 216
= 28 :32 x=6
=7:8 so, the number are 18, 24 and 42
61. The difference between a number and its two- fifth is 64. P and Q started a business investing Tk. 85,000 and Tk.
510. What is 10 percent of that number? 15,000 respectively. In what ratio the profit earned after 2
(A) 12.75 years be divided between P and Q respectively?
(B) 85 (A) 3 : 4
(C) 204 (B) 3 : 5
(D) None of these (C) 15 : 23
Solution: (D) None of these
Let the number be x Solution :
ATQ x - 2x/5 = 510 estment
3x/5 = 510 = 85000 × 2 : 15000 × 2
3x = 2550 = 17 : 3
x = 850 65. A does a work in 10 days and B does the same work in
So 10% of 850 = 85 15 days. In how many days they together will do the same
62. The profit earned by selling an article for Tk. 832 is work?
equal to the loss incurred when the same article is sold for (A) 5 days
Tk. 448. What should be the sale price for making 50% (B) 6 days
profit? (C) 8 days
(A) Tk. 920 (D) 9 days
(B) Tk. 960 Solution:
(C) Tk. 1060
(D) Tk.1200 B's one day work is 1/15
Solution: + 1/15 = 1/6

45
Collected By: Md. Raihan Ahmed
Job Information For All Students (Bcs & Bank)

Therefore, A and B together do 1/6 of the work in 1 day (B) 5


Thus, they can together finish the work in 6 days (C) 8
66. Pipe A can fill a tank in 5 hours, pipe B in 10 hours and (D) 9
pipe C in 30 hours. If all the pipes are Solution:
open, in how many will the tank be filled? Speed of the boat in still water is 1/2(a+b)
(A) 2 = 1/2 (11 + 5)
(B) 2.5 = 1/2(16)
(C) 3 = 8 Km/hr
(D) 3.5 70. Reena took a loan of Tk. 1200 with simple interest for
Solution: as many years as the rate of interest. If she paid Tk. 432 as
A can fill in 1 hour 1/5 part interest at the end of the loan period, what was the rate of
B can fill in 1 hour 1/10 part interest?
C can fill in 1 hour 1/ 30 part (A) 3.6
They all together can fill in 1 hour (B) 6
=( 1/5 + 1/10 + 1/30) part (C) 18
= 1/3 part (D) None of these
So together they can fill in 3 hour. Solution:
67. An athlete runs 200 metres race in 24 seconds. His Let rate = R% and time = R year
speed is : Then, (1200 × R × R/100) = 432
(A) 20 km/hr 12R2= 432
(B) 24 km/hr R2 = 36
(C) 28.5 km/hr R=6
(D) 30 km/hr 71. At what rate of compound interest per annum will a
Solution: sum of Tk. 1200 become Tk. 1348.32 in 2 years?
Speed = Distance ÷ time (A) 6%
= (200 ÷ 24) m/s (B) 6.5%
= ( 200 ÷ 24) × 18/5 (C) 7%
= 30 Km/hr (D) 7.5%
68. How long does a train 110 metres long running at the Solution:
speed of 72 km/hr take to cross a bridge 132 Let the rate be R% p.a
metres in length? Then 1200 × ( 1+ R/100)2 = 1348.32
(A) 9.8 sec Or, (1 + R/100)2 = 134832/120000 = 11236/10000
(B) 12.1 sec Or, (1+ R/100)2 = (106/100)2
(C) 12.42 sec Or, 1 + R/100 = 106/100
(D) 14.3 sec Or, R= 6%
Solution: 72. The length of a room is 5.5 m and width is 3.75 m.
Speed: 72 Km/hr =( 72 × 5/18)m/s =20 m/s Find the cost of paying the floor by slabs at the rate of Tk.
Total distance to be covered = 110 + 132 = 242 m 800 per sq. metre.
Time = Distance/Speed = 242/20 s = 12.1 seconds (A) Tk. 15,000
69. In one hour, a boat goes 11 km along the stream and 5 (B) Tk. 15,550
km against the stream. The speed of the boat in still water (C) Tk. 15,600
(in km/hr) is : (D) Tk. 16,500
(A) 3

46
Collected By: Md. Raihan Ahmed
Job Information For All Students (Bcs & Bank)

Solution: (C) Abidjan


Here, length = 5.5m (D) Freetown
Width = 4.75m 77. What is the name of Syrian President Basar Al Asad's
Area = (5.5 × 3.75) sq m = 20.625 sq.m Political party?
For 1 sq m cost tk 800 (A) Baath party
20.625 sq m tk needed (800 × 20.625) = 16,500tk (B) Al Ansar party
73. In a 100 m race, A covers the distance in 36 seconds (C) Syrian Peoples party
and B in 45 seconds. In this race A beats B by: (D) National Democratic party
(A) 20 m 78. Which of the following is a communist state?
(B) 25 m (A) South Korea
(C) 22.5 m (B) China
(D) 9 m (C) Brazil
Solution: (D) Poland
A won by (45 - 36) = 9 sec 79. Before being elected as the President, Pronob
Distance covered by B in 9 sec = ( 100/45 ×9) = 20 m Mukherjee served in which ministry of India?
74. How many times do the hands of a clock coincide in a (A) Ministry of External affairs
day? (B) Ministry of Home affairs
(A) 20 (C) Ministry of Finance
(B) 21 (D) Ministry of Defense
(C) 22 80. Syria does not share its border with which of the
(D) 24 following countries?
Solution: (A) Lebanon
The hands of a clock coincide 11 times in every 12 hours (B) Iran
since between 11 and 1 they coincide only once, at 12 o' (C) Turkey
clock. The hands overlap about every 65 minutes. Not (D) Jordan
every 60 minutes. The minutes hand and hour hand 81. Which militant organization is blamed for the 2008
coincide 22 time in a day. Indian city of Mumbai attack?
75. A man buys Tk. 50 shares in a company which pays (A) Lashkar-i-Taiba
10% dividend. If the man gets 12.5% on his investment, at (B) Al Qaeda
what price did he buy the shares? (C) Tehreek-e-Taliban
(A) Tk. 37.50 (D) Lashkar-e-Jabbar
(B) Tk. 40 82. Which of the following countries first started the
(C) Tk. 48 revolutionary movement called "Arab Spring"?
(D) Tk. 52 (A) Egypt
Solution: (B) Tunisia
Dividend on 1 share = ( 10/100 ×50) = Tk 5 (C) Syria
The man gets Tk 12.5 on investment of Tk 100 (D) Libya
He Buys the shares at = (100 × 5/12.50) = 40 tk 83. During the World War II, which ethnic group was under
severe oppression in the USA?
GENERAL KNOWLEDGE (A) Indian American
76. Which one of the following is the capital of Sierra Leon? (B) Chinese American
(A) Lima (C) Japanese American
(B) Khartoum (D) Latino

47
Collected By: Md. Raihan Ahmed
Job Information For All Students (Bcs & Bank)

84. Which year did South Sudan become an independent (B) ILO
country? (C) UNICEF
(A) 2010 (D) UNESCO
(B) 2011 92. Which sector has the highest contribution to the Gross
(C) 2012 Domestic Product of Bangladesh?
(D) 2013 (A) Agriculture
85. Which country has the largest proven oil reserve in the (B) industry
world? (C) Transport
(A) Saudi Arabia (D) Service
(B) Venezuela 93. What is the previous name of Sonargaon?
(C) Russia (A) Chandradeep
(D) USA (B) Suvarna Gram
86. Which country gave USA the 'Statue of Liberty'? (C) Shundharam
(A) France (D) Bik Rampur
(B) Germany 94. Which movie of Bangladesh got Oscar nomination?
(C) Uk (A) Matir Moyna
(D) Denmark (B) Cnadra Kotha
87. Which country was the first to recognize Bangladesh as (C) Adhar
an Independent Nation? (D) Andhakar
(A) Bhutan 95. Where is the "Dulahazara Safari Park" located?
(B) India (A) Barisal
(C) USSR (B) Cox's Bazar
(D) USA (C) Gazipur
88. In which year did Bangladesh become a member of (D) Rangpur
Commonwealth? 96. Which one is the largest tribe in Bangladesh?
(A) 1975 (A) Garo
(B) 1974 (B) Chakma
(C) 1980 (C) Marma
(D) 1972 (D) None of these
89. Bangladesh is not a member of which of the following 97. Which one of the following is considered as the fourth
association? state of the nation?
(A) D-8 (A) Media
(B) CIRDAP (B) Library
(C) WHO (C) Parliament
(D) OPEC (D) University
90. Which country does not have an embassy in 98. Which one is not a defense service in Bangladesh?
Bangladesh? (A) Army
(A) Bhutan (B) Navy
(B) Maldives (C) BGB
(C) Sri Lanka (D) None of these
(D) None of these 99. Where do we have the highest annual rainfall in
91. Sundarban is declared as the World Heritage by -- Bangladesh?
(A) UNDP (A) Kaptai

48
Collected By: Md. Raihan Ahmed
Job Information For All Students (Bcs & Bank)

(B) Srimongal (A) Outshine


(C) Barisal (B) Defy
(D) Sylhet (C) Challenge
100. How many women freedom fighters received the "Beer (D) Resist
Pratik Award" for their contribution in the 6. He was in a "brown study" and did not seem to catch
liberation war of Bangladesh? my point.(Match the best alternative that
(A) 5 correspond to the quotations word)
(B) 7 (A) In his study room
(C) 2 (B) Absorbed in reading
(D) 6 (C) Absent minded
(D) In a state of shock
7. Find the Misspelt word -
(A) Amateur
(B) antagonism
1. It was SCURRILOUS attack on him.(Match the same (C) anticipeted
meaning that correspond to the quotations word) (D) bureaucracy
(A) Serious 8. Find the Misspelt word -
(B) Unjustified (A) Delirious
(C) Insulting (B) Delusive
(D) Justified (C) Hefty
2. We have sanction his leave on COMPASSIONATE (D) Heinous
ground. (Match the same meaning that correspond to the 9. Find the Misspelt word -
quotations word) (A) Ferment
(A) as an exception (B) Introvert
(B) Legal (C) inveigle
(C) Merciful (D) All correct
(D) Deserving 10. Find the misspelt word-
3. He was ENGROSSED in day's office routine.(Match the (A) Fragrous
same meaning that correspond to the quotations word) (B) Monopoly
(A) involved (C) Rational
(B) tired (D) Demolish
(C) fully occupied 11. Extant ( choose the opposite in meaning from given
(D) lost options)
4. I have come to know of your "hole-and-corner" methods (A) Existing
of dealing with people. (Match the best alternative that (B) Dynamic
correspond to the quotations word) (C) Fashionable
(A) Suspicious (D) Lost
(B) secret 12. Hyperbole ( choose the opposite in meaning from given
(C) servile options)
strict (A) Exaggeration
5. They are sure to "steal a march" upon their competitors. (B) understatement
(Match the best alternative that (C) Distortion
correspond to the quotations word) (D) Impression

49
Collected By: Md. Raihan Ahmed
Job Information For All Students (Bcs & Bank)

13. Garrulous ( choose the opposite in meaning from given Here we know the value(age) of Q (25), but we don't know
options) the age of R.
(A) Talkative Therefore, (R-Q) cannot be determined.
(B) Frank 16. At an election involving two candidates, 68 votes were
(C) Enlightening declared ivalid. The winning candidates secure 52% and
(D) Taciturn wins by 98 votes. The total numbers of vote polled-
(A) 2382
MATHAMETICS (B) 2450
14. The sum of two numbers is 22. Five times one number (C) 2518
is equal to 6 times the other. The bigger of the two (D) None of these
numbers is- Solution:
(A) 10 Let the number of valid votes be x.
(B) 12 Winning candidate=52%
(C) 15 Losing candidate =100-52 =48%
(D) 16 According to the question
Solution: 52% of x - 48% of x = 98
Let the Bigger number is x and smaller is y 4% of x = 98
4/100 × x=98
x = 98 × 25 = 2450.
Now from (i) Invalid votes = 68
6x + 6y = 132 ∴Total number of votes polled = (2450 + 68) = 2518.
Or, 6x + 5x = 132 [5x = 6y] 17. If 5% more is gained by selling an article for Tk.350
Or, 11x = 132 than by selling it for Tk.340, the cost of the article is-
Or, x = 12 (A) Tk.50
15. Q is as much younger than R as he is older than T. If (B) Tk.160
the sum of ages of R and T is 50years, What is definately (C) Tk.200
the difference between R and Q's age? (D) Tk.225
(A) 1 years Solution:
(B) 25years Let Cost price of the article = Tk X
(C) 2 years When SP is Tk 340, then gain %={(340−X)/X} × 100
(D) Data inadequate When SP is Tk 350, then gain %={(350−X)/X} × 100
Solution: Hence Additional % gain {(350−X)/X}− {(340−X)/X}×100
Given that: Hence {(350−X)/X}×100− {(340−X)/X}×100 = 5
1. The difference of age b/w R and Q = The difference of Or, 35000−100− 34000+ 100X) = 5X
age b/w Q and T. Or, 5X = 1000
2. Sum of age of R and T is 50 i.e. (R + T) = 50. Or, X = Tk 200
Question: R - Q = ?. 18. x varies inversely as square of y. Given that y=2 for
Explanation: x=1. The value of x for y=6 will be equal is-
R-Q=Q-T (A) 3
(R + T) = 2Q (B) 9
Now given that, (R + T) = 50 (C) 1/5
So, 50 = 2Q and therefore Q = 25. (D) 1/9
Question is (R - Q) = ? Solution:

50
Collected By: Md. Raihan Ahmed
Job Information For All Students (Bcs & Bank)

1 (C) Tk. 40.40


x α Inversely proportional
y2
(D) Tk. 64.10
or, x = k/y2 [ k is a constant ]
Solution:
Now when y = 2 and x = 1
Difference
Then 1 = k/4
= {1000(1+ 10/100)4 1000} {1000 × 4 × 10/100}
Or, k = 4
= (1464.1 -1000) (400)
When y = 6
= 464.1 400
Then x = 4/36
= 64.1
x = 1/9
22. The sum of two numbers is 12 and their product is 35.
19. If 9 examiners can examine a certain number of answer
What is the sum of reciprocals of these numbers?
books in 12 days, working 5 hours a day; for how many
(A) 12/35
hours a day would 4 examiners have to work in order to
(B) 1/35
examine twice the number of answer book is 30 days?
(C) 35/8
(A) 3
(D) 7/32
(B) 8
Solution:
(C) 9
Let a and b are the numbers.Then a+b is 12 and ab is 35.
(D) 10
a+b/ab = 12/35
Solution:
1/b + 1/a = 12/35
Let, required time x hour/day
23. The least number of 5 digits which is exactly divisible by
30 days= 30x hour
12, 15, and 18 is -
At first stage, 12 days = 60 hour
(A) 10010
4 examiners examines in=60 × 9/4 = 135 hours
(B) 10015
So,time needed to exmine the double answer books is =
(C) 10020
135 × 2 = 270 hour
(D) 10080
Here, 30x = 270
Soluition:
So, x=9
LCM (12, 15, 18) = 180.
20. If a person walk at 14km/hr instead of 10km/hr, he
Now, we are to find smallest five-digit multiple of 180.
would walked 20 km more. The actual distance travelled by
Smallest five-digit number = 10000.
him is-
10000/180 gives a dividend of 55 leaves a remainder of 100
(A) 50km
Therefore, we need to add (180 - 100) = 80 to 10000 in
(B) 56km
order to make the sum divisible by 180
(C) 70km
Therefore, the answer is (10000 + 80) = 10080
(D) 80km
24. A boy multiplied 423 by a number and obtained 65589
Solution:
as his answer. If both the fives in the answer are wrong
Let the actual distance travelled be x km
and all other figures are correct, the correct answer is-
x/10 = (x + 20)/14
(A) 60489
4x = 200
(B) 62189
X = 50 km
(C) 61189
21. What will be the difference between simple and
(D) 62389
compound interest at 10% per annum on a sum of Tk.1000
Solution:
after 4years?
Among the given numbers, only 60489 is a multiple of 423.
(A) Tk. 31
(B) Tk. 32.10

51
Collected By: Md. Raihan Ahmed
Job Information For All Students (Bcs & Bank)

25. The least perfect square,which is divisible by each 21, (B) 10111
36 and 66 is - (C) 10011
(A) 213444 (D) None of these
(B) 214344 29. The primary job of operating system of a computer is
(C) 214434 to-
(D) 231444 (A) Manage resource
Solution: (B) Provide utilities
L.C.M. of 21, 36, 66 = 2772. (C) Provide comunication interface
Now, 2772 = 2 x 2 x 3 x 3 x 7 x 11 (D) None of these
To make it a perfect square, it must be multiplied by 7 x 11 30. When a computer first turned on or restarted, a special
So, required number type of absoulute loader is excuted,called-
2 2 2 2
= 2 x 3 x 7 x 11 (A) compile and go loader
= 213444 (B) Boot loader
26. The average of five consecutive odd number is 61. What (C) Bootstrap loader
is the difference between the highest and lowest number? (D) None of these
(A) 2 31. ______ provides a connection oriented reliable service for
(B) 5 sending message-
(C) 8 (A) TCP
(D) None of these (B) IP
Solution: (C) UDP
+5) (x+7) (x+9) (D) None of these
Average of 5 number is 61 32. Each IP packed must contained-
So, (x + 1 + x + 3 + x + 5 + x + 7 + x + 9)/5 =61 (A) Only source adress
(5x + 25)/5 = 61 (B) Only destination adress
5x + 25 = 61 × 5 (C) Source and destination adress
5x + 25 = 305 (D) None of these
5x = 305 25 33. The last adress of IP adress represents-
x = 280/5 (A) Unicast adress
x = 56 (B) Broadcast adress
Lowest no is x+1 So,56+1 = 57 (C) Network adress
Highest no is x+9 So,56+9 = 65 (D) None of these
The difference is, 65 57 = 8 34. What does router do in network ?
(A) Forwaard a packet to all outgoing links
COMPUTER (B) Forward a packet to the next outgoing link
27. Each character on the keyboard of computer has an (C) Determine on which outgoing link a packet is to be
ASCII value Which stand for- forwarded
(A) American Standard code for Information interchange (D) None of these
(B) American Stock code for information interchange 35. In a cilen/server model, a client program-
(C) American code for information interfere (A) Asks for information
(D) None of these (B) Provide information and files
28. Which one of the following represent the binary (C) Distributes data files to other computers
equivalent of the decimal number 23? (D) None of these
(A) 01011 36. What is the use of forms in HTML?

52
Collected By: Md. Raihan Ahmed
Job Information For All Students (Bcs & Bank)

(A) To display contents on Email 44. Which one is the largest library in the world?
(B) To display animation effect (A) Library of congress
(C) To collect users input (B) The British library
(D) None of these (C) National library of india
37. Which of the following is used to transfer files? (D) Woodburn library
(A) HTTP 45. Which country of the world doesn't have its capital ?
(B) HTML (A) Nauru
(C) FTP (B) Monaco
(D) None of these (C) The Vallue
38. Which organization defines the Web standards? (D) The Marshal Island
(A) Microsoft corporation 46. Which African country First recognised Bangladesh?
(B) IBM corporation (A) Libya
(C) World Wide Web consortium (B) Senegal
(D) None of these (C) South Africa
(D) None of these
GENERAL KNOWLEDGE 47. Where is "Halda vally" located?
39. Who is the first female student of University of Dhaka ? (A) Rangamati
(A) Lila nag (B) Chittagong
(B) Sulata Ghosh (C) Bandarban
(C) Ila mitra (D) Khagrachari
(D) None of them 48. Present name of "Bardhaman House" is-
40. The submarine cable project is under the supervision of (A) Bango Bhaban
which ministry ? (B) Bangla Academy
(A) Finance (C) Shishu Academy
(B) Postal and Telecommunication (D) Shilpokola Academy
(C) Science and ICT 49. What is the name of river Meghna at the place of its
(D) Foreign origin?
41. Which constituency Rushnara Ali represents in the (A) Borak
British parliament? (B) Surma
(A) Bethenal Green Bo (C) Lusai
(B) Birmingham Lady wood (D) Kusiara
(C) Leeds 50. When did the first convocation of University of Dhaka
(D) None of these take place?
42. What is meant by "Abu Garib" ? (A) 1923
(A) A jail (B) 1924
(B) A famous philoshoper (C) 1925
(C) A Museum (D) None of these
(D) A great Muslim scientist
43. In which continent is Mauritius situated?
(A) Africa
(B) Australia BANGLA
(C) Asia 1. ?
(D) North America

53
Collected By: Md. Raihan Ahmed
Job Information For All Students (Bcs & Bank)

(D)
(C) 10. ---
(D)  , ,
2. ? , ,
(C) , ,
(D) , , 
(C) 
(D)
3. - ? 11. Bengalis are "innate" poets.
(A) excellent
(B) romantic
(C) (C) dull
(D)  (D) inborn
4. ' ' ' ' ? 12. It became an instant "truism" that politics meant
selfishness.
(A) universal belief
(C)  (B) quick judgement
(D) (C) obvious truth
5. ' ' (D) immediate guess
? 13. Select from the alternative, the word that is most
closely opposite in meaning to the word given in
capital letter : EXTANT
(C)  (A) existing
(D) (B) fashionable
6. --- (C) dynamic
(D) lost
14. Select from the alternative, the word that is most
(C)  closely opposite in meaning to the word given in
(D) capital letter : INADVERTENTLY
7. ? (A) fearful
(B) impartial
(C) divide
(C) (D) purposely
(D)  15. Select from the alternative, the word that is most
8. ' ' closely opposite in meaning to the word given in
capital letter : AUDACIOUS
 (A) manifest
(C) (B) obvious
(D) (C) venture
9. ? (D) cowardly
- 16. Select the options the pair having a similar relationship
to the first pair in capital letters. MORPHINE : SEDATES : :
(C)  (A) Drug : Addicts

54
Collected By: Md. Raihan Ahmed
Job Information For All Students (Bcs & Bank)

(B) Liquor : Intoxicates results is :


(C) Medicine : Soothes (A) 1/5
(D) Oil : Smears (B) 2/9
17. Select the options the pair having a similar relationship (C) 23/99
to the first pair in capital letters. NAIVE : INGENUOUS : : (D) 23/100
(A) Ordinary : Ingenious Solution:
(B) Old : Wise
(C) Simple : Kind 23. By how much is three-fifth of 350 greater than four-
(D) Sophisticated : Urbane seventh of 210 ?
18. Select the options the pair having a similar relationship (A) 95
to the first pair in capital letters. DOCTOR : DISEASE : : (B) 110
(A) psychiatrist : Maladjustment (C) 120
(B) Teacher : Pupils (D) None of these
(C) Scholar : Knowledge Solution:
(D) Judge : Crime 3/5 of 350 - 4/7 of 210 = 210 - 120 = 90.
19. Fill in the blanks : Education is a strong ______ of 24. 50 is divided into two parts such that the sum of their
poverty status. reciprocals is 1/12. Find the two parts.
(A) requirement (A) 30 and 20
(B) predictor (B) 30 and 25
(C) discolor (C) 30 and 40
(D) exposure (D) 30 and 35
20. Fill in the blanks : He _____ the gathering in impressive Solution:
English. Let X and y are the two parts
(A) directed 1/X+1/y=1/12. And X+y=50.
(B) approached Therefore
(C) undertaken (X+y)/XY=1/12
(D) addressed Or, xy=12(X+y)
Or, Xy=12*50=600
MATHAMETICS Or, x=600/y
21. Find the largest number of four digits exactly divisible Therefore X=30 and y=20
by 12, 15, 18 and 27. 25. The ratio between the present ages of P and Q is 5 : 7
(A) 9620 respectively. If the difference between Q's present age and
(B) 9720 P's age after 6 years is 2, what is the total of P's and Q's
(C) 9820 present ages?
(D) 9920 (A) 48 years
Solution: (B) 52 years
LCM of (12, 15, 18, 27) (C) 56 years
⇒ 4 × 3 × 5 × 3 × 3 = 540 (D) None of these
largest 4-digits number = 9999 on dividing by 540 to Solution:
number = 9999/540 Ratio of present ages of P:Q = 5:7
remainder = 279 Let us assume P's age be 5x
Required number = 9999 - 279 = 9720 and Q's age be 7x
22 is converted into a fraction, then the and P's age after 6 years be 5x +

55
Collected By: Md. Raihan Ahmed
Job Information For All Students (Bcs & Bank)

Then, given that (B) 3 : 5 : 5


7x-( 5x+6 )= 2 (C) 6 : 10 : 5
7x - 5x - 6 = 2 (D) None of these
2x = 2 + 6 Solution:
x=8÷2 Let initial investment of A is 3x and B is 5x, then C
x =4. investment is also 5x, but most important to note in this
Then, P's present age is 5(4) = 20 question is the time duration of investment
Q's present age is 7(4) = 28 Like, A invested for 12 months, B invested for 12 months
Total of P's and Q's age is 20+28 = 48. and C invested for 6 months.
26. If 35% of a number is 175, then what percent of 175 is A : B : C = (3x x 12) : (5x x 12) : (5x x 6)
that number? = 36 : 60 : 30
(A) 35% = 6 : 10 : 5.
(B) 65% 29. If 17 labourers can dig a ditch 26 m long in 18 days,
(C) 280% working 8 hours a day; how many more labourer working 9
(D) None of these hours a day?
Solution: (A) 34
Let the number be 'x' (B) 51
Given that 35% of a number is 175 (C) 68
so x × 35% = 175 (D) 85
x × 35/100 = 175 Solution:
x = 175×100/35 Let the total number of men to be engaged be x.
x = 500 is the number.. More length, More labourers (Direct Proportion)
then 175 percentage of that number is Less days, More labourers (Indirect Proportion)
500×175% = 500×175/100 = 875 More hours per day, Less labourers (Indirect Proportion)
27. An article is sold at a certain price. By selling it at 2/3 Length 26 : 39 : : 17 : x
of that price one loses 10%. Find the gain percent at Days 6 : 18 : : 17 : x
original price. Hours per day 9 : 8 : : 17 : x
(A) 25% (26 * 6 * 9 * x) = (39 * 18 * 8 * 17)
(B) 30% x = 68.
(C) 35% Number of more labourers = (68 - 17) = 51.
(D) 40%
Solution: 30. Evaluate √𝟒𝟏 − √𝟐𝟏 + √𝟏𝟗 − √𝟗

Let cp=x and sp=y (A) 30°


(2y/3)+x/10=x (B) 5
y=(27/20)x (C) 6
Now gain %=((y-x)/x)*100 (D) 6.4
putting the value of y we will get gain =35% Solution:
28. A and B started a partnership business investing some
amount in the ratio of 3 : 5. C joined them after six months √41 − √21 + √19 − √9

with an amount equal to that of B. In what proportion


should the profit at the end of one year be distributed = √41 − √21 + √19 − 3
among A, B and C?
= √41 − √21 + √16
(A) 3 : 5 : 2
= √41 − √21 + 4

56
Collected By: Md. Raihan Ahmed
Job Information For All Students (Bcs & Bank)

= √41 − √25 38. The global campaign "The Future We Want" was
= √41 − 5 launched by ---
=6 (A) the G20
(B) the UNDP
GENERAL KNOWLEDGE (C) IPCC
31. MERS is the name of a -- (D) the Rio+20
(A) Virus 39. The youngest winner of the Man Booker Prize is :
(B) Galaxy (A) MacFarlane
(C) Drug (B) Daniel Glaser
(D) President Office (C) Eleanor Catton
32. The 'Limits to Growth' Report was published by --- (D) Alastain Niven
(A) the Club of Rome 40. According to the Forbes Magazine, the biggest company
(B) the Brundtland Commission in the world in 2013 was :
(C) the Nathan Commission (A) ICBC
(D) Our Common Future (B) Exxon Mobil
33. The Crimean Peninsula became the part of Ukraine (C) General Motors
during the Soviet era in --- (D) Jp Morgan Chase
(A) 1921 41. The current Secretary General of SAARC is :
(B) 1954 (A) Ahmed Salim
(C) 1935 (B) Dipanker Bannerjee
(D) 1945 (C) Suba Chandaran
34. The G20 members represent about ---- (D) Arjun Bahadur Thapa
(A) 50% of global GDP 42. "All power in the Republic belong to the people" is
(B) 90% of global GDP enshrined in the Constitution of Bangladesh by the Article :
(C) 75% of global GDP (A) 5(2)
(D) 79% of global GDP (B) 7(1) 
35. The largest landlocked country in Europe is ---- (C) 15(3)
(A) Poland (D) 29(1)
(B) Albania 43. The First Population Census of Bangladesh was held in -
(C) Croatia (A) 1972
(D) Hungary (B) 1978
36. The world's most heavily fortified border is located (C) 1981
between ---- (D) 1974
(A) North and South Korea 44. The film Guerilla has been adapted from the novel
(B) Israel and Lebanon Nishiddo Loban is written by ---
(C) India and Pakistan (A) Rabindranath Tagore
(D) North and South Sudan (B) Syed Shamsul Huq
37. Narendra Modi became the Chief Minister of Gujrat, (C) Humayun Ahmed
India in --- (D) Mohammad Zafar Iqbal
(A) October 2001 45. Which European country first recognized Bangladesh as
(B) April 2003 a sovereign state?
(C) March 2003 (A) Albania
(D) March 2000 (B) East Germany

57
Collected By: Md. Raihan Ahmed
Job Information For All Students (Bcs & Bank)

(C) Poland (C)


(D) Romania (D) 
3. ___
COMPUTER
46. Portable program means?
(A) Program with wheels (C) -
(B) Independent from its authors (D) 
(C) Independent of platform 4. _
(D) None
47. When a computer is first turned on or restarted, a
special type of absolute loader is executed, called- (C) 
(A) Compile and go loader (D)
(B) Boot loader 5. ?
(C) Bootstrap loader , , ,
(D) None ,
48. Each IP packet must contain (C) , ,
(A) Only Source address (D) , ,
(B) Only Destination address 6. - ?
(C) Source and Destination address
(D) None
49. In a client/server model, a client program _____. (C)
(A) asks for information (D) 
(B) provides information and files 7. Transliteration-
(C) distributes data files to other computers
(D) None
50. How do the main components of the computer (C)
communicate with each other? (D) 
(A) System bus 8. ' ' -
(B) Memory
(C) Keyboard
(D) None (C) 
(D)
9. -

BANGLA 
1. ' ' __ (C)
(D)
 10. -
(C)
(D) 
2. ? (C)
(D)
11. _

58
Collected By: Md. Raihan Ahmed
Job Information For All Students (Bcs & Bank)

(C)
 (D) 
(C) 20. ' ' _
(D)
12. ?
 (C) 
(D)
(C)
(D)
13. , ! ' ' 21. He was in a brown study and did not seem to catch my
? point .
 (A) in his study room
(B) absorbed in reading
(C) (C) absent-minded
(D) (D) in a state of shock
14. _ 22. Harassed by repeated acts of injustice , he decided to
 put his foot down.
- (A) not to yield
(C) (B) resign
(D) (C) withdraw
15. ? (D) none of these
23. They set the two friends by the ear and then enjoyed
the fun.
(C) (A) made friends
(D)  (B) to provoke to quarrel
16. The window panes steamed up . - (C) to bring in agreement
(D) none of these
24. The man who has committed such a serious crime must
(C)  'get the mostly severe' punishment .
(D) (A) got the mostly severely
17. - - (B) get the most severe
 (C) have got the most severely
- (D) no improvement
(C) 25. Before I could stop him , the boy 'was throwing' the
(D) box down the stairs .
18. - (A) were throwing
(B) threw
(C) did throw
(C) (D) no improvement
(D)  26. No sooner had be entered the room 'when the lights
19. ' '- _ went out' and everybody began to scream .
(A) when the lights went out
(B) than the lights went out

59
Collected By: Md. Raihan Ahmed
Job Information For All Students (Bcs & Bank)

(C) and the lights went out (D) disinterested


(D) then the lights went out 35. HALCYON
27. The orator 'had been left' the auditorium before the (A) luminous
audience stood up. (B) gentle
(A) had been left (C) motionless
(B) was left (D) peaceful
(C) had left 36. VENERATE
(D) no improvement (A) defame
28. They 'feel very proudly' that their team had won the (B) abuse
match . (C) respect
(A) feels very proudly (D) accuse
(B) felt very pride 37. OBDURATE
(C) felt very proud (A) careless
(D) no improvement (B) contrary
29. DELETERIOUS (C) callous
(A) impulsive (D) stubborn
(B) salubrious 38. ENIGMATIC
(C) pathetic (A) pithy
(D) inclusive (B) puzzling
30. USURP (C) complicated
(A) rise rapidly (D) illusive
(B) use fully 39. The Bill was passed by the Parliament in the teeth of
(C) produce quickly opposition
(D) own rightfully (A) in the face of
31. DISCORDANT (B) by force of
(A) harsh (C) cooperation
(B) different (D) assistance
(C) harmonious 40. I have a bone to pick with you in this matter .
(D) separate (A) am angry
32. INSOUCIANCE (B) selfish motive
(A) indifferent (C) selfless motive
(B) composure (D) desire
(C) interest
(D) slumber MATHAMETICS (Solved By Abdullah Aashique)
33. GRANDIOSE 41. The greatest number that exactly divides 105, 1001 and
(A) pretentious 2436 is :
(B) brilliant (A) 3
(C) egoistic (B) 7
(D) modest (C) 11
34. INDOLENT (D) 21
(A) casual Solution:
(B) lethargic For finding the greatest number that exactly divides 105,
(C) boring 1001 and 2436 we have to find H.C.F. of the numbers.

60
Collected By: Md. Raihan Ahmed
Job Information For All Students (Bcs & Bank)

Now, 13, find the numbers.


105 = 1 × 3 × 5 × 7 (A) 195 and 143
1001 = 1 × 7 × 11 × 13 (B) 190 and 143
2436 = 1 × 2 × 2 × 3 × 7 × 29 (C) 195 and 145
So, H.C.F. = 7 (D) None of these
42. If the number 91876 * 2 is completely divisible by 8, Solution:
then the smallest whole number in place of * will be : Let the numbers are 15x and 11x and their H.C.F is x.
(A) 1 Given, H.C.F = x = 13
(B) 2 Now, the numbers are= 15 × 13 and 11 × 13 = 195 and 14
(C) 3 46. How many digits will be there to the right of the
(D) 4 decimal point in the product of 95.75 and 0.2554 ?
Solution: (A) 5
Here, the number 91876 * 2 is completely divisible by 8, (B) 6
then the number 6 * 2 must be divisible by 8. (C) 7
If we put * = 3 then we have, 632 ÷ 8 = 72. (D) None of these
So the smallest whole number in place of * will be : 3 Solution:
43. If the sum of two numbers is 33 and their difference is Here, 95.75 = decimal places = 2 and 0.2554 = decimal
15,the smaller number is : places = 4
(A) 9 Sum of decimal places = 2 + 4 = 6.
(B) 12 So there will be 6 significant digits to the right.
(C) 15 47. The value of 1001 ÷ 11 of 13 is :
(D) 18 (A) 7
Solution: (B) 91
Let the numbers are x and y (C) 143
(D) 169
And, x Solution:
Adding eqn (i) & (ii) → 2x = 48 and x = 24 1001 ÷ 11 of 13
Putting the value of x in eqn (i) we have, y = 33 24 = 9 = 1001 ÷ 143
The smaller number is : 9 =7
44. The difference of two numbers is 11 and one-fifth of 48. The average of the two-digit numbers, which remain
their sum is 9. Find the numbers. the same when the digits interchange their positions , is :
(A) 28 and 17 (A) 33
(B) 28 and 18 (B) 44
(C) 28 and 19 (C) 55
(D) None of these (D) 66
Solution: Solution:
Let the numbers are x and y The two digit numbers remain same after interchanging the
digits are = 11, 22, 33, 44, 55, 66, 77, 88, 99.
And, x Then average
Adding eqn (i) & (ii) → 2x = 56 and x = 28 = (11+22+33+44+55+66+77+88+99) ÷ 9 = 495 ÷ 9 = 55
Putting the value of x in eqn (i) we have, y = 45 28 = 17 49. Tanya's grandfather was 8 times older to her 16 years
The numbers are : 28 and 17 ago . He would be 3 times of her age 8 years from now .
45. Two numbers are in the ratio of 15:11. If their H.C.F is Eight years ago, what was the ratio of Tanya's age to that of

61
Collected By: Md. Raihan Ahmed
Job Information For All Students (Bcs & Bank)

her grandfather ? Let sum be x


(A) 1:2 S.I. = TK.600, Time = 10 years.
(B) 1:5 Rate = (100 × 600) ÷ (x × 10) = 6000 ÷ x%
(C) 3:8 Interest for first 5 years
(D) None of these = (x × 5 × 6000) ÷ (x × 100) = TK.300.
Solution:
Interest for next 5 years
Let, 16 years ago Tanya's age was = x years
=(3x × 5 × (6000 ÷ (x × 100))) = TK.900
Therefore, Total interest = TK.(300 + 900) = TK.1200
Now, Tanya's age = x + 16 years
52. If the compound interest on a sum for 2 years at 12.5%
Tanya's grand
per annum is TK. 510 the simple interest on the same at
After 8 years after, Tanya's age will be = x + 16 + 8 = x +
the same rate for the same period of time is :
24 years
(A) TK. 400

years (B) TK. 450

ATQ, (C) TK. 460

8x + 24 = 3 (x + 24) (D) TK. 480


8x 3x = 72 24 Solution:
5x = 48 C = P(1 + r)n P
∴ x = 9.6 years 12.5 2
Or, 510 = P (1 + ) P
100
8 years ago Tanya's age was = 9.6 + 8 = 17.6 years
112.5 2
Or, 510 = P ( 100 ) P

84.8 years 1125 2


Or, 510 = P (1000) P
Required ratio = 17.6 : 84.8 = 11:53
9 2
50. The total age of A and B is years more than the total Or, 510 = P ( ) P
8
age of B and C. C is how many years younger than A ? 81P−64P
Or, 510 =
(A) 12 64
Or, P = 1920 TK.
(B) 24
Again, I = Pnr
(C) 26
12.5
(D) None of these = 1920 × 2 ×
100
Solution: = 480 TK.
ATQ, 53. 0.01 is what percent of 0.1 ?
A + B = 12 + B + C (A) 1/100
A = B + C + 12 B (B) 1/10
A = C + 12 (C) 10
51. The simple interest on a sum of money will be TK. 600 (D) 100
after 10 years. If the principal is trebled after 5 years , Solution:
what will be the total interest at the end of the tenth year ? 0.1 = 100%
(A) TK. 600 100 × 0.01
0.01 = = 10
0.1
(B) TK. 900
54. Sixty-five percent of a number is 21 less than four-fifth
(C) TK. 1200
of that number. What is the number ?
(D) TK. 1500
(A) 140
Solution:
(B) 142

62
Collected By: Md. Raihan Ahmed
Job Information For All Students (Bcs & Bank)

(C) 144 18 pumps raise 2170 tonnes of water working 7 hours a day
(D) 145 in 10 days
Solution: 18 pumps raise 2170 tonnes of water working 1 hours a day
65 4x in 10 × 7 days
ATQ, x × = 21
100 5 1 pumps raise 1 tonne of water working 1 hours a day in
13x 4x − 105
= 10 × 7 ×18
20 5 days
2170
80x 65x = 2100
16 pumps raise 1736 tonnes of water working 9 hours a day
x = 140
10 × 7 × 18 × 1736
55. An article when sold at a gain of 5% yields TK. 15 more in days = 7 days
2170 × 16 × 9
than when sold at a loss of 5% . Its cost price would be : 58. To fill a tank , 25 buckets of water is required. How
(A) TK. 150 many buckets of water will be required to fill the same tank
(B) TK. 200 if the capacity of the bucket is reduced to two-fifth of its
(C) TK. 250 present ?
(D) TK. 300 (A) 10
Solution: (B) 35
SP at 5% gain = 105 TK. (C) 62.5
SP at 5% loss = 95 TK. (D) None of these
Difference in SP = 10 TK. Solution:
100×15 Let the capacity of 1 bucket = x.
∴ CP = = 150 TK.
10 Then, capacity of tank = 25x.
56. An express train travelled at an average speed of 100
New capacity of bucket = (2/5)x
km/hr, slopping for 3 minutes after every 75 km. How long
Required no of buckets = 25x/(2x/5) = 62.5
did it take to reach its destination 600 km from the starting
59. A man can do a piece of work in 5 days. But with the
point ?
help of his son, he can do it in 3 days . In what time can
(A) 6 hrs 21 min
the son do it alone ?
(B) 6 hrs 24 min
(A) 6.5 days
(C) 6 hrs 27 min
(B) 7 days
(D) 6 hrs 30 min
(C) 7.5 days
Solution:
(D) 8 days
Total slopping in 600 km = 600 ÷ 75 = 8 No s.
Solution:
Time required for slopping = 8 × 3 = 24 Minutes
Man performs in 1 day = 1/5 part
Total time required= (600 ÷ 100) + 24 Min = 6 hrs 24 min
Man + son perform in 1 day = 1/3 part
57. If 18 pumps can raise 2170 tonnes of water in 10 days ,
son performs 1 day = 1/3 1/5 part = 2/15 part
working 7 hours a day ; in how many days will 16 pumps
Required time for son = 15/2 = 7.5 days
raise 1736 tonnes of water , working 9 hours a day ?
60. Tow workers A and B are engaged to do a work . A
(A) 6
working alone takes 8 hours more to complete the job than
(B) 7
if both worked together . If B worked along, he would need
(C) 8
4.5 hours more to complete the job than they both working
(D) 9
together . What time would they take to do the work
Solution:
together ?
(A) 4 hours

63
Collected By: Md. Raihan Ahmed
Job Information For All Students (Bcs & Bank)

(B) 5 hours C = 10 taka; B gets = 18 taka; A gets = 25 taka


(C) 6 hours A:B:C = 25:18:10
(D) 7 hours 63. Three partners A, B, C start a business . Twice A's
Solution: capital is equal to thrice B's capital and B's capital is four
Let A and B together take x hours to complete the work. times C's capital . Out of a total profit of TK. 16,500 at the
Then, A alone takes (x + 8) hours end of the year , B's share is :
B alone takes (x + 9/2) hours to complete the work (A) TK. 4000
ATq, (B) TK. 6000
1 1 1 (C) TK. 7500
+ 9 =
x+8 x+ x (D) TK. 6600
2
1 2 1
+ =
Solution:
x+8 2x + 9 x
x(4x+25) = (x+8)(2x+9)
taka
2x2 = 72
And, 2A = 3B → A = 6x taka
x=6
ATQ,
61. Two pipes A and B can fill a tank in 20 and 30 minutes
= 16500 × 4x/11x = 6000
respectively . If both the pipes are used together , then
how long will it take to fill the tank ?
64. The value of √𝟏𝟎 + √𝟐𝟓 + √𝟏𝟎𝟖 + √𝟏𝟓𝟒 + √𝟐𝟐𝟓
(A) 12 min
(B) 15 min (A) 4
(C) 25 min (B) 6
(D) 50 min (C) 8
Solution: (D) 10
A performs in 1 minute = 1/20 part Solution:
B performs in 1 minute = 1/30 part
A + B perform in 1 minute = 1/20 + 1/30 part = 5/60 part √
10 + √25 + √108 + √154 + √225
Required time = 60/5 = 12 minutes
62. A sum of TK. 53 is divided among A, B, C is such a way
that A gets TK. 7 more than what B gets and B gets TK. 8 = √10 + √25 + √108 + √154 + 15

more than what C gets. The ratio of their shares is :


= √10 + √25 + √108 + √169
(A) 16:9:18
(B) 25:18:10
= √10 + √25 + √108 + 13
(C) 18:25:10
(D) 15:8:30 = √10 + √25 + √121

Solution: = √10 + √25 + 11

Let C gets = x taka; B gets = x + 8 taka; A gets = x + 15 = √10 + √36

taka = √10 + 6

ATQ, = √16

3x + 23 = 53 =4

3x = 30 65. The largest number from among

x = 10 (A) √2

64
Collected By: Md. Raihan Ahmed
Job Information For All Students (Bcs & Bank)
3
(B) √3 When C covers 350 meters B covers (169 × 350)/182
4
(C) √4 meters = 325 meters
(D) All are equal C will beat B by = (350 325) = 25 meters
Solution: 69. If x =1+ √ 2 and y = 1 −√ 2, find the value of (x2 + y2)
√ (A) 6
∛ (B) 8

∜ (C) 10

The largest number is ∛3 (D) 12


Solution:
66. If ax = b, by = c and cz = a, then the value of xyz is :
x2 = 1 + 2 + 2√2 = 3 + 2√2;
(A) 0
(B) 1 y2 = 1 + 2 − 2√2 = 3 − 2√2

(C) 1/abc x2 + y2 = 3 + 2√2 + 3 − 2√2 = 6

(D) abc 70. The reflex angle between the hands of a clock at 10.25
Solution: is :
x
Given , a = b (A) 180°
z x
(c ) = b (B) 192. 5°

(by)zx = b (C) 195°

bxyz = b (D) 197. 5°


Solution:
xyz = 1
Here H = 10 , M = 25 and the minute hand is behind the
67. The average of the first nine -prime numbers is:
hour hand.
(A) 9
M M
(B) 11 Hence the angle = 30 (H − )+
5 2
1 25 25
(C) 11  = 30 (10 - )+
9 2 2
2
(D) 11 = 30 (10 5) + 12.5
9
Solution: = 30 (10 5) + 12.5
The required average = 30 × 5 + 12.5
= (2 + 3 + 5 + 7 + 11 + 13 + 17 + 23 + 29)/9 = 162.5º
= 110/9 But the question is to find out the reflex angle.
1
= 11 Reflex angle = 360 - 162.5 = 197.5°
9

68. In a race of 200 m, A can beat B by 31 m and C by 18


m. In a race of 350 m, C will beat B by: COMPUTER
(A) 22.75m 71. IPV4 address is
(B) 25 m (A) 16 bit
(C) 19.5 m (B) 32 bit

(D) 7.7 m (C) 64 bit

Solution: (D) None

A : B = 200 : (200 31) = 200 : 169 72. A hard disk is divided into tracks which are further
subdivided into :
A : C = 200 : (200 18) = 200 : 182
(A) Clusters
A/B = 200/169; A/C = 200/182
(B) Sectors
C/B = 182/169
(C) Vectors
When C covers 182 meters B covers 169 meters

65
Collected By: Md. Raihan Ahmed
Job Information For All Students (Bcs & Bank)

(D) None GENERAL KNOWLEDGE


73. Which one of the following represent the binary 81. Where in Australia did Captain Cook land ?
equivalent of the decimal number 23 ? (A) Great Barrier Reef
(A) 01011 (B) Great Australian Bight
(B) 10111 (C) Botany Bay
(C) 10011 (D) Moreton Bay
(D) None 82. In which country did the Man Mau uprising (1952-60)
74. The last address of IP address represents occur ?
(A) Unicast address (A) Uganda
(B) Network address (B) Kenya
(C) Broadcast address (C) Nigeria
(D) None (D) Zambia
75. The machine readable form of a program is called as ? 83. In the CAMELS bank-rating system, the letter 'A' stands
(A) Source code for :
(B) Object code (A) Average quality
(C) Executable File (B) Asset quality
(D) None (C) Adequate
76. CSS is an acronym for (D) Annual quantity
(A) Cascading Style Sheet 84. Who won six consecutive Wimbledon singles titles in the
(B) Costume Style Sheet 1980s ?
(C) Cascading System Style (A) Ivan Lendl
(D) None (B) Boris Becker
77. What is the use of Forms in HTML ? (C) Chris Evert
(A) To display contents email (D) Martina Navratilova
(B) To display animation effect 85. The highest tunnel for high-speed train in the world has
(C) To collect user's input been recently opened in :
(D) None (A) Japan
78. A computer port is used to-- (B) China
(A) Communicate with hard disks (C) Russia
(B) Download files (D) Brazil
(C) Communicate with other computer peripherals 86. The Declaration of Independence in the USA was written
(D) None by :
79. SQL stands for (A) George Washington
(A) Structure Query Language (B) Abraham Lincoln
(B) Standard Query Language (C) Dwight D. Eisenhower
(C) Standard Query Linguistics (D) Thomas Jefferson
(D) None 87. The only country in South East Asia that was never
80. Which of the following is not in OSI model ? colonized by a European power is :
(A) Physical layer (A) Thailand
(B) Internet layer (B) Nepal
(C) Transport layer (C) Myanmar
(D) None (D) Bhutan
88. The European Central Bank is based in :

66
Collected By: Md. Raihan Ahmed
Job Information For All Students (Bcs & Bank)

(A) Brussels (A) Second


(B) Frankfurt (B) Third
(C) Maastricht (C) Fourth
(D) Paris (D) Fifth
89. The series of sketches known as 'Famine Sketches' was 96. The Italian explorer who is said to have discovered
drawn by : America is :
(A) S. M. Sultan (A) Da Vinci
(B) Kamrul Hassan (B) Cortez
(C) Qaiyum Chowdhury (C) Caravaggio
(D) Zainul Abedin (D) Columbus
90. 'Bourse' is a European word for what financial 97. The Bangladeshi born UN Under Secretary General for
mechanism ? Field Support is :
(A) Foreign exchange market (A) Asma Jahangir
(B) Stock exchange (B) Amirah Haq
(C) Commodity market (C) Irene Khan
(D) Deferred payments (D) Niloufer Nina Ahmad
91. The highest mountain in Bangladesh is : 98. Which canal is known as the 'Highway to India' ?
(A) Mowdok Taung (A) Suez Canal
(B) Keokradong (B) Grand Canal
(C) Muripha Taung (C) Indus Canal
(D) Reng Tlang (D) Ravi Canal
92. Which is the financial centre and main city of 99. In which city was Martin Luther King assassinated in
Switzerland ? 1968 ?
(A) Lausanne (A) Oklahoma
(B) Berne (B) Memphis
(C) Zurich (C) New Orleans
(D) Geneva (D) Phoenix
93. What is the name of the ancient Roman city that was 100. The free trade area comprising Argentina, Brazil,
destroyed by the eruption of the volcano Vesuvius in the Paraguay and Venezuela is called :
first century ? (A) CAFTA
(A) Naples (B) MERCOSUR
(B) Pompeii (C) COMECON
(C) Rome (D) SACU
(D) Florence
94. The American state that is nearest to the former Soviet
Union is :
(A) Washington BANGLA
(B) Hawaii 1. ' '
(C) Alaska
(D) New York
95. In terms of Muslim population, the rank of Bangladesh (C)
is : (D) 

67
Collected By: Md. Raihan Ahmed
Job Information For All Students (Bcs & Bank)

2. ' '-- --- (C) 


(D)
 11. ' ' ?
(C)
(D)
3. ' '-- -- (C) 
(D)
12. ' ' ?
(C)
(D) 
4. ? (C)
(D) 
13. ?
(C)  - 
(D) -
5. ? (C) -
(D) -
14. ?
(C) 
(D) 
6. ' ' , __ (C)
(D)
 15. ?
(C)
(D) 
7. ' ' __ (C)
(D)
 16. - ?
(C) ,
(D) ,
8. ' ' __ (C) , 
(D) ,
 17. - ?
(C) ,
(D) ,
9. ' ' __ (C) ,
(D) 
 18. ?
(C) 
(D)
10. ' ' __ (C)
(D)

68
Collected By: Md. Raihan Ahmed
Job Information For All Students (Bcs & Bank)

19. ? (D) emblematic


27. ALTRUISTIC
(A) truthful
(C) (B) capable
(D)  (C) loving
20. ' ' __ (D) generous
28. AMELIORATE
(A) strengthen
(C)  (B) lessen
(D) (C) cause
(D) replace
29. concomitant
21. We must adapt ourselves ______ our circumstances . (A) assistant
(A) with (B) artificial
(B) in (C) pellucid
(C) to (D) desecrate
(D) by 30. equivocate
22. Nothing can absolve you ______ the promise you have (A) slouch
made. (B) sigh
(A) from (C) stability
(B) in (D) be truthful
(C) for 31. Irascible : Provoked :: _____ : _____
(D) with (A) Credulous : Persuaded
23. You must dispense _____ the services of your gardener . (B) Fanatic : Reasoned
(A) in (C) Gullible : Fooled
(B) of (D) Humorous : Amused
(C) at 32. Shard : Pottery :: _____ : _____
(D) with (A) Splinter : Wood
24. All of the people at this conference are _____. (B) Silver : Glass
(A) mathematic teachers (C) Slab : Concrete
(B) mathematics teachers (D) Fiber : Cloth
(C) mathematics teacher 33. Fox : cunning :: _____ : _____
(D) mathematic's teacher (A) Bus : school
25. I took an aspirin to ______ the pain in my knee. (B) Tool : dentist
(A) cited (C) Taj Mahal : beautiful
(B) deviate (D) Class : students
(C) alleviate 34. choose the correctly spelt word
(D) plagiarized (A) CONDIUT
26. I try to avoid making ______ remarks.s I don't like to (B) CONDUIT
hurt people's feelings. (C) CONNDUIT
(A) essence (D) CONDEUT
(B) derogatory 35. choose the correctly spelt word
(C) underpinning (A) PARIMETER

69
Collected By: Md. Raihan Ahmed
Job Information For All Students (Bcs & Bank)

(B) PERIMETER ∴ Illiterate poor people


(C) PERIMOTOR = 40 4 = 36
(D) PARIMETER 36
∴ required per cent
36. choose the correctly spelt word 60
42. A number is decreased by 10% and then increased by
(A) CANINE
10% . The number so obtained is 10 less than the original
(B) CANNINNE
number. what was the original number ?
(C) CANNINE
(A) 1,000
(D) CANINEE
(B) 1050
37. AN unexpected piece of good fortune
(C) 1500
(A) turnstile
(D) 2000
(B) windfall
Solution:
(C) philanthropy
10% decrease and 10% increase so total change
(D) benevolence
= -10+10- 10 10 /100 =-1%
38. One who cannot be corrected
So, 1% decrease = 10 then 100% decrease
(A) incurable
= 10 100= 1000.
(B) incorrigible
43. In a fraction , if numerator is increased by 40% and
(C) hardened
denominator is increased by 80% then what fraction of the
(D) vulnerable
original is the new fraction ?
39. Rack and ruin
(A) 1/2
(A) Successful
(B) 7/9
(B) debt
(C) 7/18
(C) destroyed
(D) None of these
(D) ransacked
Solution:
40. Iron will
Let the fraction = x/y
(A) good health
The new fraction = x + 40 100 / y + 80 100
(B) strong determination
= 140 /100 / 180 100 = 7/9(x/y)
(C) Sticks to the point
∴ New fraction = 7/9 of original fraction
(D) has high hemoglobin
44. In an examination , 34% of the students failed in
Mathematics and 42% failed in English . If 20% of the
MATHAMETICS (Solved By Mehadi Hassan Fuad )
students failed in both the subjects , then the percentage of
41. In a city , 40% of the people are illiterate and 60% are
students who passed in both the subjects was:
poor. Among the rich , 10% are illiterate. What percentage
(A) 44
of the poor population is illiterate?
(B) 50
(A) 36%
(C) 54
(B) 40%
(D) 56
(C) 60%
Solution:
(D) None of these
Let the number of percentage of student who passed in the
Solution:
both subject is x
Let the population of the city be 100.
Now,
Total illiterate people = 40
100% = 34% + 42% -20% + x
Poor people = 60
= 44%
Rich people = 40
45. A sum invested at 5% simple interest per annum grows
40 ×10
Illiterate rich people to Tk. 504 in 4 years The same amount at 10% simple
100
70
Collected By: Md. Raihan Ahmed
Job Information For All Students (Bcs & Bank)

interest per annum in years will grow to : 1170-1140=30


(A) Tk. 420 Work = day × worker
(B) Tk. 450 30 = 30 × day
(C) Tk. 525 Day = 1.
(D) Tk. 550 So,it's take one day more
Solution: 48. A garrison of 500 men had provisions for 27 days .
Let the sum be tk. X After 3 days a reinforcement of 300 men arrived . For how
So, Simple interest, many more days will the remaining food last now?
504 - x = x * 4 * 5/100 (A) 15
or x = 420 (B) 16
Now at 10% SI the interest is (C) 15.5
=(420 * 2.5 * 10)/100 (D) 18
=105 Solution:
So, the sum will be 105 + 420 = 525. Days left = 27 3 = 24
46. A person borrows Tk. 5000 for 2 years at 4% p.a. Let the remains food last = x days
simple interest . He immediately lends to another person at Now, Total men after 3 days is 500 + 300 = 800
𝟏
6 % p. a. for 2 years . F ind his gain in the transaction per So, 500 * 24 = 800 * x
𝟒
year . Or, x = 15
(A) Tk. 112.50 49. If 8 men can reap 80 hectares in 24 days , then how
(B) Tk. 125 many hectares can 36 men reap in 30 days ?
(C) TK. 150 (A) 350
(D) TK. 167.50 (B) 400
Solution: (C) 425
gain in 2 years (D) 450
= TK. [(5000 * 25 4 * 2 100) − (5000 * * 100)] Solution:
= TK. (625 - 400) 8 men in 24 days reap 80 hectors
= TK. 225 1 men in 1 days reap 80/ (8 * 24)
gain in 1 year = TK. (225 2) = 112.50 36 men in 30 days reap (80 * 36 * 30)/(8 * 24) hectors =
47. A contractor employed 30 men to do a piece of work in 450 Hectors.
38 days. After 25 days , he employed 5 men more and the 𝟓
50. A man completes of a job in 10 days . At this rate,
work was finished one day earlier. How many days he 𝟖
how many more days will it take him to finish the job?
would have been behind, if he had not employed additional
(A) 5
men?
(B) 6
(A) 1
1 (C) 7
(B) 1
4
3 (D) 7.5
(C) 1
4
1
(D) 1
2
Solution:
Work = day * worker
Total work = 30 * 38 = 1140
51. If the radius of a circle is increased by 6% , then the
25 day work done = 25 * 30 = 750
area is increased by :
35 work done in 12 days = 35 * 12 = 420
(A) 6%
ull done by both 30 + 5 worker = 750 + 420 =1170
(B) 12%

71
Collected By: Md. Raihan Ahmed
Job Information For All Students (Bcs & Bank)

(C) 12.36% 4√2 = √2 a


(D) 16.64% a = 4 cm
Now, area of square = a = (4 ) = 16
π Side of a square whose area is 2 x 16.
a = 32
π ⇒ a = √32
π π π ⇒ a 4√2
π π Now, diagonal of new square = √2a
52. If the height of a triangle is decreased by 40% and its = √2 × 4 √2
base is increased by 40% , what will be the effect on its = 8 cm
area? 55. 10% loss on selling price is what percent loss on the
(A) No change cost price ?
1
(B) 8% decrease (A) 9 %
11
2
(C) 16% decrease (B) 9 %
11
(D) 16% increase (C) 10%
Solution: (D) 11%
rea of triangle = * base * height Solution:
so According to your question, Suppose the CP = 110
ew base b1 = 1.4 * base And selling price = 100
ew height h1 = 0.6 * height the loss is of 10 which is 10 % on selling price.
ew Area = * b1 * h1= 1/2 * 1.4 * base * 0.6 *height Now count this 10 on CP which is 110.
Area = 0.84 Area 10/110 × 100 = 9 %
1
11
Area is decreased by 16 percent 56. The cost price of 20 articles is the same as the selling
53. A tank is 25 m long , 12 m wide and 6 m deep. The price of x articles . If the profit is 25% , then the value of x
cost of plastering its walls and bottom at 75 paise per sq.m is :
is : (A) 15
(A) Tk. 456 (B) 16
(B) TK. 458 (C) 18
(C) TK. 558 (D) 25
(D) Tk. 568 Solution:
Solution: Let C.P. of each article be Re. 1 C.P. of x articles = x.
Area to be plastered = [2(l+b)×h]+(l×b) S.P. of x articles = 20.
= [ 2(25 + 12) × 6] + (25 × 12) = 744sq.m Profit = (20 - x).
Cost of plastering = 744 × 75 100 = Tk.558
54. The diagonal of a square is 4√2 cm. The diagonal of ( – x)
another square whose area is double that of the first x
2000 100 = 25x
square is:
125x = 2000
(A) 8 cm
x = 16.
(B) 8√2 cm
57. At what profit percent must an article be sold so that by
(C) 4√2 cm
selling at half that price , there may be a loss of 30% ?
(D) 16 cm
(A) 25%
Solution:
(B) 36%
Diagonal of square = √2a [a = side]

72
Collected By: Md. Raihan Ahmed
Job Information For All Students (Bcs & Bank)

(C) 40% (B) Binary Information Term


(D) 42% (C) Binary Tree
Solution: (D) None
Let S.P = x 62. A compiler means
New S.P = x/2, Loss = 30%. (A) A person who complies source programs
So, C.P = (100/70 × x/2) = 5x/7. (B) the same thing as a programmer
Profit =( x − 5x/7) = 2x 7 (C) Keypunch operator
∴ Profit% =(2x/7 × 7/5x × 100)% =40% (D) None
58. The sale price of an article including the sales tax is TK. 63. The list of coded instructions is called
616 . The rate of sales tax 10% If the shopkeeper has made (A) computer program
a profit of 12% , then the cost price of the article is : (B) Algorithm
(A) Tk. 500 (C) Flowchart
(B) Tk. 515 (D) None
(C) Tk. 550 64. when a computer is first turned on or restarted ,a
(D) TK. 600 special type of absolute loader is executed ,called
Solution: (A) Compilc and go loader
110% of S.P. = 616 (B) Boot loader
S.P. = (616 x 100)/110 = 560 (C) Bootstrap loader
C.P = (110 x 560)/112 = 500 (D) None
59 An aeroplane covers a certain distance at a speed of 65. which class of IP address has the most host addresses
𝟐
240kmph in 5 hours . To cover the distance in 1 hours, it available by default ?
𝟑
must be travel at a speed of : (A) A
(A) 300 kmph (B) B & C
(B) 360 kmph (C) C
(C) 600 kmph (D) None
(D) 720 kmph 66. IPV4 address is
Solution: (A) 16 bit
= 240 * 5 = 1200 (B) 32 bit
= 1200 * 3/ = (C) 64 bit
60 A clock is started at noon. By 10 minutes past 5, the (D) None
hour hand has turned through : 67. which of the following is not OSI model ?
(A) 145 o
(A) Physical layer
(B) 150 o
(B) Internet layer
o
(C) 155  (C) Transport layer
(D) 160 o
(D) None
Solution: 68. The last address of IP address represents
In 1 hour clock hand move = 360/12 = 30 degree (A) Unicast address
From to 12 pm to 5.10 pm hr hand move (B) Network address
= 30 * 5 + (30/60 * 10) = 155 degree. (C) Broadcast address
(D) None
COMPUTER 69. To check to see if the Web server you are trying to
61. Bit stands for reach is available or is down, which command line utility
(A) Binary Digit should you use?

73
Collected By: Md. Raihan Ahmed
Job Information For All Students (Bcs & Bank)

(A) PING (C) Either I or II is sufficient


(B) ICS (D) Both I and II are sufficient
(C) Telnet 77. who is to the immediate right of P among five persons
(D) None P, Q, R, S and T facing North?
70. Web hosting service is a type of Statements :
(A) Genetic algorithm 1. R is third to the left of Q and P is second to the right of
(B) Internet hosting service R.
(C) computer programming 2.Q is to the immediate left of T who is second to the right
(D) None of P.
71. A proxy server is used for which of the following ? (A) I alone is sufficient while II alone is not sufficient
(A) To provide security against unauthorized users (B) II alone is sufficient while I alone is not sufficient
(B) To process client requests for web pages (C) Either I or II is sufficient
(C) To process client requests for database access (D) Neither I nor II is sufficient
(D) None 78. How many pages of book X did Mohsin read on Sunday
72. with respect to a network interface card , the term 10 ?
/100 refers to Statements :
(A) protocol speed 1. The book has 300 pages out of which two - thirds were
(B) minimum and maximum server speed read by him before Sunday .
(C) meagabits per seconds 2. Robert read the last 40 pages of the book on the
(D) None morning of Monday?
73. Virtual memory consists of __ . (A) I alone is sufficient while II alone is not sufficient
(A) Static RAM (B) II alone is sufficient while I alone is not sufficient
(B) Dynamic RAM (C) Either I or II is sufficient
(C) Magnetic memory (D) Both I and II are sufficient
(D) None 79. How many visitors saw the exhibition yesterday?
74. Cache memory acts between __. Statements :
(A) CPU and RAM 1. Each entry pass holder can take up to three persons with
(B) RAM and ROM him / her .
(C) CPU and Hard Disk 2. In all, 243 passes were sold yesterday.
(D) None (A) I alone is sufficient while II alone is not sufficient
75. which of the following protocol is not used in the (B) II alone is sufficient while I alone is not sufficient
Internet ? (C) Either I or II is sufficient
(A) Telnet (D) Neither I nor II is sufficient
(B) WIRL 80. what is the monthly salary of Rahman ? Statements :
(C) Gopher 1. Rahman's basic salary is Tk. 100 more Rajan's salary who
(D) None also serves in Rahman's company.
76. How is Related to K? 2. Other allowances drawn by Rajan besides his basic salary
Statements: are Tk. 2,000 per month which is TK. 50 less than Rahan's
1. R's sister J has married T's brother L, who is the only son salary .
of his parents 3. Rajan's basic salary is TK. 1550 per month.
2. K is the only daughter of L and J. (A) Only II
(A) I alone is sufficient while II alone is not sufficient (B) Only II and III
(B) II alone is sufficient while I alone is not sufficient

74
Collected By: Md. Raihan Ahmed
Job Information For All Students (Bcs & Bank)

(C) Only I and II (B) Italy


(D) All I , II and II (C) Turkey
(D) Malaysia
GENERAL KNOWLEDGE 89. which of the division of Bangladesh has no relation
81. EUROSTAR is the name of a through border with India ?
(A) Train service in the EU (A) Rajshahi
(B) Bus service in the EU (B) Khulna
(C) Air Service in the EU (C) Barisal
(D) None of the above (D) Chittagong
82. "The Great Depression " is historically linked with 90. when did the "Mobile War Museum " launch in
(A) USA Bangladesh?
(B) UK (A) 1 July, 2000
(C) France (B) 1 July 2001
(D) Japan (C) 10 July, 2001
83. The Iraqi militant group , ISIS stands for (D) 10 July , 2002
(A) Islamic state in Iraq and Syria
(B) Islamic Struggle for Islamic state ECONOMICS
(C) International struggle for Islamic state 91. The Marshall -Lerner condition states that a devaluation
(D) International struggle for Islamic Salvation will improve a country's balance of payments if the sum of
84. who among the Bangladesh girls has invested artificial elasticity of demand for exports and imports is:
human lung? (A) zero
(A) Meherunnuesa (B) equal to one
(B) Shakera Arefin (C) greater than one
(C) Ismart Jahan (D) less than one
(D) Ayesha Arefin Tumpa 92. A country with a current account surplus
85. The first woman who received the Ekushey paddak (A) is using foreign saving to supplement domestic saving in
award in Bangladesh is 1976 was: financing investment projects . 
(A) Sanjida Khatun (B) is using part of its domestic saving to lend overseas
(B) Sufia Kamal rather than finance domestic investment .
(C) Razia khan (C) the benefit that the government gets form imposing
(D) Badrunnessa tariffs .
86. which country will host the NAM summit in 2015 ? (D) the costs imposed by a tariff on the consumers of the
(A) India imported goods.
(B) Venezuela 93. There will be no crowding out if
(C) Brazil (A) the demand for money increase as real GDP increased.
(D) Malaysia (B) supply of money is totally unresponsive to changes in
87. How many countries are truculently using Euro? the interest rate.
(A) 16 (C) investment is totally unresponsive to changes in the
(B) 17 interest rate.
(C) 18 (D) investment is totally unresponsive to changes in the
(D) 19 real GDP.
88. which country is not a member of G -20? 94. In 2008 the exchange rate of the Euro appreciated
(A) China against the US dollar . A possible cause for the appreciation

75
Collected By: Md. Raihan Ahmed
Job Information For All Students (Bcs & Bank)

of the Euro was


(A) increase investment in USA by the European Union
(B) increasingly more US tourists were visiting EU countries BANGLA
(C) increasingly more US exports to EU countries 1. ' ' __
(D) the European Central Bank raised the value of the _
Euro
95. When Bank Rate increases then _
(A) interest rate decreases (C)
(B) money supply increases (D) 
(C) bank's loanable funds decreases 2. ' ' __ _
(D) cost of funds decreases
96. The Banking companies in Bangladesh are regulated 
under the provision of __ (C)
(A) Bank Companies Act, 1991 .  (D)
(B) Companies Act , 1994 . 3. ' '
(C) Bank company Amendment Act , 2003 . _
(D) Bangladesh Bank Order , 1972 .
97. which institution is the regulatory authority of capital 
market intermediaries in Bangladesh ? (C)
(A) Bangladesh Bank (D) -
(B) Securities & Exchange Commission 4. ' '_
(C) Ministry of Commerce __
(D) Bangladesh Securities & Exchange Commission
98. What is the current Bank Rate of Bangladesh Bank? 
(A) 5.0% (C)
(B) 5.5% (D)
(C) 6.0% 5. ?
(D) 6.5% ?

99. Which one is considered as the banker to government (C)
of Bangladesh ? (D)
(A) Sonali Bank Limited 6. ?
(B) Bangladesh Development Bank Limited
(C) Investment Corporation of Bangladesh
(D) Bangladesh Bank (C)
(D) 
100. When Bank Rate increases then __ 7. ?
(A) interest rate decreases
(B) money supply increases
(C) bank's loanable funds decreases (C)
(D) cost of funds decreases (D) 

76
Collected By: Md. Raihan Ahmed
Job Information For All Students (Bcs & Bank)

8. ? (C)
(D)
 17. ' ' ?
(C)
(D) 
9. ? (C)
 (D)
18. ?
(C) - 
(D) -
10. ? (C) -
(D) -
- 19. ' ' ?
(C) -
(D) - 
11. ' ' _ (C) 
(D)
20. ?
(C) 
(D) 
12. ' ' _ (C)
(D)

(C)
(D) 21. The blind man was feeling -----the table.
13. ' ' _ (A) in
 (B) for
(C) by
(C) (D) out
(D) 22. He does not work with his hands, he works---the
14. ' ' ? machine.
(A) with
(B) by
(C) (C) at
(D)  (D) on
15. ' '- ? 23. He is an amateur ---stenography.
(A) for
(B) in
(C) (C) to
(D)  (D) of
16. ' ' ? 24. The accelerated growth of public employment _____ the
 dramatic expansion of budgets and programmes.
(A) parallels

77
Collected By: Md. Raihan Ahmed
Job Information For All Students (Bcs & Bank)

(B) contains (D) Examine : Opinion


(C) revolves 32. Choose the word that best completes the relationship
(D) escapes similar to that in the given pair : Impromptu : Rehearsal : :
25. Man is still a ______ in the labour market. (A) Numerical : Calculation
(A) glut (B) Calculation : Calculator
(B) possibility (C) Offhand : Premeditation
(C) commodity (D) Makeshift : Fortitude
(D) provision 33. Choose the word that best completes the relationship
26. No one can say for sure how ____ the awards have similar to that in the given pair : Incubator : Infant : :
been. (A) Hive : Bee
(A) determined (B) Greenhouse : Calculator
(B) effective (C) Offhand : Revolver
(C) reducible (D) Goalkeeper : Goalpost
(D) effervescent 34. Choose the correct spelling.
27. Select the one that is nearest in meaning to the given (A) erroniously
word : ABASE (B) erroneusly
(A) low (C) erroneously
(B) peaceful (D) erroniosly
(C) humble 35. Choose the correct spelling.
(D) cruel (A) demmurage
28. Select the one that is nearest in meaning to the given (B) demurrage
word : ACUMEN (C) demarrage
(A) beauty (D) demurage
(B) appearance 36. Choose the correct spelling.
(C) insight (A) acquaintence
(D) distaste (B) acquantance
29. Select the one that is opposite in meaning to the given (C) acquaintance
word : admonition (D) acquentence
(A) warning 37. A person with a long experience of any occupation.
(B) threat (A) veteran
(C) fear (B) genious
(D) request (C) seasoned
30. Select the one that is opposite in meaning to the given (D) ambidexterous
word : assuage 38. Fear of being enclosed in a small closed space.
(A) resolve (A) agoraphobia
(B) decrease (B) claustrophobia
(C) separate (C) xenophobia
(D) irritate (D) paranoia
31. Choose the word that best completes the relationship 39. His verbal apology does not absolve him for the sins.
similar to that in the given pair : Sap : Vitality : : (A) does not absolve him off the sins
(A) Drain : Resolve (B) did not about the best and worst.
(B) Bore : Tedium (C) bring out best and worst
(C) Enhance : Allure (D) No improvement

78
Collected By: Md. Raihan Ahmed
Job Information For All Students (Bcs & Bank)

40. Adversities bring in the best and worst in different (B) 500
human beings. (C) 800
(A) bring out the best and the worst (D) 1000
(B) bring about the best and worst Solution:
(C) bring out best and worst Given that the student got 125 marks and still he failed by
(D) No improvement. 40 marks
The minimum pass mark = 125 + 40 = 165
MATHAMETICS Given that minimum pass mark = 33% of the total mark
41. What percentage of numbers from 1 to 70 have squares total mark =33/100 =165
that end in the digit 1 ? total mark = 16500/33 = 500
(A) 1 44. If x% of y is equal to z, What percent z is x ?
(B) 14 (A) y2/100
(C) 20 (B) y/1002
(D) 21 (C) 100/y
Solution: (D) 1002/y
The possible numbers are Solution:
1,11,21,31,41,51,61 x%y=z
9,19,29,39,49,59,69 Or, x = 100z/y
percentage = (14/70) ×100=20% LeT P% of Z = x
42. A number,when 35 is subtracted from it, reduces to its or, p = 1002/y
80 percent. What is four-fifth of that number ? 45. A sum of money at simple interest amounts to Tk. 815
(A) 70 in 3 tears and to Tk. 854 in 4 years.The sum is:
(B) 90 (A) Tk.650
(C) 125 (B) Tk.690
(D) 140 (C) Tk.698
Solution: (D) Tk. 700
Let the number be x. Solution:
Given that when 35 is subtracted from it, reduces to its S.I. for 1 year = Tk. (854 - 815) = Tk. 39.
80%. S.I. for 3 years = Tk.(39 x 3) = Tk. 117.
x - 35 = x(80/100) Principal = Tk. (815 - 117) = Tk. 698
x - 35 = x(4/5) 46. The simple interest on a sum of money at 8% per
x - 35 = 4x/5 annum for 6 years is half the sum. The sum is :
5x - 175 = 4x (A) Tk. 4800
x = 175. (B) Tk. 6000
Then 4/5th of x (C) Tk. 8000
= 4/5 * 175 (D) Data inadequate
= 4 * 35 Solution:
= 140. GIVEN:
Therefore the number is 140. Rate (R) of Simple Interest = 8%
43. A student has to obtain 33% of the total marks to pass. Simple Interest (S.I) = Half of the Sum of money
He got 125 marks and failed by 40 marks. The maximum To find:
marks are: Let the sum be P, where P (or the Sum) denotes the
(A) 300 Principle amount.

79
Collected By: Md. Raihan Ahmed
Job Information For All Students (Bcs & Bank)

Given that R = 8% (C) 7


N= 6 years (D) 49
And, given in the sum, Solution:
S.I = P/2 Let the required number days be x.
We know, Less spiders, More days (Indirect Proportion)
S . I .= (P × N × R)/100 Less webs, Less days (Direct Proportion)
P/2 = (P × N × R)/100 Spiders 1 : 7 ⋮⋮ 7 : x
But here, we see that P being in the numerator on both Webs 7 : 1 ⋮⋮ 7 : x
side, gets cancelled, leaving no variable, but only whole ⇒1 * 7 *x= 7 * 1 * 7
numbers. x= 7
Thus, Data is Incomplete in this sum. 50. If 3 men or 6 boys can do a piece of work in 10
47. 36 men can complete a piece of work in 18 days. In days,working 7 hours a day; how many days will it take to
how many days will 27 men complete the same work? complete a piece of work twice as large with 6 men and 2
(A) 12 boys working together for 8 hours a day ?
(B) 18 (A) 6
(C) 22 (B) 7.5
(D) 24 (C) 8.5
Solution: (D) 9
Let the required number of days be x. Solution:
Less men, More days (Indirect Proportion) 3 men = 6 boys (6 men + 2 boys) = 14 boys.
∴27 : 36 :: 18 : x More work, More days {Direct Proportion)
⇒ 27 × x = 36 × 18 More boys, Less days (Indirect Proportion)
⇒x=24 More hours per day, Less days (Indirect Proportion)
48. 4 mat-weavers can weave 4 mats in 4 days. At the Work 1 : 2 :: 10 : x
same rate, how many mats would be woven by 8 mat- Boys 14 : 6 :: 10 : x
weavers in 8 days? Hours per day 8 : 7 :: 10 : x
(A) 4 so (1 × 14 × 8 × x) = (2 ×X 6 × 7 × 10)
(B) 8 x = (840/112) = 15/2=7.5
(C) 12 51. The length of a rectangle is twice its breadth . If its
(D) 16 length is decreased by 5 cm and breadth is increased by 5
Solution: cm , the area of the rectangle is increased by 75 sq. cm .
Let the required number of mats be x. Find the length of the rectangle.
More men, More number of mats (Direct Proportion) (A) 20 cm
More days, More number of mats (Direct Proportion) (B) 25 cm
Weavers 4 : 8 ⋮⋮ 4 : x (C) 30 cm
days 4 : 8 ⋮⋮ 4 : x (D) 40 cm
⇒ 4 * 4 * x = 8 * 8 * 74 Solution:
⇒ x= 16 Let length = 2x and breadth = x then
49. If 7 spiders make 7 webs in 7 days, then 1 spider will (2x-5) (x+5) = (2x * x)+75
make 1 web in how many days? 5x-25 = 75
(A) 1 x=20
(B) 7/2 length of the rectangle = 40 cm

80
Collected By: Md. Raihan Ahmed
Job Information For All Students (Bcs & Bank)

52. A courtyard 25 m long and 16 m broad is to be paved so, (8x/5 ) - X = 24


with bricks of dimension 20 cm by 10 cm . The total x= 40
number of bricks required is : length= 64cm
(A) 18000 breadth = 40cm
(B) 20000 area = 64*40 = 2560cm²
(C) 25000 55. A tradesman sold and article at a loss of 20% . If the
(D) None of these selling price had been increased by Tk. 100 . there would
Solution: have been a gain of 5% . what was the cost price of the
25m * 100 = 2500cm article ?
16m * 100 = 1600cm (A) TK. 400
Now area of courtyard = 1600*2500=4000000 (B) TK. 500
Area of a brick = 20 * 10=200 (C) TK. 550
So no. of bricks required would be 4000000/200=20000 (D) Tk. 600
53. The ratio between the length and the perimeter of a Solution:
rectangular plot is 1 : 3 ,what is the ratio between the Let the cost price of the article = x
length and breadth of the plot? Given tradesman sold an article at a loss of 20%
(A) 1 : 2 i.e., ((100 20)/100)x = 0.8x
(B) 2 : 1 If the price is increased by Tk.100
(C) 3 : 2 then selling price becomes = (0.8x+100)
(D) Data inadequate He gains 5% with selling price = 0.8x+100
Solution: Profit = SP−C.P = (0.8x+100) − x
Let the length be x. = 0.2x+100 = 5/100 of C.P
Therefore, perimeter = 3x ⇒ 0.2x + 100 = 0.05x
since ⇒ 0.25x = 100
3x = 2(x +b) ⇒ x = 400
3x = 2x + 2b 56. Peter purchased a machine for Tk. 80,000 and spent tk.
x = 2b 5000 on repair and Tk. 1000 on transport and sold it with
ratio = length:breadth 25% profit . At what price did he sell the machine?
=x : b (A) Tk. 1,50,100
=2b : b (B) TK. 1,06, 250
=2:1 (C) Tk. 1,07, 500
54. The length of a rectangular plot is 60% more than its (D) TK. 1,17,500
breadth . If the difference between the length and the Solution:
breadth of that rectangle is 24 cm. what is the area of that C.P = Tk. (80000 + 5000 + 1000) = 86000
rectangle ? Profit = 25%
(A) 2400 sq. cm S.P = 125% of Tk. 86000 = Tk. 107500
(B) 2480 .cm 57. When a plot is sold for TK. 18,700 , the owner loses
(C) 2560 sq.cm 15% . At what price must the plot be sold in order to gain
(D) Data inadequate 15% ?
Solution: (A) Tk. 21,000
Let breadth= X cm (B) Tk. 22, 500
length = 160x/ 100 (C) TK. 25, 300
= 8x/5 cm (D) TK. 25, 800

81
Collected By: Md. Raihan Ahmed
Job Information For All Students (Bcs & Bank)

Solution: (C) 1200


85 : 18700 = 115 : x (D) 2000
x = 18700 × 11585 = 25300 Solution:
Hence,S.P = Tk.25300 700 is divided by 4 bt not by 400
58. The percentage profit earned by selling an article for So 700 is not a leap year
TK. 1920 is equal to the percentage loss incurred by selling
the same article for TK. 1280 .At what price should the COMPUTER
article be sold to make 25% profit? 61. Each character on the keyboard of computer has an
(A) TK. 2000 ASCII value which stands for
(B) TK. 2200 (A) American Standard Code for Information
(C) TK. 2400 Interchange
(D) Data inadequate (B) American Stock Code for Information Interchange
Solution: (C) American Standard Code for Information Interfere
Let C.P be Tk. X (D) None
1920−𝑥 𝑥−1280 62. when CPU is executing a program that is part of the
Then, × 100 = × 100
𝑥 𝑥
Operating System , it is said to be in __
1920 x=x 1280
(A) Interrupt mode
x = 1600
(B) System mode
Required S. P = 125% of Tk. 1600 = Tk. 2000
(C) Simplex mode
59. A can complete a journey in 10 hours. He travels first
(D) None
half of the journey at the rate of 21 km / hr and second
63. The primary job of the operating system of a computer
half at the rate of 24 km/hr. Find the total journey in km.
is to
(A) 220 km
(A) Manage resources
(B) 224 km
(B) Provide utilities
(C) 230 km
(C) Provide communication interface
(D) 234 km
(D) None
Solution:
64. Which one of the following represents the binary
Let time taken to travel the first half = x hr
equivalent of the decimal number 23?
Then time taken to travel the second half = (10 - x) hr
(A) 01011
Distance covered in the first half = 21x [because, distance
(B) 10111
= time*speed]
(C) 10011
Distance covered in the second half = 24(10 - x)
(D) None
Distance covered in the first half = Distance covered in the
65. Which of the following services does not normally use
second half
TCP?
So,
(A) DNS
21x = 24(10 - x)
(B) HTTP
45x = 240
(C) FTP
x = 16/3
(D) None
Total Distance = 2*21(16/3) = 224 Km [multiplied by 2 as
66. An organization is granted a block , one address is 2.
21x was distance of half way].
2.2.64/20. The organization needs 10 subnets .what is the
60. Which of the following is not a leap year ?
subnet prefix length ?
(A) 700
(A) /20
(B) 800
(B) /24

82
Collected By: Md. Raihan Ahmed
Job Information For All Students (Bcs & Bank)

(C) /23 (C) Vectors


(D) None of these (D) None
67. How long is an IPv6 address? 75. Which of the following is NOT a peripheral device ?
(A) 32 bits (A) Motherboard
(B) 128 bits (B) Scanner
(C) 64 bits (C) Monitor
(D) None of these above (D) None
68. Bride works in which layer of the OSI model? 76. In which year was Aunik born?
(A) Application layer Statements :
(B) Transport layer 1 . Aunik at present is 25 years younger to his mother.
(C) Datalink layer 2. Aunik 's brother , who was born in 1964 , is 35 years
(D) None younger to his mother .
69. DHCP is the abbreviation of (A) I alone is sufficient while II alone is not sufficient
(A) Dynamic Host Control Protocol (B) II alone is sufficient while I alone is not sufficient
(B) Dynamic Host Configuration Protocol (C) Neither I nor II is sufficient
(C) Dynamic Hyper Control Protocol (D) Both I and II are sufficient
(D) None 77. How many children does M have?
70. Websites without a database attached to them have Statements :
which of the following characteristics? 1. H is the only daughter of X who is wife of M .
(A) The ability to generate data via SQL 2. K and J are brothers of M.
(B) Static information using HTML or JAVA Script (A) I alone is sufficient while II alone is not sufficient
(C) The need to use TCP/ IP as the network protocol (B) II alone is sufficient while I alone is not sufficient
(D) None (C) Either I or II is sufficient
71. Which of the following is used to transfer files ? (D) Neither I nor II is sufficient
(A) HTTP 78. How many doctors are practicing in this town ?
(B) FTP Statements :
(C) HTML 1. There is one doctor per seven hundred residents .
(D) None 2. There are 16 wards with each ward with each ward
72. Which organization defines the Web Standards ? having as many doctors as the number of wards
(A) Microsoft Corporation (A) I alone is sufficient while II alone is not sufficient
(B) IBM Corporation (B) II alone is sufficient while I alone is not sufficient
(C) World Wide Web consortium (C) Either I or II is sufficient
(D) None (D) Neither I nor II is sufficient
73. From what location are the 1 st computer instructions 79. Who among P, Q, T, V and M is exactly in the middle
available on boot up? when they are arranged ascending order of their heights ?
(A) ROM BIOS Statements :
(B) CPU 1. V is taller than Q but shorter than M
(C) boo.ini 2. T is taller than Q and M but shorter than P
(D) None (A) I alone is sufficient while II alone is not sufficient
74. A hard disk is divided into tracks which are further (B) II alone is sufficient while alone is not sufficient
subdivided into: (C) Either I or II is sufficient
(A) Clusters (D) Both I and II are sufficient
(B) Sectors

83
Collected By: Md. Raihan Ahmed
Job Information For All Students (Bcs & Bank)

80. Five persons __ A,B, C, D and E are sitting in a row. 87. If it requires 10 minutes to boil 5 eggs then how many
who is sitting in the middle ? minutes will it take to boil eggs simultaneously ?
Statements : (A) 10 minutes
1. B is between E and C . (B) 20 minutes
2. B is to the right of E. (C) 15 minutes
3. D is between A and E . (D) 18.67 minutes
(A) Only I and II 88. In a fruit shop there six apples. You take away four of
(B) Only II and III those. How many apples do you have now?
(C) only I and III (A) 2
(D) All I, II and III (B) 6
(C) 5
GENERAL KNOWLEDGE & MENTAL ABILITY (D) 4
81. The secretariat of the SAARC food Bank is located in ___ 89. If the following number pattern continues , what
(A) Bhutan numbers will come next? 50, 30, 40, 20, 30 ,....
(B) Nepal (A) 10,20
(C) India (B) 20, 0
(D) Bangladesh (C) 30, 50
82. Which of the following countries is an observer of WTO? (D) 40, 20
(A) Russia 90. What is the area of the triangle BCD?
(B) Senegal B
(C) Luxembourg 8
(D) Sudan C 12 D
83. Nasdaq is the name of a (A) 40
(A) Stock Market in the USA (B) 42
(B) Stock Market in the UK (C) 44
(C) Stock Market in Germany (D) 48
(D) Superstore in the UK 91. Each character on the keyboard of computer has an
84. The 'Silk Road ' in the ancient period means ASCHI value which value satand for?
(A) Linking china with the West (A) American Standard code for information Interchange
(B) Linking Japan with the West (B) American stock code for information Interchange
(C) Linking India with the West (C) American Standard code for information interface
(D) Linking Indonesia with the west (D) None
85. The current per capita income of Bangladesh is :
(A) US $ 1230 ECONOMICS
(B) US$ 1044 92. The Marshall -Lerner condition states that a devaluation
(C) US$ 1190 will improve a country's balance of payments if the sum of
(D) US$ 1390 elasticity of demand for exports and imports is :
86. Rahim's father has five sons, name Ten , Twenty , Thirty (A) zero
and Forty. what is the name of the fifth son? (B) equal to one
(A) Fifty (C) greater than one
(B) Forty Five (D) less than one
(C) One 93. A country with a current account surplus
(D) None of these

84
Collected By: Md. Raihan Ahmed
Job Information For All Students (Bcs & Bank)

(A) is using foreign saving to supplement domestic saving in (C) Ministry of Commerce
financing investment projects .  (D) Bangladesh Securities & Exchange Commission
(B) is using part of its domestic saving to lend overseas 99. What is the current Bank Rate of Bangladesh Bank?
rather than finance domestic investment . (A) 5.0%
(C) is accumulating foreign debt. (B) 5.5%
(D) is accumulating capital at a pace higher than its (C) 6.0%
domestic saving rate. (D) 6.5%
94. The effective rate of protection measures 100. Which one is considered as the banker to government
(A) the percentage by which domestic producers' gross of Bangladesh?
receipts are increased by protection. (A) Sonali Bank Limited
(B) the percentage increase in domestic producers ' value (B) Bangladesh Development Bank Limited
added that is due to protection .  (C) Investment Corporation of Bangladesh
(C) the benefit that the government gets from imposing (D) Bangladesh Bank
tariffs .
(D) the costs imposed by a tariff on the consumers of the
imported goods.
95. There will be no crowding out if BANGLA
(A) the demand for money increases as real GDP increases 1. ' ' -
. 
(B) supply of money is totally unresponsive to changes in
the interest rate . (C)
(C) investment is totally unresponsive to changes in the (D)
interest rate.  2. ' ' ?
(D) investment is totally unresponsive to changes in the
real GDP.
96. In 2008 the exchange rate of the Euro appreciated (C) 
against the US dollar . A possible cause for the appreciation (D)
of the Euro was 3. ?
(A) increase investment in USA by the European Union (EU)
(B) increasingly more US tourists were visiting EU countries
(C) increasingly more US exports to EU countries (C)
(D) the European Central Bank raised the value of the (D)
Euro 4. ?
97. The banking companies in Bangladesh are regulated , , 
under the provision of __ , ,
(A) Bank Companies Act 1991 . Companies Act, `994 .  (C) , ,
(B) Bank Company Amendment Act, 2003. (D) , ,
(C) Bank Company Amendment Act , 2003, 5. ' ' ?
(D) Bangladesh Bank Order, 1972
98. Which institution is the regulatory authority of capital 
market intermediaries in Bangladesh ? (C)
(A) Bangladesh Bank (D)
(B) Securities & Exchange Commission

85
Collected By: Md. Raihan Ahmed
Job Information For All Students (Bcs & Bank)

(C) he
6. Choose the correct spelling. (D) what
(A) FAVOURIET 14. AMENABLE (Select the word closely opposite in
(B) FAVOURET meaning)
(C) FAVORETE (A) agreeable
(D) FAVOURITE (B) bound
7. Choose the correct spelling. (C) unwilling
(A) SUGGEST (D) submissive
(B) SUGAST 15. DOMAIN (Select the world that conveys nearly the same
(C) SUJEST meaning)
(D) SUGGAST (A) marketing
8. Select, from the options, the pair having a similar (B) area
relationship to the first pair : Army : Logistics : : (C) main
(A) Business : Strategy (D) fielding
(B) War : Logic
(C) Soldier : Student MATHAMETICS
(D) Team : Individual 16. The cost of carpeting a room 18m long with a carpet 75
9. Select, from the options, the pair having a similar m wide at Tk. 4.50 per metre is Tk. 810. The breadth of the
relationship to the first pair : Editor : Newspaper : : room is-
(A) Lecturer : University (A) 750 m
(B) Architect : Design (B) 8 m
(C) Nurse : Hospital (C) 7.5 m
(D) Teacher : School (D) 8.5 m
10. He felt bad ---- no reason at all. Solution:
(A) of Length of the carpet = (810/4.50) m = 180 m
(B) on Area of the room = (180 × 75) m2 = 13500 m2
(C) for :. Area of the room = Area of the carpet = 135 m2
(D) in :. Breadth of the room = (13500/18) m2 = 750 m.
11. The train of Rajshahi departed _____ the platform. 17. A shopkeeper sold article offering a discount of 5% and
(A) from earned a profit of 23.5%. What would have been the
(B) with percentage of profit earned if no discount was offered?
(C) at (A) 30
(D) on (B) 28.5
12. Fill in the blank : Leap years, ____ have 366 days, (C) 24.5
contain an extra day in February. (D) None of these
(A) that Solution:
(B) when Let cost price be Rs. 100
(C) where The selling price be Rs. x
(D) which Then,
13. Fill in the blank : Nazrul, _____ is our national poet, was 95x/100 =123.50
also a soldier. ⇒x = Tk. (12350/95)
(A) who ⇒x = Tk.130
(B) whom Now, selling price = Tk. 130

86
Collected By: Md. Raihan Ahmed
Job Information For All Students (Bcs & Bank)

Cost price = Rs. 100 He obtained marks = 125


∴ Profit % = 30% He failed by marks = 40
18. A group of 1200 persons consisting of captains and Passing marks = 125+40 = 165
soldiers is travelling in a train. For every 15 soldiers there is Let the maximum marks be x
one captain. The number of captains in the group is ---- We are given that a student has to obtain 33% of total
(A) 70 marks to pass
(B) 75 So, 33% of x = 165
(C) 85 33x/100 = 165
(D) 80 x =500
Solution: Hence the maximum marks is 500
Total persons consisting of captains and soldiers = 1200 21. In an election a candidate who gets 84% of the votes, is
persons elected by a majority of 476 votes. What is the total
15 soldiers = 1 captain number of votes polled?
so, 1 group = 15+1 = 16 person (A) 848
that means (B) 749
16 persons = 1 group (C) 700
1200 persons = 1200 / 16 = 75 groups (D) 672
so there are totally 75 captains in the group Solution:
19. A courtyard 25 m long and 16 m broad is to be paved GIVEN: A candidate who gets 84% of the votes is elected by
with bricks of dimensions 20 cm by 10 cm. The total a majority of 476 votes.
number of bricks required is Let the total number of votes polled be 'x'
(A) 25000 Number of votes in favour of the candidate is = 84% of x
(B) 20000 =84x/100
(C) 18000 Number of votes got by the losing candidate = x−84x/100
(D) None of these = (100x -84x)/100 = 16x/100
Solution: Number of extra votes in favor of winning candidate =
Here in this question first we have to find the area of the 84x/100−16x/100
the courtyard and then divide it by the area of one brick. 476 =68x/100
Now first convert meters into centimeters to properly x=(476×100 )/ 68
calculate the ans. x=7×100
25m * 100 = 2500cm x=700
16m * 100 = 1600cm Hence,the total number of votes polled are 700 .
Now area of courtyard = 1600*2500=4000000 22. The total number of digits used in numbering, the
Area of a brick = 20 * 10=200 pages of a book having 366 pages is
So no. of bricks required would be 4000000/200=20000 (A) 1305
20. A student has to obtain 33% of the total marks to pass. (B) 1098
He got 125 marks and failed by 40 marks. The maximum (C) 990
marks are (D) 782
(A) 1000 Solution:
(B) 800 Total number of digits
(C) 500 = (No. of digits in 1- digit page nos. + No. of digits in 2-
(D) 300 digit page nos. + No. of digits in 3- digit page nos.)
Solution: = (1 x 9 + 2 x 90 + 3 x 267) = (9 + 180 + 801) = 990.

87
Collected By: Md. Raihan Ahmed
Job Information For All Students (Bcs & Bank)

23. On selling a chair at 7% loss and a table at 17% gain, a (C) 10423
man gains Tk 296. If he sells the chair at 7% gain and the (D) 10241
table at 12% gain, then he gains Tk. 400. The actual price Solution:
of the table is --- Clearly, alternatively we add and subtract 10% of a term to
(A) Tk. 2400 obtain the next term of the series.
(B) Tk. 2200 Thus, 10000 + (10% of 10000)= 11000;
(C) Tk. 1800 11000 - (10% of 11000) = 9900;
(D) Tk. 1600 9900 + (10% of 9900) = 10890;
Solution: 10890 - (10% of 10890) = 9801.
Let C.P. of the chair be Rs. x and that f the table be Rs. y 9801 + (10% of 9801) = 9801+980 = 10781.
Then ,17% of y - 7% f x = 296 So, missing term = 10890- (10% of 10890)= 9801.
⇒ 17y - 7x = 29600........(i) 26. Find the missing term in each of the following series :
and 12% of y + 7% of x = 400 2, 5, 9, 19, 37, ?
⇒ 12y + 7x = 40000...........(ii) (A) 78
Solving (i) and (ii) , we et : y = 2400 and x = 1600 (B) 76
24. The simple interest on a sum of money at 8% per (C) 75
annum for 6 years is half the sum. The sum is (D) 73
(A) Tk. 4800 Solution:
(B) Tk. 6000 2 5 9 19 37 ?,
(C) Tk. 8000 2,
(D) None of these 2*2+1=5,
Solution: 5*2-1=9,
GIVEN: 9*2+1=19,
Rate (R) of Simple Interest = 8% 19*2-1=37,
Simple Interest (S.I) = Half of the Sum of money 37*2+1=75.
To find: 27. Find the missing term in each of the following series :
Let the sum be P, where P (or the Sum) denotes the 1, 6, 15, ?, 45, 66, 91
Principle amount. (A) 28
Given that R = 8% (B) 27
N= 6 years (C) 26
And, given in the sum, (D) 25
S.I = P/2 Solution:
We know, Pattern in the series is, +5, +9, .., .., +21, +25
S . I .= (P × N × R)/100 So 4 is getting increased at every term addition.
P/2 = (P × N × R)/100 Missing term in the series will be 15+(9+4)
But here, we see that P being in the numerator on both = 15+13 = 28
side, gets cancelled, leaving no variable, but only whole 28. A person borrows Tk. 5000 for 2 years at 4% p.a simple
𝟏
numbers. interest.He immediately lends it to another person at 6 %
𝟒
Thus, Data is Incomplete in this sum. p.a for 2 years.Find his gain in the transaction per year .
25. Find the missing term in each of the following series : (A) Tk. 167.50
10000, 11000, 9900, 10890, 9801, ? (B) Tk. 150
(A) 10929 (C) Tk. 125
(B) 10781 (D) Tk. 112.50

88
Collected By: Md. Raihan Ahmed
Job Information For All Students (Bcs & Bank)

Solution: Commission?
Gain in 2 years (A) Dr. Saleh Uddin Ahmed
= [ (5000 x 25/4 x 2/100) - (5000 x 4 x 2)/100 ] (B) Dr. Mohammed Faradhuddin
= Tk. (625 - 400) (C) Dr. Shirin sharmin Chowdhury
= Tk. 225. (D) Dr. Akbar Ali Khan
Gain in 1 year = Tk. 225/2 = Tk. 112.50 36. What is the name of the new police academy for which
construction has been initiated?
COMPUTER (A) Rajshahi Police Academy
29. Which of the following is an important attribute of RAM (B) Bangabandhu Sheikh Mujib National Police Academy
memory? (C) National Police Academy
(A) Random access (D) Jessore Police Academy
(B) Volatile 37. What is the tenure of the Governor of Bangladesh bank?
(C) Non-Volatile (A) 2 years
(D) Sequential access (B) 4 years
30. Data access time depends on (C) 3 years
(A) operating frequency (D) 5 years
(B) Seek time 38. Which Bank was the first to launch agent banking in the
(C) rotational delay country?
(D) All of the above (A) Mercantile Bank
31. The primary job of the operating systems (B) Eastern Bank
(A) Manage Resources (C) Bank Asia
(B) Manage Users (D) Brac Bank
(C) Manage Programs 39. Banking sector comes under which of the following
(D) Manage Commands sectors?
32. The file system 'NTFS' stands for (A) Industrial sector
(A) New Type file System (B) Agriculture sector
(B) Never Terminated file System (C) Service sector
(C) Non Terminated file systems (D) Manufacturing sector
(D) New Technology file system 40. The availability of cash and other cash like marketable
33. Unix Operating systems is a instruments that are useful in purchases and investments
(A) Multi Tasking Operating Systems are commonly known as :
(B) Time Sharing Operating System (A) Liquidity
(C) Multi User Operating system (B) Cash crunch
(D) All the above (C) Credit
(D) Markerability
GENERAL KNOWLEDGE & ECONOMICS 41. When it comes to providing locket facility to the
34. Which animal's genome was first sequenced by customer of any bank, the relationship between the
Bangladesh scientists? customer and bankers it that of :
(A) Sheep (A) Lessee and lessor
(B) Buffalo (B) Debtor and creditor
(C) Cow (C) Bailor and bailee
(D) Hen (D) Agent and principal
35. Who was the chairman of the Eighth National Pay

89
Collected By: Md. Raihan Ahmed
Job Information For All Students (Bcs & Bank)

42. Providing banking services to a customer without his ANALYTICAL ABILITY


entrance inside the bank's branch is called as : 47.
(A) Relationship banking
(B) Virtual banking
(C) Universal banking
(D) Mobile banking ?

43. Mortgage relates to which type of loan among the


following? (A) (B) 
(A) Gold loan
(B) Educational loan (C) (D)
(C) Car loan
(D) Housing loan
48.

44. Ali is younger than Sonia and Rupali is older than Tom.
Who among them is the oldest?
1. Rupali is older than Ali.
2. Sonia is older than Rupali. ?

3. Tom is the youngest among all.


(A) Only 2
(B) Only 3 (A)  (B)

(C) Only 1 and 2


(D) All 1, 2, and 3 (C) (D)

45. Five persons ----A, B, C, D and E are sitting in a row 49.

who is sitting in the middle?


1. B is between E and C.
2. B is to the right of E. ?
3. D is between A and E
(A) Only 1 and 2
(B) Only 2 and 3 (A) (B)
(C) Only 1 and 3
(D) All 1, 2, and 3
(C) (D) 
46. How many sons does X have?
1. Q and U are brothers of T. 50.
2. R is sister of P and U.
3. R and T are daughters of X.
(A) Only 1 and 2
?
(B) Only 2 and 3
(C) All 1, 2, and 3
(D) 1, 2, and 3 together are not sufficient
(A) (B) 

90
Collected By: Md. Raihan Ahmed
Job Information For All Students (Bcs & Bank)

(B) put out with


(C) (D) (C) put up
(D) put up with
8. Choose the correctly spelt word :
(A) Solipsism
(B) Solipasism
BANGLA (C) Salipsism
1. ---- (D) Solipesism
? 9. Choose the correctly spelt word :
(A) Consumarism
(B) Consumerism
(C) (C) Consumariesm
(D)  (D) Consummerism
2. ' '-- ? 10. Why do we always have to subunit ____ his authority?
(A) under
 (B) to
(C) (C) for
(D) (D) with
3. ' ' -- ? 11. The completion of the railway line has been _____ owing
to the workers strike.
(A) on
(C)  (B) up
(D) (C) off
4. ? (D) over
 12. Select, from the options, the pair having a similar
relationship to the first pair : President : Pope : :
(C) (A) Proclamation : Bull
(D) (B) Elected : Chosen
5. ' '-- ? (C) Ballot : Smoke
(D) Temporal : Secular
13. Select, from the options, the pair having a similar
(C)  relationship to the first pair : Ernest : Immoral : :
(D) (A) Land : Evil
(B) Dissolute : Lascivious
(C) Shore : Reef
6. Aunik was the sort of person who always looked on the (D) Restrained : Wanton
_______ side of life. 14. SUNDRY (Select the word that conveys nearly the same
(A) clear meaning)
(B) illuminated (A) complex
(C) bright (B) various
(D) broad (C) express
7. I refused to ______ his rudeness any longer. (D) unmatched
(A) put on 15. FUSION (Select the word closely opposite in meaning)

91
Collected By: Md. Raihan Ahmed
Job Information For All Students (Bcs & Bank)

(A) amalgam (C) 65%


(B) separation (D) None of these
(C) synthesis Solution:
(D) blend Let the number be 'x'
Given that 35% of a number is 175
MATHAMETICS so x × 35% = 175
16. A person's present age is two-fifth of the age of his x × 35/100 = 175
mother. After 8 years, he will be one-half of the age of his x = 175 × 100/35
mother. How old is the mother at present? x = 500 is the number..
(A) 48 years then 175 percentage of that number is
(B) 40 years 500×175% = 500×175/100 = 875
(C) 36 years 19. A person takes a loan of Tk. 200 at 5% simple interest.
(D) 32 years He returns Tk.100 at the end of 1 year. In order to clear his
Solution: dues at the end of 2 years, he would pay :
Let the mother's present age be x years. (A) Tk. 115
Then, the person's present age = 2/5 x years. (B) Tk. 115.50
(2x/5 + 8) = 1/2 (x + 8) (C) Tk. 105
2(2x + 40) = 5(x + 8) (D) Tk. 110
x = 40. Solution:
17. A cricket team won 3 matches more than they lost. If a The SI for 200 at the end of 1st year = (200*1*5)/100 =
win gives them 2 points and loss(-1) point, how many 10.
matches, in all, have they played if their score is 23? Hence total amount at the end of 1st year = 200+10 =
(A) 37 210.
(B) 40 But he returns back RS 100 at the end of 1st year.
(C) 17 Hence principl amount for 2nd year = (210 - 100) = 110.
(D) 20 Then SI at the end of 2nd year = (110*1*5)/100 = 5.5.
Solution: Hence total amount he needs to return at the end of 2nd
Let the no.of matches they lost be x. year = 110+5.5 = Tk 115.5.
Therefore,the no.of matches they won is x+3. 20. The difference between the local value and the face
Total score=23. value of 7 in the numeral 32675149 is ---
The equation becomes, (A) 75142
2(x+3)-1(x)=23 (B) 69993
2x+6-x=23 (C) 64851
x+6=23 (D) 5149
Therefore,x=17 Solution:
Hence,the no.of matches won is x+3=17+3=20. Here Local value of 7 = 70000
And the no.of matches lost is x=17. And face value of 7 = 7
Total no.of matches played=20+17=37. Difference = 70000 - 7 = 69993
18. If 35% of a number is 175, then what percent of 175 is 21. The salaries of A, B, C are in the ratio 2: 3: 5. If the
the number? increments of 15%, 10% and 20% are allowed respectively
(A) 35% in their salaries, then what will be the new ratio of their
(B) 280% salaries?
(A) 23 : 33 : 60

92
Collected By: Md. Raihan Ahmed
Job Information For All Students (Bcs & Bank)

(B) 10 : 11 : 20 Product of three numbers is 1944.


(C) 3 : 3 : 10 Their ratio is 3:4:6.
(D) None of these Let the common ratio be x.
Solution: So, first number = 3x.
The salaries A, B, C are in the ratio 2 : 3 : 5. Second number = 4x.
Let A = 2x , B = 3x , C = 5x Third number = 6x.
A's new salary = (100+15%) of 2x = 115/100×2x = 23x/10 Their product is 1944.
B's new salary= (100+10%) of 3x = 110/100 ×3x = 33x/10 SO.....
C's new salary = (100+20%) of 5x = 120/100×5x = 60x/100 3x × 4x × 6x = 1944.
Ratio of their new salaries = (23x/10 : 33x/10 : 60x/10) = 72x³ = 1944
23 : 33 : 60 x³ = 27
22. 12 men working 8 hours per day complete a piece of x = 3.
work in 10 days. To complete the same work in 8 days, Hence the greatest among them is 6x = 18.
working 15 hours a day, the number of men required, is 24. A, B and C started a business by investing Tk. 1,20,000,
(A) 4 Tk. 1,35,000 and Tk. 1,50,000 respectively. Find the share
(B) 8 of A, out of an annual profit of Tk. 56,700.
(C) 5 (A) 16,800
(D) 6 (B) 16,700
Solution: (C) 16,600
Method-I (D) None of these
You have to just follow a simple concept- Solution:
M1 × D1 × H1= M2 × D2 × H2 Investment ratio = 120000 : 135000 : 150000 = 8 : 9 : 10
Where M is no. of Men, D is no. of days and H is for Sum of ratio = 8 + 9 + 10 =27
Hours. Share of A = 8/27 × 56700 = 16800
Now put all the given values in the above formula 25. Village T is in which direction with respect to village R?
12 × 1 0× 8 = M2 × 8 × 15 1. T is to the North of W which is to the West of S.
M2 = 8 Men 2. T is to the North-West of S.
If you are not getting this concept then follow the basic 3. W is to the North-West of R.
method. (A) Any two of the three
II. Basic Method (B) Only 1 and 2
First you have to calculate units of work (C) Only 2 and 3
W= Men × Days × Hours (D) Only 1 and 3
W= 12 × 10 × 8= 960 units 26. Four subjects ---physics, Chemistry, Mathematics and
Now to complete these 960 units in 8 days, while working Biology ---were taught in four consecutive periods of one
15 hours a day, the no. of men required will be= hour each starting from 8.00 a.m. At what time was the
960÷(15×8)= 8 Men chemistry period scheduled?
23. Three numbers are in the ratio of 3 : 4: 6 and their 1. Mathematics period ended at 10.00 a.m, which was
product is 1944. The largest of these numbers is- preceded by Biology.
(A) 18 2. Physics was scheduled in the last period.
(B) 12 3. Mathematics period was immediately followed by
(C) 6 Chemistry.
(D) none of these (A) Only 1
Solution: (B) Only 2

93
Collected By: Md. Raihan Ahmed
Job Information For All Students (Bcs & Bank)

(C) Only 2 and 3 (A) SMTP


(D) Only 1 and either 2 or 3 (B) FTP
27. Who is the tallest among six boys P, T, N, D, Q and R? (C) TELNET
1. P is taller than D and N but not as tall as T. (D) None of these
2. R is taller than Q but not as tall as T. 34. Nibble describes a bits combination of
3. Q is not taller than T and R. (A) 4 bits
(A) Only 1 and 2 (B) 5 bits
(B) Only 2 and 3 (C) 6 bits
(C) All 1, 2 and 3 (D) 7 bits
(D) Only 1 and 3 35. What is the base of hexadecimal number system?
28. 1,6,15,?,45,66,91 (A) 2
(A) 28 (B) 8
(B) 27 (C) 10
(C) 25 (D) 16
(D) 26 36. Spot the odd one out amongst the following
29. T,R,P,N,L,?? (A) Basic
(A) J,G (B) UNIX
(B) K,H (C) Windows
(C) J,H (D) DOS
(D) K,I
30. Find the wrong number in the series : GENERAL KNOWLEDGE & ECONOMICS
3,8,15,24,34,48,63 37. Who is the World Holder for fastest 50 in ODI cricket?
(A) 34 (A) MDKJ Perera
(B) 48 (B) Jayasuriya
(C) 15 (C) MJ Guptill
(D) 24 (D) AB de Villiers
31. A shopkeeper sells 25 articles at Tk. 45 per article after 38. Currently how many member states are there in the
giving 10% discount and earns 50% profit.If the discount is United Nation?
not given, the profit gained is (A) 192
2
(A) 66 % (B) 193
3
(B) 66% (C) 190
(C) 60% (D) 191
2
(D) 60 % 39. Which constitutional amendment establishes Islam as
3
the state religion of Bangladesh?
COMPUTER (A) 8th
32. ________ is a system of counting using only two numbers (B) 7th
0 and 1. (C) 5th
(A) Banary (D) 4th
(B) HTML 40. Who was the first Governor General of India?
(C) Decimal (A) Mahatma Gandhi
(D) Mouse (B) Lord Mountbatten
33. Which protocol provides e-mail facility among different (C) Vallabhbhia Patel
hosts? (D) Jawaharlal Nehru

94
Collected By: Md. Raihan Ahmed
Job Information For All Students (Bcs & Bank)

41. The 'Sangsaptak' sculpture is located at?


(A) Dhaka University (C) (D)
(B) Chittagong University
(C) Jahangirnagar University 48. What shape or pattern belongs to the group

(D) Agriculture University


42. Financial inclusion means provision of
(A) Ration at affordable cost to persons not yet given the
same
(A) (B)
(B) House at affordable cost to persons not yet given the
same
(C) Food at affordable cost to persons not yet given the (C)  (D)
same
(D) Financial services namely payments, remittances, 49. Count the number of squares in the given figure ?
savings, loans and insurance at affordable cost to persons
not provided by the banks. 
43. What happens when an economy is in recession?
(A) GDP growth slows down
(B) Unemployment rate rises
(C) Bankrutcies rise
(D) All 1, 2, and 3
44. Which of the following is known as plastic money? (A) 17 (B) 16
(A) credit cards (C) 13 (D) 12
(B) bearer cheques
(C) demand drafts 50. Find the Matching pair

(D) gift cheques


45. Which of the following organizations issue the rules of
global trade?
(A) IMF
(B) Foreign trade (A) (B)

(C) World Trade Organization


(D) G-20
46. Which sister organization of the world Bank provides
long term loans at zero interest to the poorest developing (C) (D) 

countries?
(A) International Developing Association
(B) IMF
(C) Asian Development Bank
(D) International Finance Corporation
BANGLA
1. ?
ANALYTICAL ABILITY
47. Find the odd one

(C) 
(A) (B) 
(D)

95
Collected By: Md. Raihan Ahmed
Job Information For All Students (Bcs & Bank)

2. ' ' ?

(C)
(C)  (D) 
(D) 11. ---
3. ?
 
(C)
(C) (D)
(D) 12.
4. ? ?
-
-
(C)  (C) -
(D) (D) - 
5. 'Birds of a feather flock together'-- 13. 'parasite' ---
?

 (C) 
(C) , (D)
(D) 14. ?
6. ' ' ' ' ? , 
 ,
(C) ,
(C) (D) ,
(D) 15. ' ' ---
7. ?

(C) 
(C) (D)
(D)  16. ' '-- ----
8. ?

(C)
(C)  (D) 
(D) 17. ' ' ---
9. 'depreciation'-- -- 

(C)
(C)  (D)
(D) 18. ' ' ------
10. ?

96
Collected By: Md. Raihan Ahmed
Job Information For All Students (Bcs & Bank)

(C)  (B) acommodate


(D) (C) accommodate
19. ? (D) accommodat
(A) green room 27. Choose the correctly spelt word :
(B) pass-word (A) vegeterian
(C) municipality (B) vegetarian
(D) parcel (C) vegetarean
20. ' ' ? (D) vegitarean
28. Choose the correctly spelt word :
(A) immence
(C) (B) imence
(D)  (C) immense
21. ? (D) immenze
- 29. Choose the correctly spelt word :
- (A) privilige
(C) -  (B) privilege
(D) - (C) previlage
22. ' - ' -- (D) privelage
- ? 30. Choose the correctly spelt word :
(A) supersede
(B) superseed
(C)  (C) supercede
(D) (D) supperssed
23. 31. Choose the word nearest in meaning to it. APEX
, --- (A) inborn
 (B) category
(C) top
(C) (D) banner
(D) 32. Choose the word nearest in meaning to it. FULSOME
24. ' ' --- (A) smooth
(B) generous
(C) liberal
(C) (D) excessive
(D)  33. Choose the word nearest in meaning to it. DETERRENT
25. ? (A) restriction
(B) anchor
(C) harness
(C) (D) chain
(D)  34. Choose the word nearest in meaning to it. WAIVE
(A) restrict
(B) relax
26. Choose the correctly spelt word : (C) permit
(A) accomodate (D) admit

97
Collected By: Md. Raihan Ahmed
Job Information For All Students (Bcs & Bank)

35. Choose the word nearest in meaning to it. (A) Rain : River
DIPLOMATICALLY (B) Wind : Sleet
(A) slyly (C) Snow : Blizzar
(B) slowly (D) Ice : Floe
(C) tactfully 43. BELT : TROUSERS : :
(D) clearly (A) Braces : Garters
(B) Trunk : Tree
Question(36-40): Select the word that is most closely (C) Pillar : Society
opposite in meaning to the capitalized word. (D) Cables : Bridge
36. PROFOUND 44. LUNGS : BLOOD : :
(A) superficial (A) Heart : Circulation
(B) obscure (B) Arteries : Veins
(C) intense (C) Carburetor : Car
(D) hidden (D) Carburetor : Gasoline
37. SCARCE 45. HABITS : INSTINCTS : :
(A) thick (A) Work : Play
(B) enlarge (B) Training : Heredity
(C) plentiful (C) Acquired : Cultivated
(D) mediocrity (D) Natural : Unusual
38. MOISTURE 46. Fill in the blanks : We didn't _____ the programme to be
(A) dryness such a huge success.
(B) delicate (A) except
(C) dampness (B) expect
(D) defect (C) access
39. EXTRAVAGANT (D) accept
(A) expensive 47. Fill in the blanks : We should ______ opportunities as
(B) thrifty they arise.
(C) unlimited (A) cease
(D) proud (B) size
40. IMPRECISION (C) sneeze
(A) accuracy (D) seize
(B) wrong 48. Fill in the blanks : The students aren't prepared _____
(C) mistake the examination.
(D) erroneousness (A) to listen
(B) to work
Question(41-45): Select from the answer options, the pair (C) to take
of words having a similar relationship to the given pair. (D) to give
41. CONVICT : IMPRISONMENT : : 49. Fill in the blanks : I suffer from no ______ about my
(A) Student : School capabilities.
(B) Exile : Banishment (A) doubts
(C) Delinquent : Orphanage (B) hallucidations
(D) Prisoner : Court (C) imaginations
42. WATER : FLOOD : : (D) illusions

98
Collected By: Md. Raihan Ahmed
Job Information For All Students (Bcs & Bank)

50. Fill in the blanks : Man is essentially a ______ animal and Solution:
tends to associate with others. HCF of (91-43), (183- 91) and (183- 43) is
(A) sentimental HCF of 48, 92, 140 = 4
(B) gregariopus 55. 0.002 × 0.5 = ?
(C) selfish (A) 0.0001
(D) perverse (B) 0.001
(C) 0.01
MATHAMETICS (Solved By Raihana Pervin Rontu) (D) 0.1
51. The difference between the place value and the face Solution:
value of 6 in the numeral 856973 is -- 0.002 × 0.5 = 0.001
(A) 973 56. (8/88) × 8888088 = ?
(B) 6973 (A) 808008
(C) 5994 (B) 808080
(D) None of these (C) 808088
Solution: (D) 8008008
Place value of 6 = 6000 and face value of 6 is 6 Solution:
Difference = 6000 6 = 5994 (8/88) × 8888088 = 808008
52. 4500 × ? = 3375 57. A man has some hens and cows. If the number of
(A) 2/5 heads be 48 and the number of feet equals 140, then the
(B) 3/4 number of hens will be ----
(C) 1/4 (A) 22
(D) 3/5 (B) 23
Solution: (C) 24
4500/3375 = 4/3 (D) 26
Or, 4550 * 3/4 = 3375 Solution:
53. Which one of the following numbers is exactly divisible L x
by 11? 4(48-x) + 2x = 140
(A) 235641 Or, 192 4x + 2x = 140
(B) 245642 Or, 2x = 52
(C) 315624 Or, x = 26
(D) 415624 58. The mean of 50 observations was 36. It was found later
Solution: that an observation 48 was worngly taken as 23. The
Option test correct new mean is :
2+3+5+6+4+1 = 21, which is not divisible by 11 (A) 35.2
2+4+5+6+4+2 = 23, which is not divisible by 11 (B) 36.1
3+1+5+6+2+4 = 21, which is not divisible by 11 (C) 36.5
4+1+5+6+2+4 = 22, which is divisible by 11 (D) 39.1
54. Find the greatest number that will divide 43, 91 and Solution:
183 so as leave the same remainder in each case. 48 will be replaced by 23
(A) 4 So, (48 23)/50 = 25/50 = 1/2 = .5
(B) 7 The correct new mean = 36 + (.5) = 36.5
(C) 9 59. If doubling a number and adding 20 to the result gives
(D) 13 the same answer as multiplying the number by 8 and taking

99
Collected By: Md. Raihan Ahmed
Job Information For All Students (Bcs & Bank)

away 4 from the product, the number is : Solution:


(A) 2 1/3 * x/100 = 1/2
(B) 3 Or, x = 3*100/2 = 150%
(C) 4 64. After deducting a commission of 5%, a T.V set costs Tk.
(D) 6 9595. Its marked price is :
Solution: (A) Tk. 10,000
2x + 20 = 8x 4 (B) Tk. 10,075
Or, 3x = 12 (C) Tk. 10,100
Or, x =4 (D) Tk. 10,500
60. The sum of three consecutive multiples of 3 is 72. What Solution:
is the largest number? 95 tk purchase price then marked price 100 tk
(A) 21 9595 tk purchase price then marked price 9595*100/95
(B) 24 =10100 tk
(C) 27 65. A man buys an article for 10% less than its value and
(D) 36 sells it for 10% more than its value. His gain of loss percent
Solution: is :
By checking options (A) no profit, no loss
21+18+15 = 54 (B) 20% profit
24+21+18 = 63 (C) less than 20% profit
27+24+21 = 72 (D) more than 20% profit
61. Rahim is 40 years old and Karim is 60 years old. How Solution:
many years ago was the ratio of their ages 3 : 5 ?
(A) 5 years Cost = 90% of x = 90/100 * x = 9x/10
(B) 10 years Selling price = 110% 0f x = 110/100 * x = 11x/10
(C) 20 years Gain = 11x/10 9x/10 = 2x/10
(D) 37 years In percentage, gain
Solution: = (2x/10)/(9x/10) *100 = 200/9 = 22.22
3:5 66. If x/5 = y/8, then (x+5) : (y+8) is equal to :
(40-x)/(60-x) = 3/5 (A) 3 : 5
Or, 2x = 20 (B) 13 : 8
Or, x =10 (C) 8 : 5
62. 49 × 49 × 49 × 49 = 7^? (D) 5 : 8
(A) 4 Solution:
(B) 7 x/5 = y/8
(C) 8 Or, (x+5)/5 = (y+8)/8
(D) 16 Or, (x+5)/(y+8) = 5/8
Solution: 67. A and B started a business jointly. A's investment was
2 2 2 2= 2+2+2+2 = 8
7 * 7 *7 *7 7 7 thrice the investment of B and the period of his investment

63. 1/2 is what percent of 1/3? was two times the period of investment of B. If B received

(A) 50% Tk. 4000 as profit, then their total profit is :

(B) 150% (A) Tk. 16,000

(C) 250% (B) Tk. 20,000

(D) None of these

100
Collected By: Md. Raihan Ahmed
Job Information For All Students (Bcs & Bank)

(C) Tk. 24,000 Shortcut:


(D) Tk. 28,000 8 18
Solution
14

Then A will get 3*2 = 6 times more profit than B 4 6


A will get 4000* 6 = 24000 as profit 2 3
Total profit = 24000 + 4000 = 28000 tk Quantity sold at 18% profit = ( 3/5)*1000 = 600
70. The simple interest on a certain sum of money at the
68. A and B can do a piece of work in 5 days; B and C can rate of 5% p.a. for 8 years is Tk. 840. At what rate of
do it in 7 days; A and C can do it in 4 days. Who among interest the same amount of interest can be received on the
these will take the least time if put to do it alone? same sum after 5 years?
(A) A (A) 6%
(B) B (B) 8%
(C) C (C) 9%
(D) None of these (D) 10%
Solution: Solution:
(A+B) + (A+C) can do the work 1/5 + 1/4 in 1 day As interest amount is same then
Or, 2A + (B+C) can do the work 9/20 in 1 day (840 × 5 × 8)/100 = (840 × r × 5)/100
Or, A can do the work (9/20 1/7)/2 = 43/280 in 1 day Or, r = 8%
So, B can do the work 1/5 43/280 = 13/280 in 1 day 71. In a dairy farm, 40 cows eat 40 bags of husk in 40
And C can do the work 1/4 43/280 = 27/280 in 1 day days. In how many days one cow will eat one bag
So A can do it faster. of husk?
(A) 1
69. A merchant has 1000 kg of sugar, part of which he sells (B) 1/40
at 8% profit and the rest at 18% profit. He gains 14% on (C) 40
the whole. The quantity sold at 18% profit is : (D) 80
(A) 400 kg Solution:
(B) 560 kg 40 cows eat 40 bags husk in 40 days
(C) 600 kg 1 cow eat 40 bags husk in 40 × 40 days
(D) 640 kg 1 cow eat 1 bag husk in (40 × 40)/40 = 40 days
Solution: 72. Below, out of the four figures marked A, B, C and D,
Let the sugar sold at 8% gain = x three are similar in a certain manner , However, one figure
Therefore, Sugar sold at 18% gain = (1000 - x) is not like the other three. Choose the figure which is
Let CP of sugar = Rs.y per kg different from the rest.
Total CP = Rs.100y
Therefore,
(108/100 × xy) + [118/100(1000 - x)y ] = 114/100 × 1000y (1) (2) (3) (4)
108xy + 118000y - 118xy = 114000y (A) 1
10x = 4000 (B) 2
x = 400 (C) 3
Quantity sold at 18% profit = 1000 - 400 = 600 kg (D) 4

101
Collected By: Md. Raihan Ahmed
Job Information For All Students (Bcs & Bank)

73. A shopkeeper purchased 70 kg of potatoes for Tk. 420 (D) Padma


and sold the whole lot at the rate of Tk. 6.50 per kg. .what 79. 'Tajmahal Mosque' is located in ----
will be his gains percent? (A) Dhaka
1
(A) 4 % (B) Delhi
6
1
(B) 6 % (C) Lahore
4
1
(C) 8 % (D) Agra
3
(D) 20% 80. What is the position of Bangladesh in ODI Cricket

Solution: ranking according to ICC?

He purchased potatoes = 420/70 = 6 tk/ kg (A) 7th

So, gain 6.50 6 = 0.50 tk/kg (B) 8th

From 6 tk gains 0.50 tk (C) 9th

From 100 tk gains (0.5 × 100)/6 = 25/3% = 8 %


1
(D) 10th
3
74. A car moves at the speed of 80 km/hr. what is the 81. Which one is the largest Muslim country in the world by

speed of the car in metres per second ? population?

(A) 8 m /sec (A) Indonesia


1
(B) 20 m/sec (B) Pakistan
9
2
(C) 22 m/sec (C) Bangladesh
9
(D) None of these (D) Turkey

Solution: 82. Which country banned smoking first?

80 km/hr = (80 × 1000)/3600 = 200/9 m/sec (A) Sri lanka

75. √.0025 × √2.25 × √.0001 =? (B) Bhutan

(A) 0.000075 (C) Sweden

(B) 0.00075 (D) Canada

(C) 0.0075 83. What is the name of Boarder Security Force of

(D) 0.075 Myanmar?

Solution: (A) NASAKA

√.0025 × √2.25 × √.0001= 0.05 × 1.5 × 0.01 = 0.00075 (B) BGP


(C) MSF

GENERAL KNOWLEDGE (D) MBP

76. How many tribes live in the Chittagong Hill Tracts? 84. Who is the secretary general of OIC?

(A) 11 (A) Iusuf al osaimin

(B) 12 (B) Ekmeleddin Ihsanoglu

(C) 13 (C) Prince Salman

(D) 14 (D) Iyad Bin Amin Madani

77. Awami Muslim League was formed in ----- 85. Which country is known as the sugar bowl of the world?

(A) 1948 (A) Brazil

(B) 1949 (B) Sri Lanka

(C) 1950 (C) Cuba

(D) 1952 (D) Germany

78. Madaripur town is located on the river ----- 86. The oldest tennis tournament in the world is -----

(A) Arial Khan (A) Wimbledon

(B) Modhumati (B) French Open

(C) Rupsha (C) Australian Open

102
Collected By: Md. Raihan Ahmed
Job Information For All Students (Bcs & Bank)

(D) US Open (C) Sheikh Azizur Rahman


87. The first general election was held in independent (D) Sheikh Lutar Rahman
Bangladesh on ---- 95. The birth place of professor Amartya Sen was in ----
(A) 7 March 1973 (A) Kolkata
(B) 26 January 1973 (B) Comilla
(C) 1 June 1973 (C) Chittagong
(D) 16 July 1973 (D) Manikgonj
88. What is the name of the director of the film 'Muktir 96. What was the previous name of Comilla?
Gaan'? (A) Tripura
(A) Zahir Raihan (B) Nasirabad
(B) Tareq Masud (C) Sudharam
(C) Alamgir Kabir (D) Subarnagram
(D) Khan Ataur Rahman 97. Which day is being observed as the 'Mujibnagar Day' in
89. What is the length of Bangabandhu Bridge? Bangladesh?
(A) 4.5 km (A) 17 April
(B) 4.2 km (B) 4 April
(C) 4.8 km (C) 4 November
(D) 5.1 km (D) 3 December
90. What is the name of the first woman speaker of Jatiya 98. The first European country to recognize Bangladesh----
Sangshad of Bangladesh? (A) East Germany
(A) Dr. Fazana Islam (B) West Germany
(B) Dr. Shirin Sharmin Chowdhury (C) France
(C) Nazmun Ara Sultana (D) UK
(D) Sahara Khatun 99. Who introduced the Bengali Calendar?
91. Which one was the first foreign bank in Bangladesh? (A) Emperor Asoka
(A) Standard Chartered (B) Emperor Akbar
(B) City N.A. (C) Emperor Alamgir
(C) HSBC (D) Raja Lakshman Sen
(D) Arab Bangladesh 100. In which district is the Payra Sea Port located?
92. Where is the mausoleum of Bir Sreshtha Mostafa (A) Khulna
Kamal? (B) Patuakhali
(A) Akhaura, Brahmanbaria (C) Barguan
(B) Muksudpur, Gopalganj (D) Cox's Bazar
(C) Charfashion, Bhola
(D) Bikrompur, Munshiganj
93. Where was the residence of 'Bangabondhu' located?
(A) Banani BANGLA
(B) Baridhara 1 ' '
(C) Dhanmondi
(D) Gulshan
94. What is the official name of litterateur Shaukat Osman? (C) 
(A) Abul Fazal (D)
(B) Ahmed Sharif

103
Collected By: Md. Raihan Ahmed
Job Information For All Students (Bcs & Bank)

2 ' ' ?

(C) 
(C) (D)
(D) 11 ' '- ' '
3 ? ?

(C)  (C) 
(D) (D)
4 ? 12 ' '-
,
,
(C) , 
(D) , (C)
5 ' ' ? (D) 
 13 ' '-

(C)
(D) (C)
6 ' ' ? (D) 
 14 ' ' ?

(C)
(D) (C) 
7 ' '- (D)
? 15 ' ' ?


(C)  (C)
(D) (D)
8 ' ' ?

 16. At the end of the play about women's liberation, the


(C) leading lady cautioned the audience not to judge
(D) womanhood by the way she dresses .
9 ' ' ? (A) she dresses
(B) she dressed
 (C) it dresses
(C) (D) they dressed
(D) 17. As no one knows the truth as fully as him , no one but
10 ' ' ? him can provide the testimony .
(A) as fully as his , no one but him

104
Collected By: Md. Raihan Ahmed
Job Information For All Students (Bcs & Bank)

(B) as fully as he , no one but him (C) decorative


(C) as fully as he does , not one but him (D) Showy
(D) as fully as he does , no one but he alone 26. INDIGENT
18. His speech was optimistic , but at the end of it he (A) inferior
stroke a note of caution . (B) wealthy
(A) strokes a note of caution . (C) subordinate
(B) strut for a note of caution . (D) insolvent
(C) stroked a note of caution . 27. SPOIL
(D) struck a note of caution .  (A) improve
19. Men have been known how important the sun is to (B) blemish
them. (C) destroy
(A) Men have been knowning (D) blight
(B) Men have to known long 28. AVARICE
(C) Men had long known (A) cupidity
(D) Men have long known (B) conservation
20. Please give him medicines if his temperature will rise . (C) indulgence
(A) if his temperature will rise . (D) generosity
(B) if his temperature would rise . 29. INERT
(C) if his temperature rise . (A) tight
(D) unless his temperature rise . (B) immobile
21. MELEE (C) moving
(A) kindness (D) void
(B) brawl 30. TENDER
(C) simple song (A) difficult
(D) primitive dance (B) leonine
22. LIMPID (C) tepid
(A) moist (D) hardened
(B) dear 31. Animals who eat flesh of another animal
(C) transparent (A) maneater
(D) dark (B) beast
23. OBTUSE (C) carnivorous
(A) difficult (D) cannibal
(B) interfering 32. Cessation of arms before a formal treaty is signed
(C) blunt during war
(D) concealed (A) truce
24. MYSTICAL (B) armistice
(A) imaginary (C) accord
(B) vague (D) retreat
(C) prophetic 33. Something that becomes outdated
(D) spiritually symbolic (A) old
25. OSTENTATION (B) ancient
(A) protruding (C) obsolete
(B) wealthy (D) useless

105
Collected By: Md. Raihan Ahmed
Job Information For All Students (Bcs & Bank)

34. Official misconduct (A) +


(A) malefactor (B) -
(B) malfeasance (C) ×
(C) maltreatment (D) ÷
(D) maladministrations Solution:
35. Likely to break apart easily 2?6 12/4 + 2 = 11
(A) breakable Or, 2?6 3 + 2 = 11
(B) thin Or, 2?6 - 1 = 11
(C) brittle Or, 2?6 = 11 + 1 = 12
(D) harsh So, 2 × 6 = 12
40 The average of five consecutive odd numbers is 61.
MATHAMETICS (Solved By Raihana Pervin Rontu) What is the difference between the highest and lowest
36 The smallest 6-digit number exactly divisible by 111 is numbers ?
(A) 111111 (A) 2
(B) 110011 (B) 5
(C) 100011 (C) 8
(D) 110101 (D) None of these
Solution: Solution:
Smallest number from the option is 10011
So, we have to check the option 1st Average of 5 number is 61
100011/111 = 91, so this is divisible by 111. So, (x + 1 + x + 3 + x + 5 + x + 7 + x + 9)/5 =61
37 The ratio of two numbers is 3:4 and their H.C.F. is 4 . (5x + 25)/5 = 61
Their L.C.M. is : 5x + 25 = 61 × 5
(A) 12 5x + 25 = 305
(B) 16 5x = 305 25
(C) 24 x = 280/5
(D) 48 x = 56
Solution: Lowest no is x+1 So,56+1 = 57
The ratio of numbers 3:4 and their HCF is 4, so the Highest no is x+9 So,56+9 = 65
numbers are 3 * 4 = 12 and 4 * 4 = 16 The difference is, 65 57 = 8
We know, Shortcut:
LCM * HCF = multiplication of the two numbers As the numbers are consecutive odd numbers, so for any
LCM * 4 = 12 * 16 consecutive odd numbers difference between 2 numbers =
LCM = 12 * 16/4 = 48 2, for five numbers difference will be 4*2 = 8, difference is
38 What decimal of an hour is a second ? counted for four places.
(A) 0.0025 41 Two numbers are such that the ratio between them is
(B) 0.0256 4:7 . If each is increased by 4, the ratio becomes 3:5 . The
(C) 0.00027 larger number is :
(D) 0.000126 (A) 36
Solution: (B) 48
1/(60 * 60) = 1/3600 = 0.00027 (C) 56
39 What mathematical operation should come at the place (D) 64
of '?' in the equation : 2 ? 6 - 12 ÷ 4 + 2 = 11

106
Collected By: Md. Raihan Ahmed
Job Information For All Students (Bcs & Bank)

Solution: Or, x = 15
(4x +4)/(7x + 4) = 3/5 Present age of A = 3 × 15 = 45 and B = 15
Or, 20x + 20 = 21x + 12 45 Aunik, Kamal and Jamal invested TK. 8000, TK. 4000
Or, x = 8 and TK. 8000 respectively in a business . Aunik left after six
Larger number = 7 × 8 = 56 months. If after eight months, there was a gain of TK.
42 A person's present age is two-fifth of the age of his 4005, then what will be the share of Kamal ?
mother. After 8 years , he will be one-half of the age of his (A) TK. 890
mother . How old is the mother at present ? (B) TK. 1335
(A) 32 years (C) TK. 1602
(B) 36 years (D) TK. 1780
(C) 40 years Solution:
(D) 48 years Aunik : Kamal : Jamal
Solution: = (8000 × 6) : (4000 × 8) : (8000 × 8)
Let, age of mother = x and age of person = 2x/5 = 48000 : 32000 : 64000
(x + 8)/(2x/5 + 8) = 2/1 =3:2:4
Or, x + 8 = 4x/5 + 16 So, share of Kamal = 4005 × (2/9) = 445 × 2 = 890
Or, x 4x/5 = 16 - 8 46 15 men take 21 days of 8 hours each to do a piece of
Or, x/5 = 8 work. How many days of 6 hours each would 21 women
Or, x = 40 take , if 3 women do as much work as 2 men ?
43 If 25% of a number is subtracted from a second (A) 18
number, the second number reduces to its five-sixth .What (B) 20
is the ratio of the first number to the second number ? (C) 25
(A) 1:3 (D) 30
(B) 2:3 Solution
(C) 3:2
(D) None of these Then, (21 × 8 × 15)/(x × 6 × 21) = 2/3
Solution: Or, x = 30
Let, first number = x and second number = y 47 A is 30% more efficient than B . How much time will
Now, y 25%of x = 5y/6 they, working together , take to-complete a job which A
Or, y 25x/100 = 5y/6 alone could have done in 23 days ?
Or, y- 5y/6 = x/4 (A) 11 days
Or, y/6 = x/4 (B) 13 days
x/y = 4/6 = 2/3 (C) 21 days
44 The ages of A and B are in the ratio 3:1 . Fifteen years (D) None of these
hence , the ratio will be 2:1 . Their present ages are : Solution
(A) 30 years , 10 years 30% of 23 = 23 × 30/100 = 69/10
(B) 45 years , 15 years A can do the work in 1 day 1/23
(C) 21 years , 7 years B can do the work in 1 day 1/(23 + 69/10)
(D) 60 years , 20 years Together they can do in 1 day = (1/23) + (1/(23 +69/10))
Solution: = 1/23 + 10/299 = (13+10)/299 = 26/299 = 1/13
Let, present age 3x and x of A and B respectively. So, in 13 days they can complete the work.
After 15 years, (3x+15)/(x+15) = 2/1 48 One pipe can fill a tank three times as fast as another
Or, 3x+15 = 2x+30 pipe. If together the two pipes can fill the tank in 36

107
Collected By: Md. Raihan Ahmed
Job Information For All Students (Bcs & Bank)

minutes, then the slower pipe alone will be able to fill the (A) TK. 1020.80
tank in : (B) TK. 1025
(A) 81 min (C) TK. 1052
(B) 108 min (D) None of these
(C) 144 min Solution:
(D) 192 min SI = 956 800 = 156
Solution: 156 = 800 × 3 × r/100
Slow pipe is 3 times slower than fast pipe. Or, r = 156/24 = 6.5%
So, it fills 1/4 of the tank. New rate = 6.5 + 4 = 10.5%
The slower pipe alone will be able to fill the tank in 36*4 = Interest = 800 × 10.5 ×3/100 = 252
144 min ∴ Amount = 800 + 252 = 1052 Tk
49 A person has to cover a distance of 6 km in 45 minutes. 52 The percentage increase in the area of a rectangle , if
If he covers one-half of the distance in two-thirds of the each of its sides is increased by 20%, is :
total time; to cover the remaining distance in the remaining (A) 40%
time , his speed (in km/hr ) must be : (B) 42%
(A) 6 (C) 44%
(B) 8 (D) 46%
(C) 12 Solution:
(D) 15 Let the original length = x and breadth = y and area = xy
Solution: New length = 120x/100 and
2/3 × 45 = 30 new breadth = 120y/100 and
1/2 × 6 = 3 New area = (120x/100) ×(120y/100) = 144xy/100 =36xy/25
So, he has to cover 3 km in (45-30) = 15 minutes = 1/4
The difference between the original area and new-area is
hour
His speed must be 3/(1/4) = 12 km/hr = (36/25)xy - xy

50 How many seconds will a 500 meter long train take to = xy(36/25 - 1)
cross a man walking with a speed of 3 km/hr in the
= xy(11/25) or (11/25)xy
direction of the moving train if the speed of the train is 63
km/hr ? Percentage = (11xy/25)/xy × 100 = 44%
(A) 25 53 Cyclone is related to Anticyclone in the same way as
(B) 30 Flood is related to ............ ? ...............
(C) 40 (A) Devastation
(D) 45 (B) Havoc
Solution: (C) River
Because of same direction the average speed = 63 3 =60 (D) Drought
km/hr 54 Root : Stem : Branch : ?
Train = 500 m = 1/2 km (A) Wood
60 km can be crossed within 1 hr or 60 × 60 seconds (B) Leaf
1/2 km can be crossed within (60 × 60)/(60 × 2) = 30 (C) Tree
seconds (D) Fertilizer
51 TK. 800 becomes TK. 956 in 3 years at a certain rate of 55 Choose out the odd one.
simple interest. If the rate of interest is increased by 4%, (A) IMF
what amount will TK. 800 become in 3 years ? (B) SAARC

108
Collected By: Md. Raihan Ahmed
Job Information For All Students (Bcs & Bank)

(C) UNICEF (C) 12


(D) WHO (D) 23
56 Choose the number pair/group which is different from Solution:
others . No of student=60
(A) 50-66 No. of boys and girls= x + 2x =60
(B) 32-48 No. of boys = 20
(C) 64-88 No. of girls = 40
(D) 63-77 No. of the boy ranked ahead of kamal if there are 9 girls
Solution: ahead of him = 17 9=8
Only odd number group is 63-77 No. of boys ranked from the bottem are 20 8 = 12
57 If A = 26, SUN = 27, then CAT = ? 62 A, B and C are sisters . D is the brother of E . and E is
(A) 24 the Daughter of B. How is A related to D ?
(B) 27 (A) Sister
(C) 57 (B) Cousin
(D) 58 (C) Niece
58 How many sons does X have ? (D) Aunt
I. Q and U are brothers of T. 63 2√ 27 − √ 75 + √ 12) is equal to :
II. R is sister of P and U. (A) √3
III. Rand Tare daughters of X (B) 2√3
(A) Only I and II (C) 3√3
(B) Only II and III (D) 4√3
(C) All I , II and III Solution:
(D) I, II and III together are not sufficient 2√ 27 − √ 75 + √ 12
59 If A + B = 2C and C + D = 2A , then = 2√(3*9) − √(3*25) + √(3*4)
(A) A + C = B + D = 2*3√3 − 5√3 + 2√3
(B) A + C = 2D = 6√3 − 5√3 + 2√3
(C) A + D = B + C = 3√3
(D) A + C = 2B 64 If m and n are whole numbers such that mn = 121,
Solution: then the value of (m − 1)n+1 is :
As, A + B + C + D = 2A + 2C = A + C + A + C (A) 1
Or, B + D = A + C (B) 10
60 Arrange the following in a logical order : 1. Euphoria 2. (C) 121
Happiness 3. Ambivalence 4. Ecstasy 5. Pleasure (D) 1000
(A) 1, 4, 2, 5,3 Solution:
(B) 2,1, 3, 4, 5 Mn = 121 = 112
(C) 3, 2, 5, 1, 4 So, m = 11 and n =2
(D) 4, 1, 3, 2, 5 (m-1)n+1 = (11-1)2+1 = 103 =1000
61 In a class of 60, where girls are twice that of boys , 65 If selling price of an article is of its cos t price 3/4 , the
Kamal ranked seventeenth from the top . If there are 9 girls profit in the transaction is :
2
ahead of Kamal , how many boys are after him in rank ? (A) 16 %
3
1
(A) 3 (B) 20 %
2
(B) 7 (C) 25 %
1
2
1
(D) 33 %
3

109
Collected By: Md. Raihan Ahmed
Job Information For All Students (Bcs & Bank)

Solution: (D) Both A and B

Selling price = x 73 The output quality of a printer is measured by


(A) Dot per sq. inch
Profit (x – 3x/4) = x/4
(B) Dot per inch
Profit in the transaction (C) Dots printed per unit time
1 (D) All of them
= (x/4)/(3x/4) × 100 = 100/3 = 333 %
74 Which of the following storage devices can store
maximum amount of data?
COMPUTER
(A) Floppy Disk
66 The basic operations performed by a computer are
(B) Hard Disk
(A) Arithmetic operation
(C) Compact Disk
(B) Logical operation
(D) magneto Optic Disk
(C) Storage and relative
75 Which of the following is not an input device ?
(D) All of them
(A) OCR
67 The two major types of computer chips are
(B) COM ( Computer Output to Microfilm ) 
(A) External memory chip
(C) Voice recognition device
(B) Primary memory chip
(D) Optical scanners
(C) Microprocessor chip
76 Properly arranged data is called
(D) Both B and C
(A) Field
68 The brain of any computer system is
(B) Words
(A) ALU
(C) Information
(B) Memory
(D) File
(C) CPU
77 What is required when more than one person uses a
(D) Control unit
central computer at the same time ?
69 Storage capacity of magnetic disk depends on
(A) Light pen
(A) tracks per inch of surface
(B) Mouse
(B) bits per inch of tracks
(C) Digitizer
(C) disk pack in disk surface
(D) Terminal
(D) All of them
78 Which of the following produces the best quality
70 The two kinds of main memory are :
graphics reproduction ?
(A) Primary and secondary
(A) Laser printer
(B) Random and sequential
(B) InkJet printer
(C) ROM and RAM
(C) Plotter
(D) All of them
(D) Dot matrix printer
71 CD-ROM is a
79 Which of the following is not processing ?
(A) Semiconductor
(A) arranging
(B) Memory register
(B) manipulating
(C) Magnetic Memory
(C) calculating
(D) None of these
(D) gathering
72 Which of the following IC was used in third generation
80 Instructions and memory address are represented by
of computers ?
(A) Character code
(A) SSI
(B) Binary codes
(B) MSI
(C) LSI

110
Collected By: Md. Raihan Ahmed
Job Information For All Students (Bcs & Bank)

(C) Binary word 88 What is the local name of Saint Martin Island ?
(D) Parity bit (A) Narikel Monjira
(B) Narikel Jin Jira
GENERAL KNOWLEDGE (C) Nariker Batasha
81 Which of the following is situated to the East of (D) Narikel Bagan
Bangladesh ? 89 The only Floklore Museum of Bangladesh is located at
(A) Tripura (A) Naogaon
(B) Sikim (B) Bajitpur
(C) Mongolia (C) Sonargoan
(D) Meghalaya (D) Netrokona
82 Which of the following organization has head office 90 Who made Hossaini Building (Imam Ban) ?
located at Dhaka ? (A) Mir Hamza
(A) UNICEF (B) Mir Jafar
(B) CIRDAP (C) Mir Makbul
(C) SAARC (D) Mir Murad
(D) BIMSTEC & CIRDAP 91 Which of the following country does not belong to G-8 ?
83 Which film is made on the partition of 1974 ? (A) Switzerland
(A) Juddhashishu (B) France
(B) Abar Tora Manush Ha (C) Germany
(C) Chitra Naidr Pare (D) Italy
(D) Nadir Naam Madhumati 92 Where is Birmingham Palace situated ?
84 Who was the captain of the test match of Bangladesh ? (A) London
(A) Akram Khan (B) Birmingham
(B) Naymur Rahman (C) Glasgow
(C) Gazi Ashraf Hossain (D) Cardiff
(D) Khaled Masud 93 What is the name of Israel's intelligence agency ?
85 Which one is the biggest religious and social festival of (A) SISMI
the Chakmas ? (B) Mossad
(A) Bijhu (C) ASIS
(B) Wangala (D) CSIS
(C) Sandrey 94 The recent Nepal earthquake in April 2015 is also
(D) Sangrai known as the
86 What is the name of the biggest NGO in the world ? (A) Pokhra earthquake
(A) Grameen Bank (B) Chitwan earthquake
(B) BRAC (C) Gorkha earthquake
(C) Proshika (D) NagarBhum earthquake
(D) ASA 95 When was "Missionaries of Charity" established by
87 Which Bangladeshi player's name has been included in Mother Teressa ?
the Guinness Book of Record ? (A) 1950
(A) Salma Khatun (B) 1951
(B) Rani Hamid (C) 1953
(C) Sharmin Akhter (D) 1954
(D) Zobaira Linu

111
Collected By: Md. Raihan Ahmed
Job Information For All Students (Bcs & Bank)

96 Which one is the last member state of the United (C)


Nations ? (D)
(A) East Timor 3 ?
(B) South Sudan :
(C) Sierra Leon :
(D) Andorra (C) :
97 Who is the painter of the famous painting 'The (D) : 
Persistence of Memory' ? 4
(A) Leonardo da Vinci 
(B) Michel Angelo
(C) Edvard Munch (C)
(D) Salvador Dali (D)
98 Who became the first cricketer to score four successive 5 ?
one day international centuries during the World Cup 2015 , , ,
? , , , 
(A) Virat Kohli (C) , , ,
(B) kumar Sangakkara (D) , , ,
(C) Martin Guptill 6 ' , '-
(D) Hashim Amla
99 Who is known as 'the father of Green Revolution' ? (A) The devil wouldn't listen to the scripture
(A) Norman Borlaug (B) Run with the hare and hunt with the hounds
(B) Hanry A Wallace (C) Birds of a feather flock together
(C) Rachel Carson (D) None of these
(D) Joseph Watson 7 ' '
100 In which university does the physicist of Bangladesh
Dr. M Zahid Hasan work ?
(A) Princeton (C) 
(B) Harvard (D)
(C) Chicago 8
(D) Cornel 

(C)
(D)
BANGLA 9 ' ' ?
1 '
'- ?
 (C)
(D) 
(C) 10
(D)
2
(C)
 (D) 

112
Collected By: Md. Raihan Ahmed
Job Information For All Students (Bcs & Bank)

11 ? (C)
(D)
20 __
(C) 
(D)
12 ' '
 (D) 

(C)
(D) 21. The gift of the gab means
13 ' '- (A) an unexpected gain
(B) a X-mas gift
 (C) Thought provoking oration
(C) (D) fluency of speech
(D) 22. A fools ' paradise means
14 ' '- ' ' (A) false hopes for a foolish person
(B) an imaginary idea
(C) an unexpected gain
(C) (D) a foolish person
(D)  23. To cry wolf
15 'Morphology' - (A) to overcome someone
 (B) to turn pale
(C) to ruin overself
(C) (D) to give false alarm
(D) 24. A hard nut to crack is
16 ' ' (A) one who is very obstinate
 (B) very difficult child
(C) problem that is hard to be solved
(C) a walnut or a fruit
(D) 25. To the ends of the earth
17 (A) upto a certain limit
(B) till losing one's interest
(C) everywhere
(C)  (D) till losing one's patience
(D) 26. On the results of the survey _____ and the type of
18 campaign , we shall wage .
 (A) depend the extend
(B) depend the extent
(C) (C) depending the extend
(D) (D) depends the extend
19 ? 27. Neither the salesmen nor the marketing manager ______
of the system .
 (A) is in favour

113
Collected By: Md. Raihan Ahmed
Job Information For All Students (Bcs & Bank)

(B) is favouring (B) Tepid : Seething


(C) are in favour (C) Intimate : Famous
(D) are for favour (D) Turbid : Swollen
28. The decoration of the new office block, including the
furniture and curtains, ______. Choose the opposite word : (36-40)
(A) is more pleasing 36. ACCRETION
(B) is most pleasing (A) stack
(C) are more pleasing (B) addition
(D) have most pleasing (C) procession
29. Please vote for the member ______ has done the most (D) erosion
for our village. 37. APPLAUD
(A) who you believe (A) criticise
(B) who you believed (B) request
(C) that you believe (C) pray
(D) whom you believe (D) flatter
30. The chief competitor , as well as ourselves , prices 38. DEFAULT
this summer . (A) unlimited
(A) are obliged in rising (B) principal
(B) is obliged to rising (C) payment
(C) is obliged to raise (D) evasion
(D) are obliged to raise 39. PROFOUND
31. Fortuitous : Inherent :: ? : ? (A) obscure
(A) Legible : Indelible (B) superficial
(B) Rugged : Endurable (C) intense
(C) Envious : Desire (D) hidden
(D) Gregarious : Introverted 40. TRANSMIT
32. Alleviate : Aggravate :: ? : ? (A) show
(A) Elastic : Rigid (B) reply
(B) Elevate : Agree (C) televise
(C) Joke : Worry (D) withhold
(D) Level : Grade
33. Desultory : Methodical :: ? : ? MATHAMETICS (Solved By Tanisha tabassum)
(A) Integral : Unified 41 The total of the ages of Ali , Gazi and Masud is 93
(B) Dissipated : Concentrated years. Ten years ago, the ratio of their ages was 2 : 3 : 4 .
(C) Unborn : Died What is the present age of Masud ?
(D) Villian : Mighty (A) 24 years
34. Exempt : Obliged :: ? : ? (B) 38 years
(A) Affluent : Fluent (C) 34 years
(B) Steadfast : Reputed (D) 32 years
(C) Valiant : Mighty Solution:
(D) Immune : Susceptible The total of the ages of Ali, Gazi and Masud is 93 years.
35. Opaque : Transparent :: ? : ? Ten years ago, the ratio was 2:3:4.
(A) Concentrated : Dissipated ATQ,

114
Collected By: Md. Raihan Ahmed
Job Information For All Students (Bcs & Bank)

2x + 3x + 4x = 93-30 (C) 48
9x = 63 (D) 20
x =7 Solution:
4x = 28 The hands of a clock point in opposite directions (in the
Masud's age=38 same straight line) 11 times in every 12 hours. (Because
42 What will be the least number which when doubled between 5 and 7 they point in opposite directions at 6
with be exactly divisible by 12, 18, 21 and 30 ? o'clcok only). So, in a day, the hands point in the opposite
(A) 630 directions 22 times.
(B) 1260 46 Tow numbers A and B are such that the sum of 5% of A
(C) 2520 and 4%of B is two-third of the sum of 6% of A and 8%of B.
(D) 196 Find the ratio of A : B.
Solution: (A) 1 : 1
LCM of 12,18,21 and 30 is 1260 (B) 3 : 4
The number is 630 (C) 4 : 3
43 A train 800 meters long is running at a speed of 78 (D) 2 : 3
km/hr . If it crosses a tunnel in 1 minute, then the length Solution:
of the tunnel ( in meters ) is : 5% of A + 4% of B = 2 /3(6% of A + 8% of B)
(A) 360 A:B=4:3
(B) 500 47 If the number 5 * 2 is divisible by 6, then * = ?
(C) 540 (A) 3
(D) 130 (B) 6
Solution: (C) 7
Let the tunnel is x meter (D) 2
In 1 min the train crosses x + 800 metre Solution:
Speed = 78km/hr = 78000m/60min = 1300m/min As we can see it is an even number, so to be divisible by 6
ATQ, it needs to be divisible by 3.
x + 800 = 1300 If the sum of the digits of a number is divisible by 3, then
x = 500 the total number is divisible by 3.
44 If 4 (A's capital) = 6 (B's capital) , So putting 2 in the place of * the sum of the digit will be
then out of a profit of TK. 4650, C will receive : 5+2+2 = 9, which is divisible by 3
(A) TK. 900 Therefore * = 2
(B) TK. 1550 48 3 pumps , working 8 hours a day, can empty a tank in
(C) TK. 2250 2 days. How many hours a day must 4 pumps work to
(D) TK. 465 empty the tank in 1 day ?
Solution: (A) 10
Let, 4A = 6B = 10C = k (B) 11
Then A = k/4, B = k/6, C = k/10 (C) 12
A : B : C = k/4 : k/6 : k/10 = 15 : 10 : 6 (D) 9
C gets 4650 × 6/31 = 900 Solution:
45 How many times in a day, are the hands of a cock in Total hours needed for a pump to empty = 3 * 8 * 2 = 48
straight line but opposite in direction ? hours
(A) 22 4 pumps can empty it in one day by working for 48/4 = 12
(B) 24 hours

115
Collected By: Md. Raihan Ahmed
Job Information For All Students (Bcs & Bank)

49 Which of the following trains is the fastest ? Additional interest = 225 202.50 = Tk. 22.50
(A) 25m/sec 53 2- [ 2 - { 2 - 2 ( 2 +2 ) } ] = ?
(B) 1500m/min (A) -4
(C) 90km/hr (B) 6
(D) None of these (C) 4
Solution: (D) None of these
All of them same speed Solution:
50 A student was asked to find the arithmetic mean of the 2 - [ 2 - { 2 - 2 ( 2 +2 ) } ]
numbers 3, 11, 7, 9, 15, 13, 8, 19, 17, 21, 14, and x. He = 2- [ 2 - { 2 - 2( 4 ) } ]
found the mean to be 12. What should be the number in = 2- [ 2 - { 2 - 8 } ]
place of x ? = 2- [ 2 - { 2 {-6}]
(A) 7 =2 [ 2- {2 + 6}]
(B) 17 =2 [2 8]
(C) 31 =2 [-6]
(D) 3 =2+8
Solution: = 10
Clearly, we have 54 If 0.4 of a number is equal to 0.06 of another number,
(3 +11 +7+ 9 +15 +13 +8 +19 +17+21+14+ x)/12 = 12. the ratio of the numbers is :
or 137 + x = 144 (A) 3:4
or x = 144 - 137 = 7. (B) 3:20
51 Find the smallest number by which 5808 should be (C) 20:3
multiplied so that the product becomes a perfect square . (D) 2:3
(A) 3 Solution:
(B) 7 o.4 x = 0.06y
(C) 11 x/y = 0.06/0.4 = 3/20
(D) 2 55 The breadth of a rectangular field is 60% of its length .
Solution: If the perimeter of the field is 800 m, what is the area of
5808 = 2 x 2 x 2 x 2 x 3 x 11 x 11 the field ?
= 2^2 x 2^2 x 3 x 11^2. (A) 37500 sq.m
To make it a perfect square, it must be multiplied by 3. (B) 40000 sq.m
52 The interest on a certain deposit at 4.5% p.a. is TK. (C) 48000 sq.m
202.50 in one year. How much will the additional interest in (D) 18750 sq.m
one year be on the same deposit at 5% p.a. ? Solution:
(A) TK. 22.50 Length =100x
(B) TK. 25 Breadth = 60x
(C) TK. 20.25 Perimeter=800
(D) TK. 42.75 Atq,
Solution: 2(100x + 60x) = 800
I = pnr 320x = 800
202.50 = p ×1 × 4.5% x = 80/32 =5/2
P = 202.5/4.5% = 4500 Area = 6000x^2 = 6000.25/4 =37500
So the new Interest will be, 56 Peter purchased a machine for TK. 80,000 and spent
4500 × 1 × 5% = 225 TK. 5,000 on repair and TK. 1,000 on transport and sold it

116
Collected By: Md. Raihan Ahmed
Job Information For All Students (Bcs & Bank)

with 25% profit , At what price did he sell the machine ? (C) 78
(A) TK.1,06,250 (D) 73
(B) TK.1,07,500 Solution:
(C) TK.1,17,500 XYZ
(D) TK. 1,05,100 Z = y + 2x
Solution: So the next number is 75
Total spent on the machine = 80000 + 5000 +1000 =86000 62 Rahim ranks seventh from the top and twenty-sixth
Selling price = 86000 + 25% of 86000 = Tk. 107500 from the bottom in class . How many students are there in
57 A takes twice as much time as B or thrice as much time the class ?
as C to finish a piece of work . Working together , they can (A) 32
finish the work in 2 days . B can do the work alone in : (B) 33
(A) 6 days (C) 34
(B) 8 days (D) 31
(C) 12 days 63 When you see a blind man trying to cross the road ,
(D) 4 days you
Solution: (A) go and help him
Suppose A, B and C takes x , x/2 and x/3 days respectively (B) ignore and move on
to finish the work (C) ask someone to help him
Atq, wait till he crosses the road
1/x + 2/x + 3/x = 1/2 64 Z, U, Q, ?, L
6/x = 1/2 (A) K
x = 12 (B) M
So, B takes (12/2) = 6 days to finish the work. (C) N
58 Which of the following fractions is greater than 3/4 and (D) I
less than 5/6 ? 65 If A > B, B > C and C > D, then which of the following
(A) 2/3 conclusions is definitely wrong ?
(B) 4/5 (A) A > C
(C) 9/10 (B) D > A
(D) 1/2 (C) A > D
59 Arrange the following in a meaningful sequence : (D) B > D
(A) Consultation 66 Which letter in the alphabet is as far from G as T is
(B) Illness from M ?
(C) Treatment (A) N
(D) Recovery (B) O
60 If X is brother of son of Y's son, then how is X related (C) P
to Y ? (D) M
(A) Cousin 67 Among P, Q, R, S and T, Q is the second tallest and S is
(B) Grandson immediate taller than the shortest . Who among them is in
(C) Son the middle when they stand in the order of their heights ?
(D) Brother (1) T is not the shortest
61 2, 5, 9, 19, 37, ? (2) R is taller than S but shorter than Q .
(A) 75 (3) P ranks third in height above S when all are arranged in
(B) 76 the order of height .

117
Collected By: Md. Raihan Ahmed
Job Information For All Students (Bcs & Bank)

(A) Only 1 and 2 COMPUTER


(B) Only 2 and 3 71 Which of the following is not a micro computer ?
(C) Either 2 only or 1 and 3 only (A) Tablet PCs
(D) Only 2 (B) Laptop PCs
68 How many days are there in x weeks x days? (C) Desktop PCs
(A) 8x (D) None of these
(B) 14x 72 Which operation is not performed by computer
(C) 7 (A) Processing
2
(D) 7x (B) Controlling
Solution: (C) Understanding
1 week has 7 days (D) Inputting
x weeks have 7x days 73 MICR stands for
Total = 7x + x = 8x days (A) Magnetic Ink Cases Reader
69 Pipes A and B can fill a tank in 5 and 6 hours (B) Magnetic Ink Character Reader
respectively. Pipe C can empty it in 12 hours. If all the (C) Magnetic Ink Code Reader
three pipes are opened together, then the tank will be filled (D) None of them
in: 74 Which is not a computer classification ?
8
(A) 2 hours (A) maxframe
11
9
(B) 3 hours (B) mainframe
17
1
(C) 4 hours (C) mini
2
(D) 1
13
hours (D) notebook
17
Solution: 75 Which of the following is associated with error detector

A in 1 hour fills 1/5 part ?

B in 1 hour fills 1/6 part (A) Even parity bit

C in one hour empties 1/12 Part (B) Odd parity bit

Together they fill in 1 hour 1/5 + 1/6 - 1/12 = (12 + 10- (C) Both of A & B

5)/60 = 17/60 part (D) None of them

Total Time = 60/17 = 3


9
hours 76 Signals can be analog or digital and a computer that
17
70 processes the both type of signals is known as
(A) Digital computer

5 6 6 7 4 8 (B) Hybrid computer


12 21 ? (C) Analog computer

4 10 10 Mainframe computer

(A) 30 77 The word length of a computer is measured in

(B) 32 (A) Millimeters

(C) 22 (B) Meters

(D) None of these (C) Bits

Solution: (D) Bytes

5 × 6 × 4 = 120/10 = 12 78 A computer program that converts an entire program

6 × 7 × 5 = 210/10 =21 into machine language is called a/an

4 × 8 × 10 = 320/10 =32 (A) Simulator


(B) Compiler
(C) Interpreter

118
Collected By: Md. Raihan Ahmed
Job Information For All Students (Bcs & Bank)

(D) Commander (A) Caretaker government System


79 Which unit is known as nerve center of computer ? (B) Reserved Seats for Women
(A) CU (C) Islam as State Religion
(B) Memory (D) Secularism
(C) ALU 87 The capital city of Myanmar is :
(D) Registers (A) Yangon
80 Floppy disks which are made from flexible plastic (B) Naypi Daw
material are also called ? (C) Manadalay
(A) High-density disks (D) Bagan
(B) Diskettes 88 Which one is constitutional organization ?
(C) Hard disks (A) Bangladesh Atomic Energy Commission
(D) Templates (B) Bangladesh Privatization Board
(C) Bangladesh Election Commission
GENERAL KNOWLEDGE (D) Bangladesh Investment Board
81 Which institution first published the complete map of 89 The Cactus Curtain separates Guantanamo Naval Base
Bangladesh's sea boundary ? from :
(A) Marine Academy (A) Cuba
(B) Dhaka University (B) Mexico
(C) Institute of Marine Sciences and Fisheries (C) Barbados
(D) None of these (D) Samoa
82 Ramon Magsaysay Award is given from 90 The Louvre museum is in :
(A) Malaysia (A) Rome
(B) Australia (B) Vienna
(C) Philippines (C) Geneva
(D) Indonesia (D) Paris
83 The present per capita income of Bangladesh is 91 How many were accused in the Agartala conspiracy case
(A) 1300 US dollar including Banglabandhu?
(B) 1314 US dollar (A) 35 persons
(C) 1200 US dollar (B) 39 persons
(D) None of these (C) 51 persons
84 Who is the designer of the National Flag of Bangladesh (D) 19 persons
? 92 What is the name of Malaysian Currency?
(A) Zainul Abedin (A) Dinar
(B) Kamrul Hasan (B) Ringgit
(C) Badrul Hasan (C) Rial
(D) All of them (D) None of these
85 Who scored the first ever per feet '10' gymnastics 93 World Population Day is observed on
(A) Nadia Comaneci (A) 11 July
(B) Olga Korbut (B) 10 December
(C) Valentina Turisheva (C) 17 October
(D) Svetlana Khorkina (D) 21 June
86 The 15th Amendment of the Bangladesh Constitution 94 What kind of organization of the Bangladesh
abolished ............. government is DAE?

119
Collected By: Md. Raihan Ahmed
Job Information For All Students (Bcs & Bank)

(A) Agriculture (B) DRAM


(B) Education (C) ROM
(C) Drama (D) All of them
(D) Drag 2. Which of the items below are considered removable
95 The Masai Mara National Reserve in Kenya is contiguous storage media ?
with what famous national park of (A) Tanzania? (A) Removable hard disk cartridges
(A) Kruger (B) (Magneto-optical) disk
(B) The Serengeti (C) Flexible disks cartridges
(C) Okavango (D) All of them
(D) Faro 3. Which of the following is not a form of data ?
96 First medicine park of Bangladesh is being established (A) numbers and characters
in (B) images
(A) Savar (C) sound
(B) Gajaria (D) None of them
(C) Bhaluka 4. The control unit of a microprocessor
(D) Khulna (A) stores data in the memory
97 Usain Bolt is a citizen of (B) accepts input data from keyboard
(A) Sudan (C) performs arithmetic/logic function
(B) Jamaica (D) None of them
(C) Barbados 5. Which of the following is internal memory ?
(D) England (A) Disks
98 The last film of Humayun Ahmed was (B) Pen Drives
(A) Sarban Megher Dine (C) RAM
(B) Nandita Narake (D) CDs
(C) Ghetuputra Kamala 6. On which aspect the analog computers are better than
(D) Common Gender digital ?
99 Who is called the 'Father of the Green Revolution '? (A) Speed
(A) Henry A Wallace (B) Accuracy
(B) Norman Borlaug (C) Reliability
(C) Rachel Carson (D) Automatic
(D) Joseph Watson 7. How many numbers could ENIAC store in its internal
100 How much is the percentage of technocrat ministers memory ?
allowed in the cabinet of Bangladesh Government ? (A) 100
(A) 10 Percent (B) 20
(B) 20 Percent (C) 80
(C) 1 percent (D) 40
(D) None of these 8. Binary circuit elements have
(A) One stable state
(B) Two stable state
(C) Three stable state
COMPUTER (D) None of them
1. Which of the following memories needs refreshing ? 9. Which of the following is used for manufacturing chips ?
(A) SRAM (A) Control bus

120
Collected By: Md. Raihan Ahmed
Job Information For All Students (Bcs & Bank)

(B) Control unit Analog computers


(C) Parity unit (D) AT computers
(D) Semiconductor 18. Central Processing Unit is combination of
10. Which of the following terms is the most closely related (A) Control and storage
to main memory ? (B) Control and output unit
(A) Non volatile (C) Arithmetic logic and input unit
(B) Permanent (D) Arithmetic logic and control unit
(C) Control unit 19. What does the disk drive of a computer do ?
(D) Temporary (A) Rotate the disk
11. The word length of a computer is measured in (B) Read the disk
(A) Bytes (C) Load a program from the disk into the memory
(B) Millimeters (D) Both B and C
(C) Meters 20. Which of the following produces the best quality
(D) Bits graphics reproduction?
12. Which unit holds data permanently ? (A) Laser printer
(A) Input unit (B) Ink Jet printer
(B) Secondary unit (C) Plotter
(C) Output unit (D) Dot matrix printer
(D) Primary Memory unit 21. What type of device is computer keyboard ?
13. Before a disk can be used to store data , it must be (A) Memory
(A) Formatted (B) Output
(B) Reformatted Storage
(C) Addressed (D) Input
(D) None of them 22. Which of the following is not true ?
14. Magnetic tape can serve as (A) Transistors are much smaller
(A) Secondary storage media (B) Transistors produce low heat
(B) Output media (C) Transistors were less reliable
(C) Input media (D) Transistors were used in radios and other electronic
(D) All of them devices
15. Which technology is used in computer disks ? 23. Typical data transfer rates in LAN are of the order of
(A) Mechanical (A) Bits per sec
(B) Electrical (B) Kilo bits per sec
(C) Electro Magnetic (C) Mega bits per sec
(D) Laser (D) None of them
16. Which of the following is a storage device ? 24. Which was the most popular first generation computer ?
(A) Tape (A) IBM 1650
(B) Hard Disk (B) IBM 360
(C) Floppy Disk (C) IMB 1130
(D) All of them (D) IBM 2700
17. Which of the following is a class of computers based on 25. The computer code of the interchange of information
model ? between terminals is
(A) Digital computers (A) ASCII
(B) Hybrid computers (B) BCD

121
Collected By: Md. Raihan Ahmed
Job Information For All Students (Bcs & Bank)

(C) EBCDIC 33. 'Mohasen' is related to


(D) All of them (A) Cyclone
(B) Tornedo
GENERAL KNOWLEDGE (C) Earthquake
26. What is the tenure of the Governor of Bangladesh Bank (D) Flood
? 34. The British Prime Minister is :
(A) 2 years (A) Tony Blair
(B) 3 years (B) Edward Heath
(C) 4 years (C) Brendan Cameron
(D) 5 years (D) David Cameron
27. Which international weekly recognized Bangabandhu 35. Which sports star is the highest medal winner in
Sheikh Mujibur Rahman as 'Poet of Polities' ? Olympics ?
(A) Time (A) Laris Latynina
(B) The Newsweek (B) Ben Johnson
(C) Outlook (C) Mark Spitz
(D) The Week UK (D) Michael phelps
28. Bangladesh shares land boundary with 36. What is the world's smallest landlocked country ?
(A) India and Nepal (A) Lesotho
(B) Nepal and Bhutan (B) Vatican City
(C) Myanmar and Bhutan (C) Nepal
(D) India and Mayanmar (D) Bhutan
29. Which one is the longest river of Bangladesh ? 37. ulian Assange is best known for starting which
(A) Padma controversial website in 2006 ?
(B) Meghna (A) youtube
(C) Jamuna (B) facebook
(D) Surma (C) wikileaks
30. In the Indian sub-continent the first woman graduate (D) wikipedia
with honours was 38. What is the official press agency of the government of
(A) Shantisudha bose China ?
(B) Kamini Roy (A) Renter's
(C) Parul Ghosh (B) Xinhua
(D) Abala Bose (C) Tianshan
31. Which Bangladeshi person received the title 'Knight' (D) Shihezi
from the French government ? 39. France's high speed rail service is known by what three-
(A) Dr. Muhammad Shahidullah letter name ?
(B) Partha Pratim Majumder (A) FFT
(C) Artist Shahabuddin Ahmed (B) TGV
(D) All of them (C) MRT
32. Avro Key-board was made by (D) RTS
(A) Mehdi Hasan Khan 40. Red Barn, now a historic site in the US, was the original
(B) Muhammad Zafar Iqbal manufacturing site of which company ?
(C) Bangladesh Computer Research Center (A) Boeing
(D) Maksudul Alam (B) Coca Cola

122
Collected By: Md. Raihan Ahmed
Job Information For All Students (Bcs & Bank)

(C) Kodak
(D) Red Rooster
(C) 
(D)
9
BANGLA 
1. ?
 (C)
(D)
(C) 10
(D)
2. -
(C) 
(D)
11
(C)
(D) 
3. The car turned turtle - (C)
 (D) 
12.
(C) 
(D)
4 (C)
(D)
13.
(C)
(D)  
5. 'Co-opted' - (C)
(D)
14
(C) 
(D) 
6 (C)
(D)
 15
(C)
(D)
7 (C) 
(D)
16
(C)
(D)  
8. , । ? (C)

123
Collected By: Md. Raihan Ahmed
Job Information For All Students (Bcs & Bank)

(D) ,
17. (C) ?
(D) ?
 (E) 
(C)
(D)
18. Select the word or phrase that is most
closely opposite in meaning to the capitalized word.
26. ACRIMONIOUS
(C) (A) legal
(D)  (B) severe
- (C) harmonious
(D) cursive
27. ANOMALOUS
(C) (A) usual
(D)  (B) vicious
20 (C) connected
(D) capacious
 28. INTRACTABLE
(C) (A) wayward
(D) (B) easygoing
21. (C) bleak
(D) abstinent
29. SEGREGATE
(C)  (A) abolish
(D) (B) confuse
22 (C) darken
(D) compile
30. RECLUSIVE
(C)  (A) gregarious
(D) (B) obscure
23 (C) joined
, , (D) urban
, ,
(C) , , Choose the one that best expresses the
(D) , ,  meaning of the given idiom/phrase.
31. Sunita has 'bitten off more than she can chew'.
 (A) trying to do too much
(B) over and above
(C) (C) beyond her capacity
(D) (D) bad fall
25 32. I have 'a bone to pick' with you in this matter.
(A) desire

124
Collected By: Md. Raihan Ahmed
Job Information For All Students (Bcs & Bank)

(B) selfish motive furniture and curtains, _______.


(C) selfless motive (A) is more pleasing
(D) am angry (B) are more pleasing
33. So far as hazards to pollution are concerned, the traffic (C) have most pleasing
policeman 'bear the brunt'. (D) is most pleasing
(A) face the consequences 40. The chief competitor, as well as ourselves, ______ prices
(B) are exposed to the danger this summer.
(C) suffer the most (A) is obliged to raise
(D) have to run the risk (B) is obliged to rising
34. His most trusted friend proved to be 'a snake in the (C) are obliged to raise
grass'. (D) are obliged in rising
(A) an unreliable persons
(B) low and mean Select from the alternatives, the word
(C) a hidden enemy that conveys the same meaning as the word given in capital
(D) have it both ways letters.
35. You cannot 'have your cake and eat it' too. 41. KNOTTY
(A) enjoy for ever (A) terrible
(B) enjoy without payment (B) mysterious
(C) absolve yourself of guilt (C) difficult
(D) have it both ways (D) confusing
42. ERRANT
Select from the answer choices given (A) unreliable
under each sentence to make the sentences grammatically (B) sinful
correct. (C) shifting
36. Neither the salesmen nor the marketing manager (D) confusing
of the system. 43. BUCOLIC
(A) is favouring (A) rural
(B) are for favour (B) simple
(C) are in favour (C) hidebound
(D) is in favour (D) quite
37. Please vote for the member _______ has done the most 44. CASTIGATE
for our village. (A) deligate
(A) who you believe (B) criticise
(B) who you believed (C) evaluate
(C) that you believe (D) enforce
(D) whom you believe 45. CORROBORATE
38. On the results of the survey ______ and the type of (A) disprove
campaign, we shall wage. (B) refute
(A) depends the extent (C) verify
(B) depend the extent (D) disapprove
(C) depending the extent
(D) depend the extend Choose the pair that best expresses the
39. The decoration of the new office block, including the relationship existing between the given pair.

125
Collected By: Md. Raihan Ahmed
Job Information For All Students (Bcs & Bank)

46. CONTRADICT : CONTRAVENE So their ages 16, 28, 36


(A) profane : vulgarity 52 Find the surface area of a 10cm × 4cm × 3cm brick.
(B) confuse : satisfy (A) 48 sq.cm
(C) compensate : damage (B) 124 sq.cm
(D) deny : refute (C) 164 sq.cm
47. DEPRECATE : CREDIT (D) 180 sq.cm
(A) unfair : biased Solution:
(B) bait : heckle We know surface area = 2 (ab + bc +ca)
(C) enhance : disparage = 2(40 + 12 + 30 )
(D) discourage : force = 164 sq. cm
48. BLURRED : CONFUSED 53 By how much is three-fifth of 350 greater than four-
(A) dangerous : adequate seventh of 210 ?
(B) muddled : unclear (A) 95
(C) scam : clarity (B) 110
(D) abatement : significant (C) 120
49. HARM : DAMAGE (D) None of these
(A) injure : incapacitate Solution:
(B) sweet : sour 3/5 of 350 = 210
(C) stout : weak 4/7 of 210 = 120
(D) hook : crook Difference = 210 120 = 90
50. BIASED : PARTIAL 54 In what ratio must a grocer mix two varieties of tea
(A) built-in : included worth TK. 60 a kg and TK. 65 a kg so that by selling the
(B) axle : wheel mixture at TK. 68.20 a kg he may gain 10% ?
(C) partisan : prejudiced (A) 3:2
(D) leaning : yield (B) 3:4
(C) 3:5
MATHAMETICS (D) 4:5
51 The present ages of three persons are in proportions Solution:
4:7:9. Eight years ago, the sum of their ages was 56. Find Let the ratio of the mixture be x : y
their present ages(in years). Selling price 68.20 then cost price=100/110 × 68.20= 62 tk.
(A) 8,20,28 Atq,
(B) 16,28,36 60x + 65y = 62(x + y)
(C) 20,35,45 2x = 3y
(D) None of these x/y = 3/2
Solution: 55 The sum of two numbers is 15 and the sum of their
Let the present ages of three persons are 4x, 7x, 9x squares is 113. Find the numbers.
ATQ, (A) 6 and 9
4x 8 + 7x 8 + 9x -8 = 56 (B) 7 and 8
20x - 24 = 56 (C) 10 and 5
20x = 54 + 24 (D) None of these
20x = 80 Solution:
x=4 Let the numbers are x & y
ATq,

126
Collected By: Md. Raihan Ahmed
Job Information For All Students (Bcs & Bank)

x + y = 15 and x2 + y2 = 113 in 1 hour and goes 1 km along the current in 10 minutes.


Now, How long will it take to go 5 km in stationary water ?
2 2
x + y = 113 (A) 40 minutes
2 2
(15 y) + y = 113 [x + y =15] (B) 1 hour
225 30y + y2 + y2 = 113 (C) 1 hour 15 minutes
2
2y -30y + 112 = 0 (D) 1 hour 30 minutes
2
y 15y + 56 = 0 Solution:
(y -7) (y -8) = 0 Rate downstream = (1/10 × 60)km/hr = 6km/hr
So y = 7 or y = 8 Rate upstream =2km/hr
And x = 7 or x = 8 Speed in still water = 1/2(6 + 2)km/hr = 4km/hr
1
Therefore the numbers are 7 and 8 ∴Required time = 5/4hrs = 1 hrs = 1hr 15min.
4
56 Which of the following numbers does not lie between 60 How many prime numbers are less than 50 ?
4/5 and 7/13 ? (A) 14
(A) 1/2 (B) 15
(B) 2/3 (C) 16
(C) 3/4 (D) 18
(D) 5/7 Solution:
Solution: The total number of prime numbers less than 50 are 15
4/5 = 0.8 and 7/13 = 0.54 and are enumerated below.
And 1/2 = 0.5 2, 3, 5, 7, 11, 13, 17, 19, 23, 29, 31, 37, 41, 43, 47
So ½ is not between them 61 Two numbers are in the ratio 1:2. If 7 is added to both,
57 A clock is started at noon. By 10 minutes past 5, the their ratio changes to 3:5. The greatest number is :
hour hand has turned through : (A) 24
(A) 145 degree (B) 26
(B) 150 degree (C) 28
(C) 155 degree (D) 32
(D) 160 degree Solution:
Solution: Let the numbers are x and 2x
In 1 hour clock hand move = 360/12 = 30 degree ATQ,
From to 12 pm to 5.10 pm hr hand move (x + 7)/(2x + 7) = 3/5
= 30 * 5 + (30/60 * 10) = 155 degree. Or, 5x + 35 = 6x + 21
58 Running at the same constant rate, 6 identical machines Or, x = 14
can produce a total of 270 bottles per minute. At this rate, The greatest number is = 2 × 14 = 28
how many bottles could 10 such machines produce in 4
62 The simple interest on a certain sum of money for 2 ½
minutes ?
years at 12% per annum is TK. 40 less than the simple
(A) 648
interest on the same interest on the same sum for 3 ½
(B) 1800
years at 10% per annum. Find the sum.
(C) 2700
(A) TK. 700
(D) 10800
(B) TK. 800
Solution:
(C) TK. 900
1 machine can produce = 270/6 = 45 bottles per minute
(D) None of these
10 machine can produce in 4 minutes =10 × 45 × 4 = 1800
Solution:
59 A boatman goes 2 km against the current of the stream
Let the sum be P

127
Collected By: Md. Raihan Ahmed
Job Information For All Students (Bcs & Bank)

ATQ, (A) 99


[P × 7/2 × 10/100] [P × 5/2 × 12/100] = 40 (B) 89
7P/20 3P/10 = 40 (C) 79
7P 6P = 40 × 20 (D) 69
P = 800 Tk We know, Product of two numbers = H.C.F × L.C.M
63 The sum of two numbers is 28/25 of the first number. So the number is = 11 × 693/77 = 99
The second number is what percent of the first ? 67 Find the compound interest on Tk. 15,625 for 9 months
(A) 12% at 16% per annum compounded quarterly.
(B) 14% (A) Tk. 1851
(C) 16% (B) Tk. 1941
(D) 18% (C) Tk. 1951
Solution: (D) Tk. 1961
Let the first number be X and second number be Y Solution:
ATQ, P= Rs 15625
X + Y = 28x/25 n= 9 months= 3 quarters
Y = 28X/25 - X R= 16% p.a.= 4% per quarter
Y = 3X/25 Amount = [15625(1 + 4/100)3]
The second number = 3X/25 × 100/X = 12% of the first = 15625 × 26/25 × 26/25 × 26/25
64 A water tank is two-fifth full. Pipe A can fill a tank on = 17576 Tk
10 minutes and pipe B can empty it in 6 minutes. If both C.I.= (17576-15625)
the pipes are open, how long will it take to empty or fill the C.I.= Tk. 1951
tank completely ? 68 What is the smallest number to be subtracted from
(A) 6 min to empty 549162 in order to make it a perfect square ?
(B) 6 min to fill (A) 28
(C) 9 min to empty (B) 36
(D) 9 min to fill (C) 62
Solution: (D) 81
A can fill in 1 minute 1/10 part Solution:
B can empty in 1 minute 1/6 part √549162 = 741.0546
(A + B) can empty in 1 minute = (1/6 1/10) = 1/15 part Therefore 741 × 741 = 549081
1/15 part can empty in 1 minute Hence, 549162-549081 = 81
2/5 part can empty in = 15 × 2/5 = 6 minute 81 is the smallest number
65 A train 240m long passed a pole in 24 seconds. How 69 The sale price of an article including the sales tax is Tk.
long will it take to pass a platform 650m long ? 616. The rate of sales tax is 10%. If the shopkeepers has
(A) 65 sec made a profit of 12%, then the cost price of the article is :
(B) 89 sec (A) Tk. 500
(C) 100 sec (B) Tk. 515
(D) 150 sec (C) Tk. 550
Solution: (D) Tk. 600
Speed of the train = 240/24 = 10 m/s Solution:
Require time to pass the platform =(240 + 650)/10= 89 sec Let the sale price be x
66 The H.C.F of two number is 11 and their L.C.M is 693. 10x/100 + x = 110x/100 or 11x/10
If one of the numbers is 77 find the other. 11x/10 = 616

128
Collected By: Md. Raihan Ahmed
Job Information For All Students (Bcs & Bank)

x= 616 ×10/11 (A) 1


Value of x =Tk 560 (B) 2
Let CP be 100 (C) 3
Profit= 12 (D) None of these
SP= 112 Solution:
If SP is Tk 112 then CP = 100 Given, x = ya
If SP is Tk 560 then CP = 100/112 × 560 = Tk. 500 Or, x = (zb)a
70 A person crosses a 600m long street in 5 minutes. What Or, x = (xc)ab
is his speed in km per hour ? Or, x = xabc
(A) 3.6 Or, abc = 1
(B) 7.2 74 The difference between the length and breadth of a
(C) 8.4 rectangle is 23 m. If its perimeter is 206 m, then its area is:
(D) 10 (A) 1520 m2
Solution: (B) 2420 m2
Speed in km hour = 600/1000 × 60/5 = 7.2 kmph (C) 2480 m2
71 The average of 20 numbers is zero. Of them, at the (D) 2520 m2
most, how many may be greater than zero ? Solution:
(A) 0 Let the length be x and breadth y
(B) 1 ATQ,
(C) 10 X y = 23
(D) 19 Or, x = y + 23
Solution: Again,
Average of 20 numbers = 0. 2(X + y) = 206
Sum of 20 numbers (0 x 20) = 0. Or, 2(y + 23 + y) = 206
It is quite possible that 19 of these numbers may be Or, 4y + 46 = 206
positive and if their sum is a, then 20th number is (-a). Or, 4y = 160
72 Worker A takes 8 hours to do a job . Worker B takes 10 Or, y = 40
hours to do the same job. How long should it take both A And x = 40 + 23 = 63
and B, working together but independently , to do the Area of the rectangle = 63 × 40 = 2520 m2
same job? 75 How many days are there in x weeks x days ?
4
(A) 4 days (A) 7x2
9
4
(B) 4 days (B) 8x
7
3
(C) 4 days (C) 14x
8
(D) None of these (D) 7

Solution: Solution:

Job done by A in 1 hour = 1/8 One week = 7 days

Job done by B in 1 hour = 1/10 X weeks = 7x days

Part of job completed when they both do the work together So total days = 7x + x = 8x

in 1 hour = 1/8 + 1/10 = ( 5 + 4 )/40 = 9/40


So, 9/40 part of job is done in = 1 hour COMPUTER
4
Whole part of job is done in = 1 × 40/9 = 4 days 76 1 nibble equals to
9
73 If x = ya , y = zb and z = xc , then find the value of (A) 1 bit

abc. (B) 2 bits

129
Collected By: Md. Raihan Ahmed
Job Information For All Students (Bcs & Bank)

(C) 4 bits (C) Touch Pad


(D) 8 bits (D) Optical Scanners
77 Which Disk Format can access by MS-Dos 6.0 ? 85 Which of the following Language that Computer can
(A) FAT Understand & Execute ?
(B) OSX (A) Machine Language
(C) NTFS (B) C Programming Language
(D) EXT2 (C) Java Programming Language
78 Which of the following memories need refresh ? (D) None of these
(A) DRAM
(B) SRAM GENERAL KNOWLEDGE
(C) ROM 86 CEDAW is
(D) ALL (A) War stopping treaty between Argentina and Britain
79 Which of the following is the most commonly used (B) A treaty for the rights of children
encoding standard of Unicode ? (C) A treaty for the rights of women
(A) UTF-6 (D) A Canadian organization against war
(B) UTF-7 87 First Afghan war took place in
(C) UTF-8 (A) 1848
(D) UTF-9 (B) 1843
80 MICR stands for (C) 1833
(A) Magnetic Ink Case Reader (D) 1839
(B) Magnetic Ink Code Reader 88 The great Victoria Desert is located in
(C) Magnetic Ink Character Reader (A) Australia
(D) None of these (B) Canada
81 What is the full form of AT in the IBM PC-AT ? (C) West Africa
(A) Applied Technology (D) North America
(B) Advance Technology 89 In which country Bangla is the second national
(C) Additional Technology language?
(D) None of these (A) Ruanda
82 In which mode Windows starts with only the core (B) Sudan
drivers & services ? (C) Sierra Leone
(A) Normal Mode (D) Giana
(B) Safe Mode 90 When was Berlin Wall demolished ?
(C) Quick Mode (A) 1989
(D) Repair Mode (B) 1990
83 Physical connection between Microprocessor Memory (C) 1992
and other parts is called - (D) 1988
(A) Address bus 91 The D-8 is an organization of eight ______ countries
(B) Path (A) Developed
(C) Hub (B) Africa
(D) None of these (C) Asian
84 Which of the following is not an Input Device ? (D) Developing
(A) Touch Screen 92 Who was the leader of 'Tebhaga Andolon' of
(B) Mouse Pad Bangladesh?

130
Collected By: Md. Raihan Ahmed
Job Information For All Students (Bcs & Bank)

(A) Ila Mitra When was the constitution of Bangladesh adopted in


(B) Sumitra Devi the National Parliament of Bangladesh ?
(C) Taramon Bibi (A) 2 October, 1971
(D) Pritilata Waddedar (B) 16 December, 1972
93 Which Indian state was known as Mysore in the past ? (C) 4 November, 1972
(A) Kerala (D) 4 January, 1973
(B) Maharashtra
(C) Karnataka
(D) Modhya Pradesh
94 Who was the top scorer of 2014 FIFA World Cup ? BANGLA
(A) Arjen Robben
(B) Thomas Muller
(C) Lionel Messi (B)
(D) James Rodriguez (C) 
95 Organization of Islamic Cooperation(OIC) has _____ (D)
official languages. ?
(A) 1
(B) 2 (B) 
(C) 3 (C)
(D) 4 (D)
96 The chairmanship/presidency of the UN Security Council
rotate among the council members.
(A) every month (B)
(B) every 3 months (C) 
(C) every year (D)
(D) every 6 months
97 In which year did Bangladesh become a member of 
Commonwealth ? (B)
(A) 1972 (C)
(B) 1973 (D)
(C) 1974
(D) 1975
Which daily newspaper of the world has the highest (B)
circulation ? (C) 
(A) Times of India (D)
(B) New York Times
(C) Ashahi Shimbun
(D) Herald Tribune (B) 
Statue of peace is situated in which country ? (C)
(A) USA (D)
(B) UK 'Breaking a butterfly on the wheel'-
(C) Germany
(D) Japan

131
Collected By: Md. Raihan Ahmed
Job Information For All Students (Bcs & Bank)

 (C) 
(B) (D)
(C)
(D)
(B)
(C) 
(B) (D)
(C) 
(D) 
(B)
(C)
 (D)
(B)
(C) 
(D) (B)
(C)
 (D)
(B)
(C)
(D) (B) 
(C)
(D)
(B)
(C) 
(D) (B)
(C)
(D) 
(B)
(C)  
(D) (B)
(C)
(D)
(B) 'Lease'
(C) 
(D) (B)
(C) 
(D)
(B)
(C) 
(D)  (B) ;
(C)
(D)
(B)

132
Collected By: Md. Raihan Ahmed
Job Information For All Students (Bcs & Bank)

Question(31-35): Select a pair of words to replace the


(B)  question marks .
(C) 31. ESCAPE : ABSCOND :: ? : ?
(D) (A) Freedom : Independence
(B) Exult : Jubilate
(C) Weaken : Strengthen
(B) (D) Endless : Eternal
(C)  32. BURDEN : ENCUMBER :: ? : ?
(D) (A) Weariness : Tireless
(B) Synopsise : Recapitulate
(C) Reptile : Snake
Question(26-30): Select from the alternatives, the word (D) Behead : Summary
that conveys more or less the same meaning as the 33. ALLEVIATE : AGGRAVATE :: ? : ?
italicised word in the sentence. (A) Elastic : Rigid
26. No one welcome him to a party for he is so 'garrulous'. (B) Elevate : Agree
(A) talks too much (C) Joke : Worry
(B) too proud (D) Level : Grade
(C) to have ego 34. BENEVOLENT : KIND :: ? : ?
(D) behaves indecently (A) Imprudent : Reverential
27. He was 'engrossed' in days 's office routine. (B) Unclear : Muddy
(A) involved (C) Sagacity : Incautious
(B) tired (D) Redundant : Cheerful
(C) lost 35. FORTUITOUS : INHERENT :: ? : ?
(D) fully occupied (A) Rugged : Endurable
28. If you want real 'solace' go to Cox's Bazar (B) Legible : Indelible
(A) holiday (C) Envious : Desire
(B) happiness (D) Gregarious : Introverted
(C) consolation
(D) rest Question(36-40): Choose the one which best replaces the
italicised phrase
29. Bengalis are 'innate' poets. 36. ' If I would have' realised the nature of the job earlier, I
(A) excellent would not have accepted it.
(B) inborn (A) If I would
(C) dull (B) no improvement
(D) romantic (C) In case I would have
(D) Had I
30. The use of bullock carts is a 'superannuated' mode of 37. I would have waited for you at the station if 'I knew '
transportation. that you would come.
(A) cheaper (A) I known
(B) quicker (B) was Knowing
(C) obsolete (C) no improvement
(D) rural (D) had known

133
Collected By: Md. Raihan Ahmed
Job Information For All Students (Bcs & Bank)

38. The tea estate is 'in such a mess there' is no one to set (D) omnivorous
things right. 45. A place that provides refuge
(A) in such a mess that there (A) sanatorium
(B) in a such mess that there (B) asylum
(C) in a mess there (C) shelter
(D) no correction required (D) orphanage
39. As no one knows the truth 'as fully as him', no one but
him can provide the testimony. Question(46-50): Select the word or phrase that is most
(A) as fully as he, no one but him closely opposite in meaning to the capitalized word :
(B) as fully as his, no one but him 46. BURIED
(C) as fully as he does, not one but he (A) marked
(D) as fully as he does, no one but be alone (B) shown
40. Speculations and 'hypothesizing' are the most essential (C) obscured
and well-known aspects of inventions. (D) hidden
(A) hypothesizing needs 47. PREAMBLE
(B) Hypothesized (A) preface
(C) hypothesis (B) commencement
(D) no improvement (C) tender
(D) postscript
Question(41-45): Choose the one that can be substituted 48. PUNCTILIOUS
for the given words/phrase : (A) former
41. A general pardon granted by the government to political (B) observant
offenders (C) careless
(A) pardon (D) moderate
(B) excuse 49. SYCOPHANCY
(C) amnesty (A) advise candidly
(D) honesty (B) shout harshly
42. A speech delivered without preparation (C) push forcefully
(A) extempore (D) speak harmoniously
(B) straightforward 50. ACCRETION
(C) verbose (A) addition
(D) maiden (B) erosion
43. A person who tries to deceive people by claiming to be (C) procession
able to do wonderful things (D) stack
(A) impostor
(B) trickster MATHAMETICS
(C) magician What will be the compound interest on a sum of Tk.
(D) mountebank 25000 after 3 years at the rate of 12 p.c.p.a. ?
44. One who eats everything (A) Tk. 9000.30
(A) carnivorous (B) Tk. 9720
(B) gourmet (C) Tk. 10483.20
(C) omnipotent (D) Tk. 10123.20

134
Collected By: Md. Raihan Ahmed
Job Information For All Students (Bcs & Bank)

Solution: 5x + 2x = 70 × 5
Compound interest I = C P 7x = 350
n
= P[(1 + r) ] P x = 50
3
= 25000 [ (1 + 12/100) ] - 25000 So the shorter piece = 2 × 50/5 = 20 cm
= 10123.20 Tk. The product of two numbers is 4107. If the H.C.F. of
In a school, 10% of the boys are some in number as these numbers is 37, then the greater number is:
1/4th of the girls. What is the ratio of boys to girls in that (A) 101
school ? (B) 107
(A) 2:1 (C) 111
(B) 3:2 (D) 185
(C) 4:3 Solution:
(D) 5:2 The greater number is = 4107/37 = 111
Solution: What will be the ratio of simple interest earned by
Let number of boys in the school be x certain amount at the same rate of interest for 6 years and
And number of girls in the school be y that for 9 years ?
ATQ, (A) 1:3
10x/100 = y/4 (B) 1:4
x/y = 5/2 (C) 2:3
If a number is decreased by 4 and divided by 6, the (D) None of these
result is 8. What would be the result if 2 is subtracted from Solution:
the number and then it is divided by 5 ? Simple interest in 6 years = P × 6 × r
2
(A) 9 Simple interest in 9 years = P × 9 × r
3
(B) 10 P×6×r
Ratio of simple interest = =2:3
1 P×9×r
(C) 10
5 The average of six numbers is x and the average of
1
(D) 11 three is y. If the average of the remaining three is z, then :
5
Solution: (A) x = y+z
Let the number is x (B) x = 2y+2z
ATQ, (C) 2x = y+z
(x 4)/6 = 8 (D) none of these
xc= 52 Solution:
Now = (52 2)/5 = 10 Average of 6 number is x then total = 6x
A 70cm long wire is to be cut into two pieces such that Average of 3 number is y then total = 3y
one piece will be 2/5 as long as the other. How many Average of remaining 3 number is z then total = 3z
centimetres will the shorter piece be ? Atq,
(A) 10 6x = 3y + 3z
(B) 14 2x = y + z
(C) 20 In what ratio must a grocer mix two varieties of pulses
(D) 28 costing Tk. 15 and Tk. 20 per kg respectively so as to get a
Solution: mixture worth Tk.16.50 per kg ?
Let Bigger piece be x cm (A) 3:7
Then Shorter piece be 2x/5 cm (B) 5:7
ATQ, (C) 7:3
x + 2x/5 = 70 (D) 7:5

135
Collected By: Md. Raihan Ahmed
Job Information For All Students (Bcs & Bank)

Solution: (C) 1200


Let varities should be mixed in x : y ratio (D) 2000
ATQ, Solution:
15x + 20y = 16.50(x + y) 700 is divided by 4 bt not by 400
16.50x 15x = 20y 16.50y So 700 is not a leap year
1.5x = 3.5y The ratio of the father's age to his son's age is 7:3. The
x/y = 35/15 product of their ages is 756. The ratio of their ages after 6
x:y=7:3 years will be :
How long will a boy take to run round a square field of (A) 2:1
side 35 metres, if he runs at the rate of 9km/hr ? (B) 5:2
(A) 50 sec (C) 11:7
(B) 52 sec (D) 13:9
(C) 54 sec Solution:
(D) 56 sec
Solution: Atq,
Perimeter of the square = 4 × 35 = 140 metres 7x × 3x = 756
9 km/h = 9 × 5/18 = 5/2 m/s 21x2 = 756
Required time = 140 × 2/5 = 56 sec x2 = 36
The smallest 5-digit number exactly divisible by 41 is : x=6
(A) 10004 Ratio of their ages after year
(B) 10025 = (7x + 6)/(3x + 6)
(C) 10041 = 48/24
(D) 10045 =2:1
A,B,C hired a car for Tk. 520 and used it for 7,8 and 11 Two pipes A and B can fill a tank in 6 hours and 4
hours respectively. Hire charges paid by B were : hours respectively. If they are opened on alternate hours
(A) Tk. 140 and if pipe A is opened first, in how many hours, the tank
(B) Tk. 160 shall be full ?
(C) Tk. 180 (A) 4
(D) Tk. 220 (B) 4.5
Solution: (C) 5
Ratio of using = 7 : 8 : 11 (D) 5.5
Sum of ratio = 7 + 8 + 11 = 26 Solution:
So hire charges paid by B = 520 × 8/26 = 160 Tk A can fill in one hour 1/6 part
The square root of 64009 is : B can fill in one hour 1/4 part
(A) 253 (A + B) can fill in 2 hour = 1/6 + 1/4 = 5/12 part
(B) 347 In 4 hour they can fill ( 2 × 5/12 ) = 10/12 = 5/6 part
(C) 363 Remaing (1 - 5/6) = 1/6 part A will fill in 1 hour
(D) 803 Total time = 4 + 1 = 5 hour
Solution: In a camp, there is a meal for 120 men or 200 children.
253 × 253 = 64009 If 150 children have taken the meal, how many men will be
Which of the following is not a leap year ? catered to with the remaining meal ?
(A) 700 (A) 20
(B) 800 (B) 30

136
Collected By: Md. Raihan Ahmed
Job Information For All Students (Bcs & Bank)

(C) 40 5% of 40 is greater than 25% of a number by 2


(D) 50 6 - 0.25x = 2
Solution: 0.25x = 6 - 2
Meal remain for = (200 150 ) = 50 children 0.25x = 4
Now 200 children = 120 men x = 4 ÷ 0.25
50 children = (50 × 120)/200 = 30 men x = 16
7 18
1000) ÷ 10 =? A can finish a work in 18 days and B can do the same
(A) 10 work in half the time taken by A. Then, working together,
(B) 100 what part of the same work they can finish in a day ?
(C) 1000 (A) 2/5
(D) 10000 (B) 1/6
Solution: (C) 1/9
7 18
(1000) ÷ 10 (D) 2/7
1021 Solution:
=
1018 A's 1 day work = 1/18
= 103
B's 1 day work = 1/9 [because B take half time than A]
= 1000
(A+B)'s one day work =
In a 200 metres race A beats B by 35 m or 7 seconds.
(1/18 +1/9) = (1+2)/18 = 1/6
A's time over the course is :
So in one day 1/6 work will be done.
(A) 33 sec
At 3.40, the hour hand and the minute hand of a clock
(B) 40 sec
form an angle of :
(C) 47 sec
(A) 120 degree
(D) None of these
(B) 125 degree
Solution:
(C) 130 degree
B runs 35 m in 7 sec.
(D) 135 degree
B covers 200 m in (7/35 × 200) = 40 sec.
Solution:
B's time over the course = 40 sec.
Angle traced by hour hand in 12hrs = 360o
A's time over the course (40 - 7) sec = 33 sec.
Angle traced by it in 113hrs = (360/12 × 11/3)∘ = 110o
617 + 6.017 + 0.617 + 6.0017 = ?
Angle traced by min. hand in 60 min = 360o
(A) 6.2963
Angle traced by it in 40min. =(360/60 × 40)∘=240o
(B) 62.965
∴Requiredangle=(240−110)∘=130∘
(C) 629.6357
The length of a rectangular hall is 5m more than its
(D) None of these
breadth. The area of the hall is 750m^2. The length of the
If 15% of 40 is greater than 25% of a number by 2,
hall is :
then the number is :
(A) 15m
(A) 12
(B) 22.5m
(B) 16
(C) 25m
(C) 24
(D) 30m
(D) 32
Solution:
Solution:
Let breadth = x m
5% of 40 = 0.15 x 40 = 6
Then, length = (x+5)m
Let x be the number
Area of a rectangle = Length x Breadth
Find 25% of x:
x(x+5) = 750
25% of x = 0.25x

137
Collected By: Md. Raihan Ahmed
Job Information For All Students (Bcs & Bank)

x² + 5x - 750= 0 Which of the following is an important attribute of RAM


(x+30)(x-25)= 0 memory ?
x = 25 or x = -30 (A) Random access
Hence, breadth x = 25m (B) Volatile
Length = x + 5 = 25 + 5 = 30m. (C) Non-Volatile
When a commodity is sold for Tk. 34.80, there is a loss (D) Sequential access
of 2%. What is the cost price of the commodity ? The Internet is an example of -
(A) Tk. 26.10 (A) Packet switched network
(B) Tk. 43 (B) Circuit swithched network
(C) Tk. 43.20 (C) Cell switched network
(D) Tk. 46.40 (D) All of them
(E) Tk. 35.51 The primary job of the operating system is -
Solution: (A) Manage commands
Cost price = 34.80 × 100/98 = 35.51 Tk (B) Manage Users
What comes next in the series given? (C) Manage Resources
(D) Manage Programs
Which of following provides reliable communications ?
(A) (A) IP
(B) TCP
(B) (C) UDP
(D) All of them
(C) Data access time depends on -
(A) Seek time
(D) (B) Rotational delay
(C) Operating frequency
(D) All of them
Which of the following refers to the correctness and
What is the full form of RAID ? completeness of the data in a database ?
(A) Random Access of Independent Disks (A) Data integrity
(B) Redundant Array of Important Disks (B) Data security
(C) Redundant Array of Independent Disks (C) Data constraint
(D) Random Access of Important Disks (D) Data independence
Unix Operating System is a - Which memory allocation policy allocate the largest hole
(A) Multi User Operating System to the process ?
(B) Time Sharing Operating System (A) Best-Fit
(C) Multi-Tasking Operating System (B) First-Fit
(D) All of them (C) Worst-Fit
Which of the following is the most commonly used (D) None of them
HTTP methods ?
(A) PRE and POST GENERAL KNOWLEDGE
(B) GET and SET F.R. Khan was -
(C) GET and POST (A) A film director
(D) ASK and REPLY (B) A music director

138
Collected By: Md. Raihan Ahmed
Job Information For All Students (Bcs & Bank)

(C) An architect (C) Sibnarayan Ray


(D) A writer (D) A.N.A. Saha
G-15 is an economic grouping of - The ozone layer restricts -
(A) Third World Nations (A) Visible light
(B) Second World Nations (B) Ultraviolet radiation
(C) First World Nations (C) X-rays and gamma rays
(D) Fourth World Nations (D) Infrared radiation
First China War was fought between - Which of the following is the first private bank of
(A) China and France Bangladesh ?
(B) China and Britain (A) Arab-Bangladesh Bank
(C) China and Egypt (B) Prime Bank
(D) China and Greek (C) IFIC Bank
Durand Cup is associated with the game of - (D) Jamuna Bank
(A) Cricket Eritrea, which became the 182nd member of the UN, is
(C) Hockey in the continent of -
Football (A) Africa
(D) Volleyball (B) Asia
Where was Six-point programme declared ? (C) Europe
(A) Karachi (D) Australia
(B) Lahore Fathometer is used to measure -
(C) Dhaka (A) Earthquakes
(D) Chittagong (B) Rainfall
For which of the following disciplines is Nobel Prize (C) Sound intensity
awarded ? (D) Ocean depth
(A) Physiology and Medicine How many times has Brazil won the World Cup Football
(B) Physics and Chemistry Championship ?
(C) Literature, Peace and Economics (A) Once
(D) All of them (B) Twice
Under which sector was Mujibnagar during the war of (C) Four times
liberation of Bangladesh ? (D) Five times
(A) 2 Each year World Red Cross and Red Crescent Day is
(B) 8 celebrated on -
(C) 10 (A) May 8
(D) 11 (B) June 8
EACAP stands for - (C) May 18
(A) European Society Council for Africa and Pacific (D) June 18
(B) Economic and Social Commission for Asia and Pacific
(C) Economic and Social Commission for Africa and Pacific
(D) None of the above
Who is the designer of official Logo of Bangladesh BANGLA
Government ?
(A) Qumrul Hasan
(B) Major James Ranel 

139
Collected By: Md. Raihan Ahmed
Job Information For All Students (Bcs & Bank)

(C) (D)
(D) 'Killing two birds with one stone' -

, ,
, ,
(C) , ,  (C)
(D) , , (D) 

(C) 
(D)  (C)
(D)

(C) 
(D) (C) 
(D)

 
(C)
(D) (C)
(D)


(C) 
(D) (C)
(D)
'retrospective' -

(C) 
(D)  (C)
(D)
-

 
(C) (C)
(D) (D)
-
?

(C) 
(C)  (D)

140
Collected By: Md. Raihan Ahmed
Job Information For All Students (Bcs & Bank)

(C)
(D)

(C) 
(D) Select the word or phrase that is most
closely opposite in meaning to the capitalized word.
26. EXECRABEL
 (A) repulsive
(C) (B) unhappy
(D) (C) blamed
'depreciation' - (D) laudable
27. FRUGALITY
(A) thrift
(C)  (B) generosity
(D) (C) degradation
(D) providence
28. APPLAUD
(A) request
(C)  (B) criticise
(D) (C) pray
(D) flatter
29. TRIUMPH
 (A) excitement
(C) (B) failure
(D) (C) gain
(D) joy
30. ABERRATION
 (A) typical behaviour
(B) correct manners
(C) (C) straight aim
(D) (D) full truthfulness

Select from the options, the pair having a


similar relationship to the first pair.
31. PROMISE : FULFILL ::
(C)  (A) Pawn : Redeem
(D) (B) Pledge : Deny
(C) Law : Enforce
(D) Confession : Hedge
32. MANUMIT : ENSLAVE ::
(A) Large : Vivid
 (B) Gigantic : Massive
(C) Repel : Attract

141
Collected By: Md. Raihan Ahmed
Job Information For All Students (Bcs & Bank)

(D) Barren : Infertile (C) geologist


33. ERNEST : IMMORAL :: (D) seismologist
(A) Land : Evil
(B) Dissolute : Lascivious Choose the one that best expresses the
(C) Restrained : Wanton meaning of the given idiom/phrase in italics.
(D) Shore : Reef 41. An honest person never 'plays fast and loose' with his
34. ISOLATIONIST : ALOOF :: friends.
(A) Altruist : Selfish (A) ignores
(B) Mist : Mournful (B) deceives
(C) Scholar : Proud (C) to be inconsistent
(D) Bigot : Tolerant (D) delay tactics
35. PERMANENT : EVANESCENT :: 42. After the death of his father, Rahim is 'playing ducks
(A) Durable : Fleeting and drakes' with his inherited property and is sure to ruin
(B) Lasting : Glittering himself.
(C) Eternal : Everlasting (A) gambling
(D) Hairdo : Bleach (B) to spend lavishly
(C) horse trading
Choose the one that can be substituted (D) bad tacties to fool others
for the given words/sentences in the following questions. 43. It has been proved 'upto the hilt that' Rahman's
36. One who sacrifices his life for a cause intentions are bad .
(A) patriot (A) completely
(B) martyr (B) up to final decision
(C) soldier (C) final outcome
(D) revolutionary (D) none of these
37. A person interested in reading books and nothing else 44. She 'took into her head' to leave her job in no time and
(A) book-keeper go to her parents.
(B) scholar (A) half-heartedly decided
(C) bookworm (B) a sudden idea
(D) student (C) after careful thought
38. The murder of the king is called a-- (D) none of these
(A) homicide 45. After the retirement, Mr. Karim is thinking of 'resting on
(B) matricide his oars'.
(C) patricide (A) to depend on his savings
(D) regicide (B) rest after hard work
39. Taking disrespectfully of sacred things is called- (C) do a mild business to keep busy
(A) blasphemy (D) none of these
(B) heresy 46. Choose the correctly spelt word :
(C) atheism (A) DOLOROUS
(D) apostasy (B) DOLUROUS
40. A person who studies the formation of the earth (C) DOLOREUS
(A) meteorologist (D) DELORIOUS
(B) anthropologist 47. Choose the correctly spelt word :
(A) SEDATE

142
Collected By: Md. Raihan Ahmed
Job Information For All Students (Bcs & Bank)

(B) SEDOTE Or, n =12


(C) SCEDATE The sum of two numbers is equal to thrice difference. If
(D) SEEDATE the smaller of the numbers is 10 find the other number.
48. Choose the correctly spelt word : (A) 15
(A) SUGAST (B) 20
(B) SUJEST (C) 40
(C) SUGGEST (D) none of these
(D) SUGGAST Solution:
49. Choose the correctly spelt word : Let the larger number is = x and smaller number is = y
(A) CANINE Atq,
(B) CANNINNE x + y = 3(x y)
(C) CANNINE Or, 3x x = 3y + y
(D) CANINEE Or, 2x = 4y
50. Choose the correctly spelt word : Or, x = 40/2
(A) JOVIAL Or, x = 20
(B) JOVEAL The average age of 8 men is increased by 2 years when
(C) JOVIEL one of them whose age is 24 years is replaced by a woman.
(D) JOIVEL What is the age of the woman ?
(A) 35 years
MATHAMETICS (B) 28 years
If y = 5, then what is the value of 10y√𝒚𝟑 − 𝒚𝟐 (C) 32 years
(A) 50√2 (D) 40 years
(B) 500 Solution:
(C) 200√5 Let the sum of age of 8 men = 8x and the age of new
(D) 100 women = y years
Solution: According to the question,
Given y = 5 8x - 24 + y = 8 (x + 2)
Now, 8x - 24 + y = 8x + 16
10y√𝑦 3 − 𝑦 2 y = 16 + 24
= 10(5) √53 − 52 y = 40 years.
= 50√100 50% of a% of b is 75% of b% of C. Which of the
= 50 × 10 following is C ?
= 500 (A) 1.5b
If √𝟐𝒏 = 64 then the value of n is : (B) 0.667a
(A) 12 (C) 0.5a
(B) 6 (D) 1.25a
(C) 4 Solution:
(D) 2 (50/100) × (a/100) × b = (75/100)× (b/100)×c
Solution: ab/(2×100) = 3bc/(4×100)
Given √2𝑛 = 64 c=( ab × 4×100)/(2×100× 3b)
𝑛
Or, 2 = 26
2 c= 2a/3
c= 0.6666a
Or, n/2 = 6
c= 0.667a

143
Collected By: Md. Raihan Ahmed
Job Information For All Students (Bcs & Bank)

A makes an article for Tk. 120 and sells it to B at a 3.14 × 106 = 3.14 × 1000000 = 3140000
profit of 25% B sells it to C who sells it for Tk. 198 making If x = 1-q and y = 2q+1, then for what value of q, x is
a profit of 10%. What profit percent did B make ? equal to y ?
(A) 25% (A) 0
(B) 20% (B) -1
(C) 16.55% (C) 0.5
(D) 15% (D) 2
Solution: Solution:
A's sp= 125% of 120=150 Given x = 1-q and y = 2q+1
C sell it 198 tk which make 10% profit If x = y
So, C's cost= 110% = 198, Then, 1- q = 2q + 1
100% = 180 Or, 3q = 0
Finally B purchased it for 150 and sell it to C 180tk So, Or, q = 0
fit percent = 20% 3 years ago, the average age of a family of 5 members
A sum of Tk. 600 amounts to Tk. 720 in 4 years at was 17 years. A baby having been born the average age of
Simple Interest. What will it amount to if the rate of the family is the same today. The present age of the baby is
interest is increased by 2% ? (A) 1 year
(A) Tk.648 (B) 1 year 6 months
(B) Tk.768 (C) 2 years
(C) Tk. 726 (D) 3 years
(D) Tk. 792 Solution:
Solution: Let age of the baby is x.
Interest = Amount Principal = 720 600 = 120 Tk. 3 years ago total age of the family = 5 *17 = 85 years.
Simple Interest = P*N*R/100 Total age of the 5 member at present time,
120 = 600 * 4 * R/100 = 24R = 85 + 3*5 = 100 years
R = 5% p.a. Total age of the family at present time including baby,
New Interest = 600 *7 * 4/100 = 168 = 100 + X.
Amount = Interest + Principal = 600 + 168 = Tk. 768 The average of the family including baby at present time,
The sum of first five prime numbers is : = 17 years.
(A) 11 (100 +X)/6 = 17
(B) 18 100 +X = 102
(C) 26 X = 102 - 100 = 2 years.
(D) 28 There are two numbers such that the sum of twice the
Solution: first and thrice the second is 39. While the sum of thrice
First 5 prime numbers are 2, 3, 5, 7, 11 first and twice the second is 36. The larger of the two is :
Sum = 2 + 3 + 5 + 7 + 11 = 28 (A) 6
6
Which is the following is equal to 3.14 × 10 ? (B) 8
(A) 314 (C) 9
(B) 3140 (D) 12
(C) 3140000 Solution:
(D) none of these Let the first digit be X & second be Y.
Solution: 2x+3y=3

144
Collected By: Md. Raihan Ahmed
Job Information For All Students (Bcs & Bank)

From (i) & (ii) (C) Tk. 72


X=6,y=9 (D) Tk. 88.25
Larger of the two is 9 Solution:
A person was asked to state his age in years. His reply C.P. = Tk. (100/122.5 x 392 ) = Tk. (1000/1225 x 392)
was, 'Take my age three years hence, multiply it by 3 and = Tk. 320
then subtract three times my age three years ago and you Profit = Tk. (392 - 320) = Tk. 72.
will know how old I am' What was the age of the person ? The least whole number which when subtracted from
(A) 18 years both the terms of the ratio 6 : 7 gives a ratio less than
(B) 20 years 16:21 is :
(C) 24 years (A) 2
(D) 32 years (B) 3
Solution: (C) 4
Let The Present age be x years. (D) 6
Then, Solution:
3(x + 3) -3(x - 3) = x Let the whole number is X.
⇒ 3x +9 -3x + 9 = x Now, according to question,
⇒ x = 18 (6 -X) /(7 -X) < 16/21.
In an election, 30% of the voters voted for candidate A 21 *(6 -X) < 16 *(7 -X)
whereas 60% of the remaining voted for candidate B. The 126 - 21X < 112 - 16X
remaining voters did not vote. If the difference between 126 - 112 < - 16X + 21X
those who voted for candidate A and those who did not 14 < 5X
vote was 1200, how many individuals were eligible for 5X > 14
casting vote in that election ? X > 2.8
(A) 10,000 So, Least such whole number would be 3.
(B) 45,000 10 man, working 6 hours a day can complete a work in
(C) 60,000 18 days. How many hours a day must 15 men work to
(D) 72,000 complete the same work in 12 days ?
Solution: (A) 6
Let the number of persons eligible to vote be x (B) 10
Then, votes who voted for A = 30% of x (C) 12
Votes who voted for B = 60% of (70% of x) (D) 15
= (60/100 × 70/100 × 100) % of x Solution:
= 42% of x More Men, Less Hours {Indirect Proportion}
Voters who did not vote Less Days, More Hours {Indirect Proportion}
= [100 - (30 + 42)]% of x = 28% of x Men 15 : 10 ⋮⋮ 6 : x
∴ 30% of x - 28% of x = 1200 days 12 : 18 ⋮⋮ 6 : x
⇒ 2% of x = 1200 ⇒ 15 * 12 * x = 10 * 18 * 6
⇒ x = (1200 × 100/2) ⇒x = 6
⇒ x = 60000 Two taps A and B can fill a tank in 5 hours and 20
A shopkeeper expects a gain of 22.5% on his cost price. hours respectively. If both the taps are open then due to a
If in a week, his sale was of Tk. 392, what was his profit ? leakage, it took 30 minutes more to fill the tank. If the tank
(A) Tk. 18.20 is full, how long will it take for the leakage alone to empty
(B) Tk. 70 the tank ?

145
Collected By: Md. Raihan Ahmed
Job Information For All Students (Bcs & Bank)

(A) 4.5 hrs Solution:


(B) 7 hrs Let original length = x and original breadth = y.
(C) 18 hrs Decrease in area = xy - ( 80x/100 × 90y/100 ) = (7/25)xy
(D) 36 hrs Decrease % = (7/25)xy × (1/xy) × 100 = 28%
Solution:
Two taps A and B can fill a tank in 5 hours and 20 hours COMPUTER
respectively. Which of the following is not a class of computers
Tap A fills in 1 hr = 1/5 based on size ?
Tap B fills in 1 hr = 1/20 (A) Mainframe Computers
Tap A & B together fill in 1hr = 1/5 + 1/20 = (4 + 1)/20 = (B) Mini Computers
1/4 (C) Micro Computers
Tap A & B together fill tank in = 1/(1/4) = 4 Hr (D) Super Computers
Let say leakage can empty tank alone in = L hrs CD-ROM stands for
Leakage in 1 hr = 1/L (A) Compactable Read Only Memory
Tap A + Tap B + Leakage in 1 hr = 1/4 - 1/L (B) Compact Data Read Only Memory
Tap A + Tap B + Leakage fill tank in = 4 hr + 30 minutes = (C) Compactable Disk Read Only Memory
9/2 hrs (D) Compact Disk Read Only Memory
Tap A + Tap B + Leakage fill tank in 1 hr = 2/9 Who invented the microprocessor ?
1/4 - 1/L = 2/9 (A) Marcian E Huff
⇒ 9 - 36/L = 8 (B) Herman H Goldstein
⇒ 1 = 36/L (C) Jeseph Jacquard
⇒ L = 36 (D) ALL
A train can travel 50% faster than a car. Both start lost Easily relocatable language is
about 12.5 minutes while stopping at the stations. The (A) Machine Language
speed of the car is : (B) Assembly Language
(A) 100 kmph (C) High Level Language
(B) 110 kmph (D) Medium Level Language
(C) 120 kmph Which is the limitation of high level language ?
(D) 130 kmph (A) Lower efficiency
Solution: (B) Machine dependence
Let speed of the car be x kmph. (C) Machine level coding
Then, speed of the train = 150x/100 = (3x/2) kmph. (D) none of these
75/x - 75/(3x/2) = 125/(10 x 60) A characteristic of card systems is :
75/x - 50/x = 5/24 (A) Slowness in processing data
x = (25 x 24)/5 (B) Using cards as records of transactions
x = 120 kmph. (C) Needing a larger DP staff
A towel, when bleached, was found to have lost 20% of (D) All of them
its length and 10% of its breadth. The percentage of Which of the following memories need refresh ?
decrease in area is : (A) SRAM
(A) 10% (B) DRAM
(B) 10.08% (C) ROM
(C) 20% (D) All
(D) 28% An advantage of the database management approach is

146
Collected By: Md. Raihan Ahmed
Job Information For All Students (Bcs & Bank)

(A) data is dependent on programs University -


(B) data redundancy increases (A) Dr. A.R. Mallik
(C) data is integrated and can be accessed by multiple (B) Professor Abdul Fazal
programs (C) Dr. M Innas Ali
(D) None of these (D) Dr. Abdul Karim
Data is independence means The Faraizi Movement was launched by -
(A) data is defined separately and not included in programe (A) Mohammad Ali
(B) programs are not dependent of the physical attributes (B) Haji Shariatullah
of data (C) Syed Amir Ali
(C) programs are not dependent on the logical attributes of (D) Dudu Mia
data The constitution Drafting Committee formed in 1972
(D) both b and c had -
Key to represent relationship between tables is called (A) 31 members
(A) primary key (B) 32 members
(B) secondary key (C) 33 members
(C) foreign key (D) 34 members
(D) none of these Metamorphoses was written by
(A) Ovid
GENERAL KNOWLEDGE (B) Aesop
Who is the highest wicket-taking bowler for Bangladesh (C) Virgil
in ODI cricket ? (D) Homer
(A) Mashrafe Mortoza The capital city of Albania is
(B) Shakib Al Hasan (A) Astana
(C) Abdur Razzak (B) Tirana
(D) None of them (C) Almaty
When did Bangladesh first participate in the UN Peace- (D) Bukhara
keeping Mission ? Who was the oldest person to sign the US Declaration
(A) 1987 of Independence ?
(B) 1989 (A) Abraham Lincoln
(C) 1988 (B) Benjamin Franklin
(D) 1990 (C) George Washington
The name of the first post-liberation war sculpture of (D) Thomas Jefferson
Bangladesh is - The Nigerian currency is
(A) Aparajeyo Bangla (A) Shilling
(B) Jagroto Chowrongi (B) Dirham
(C) Shabash Bangladesh (C) Naira
(D) None of these (D) Lira
In Bangali Literature, Binoy Mukherjee is known as - The French Revolution began in which year ?
(A) Jajabor (A) 1957
(B) Nillohit (B) 1759
(C) Parshuram (C) 1789
(D) Shanoto Pathok (D) 1776
The name of the first Vice-Chancellor of Chittagong The Statue of Liberty was a gift to the United states of

147
Collected By: Md. Raihan Ahmed
Job Information For All Students (Bcs & Bank)

America from which country ?


(A) France
(B) Germany
(C) Japan BANGLA
(D) United Kingdom 1. ' ' ?
The smallest ocean in the world is (A) 
(A) Arctic (B)
(B) Antarctic (C)
(C) Mariana (D)
(D) Indian Ocean 2. ?
The national flower of USA is (A)
(A) rose (B)
(B) sunflower (C) 
(C) tulip (D)
(D) daffodil 3. ?
In Greek mythology, Nike is the goddess of (A)
(A) health (B) 
(B) speed (C)
(C) victory (D)
(D) wealth 4. ?
Magyr is the language of (A) 
(A) Macedonia (B)
(B) Hungary (C)
(C) Romania (D)
(D) Monaco 5. - ?
When were women first allowed to compete in the (A)
Olympics ? (B)
(A) 1900 (C) 
(B) 1904 (D)
(C) 1908 6. - ?
(D) 1912 (A)
Which country was suspended from the Arab League for (B)
10 years from 1979 ? (C)
(A) Egypt (D) 
(B) Iraq 7. ?
(C) Jordan (A) 
(D) Syria (B)
Where was Jibanananda Das Born ? (C)
(A) Calcutta (D)
(B) Mehinipur 8. - ' ' ?
(C) Barisal (A)
(D) Faridpur (B) 

148
Collected By: Md. Raihan Ahmed
Job Information For All Students (Bcs & Bank)

(C) (D)
(D) 17. ' '- ?
9. - ? (A)
(A) (B)
(B) (C)
(C)  (D) 
(D) 18. ' - '- ?
10. ' ' ? (A)
(A) (B) 
(B)  (C)
(C) (D)
(D) 19. ' , , '- ' '
11. ' ' ? ?
(A) (A)
(B) (B) 
(C) (C)
(D)  (D)
12. ' ' ? 20. :
(A) (A)
(B) (B)
(C)  (C) 
(D) (D)
13. ? 21. ?
(A) (A)
(B) (B)
(C) (C) 
(D)  (D)
14. ' ' ? 22. ?
(A) (A) 
(B) (B)
(C) (C)
(D)  (D) .
15. ? 23. ?
(A) (A)
(B) (B)
(C)  (C)
(D) (D) 
16. ' ' ' '
? 24. ' ' ?
(A) (A)
(B) (B)
(C)  (C) 

149
Collected By: Md. Raihan Ahmed
Job Information For All Students (Bcs & Bank)

(D) (D) centenary


32. A person interested in reading books and nothing else .
25. ? (A) book-keeper
(A) (B) bookworm
(B) (C) scholar
(C)  (D) student
(D) 33. A person who studies the formation of the earth .
(A) geologist
ENGLISH (B) anthropologist
Question(26-30): Choose the word which is most nearly (C) meteorologist
opposite in meaning to word in capital letters. (D) seismologist
26. VIRTUE 34. A tank in which fish are kept is called an -
(A) Untrue (A) aquacade
(B) Defeat (B) aquarium
(C) dVice (C) aquatint
(D) Fool (D) aqua tone
27. ASSENT 35. One who sacrifices his life for a cause -
(A) Separation (A) patroit
(B) Self-interest (B) revolutionary
(C) Dissent (C) soldier
(D) Confusion (D) martyr
28. RELENTLESS
(A) kind Question(36-40): Choose the word that conveys the same
(B) Yielding meaning as the word given in capital letters
(C) Gentle 36. OPULENT
(D) Sensitive (A) hard-working
29. EVASIVE (B) rich
(A) Unclear (C) obscure
(B) Explicit (D) comfortable
(C) Categorical 37. TRANSFORM
(D) Indefinite (A) transfer
30. HOSTILITY (B) change
(A) Partnership (C) continue
(B) Friendship (D) transact
(C) Enmity 38. RECAPITULATION
(D) Relationship (A) introduction
(B) recall
Question(31-35): Choose the one that can be substituted (C) withdrawal
for the given word/phrase. (D) capture
31. Yearly celebration of a date or an event . 39. COMPLACENT
(A) anniversary (A) recommend
(B) birthday (B) satisfied
(C) jubilee (C) witty

150
Collected By: Md. Raihan Ahmed
Job Information For All Students (Bcs & Bank)

(D) confuse (A) are


40. VISCERAL (B) have been
(A) cloudy (C) are still
(B) heavenly (D) will be
(C) bodily 49. The stranger _____ in through the window last night .
(D) intelligent (A) creeping
41. Choose the correctly spelt word: (B) crept
(A) vaccum (C) creeps
(B) vacum (D) creep
(C) vacuum 50. Do not look _____ on women.
(D) vaccuum (A) right
42. Choose the correctly spelt word: (B) left
(A) Sanquin (C) up
(B) Sanguin (D) down
(C) Sanguine
(D) Sankuine MATHEMATICS
43. Choose the correctly spelt word: 51. The ratio of boys and girls in a club is 3 : 2 . Which of
(A) Miscellaneous the following could be the actual number of members ?
(B) Miscellanous (A) 16
(C) Misellaneous (B) 18
(D) Miscelaneous (C) 25
44. Choose the correctly spelt word: (D) 24
(A) orthopeadic Solution:
(B) orthopedic Sum of ratio = 3+2 = 5
(C) orthopadic Only 25 can be divisible by 5.
(D) orthopaedic 52. 30% apples out of 450 are rotten . How many apples
45. Choose the correctly spelt word: are in good condition?
(A) Brodcaster (A) 125
(B) Broodcaster (B) 315
(C) Boardcaster (C) 240
(D) Broadcaster (D) 180
46. I _____ him on this point. Solution:
(A) admitted to apples in good condition are,
(B) agreed with = (450 30% of 450) = 450 135 = 315
(C) agree to 53. The present age of son is half of the present age of his
(D) agreeing with mother. Ten years ago, his mother's age was thrice the age
47. The specialist _____ you want to meet is available only of her son . What is the present age of the son?
on Thursday . (A) 25 years
(A) whose (B) 20 years
(B) who (C) 30 years
(C) whom (D) 40 years
(D) which Solution:
48. They _____ living here since 2000. M= x

151
Collected By: Md. Raihan Ahmed
Job Information For All Students (Bcs & Bank)

Son= x/2 And, c= 46-10= 36


10 years ago Now, (b+c)-a= (46+36)-48= 82-48= 34
M= x-10 56. The least number which should be added 2497 so that
S= x/2-10= (x-20)/2 the sum is exactly divisible 5,6,4 and 3 is -
3(x-20)/2= x-10 (A) 23
Or, 3x-60= 2x-20 (B) 13
Or, x= 40 (C) 3
So, present age of son= 40/2= 20 (D) 33
54. The average of two numbers is 6.5 and square root of Solution:
their product is 6 . What are the number? LCM of 5,6,4,3= 60
(A) 11 and 2 2497/60= Remainder 37
(B) 8 and 5 The least number =(60-37)= 23
(C) 10 and 3 57. The face value of 8 in the number 458926 is -
(D) 9 and 4 (A) 8000
Solution: (B) 1000
Avg. Of two number x & y= 6.5 (C) 8
Sum of x+y= 6.5×2= 13 (1) (D) 8926
ATQ, Solution:
√x√y= 6 The value of a digit depends on its place, or position, in the
Or, xy= 36 number.
Or, x= 36/y The face value is the actual value of the digit, i.e, the face
From (1) value of 5 is 5 and 0 is 0 and so on. In this case, the face
36/y+y= 13 value of 8 is 8.
2
Or, 36+y -13y= 0 58. The total cost of flooring a room at Tk. 8.50 per square
2
Or, y -13y+36= 0 metre is TK. 510. If the length the room is 8 m , its breadth
2
Or, y -9y-4y+36=0 is -
Or, y(y-9)-4(y-9)= 0 (A) 8.5 m
Or, y= 4 And 9 (B) 7.5 m
55. a is greater than b by 2 and b is greater than c by 10 . (C) 10.5 m
If a + b+ c = 130 , then (b+c) a=? (D) 12.5 m
(A) 42 Solution:
(B) 38 8.50 tk cost 1 square
(C) 34 So, 510 tk. Cost = 510/8.50 = 60 square
(D) 44 Let, breadth = x
Solution: So, 8x = 60
a= b+2 Or, x = 7.5
b= c+10 59. A can do 3 piece of work in 4 hours, B and C together
c= b-10 in 3 hours , and A and C together in 2 hours. How long will
a+b+c= 130 B alone take to do it?
Or, b+2+b+b-10= 130 (A) 8 hours
Or, 3b= 138 (B) 12 hours
Or, b= 46 (C) 10hours
So, a= 46+2= 48 (D) 24 hours

152
Collected By: Md. Raihan Ahmed
Job Information For All Students (Bcs & Bank)

Solution: (B) 37


1/A +1/B + 1/C = 1/4 + 1/3 = 7/12 (C) 28
1/A + 1/C = 1/2 (D) 51
Or, 1/C = 1/2 - 1/4= 1/2 Solution:
So, Let, numbers are a & b.
1/B = 7/12 - 1/2 = (7 - 6)/12 = 1/12 Now,
Time taken = 12 hrs (a+b)2 = (a-b)2 + 4ab
Or, (a+b)2 = 52 + (4×336)
60. A speed of 14 meters per second is the same as- Or, (a+b)2 = 25 + 1344
(A) 50 km/hr Or, (a+b)2 = 1369
(B) 46.6 km/hr Or, a+b = 37
(C) 28 km/hr
65. Evaluate √𝟐𝟒𝟖 + √𝟓𝟐 + √𝟏𝟒𝟒
(D) 70 km/hr
(A) 20
Solution:
(B) 16
14 × (18/5) = 252/5 = 50.4
(C) 24
61. Oranges are bought at 5 for Tk. 10 and sold at 6 for Tk.
(D) 30
15 . The gain percent is -
Solution:
(A) 50%
(B) 40% Now √248 + √52 + √144

(C) 25% = √248 + √52 + 12

(D) 35% = √248 + √64


Solution: = √248 + 8
CP = 10/2 = 5 = √256
SP = 15/6 = 2.5 = 16
Gain = (2.5 - 2) = 0.5 66. (0. 75 × 4. 4 × 2. 4) ÷ 0. 6 =?
Gain % = (0.5 × 100)/2 = 25% (A) 4.752
62. By selling an article for Tk. 100 a man gains Tk. 15 (B) 12
.Then, his gain % is - (C) 15.84
(A) 15% (D) 13.2
2
(B) 12 % Solution:
3
(C) 17 %
1 (0.75 × 4.4 × 2.4) ÷ 0.6
4
11 = 7.92 ÷ 0.6
(D) 17 %
17
= 13.2
Solution:
67. 48.95 - 32.006 =?
the cost price = 100 15 = 85 tk.
11 (A) 16.089
gain % = (15 × 100)/85 = 17 %
17
(B) 16.35
63. (0. 04)2 ÷ ((0. 008) × (0. 2)6) = (0. 2)?
(C) 16.944
(A) 5
(D) 16.89
(B) 6
68. P and Q started a business in the ratio of 2:3 . After 1
(C) 8
year P left the business but Q continues. After 2 years he
(D) None of these
had the profit of TK. 26000. What is the profit of Q?
64. Two numbers differ by 5. If their product is 336, then
(A) Tk. 10400
the sum of the two numbers is -
(B) Tk. 1300
(A) 21

153
Collected By: Md. Raihan Ahmed
Job Information For All Students (Bcs & Bank)

(C) Tk. 15600 (B) 40%


(D) None of these (C) 80%
Solution: (D) 12.5%
P : Q = (2 × 1) : (3 × 2) = 2 : 6 = 1 : 3 Solution:
5 × 100/4 = 125%
69. How much time will it take for an amount of Tk 450 to 73. The average of 6 numbers is 7. The average of three
yield Tk. 81 as interest at 4.5% per annum of simple numbers of them is 5. What will be the average of
interest? remaining numbers?
(A) 3.5 years (A) 15
(B) 4.5 years (B) 30
(C) 4 years (C) 42
(D) 5 years (D) 9
Solution: Solution:
ATQ, Sum of 6 numbers is = 6 × 7 = 42
(450 × 4.5 × n)/100 = 81 Sum of 3 numbers is = 3 × 5 = 15
Or, 450 × 4.5 × n = 8100 So, the average of remaining numbers
Or, 2025n = 8100 = (42 - 15)/3 = 27/3=9
Or, n = 4 year 74. In a group of 52 persons , 16 drink tea but not coffee
70. A Person crosses a 600 m long street in 5 minutes. and 33 drink tea. How many drink coffee but not tea?
What is his speed in km per hour? (A) 3
(A) 7.2 (B) 7
(B) 3.6 (C) 19
(C) 8.4 (D) 17
(D) 10 Solution:
Solution: There are total 52 people out of which 33 drink tea.
5 minutes need to cross 600 m Let, people drink coffee but not tea = x
1 minutes need to cross 600/5 m So,
60 minutes need to cross (600/5) ×60 m = 7200m= 7.2 km 33 + x = 52
71. If 8 men can reap 801 hectares in 24 days, then dhow Or, x = 19
many hectares can 36 men reap in 30 days? 75. The least prime number is .
(A) 350 (A) 0
(B) 400 (B) 1
(C) 450 (C) 3
(D) 425 (D) 2
Solution:
8 men need 24 days to reap 80 hectares GENERAL KNOWLEDGE & COMPUTER
1 men need 1 days to reap 80/(24 × 8) hectares 76. Who was the main singer of the ' Concert for
36 men need 30 days to reap Bangladesh' in 1971 ?
= (80 × 36 × 30/(24 × 8) hectares = 450 hectares (A) Runa Laila
72. The ratio 5 :4 expressed as a percent equals (B) George Harrison
(A) 125% (C) Mark Anthony
(D) Bappi Lahiri
77. A Groupware is a -

154
Collected By: Md. Raihan Ahmed
Job Information For All Students (Bcs & Bank)

(A) Hardware been appointed as the new Indian Ambassador to USA-


(B) Software (A) Pinak RanJan Chakravarty
(C) Network (B) Riva anguly Das
(D) Foilmware (C) Harsha Varshan Shringla
78. As per World Bank criteria, Bangladesh has been (D) None of them
graduated to the lower-middle income status in the year- 86. What is the global rank of Bangladesh in terms of
(A) 2013 population ?
(B) 2015 (A) 7th
(C) 2014 (B) 9th
(D) 2016 (C) 8th
79. 'RAM' stands for- (D) 10 th
(A) Read access Memory 87. During the War of Liberation, Barishal was under sector
(B) Random Access Memory number -
(C) Read Arithmetic Memory (A) 1
(D) Random Arithmetic Memory (B) 6
80. Which district of Bangladesh was a part of Assam? (C) 9
(A) Chittagong (D) 8
(B) Khulna 88. which of these organizations has been awarded the
(C) Feni Nobel Prize three times?
(D) Sylhet (A) International Committee of Red Cross
81. According to the Article 48 of Constitution of (B) Amnesty International
Bangladesh, what should be the minimum age of the (C) Transparency International
president? (D) United Nations Organization
(A) 30 89. Antivirus software is a type of -
(B) 32 (A) System software
(C) 40 (B) Utility Software
(D) 35 (C) Application Software
82. Which river of Bangladesh has its origin in Tibet ? (D) Publishing
(A) Padma 90. The currency of Japan is known as -
(B) Brahmaputra (A) Yen
(C) Surma (B) Dollar
(D) Meghna (C) Renminbi
83. Central Processing Unit (CUP) consists of - (D) Ringgit
(A) control unit 91. Who has been awarded posthumous 'Ekushey Padak' in
(B) arithmetic & logic unit 2018 ?
(C) main store (A) Humayun Ahmed
(D) all of the above (B) Humayun Faridi
84. Who was the promoter of Chinese 'One Road ' initiative? (C) Humayun Kabir
(A) Jing Zemin (D) Humayun Azad
(B) Hanyu Pinyin 92. The Procedure for starting or re-starting a computer is
(C) Sinzo Abe called-
(D) Xi Jinping (A) Warm Boot
85. The High Commissioner of India Bangladesh who has (B) cold Boot

155
Collected By: Md. Raihan Ahmed
Job Information For All Students (Bcs & Bank)

(C) Booting (A) Vivian Richards


(D) Loading of Windows (B) AB De Villiers
93. What is the name of the Central Bank of India? (C) Shahid Afridi
(A) Bank of India (D) Corey Anderson
(B) Reserve Bank of India
(C) Central Bank of India
(D) None of these
94. Which of the following is the deepest river in the world
? BANGLA
(A) Volga 1. ?
(B) Nile (A) 
(C) Congo (B)
(D) Amazon (C)
95. General A.K Niazi surrendered at - (D)
(A) Suhrawardy Udyan 2. -
(B) Paltan Maidan (A)
(C) Osmani Udyan (B) 
(D) Lalbagh (C)
96. Who was the Finance Minister of Mujibnagar (D)
Government ? 3. Guilty -
(A) Professor Yousur ALi (A)
(B) AHM Kamaruzzaman (B) 
(C) Captain M Mansur Ali (C)
(D) Tajuddin Ahmad (D)
97. Two valiant freedom fighters who have been awarded 4. ?
'Bir Protik' title- (A)
(A) Sultana Kabir & Salma Khan (B) 
(B) Anzuman Ara & Kaniz Fatema (C)
(C) Dr. Setara Begum & Taramon Bibi (D)
(D) None of them 5. -
98. A megabyte is actually equal to ___ kilobytes. (A) 
(A) 100 (B)
(B) 1000 (C)
(C) 1028 (D)
(D) 1024 6. ?
99. Which of the following Indian states has a boundary (A)
with Myanmar as well as Bangladesh ? (B) 
(A) Manipur (C)
(B) Tripura (D)
(C) Mizaram 7. ?
(D) Nagaland (A) -
100. Which cricketer has scored fastest century in ODI (B) -
cricket?

156
Collected By: Md. Raihan Ahmed
Job Information For All Students (Bcs & Bank)

(C) -  (D)
(D) - 16. - ?
8. ? (A) -
(A)  (B) -
(B) - (C) - 
(C) (D) -
(D) - 17. ?
9. ? (A) 
(A) (B)
(B)  (C)
(C) (D)
(D) 18. ?
- (A)
? (B) 
(A) (C)
(B)  (D)
(C) 19. ?
(D) (A) -
11. ? (B) -
(A) (C) - 
(B) (D) -
(C)  20. ?
(D) (A)
12. ? (B)
(A) (C) 
(B) (D)
(C)  21. ?
(D) (A)
13. ? (B) 
(A) (C)
(B) (D)
(C) 22. ?
(D)  (A) 
14. ? (B)
(A) (C)
(B) (D)
(C)  23. - ?
(D) (A) 
? (B)
(A) (C)
(B)  (D)
(C) 24. ?

157
Collected By: Md. Raihan Ahmed
Job Information For All Students (Bcs & Bank)

(A) 32. In each set a word has been miss pelt . Find the miss
(B)  pelt word.
(C) (A) FEASIBEL
(D) (B) FASCIEBLE
25. ? (C) FEASIBLE
(A) (D) FEASEBLE
(B) 33. In each set a word has been miss pelt . Find the miss
(C) pelt word.
(D)  (A) HAABITS
(B) HABITTS
ENGLISH (C) HABBTS
26. A good judge never jumps _____ the conclusion. (D) HABITS
(A) to 34. In each set a word has been miss pelt . Find the miss
(B) on pelt word.
(C) at (A) NAVIJATION
(D) for (B) NAVIGATION
27. The mother was anxious ____ the safety of her son. (C) NAVITATION
(A) about (D) NEVIGATION
(B) on 35. In each set a word has been miss pelt . Find the miss
(C) for pelt word.
(D) upon (A) SUGGEST
28. Give an example pertinent _____ the case. (B) SUBJECT
(A) with (C) SUGAST
(B) on (D) SUGGAST
(C) to
(D) for Question(36-40): Choose the word that conveys the same
29. It is not always easy to sympathies _____ an unfortunate meaning as the word given in capital letters
man 36. FLUX
(A) for (A) overflow
(B) with (B) merge
(C) by (C) change
(D) at (D) soften
30. It is dangerous to intrude ____ the enemy's camp. 37. COPIOUS
(A) in (A) liberal
(B) on (B) heavy
(C) into (C) abundant
(D) through (D) broad
31. In each set a word has been miss pelt . Find the miss 38. KNOTTY
pelt word. (A) terrible
(A) PERIMETER (B) difficult
(B) PARMETER (C) confusing
(C) PERIMOROR (D) mysterious
(D) PARIMETRE 39. GLIB

158
Collected By: Md. Raihan Ahmed
Job Information For All Students (Bcs & Bank)

(A) artful (C) adolesence


(B) persuasive (D) old age
(C) flattering 47. One who cats everything
(D) informal (A) omnivorous
40. CURB (B) herbivorous
(A) medicinal (C) inserctivorous
(B) restriction (D) carnivorous
(C) participation 48. One who conceals his idnetity as a writer under an
(D) hunger assumed pen-name
(A) pompous
Question(41-45): Choose the word which is most nearly (B) plagiarist
opposite in meaning to word in capital letters. (C) philistine
41. AUTONOMOUS (D) pseudonym
(A) absolute 49. AN organ of body cut off by surgery
(B) separate (A) imitatin
(C) dependent (B) amputation
(D) self-directed (C) adaptation
42. ABYSMAL (D) impartial
(A) etrnal 50. Person who looks at the dark side of everything
(B) beneath (A) optimist
(C) immeasurable (B) asealist
(D) super (C) Pessimist
43. FERAL (D) Naturalism
(A) grim
(B) tame MATHEMATICS
(C) unyieding 51. √√𝟏𝟕𝟗𝟓𝟔 + √𝟐𝟒𝟎𝟐𝟓 = ?
(D) natural (A) 19
44. ABSTINENCE (B) 155
(A) afraid (C) 17
(B) self-denial (D) 256
(C) indulgence Solution:
(D) devotion Now √√17956 + √24025
45. INTIMATE = √134 + 155
(A) declare = √289
(B) distance = 17
(C) close 52. Which of the following fraction is the largest?
(D) release (A) 13/16
(B) 7/8
Question(46-50): Choose the one that can be substituted (C) 31/40
for the given word/phrase. (D) 63/80
46. Stage of growth between boyhood and youth 53. The sum of three consecutive multiples of 3 is 72. What
(A) infancy is the largest number?
(B) teenager (A) 21

159
Collected By: Md. Raihan Ahmed
Job Information For All Students (Bcs & Bank)

(B) 24 = 48/60 × 1/5


(C) 36 = 4/25
(D) 27 = 0.16
Solution: 58. A boat goes 8 km in one hour along the stream and 2
Let the numbers be 3x, 3x + 3 and 3x + 6. km in one hour agaist the stream. The speed In km/hr of
Then, the stream is
3x + (3x + 3) + (3x + 6) = 72 (A) 2
9x = 63 (B) 4
x=7 (C) 3
Largest number = 3x + 6 = 27. (D) 5
3 2 3 ?
54. 9 × (81) ÷ (27) = (3) Solution:
(A) 5 Speed of boat = x .
(B) 4 Speed of the stream = y.
(C) 3 x+y=8
(D) 6 x-y=2
Solution: 2x = 10 or x = speed of the boat is 5 km/h.
3 2 3 ?
9 × (81) ÷ (27) = (3) 5 + y =8
Or, (32)3 × (34)2 ÷ (33)3 = (3)? y = 3 km/h.
6 8 9 ?
Or, 3 × 3 ÷ 3 = (3) 59. A sum of money at simple interest amounts to Tk. 815
6 -1 ?
Or, 3 × 3 = (3) in 3 years and to Tk. 854 in 4 years. The sum is-
5 ?
Or, 3 = (3) (A) Tk. 650
Or, ? = 5 (B) Tk. 698
55. Which of the following is the lowest ratio? (C) Tk. 690
(A) 15 : 23 (D) Tk, 700
(B) 7 : 15 Solution:
(C) 17 :25 (4-3)= 1 year interest = (854 815) = 39 tk
(D) 21 : 39 3 year interest = 3 × 39 = 117 tk
56. 0. 213 ÷ 0. 00213 =? The sum = (815 117) = 698 tk
(A) 1 60. The average of 2,7 and x is 5 and the average of 18, 1,
(B) 100 6 , 5 and y is 10 . What is the value of y?
(C) 10 (A) 5
(D) None of these (B) 10
57. A can complete a certain work in 4 minutes, B in 5 (C) 30
minutes, C in 6 minutes, D in 10 minutes and E in 12 (D) 20
minutes. The average number of units of work completed Solution:
by them per minute will be In case 1, The average of 2,7,6 and x is 5
(A) 0.40 (2 + 7 + 6 + x)/4 = 5
(B) 0.16 (15 + x)/4 = 5
(C) 0.80 15 + x = 20
(D) None of these x= 5
Solution: In case 2, the average of 18, 1, 6,x and y is 10
Required average, So, (18 + 1 + 6 + x + y)/5 = 10
= (1/4 + 1/5 + 1/6 + 1/10 + 1/12)/5 25 + x + y = 10

160
Collected By: Md. Raihan Ahmed
Job Information For All Students (Bcs & Bank)

25 + 5 + y = 10 [putting the value of x=5] men would be required to do the same work in 20 days?
y = 20 (A) 24
61. Which of the following has most number of divisors? (B) 12
(A) 99 (C) 36
(B) 101 (D) 48
(C) 182 Solution:
(D) 176 Finish the work in 16 days men needed = 30
Solution: Finish the work in 20 days men will need=(16 × 30)/20= 24
99 = 1 × 3 × 3 × 11 65. A man riding his bicycle covers 150 metres in 25
101 = 1 × 101 seconds. is his speed in km per hour?
176 = 1 × 2 × 2 × 2 × 2 × 11 (A) 20
182 = 1 × 2 × 7 × 13 (B) 23
So, divisors of 99 are 1, 3, 9, 11, 33, 99 (C) 21.6
Divisors of 101 are 1 and 101 (D) 25
Divisors of 176 are 1, 2, 4, 8, 11, 16, 22, 44, 88 and 176 Solution:
Divisors of 182 are 1, 2, 7, 13, 14, 26, 91 and 182 Speed = 150/25; = 6m/s.
Hence, 176 has the most number of divisors. Speed in km/hr = 6 × 18/5 km/hr; = 21.6 km/hr.
62. The sum of twice a number and three times of 42 is 66. (? − 968) ÷ 79 × 4 = 512
238. what is the sum of trice the number and two times of (A) 10185
42? (B) 10190
(A) 252 (C) 11080
(B) 250 (D) 11075
(C) 245 Solution:
(D) 264 (? 968) ÷ 79 × 4 = 512
Solution: (? 968) ÷ 79 = 512/4
Let the number is x (? 968) ÷ 79 = 128
ATQ, ? 968 = 128 × 79
2x + (42 × 3) = 238 ? = 10112 + 968
Or, 2x = 238 126 ? = 11080
Or, x = 112/2 67. 19. 99 × 9. 9 + 99. 9 =?
Or, x = 56 (A) 129.79
Now = (56 × 3) + (2 × 42) = 252 (B) 296.910
63. Find the multiple of 11 in the following numbers. (C) 1009
(A) 112144 (D) 297.80
(B) 447355 Solution:
(C) 978626 19.99 × 9.9 + 99.9
(D) 869756 = 197.901 + 99.9
Solution: = 297.801
If the difference of the sum of number of odd position and = 297.80
even position = 0 then it will be divisible by 11 68. If you subtract -1 from +1 what is be the result ?
In 978626, we have (6 + 6 + 7) - (2 + 8 + 9) = 0 (A) -2
Hence, 978626 is completely divisible by 11 (B) 2
64. 30 men can do a piece of work in 16 days. How many (C) 1

161
Collected By: Md. Raihan Ahmed
Job Information For All Students (Bcs & Bank)

(D) 0 Profit = 7%
Solution: ∴S.P.= Tk.(107/100 × 7840) = Tk. 8388.80
1 (-1) = 2 73. A train 108 m long moving at a speed of 50 km/hr
69. The ages of A and B are in the ratio 6 : 5 and sum their crosses a train 112 m long coming from opposite direction
ages is 44 years. What will be the ratio of their ages after 8 in 6 seconds.The speed of the second train is
years? (A) 48 km/hr
(A) 8:7 (B) 54 km/hr
(B) 7:6 (C) 82 km/hr
(C) 9:8 (D) 66 km/hr
(D) 3:4 Solution:
Solution: Let speed of the second Train be x km/hr
Let ages be 6x and 5x
so, 6x + 5x = 44 Relative speed
or, 11x = 44 = (x + 50) km/hr
or, x =44/11 = (x +50)× 5/18 m/s
Or, x = 4 = (250 + 5x)/18 m/s
So, the ages = 24 and 20 Distance covered = 108 + 112 = 220m
Ratio after 8 yr = (24 + 8)/(20 + 8) = 32/28 = 8/7 ATQ,
70. Half of 1 percent written as a decimal is (220 × 18)/(250 + 5x) = 6
(A) 0.05 250 + 5x = 660
(B) 0.005 x = 82 km/hr
(C) 0.02 74. If the square of an odd natural number is divided by 8,
(D) 0.2 then the remainder will be
Solution: (A) 1
1/2 × 1/100 = 0.005 (B) 2
71. 6 × 3(3 − 1) = ? (C) 4
(A) 19 (D) 3
(B) 20 Solution:
(C) 53 The odd number can be represented as : 2n + 1, where n
(D) 36 is any natural number
Solution: Now square of odd number = (2n + 1) ^ 2 = (4 n ^ 2 ) +
6 × 3(3 − 1) (4 n) + 1 = [ 4 n (n + 1) ] + 1
= 6 × 3(2) Now n(n + 1 ) is always an even number, let it be 2m
=6×6 (where m is any natural number)
=36 So above equation can be written as :-
72. The cost price of an article is Tk. 7840. What should be 4n × 2m + 1 = 8 nm + 1
the selling price of the article so that there is a profit of 7% So the remainder Will be 1 when divided by 8..
(A) Tk. 8388.80 75. P and Q started a business investing Tk. 85000 and tk
(B) Tk. 8300 15000 respectively. In what ration the profit earned after 2
(C) Tk. 8000 years be divevded between P and Q respectively ?
(D) Tk. 8500.50 (A) 3:4
Solution: (B) 3:5
C.P. = Tk.7840

162
Collected By: Md. Raihan Ahmed
Job Information For All Students (Bcs & Bank)

(C) 17:3 (D) Bhawal National Park


(D) 15:23 83. Which is the smallest district in Bangladesh in items of
Solution: area?
P : Q = 85000 : 15000 = 86 : 15 = 17 : 3 . (A) Meherpur
(B) Narayangonj
GENERAL KNOWLEDGE & COMPUTER (C) barguna
76. UNESCO was established in- (D) None of these
(A) 1919 84. When was the Constitution Bangladesh adopted?
(B) 1945 (A) 4 November 1972
(C) 1957 (B) 26 March 1972
(D) None of these (C) 16 December 1972
77. The total number of members of UN Security Council is (D) 10 January 1972
(A) 5 85. What is the ratio of the length and breadth of
(B) 10 Bangladesh flag?
(C) 20 (A) 10:5
(D) 15 (B) 10:6
78. Which country will host the 2019 Cricket World Cup? (C) 9:6
(A) England & Wales (D) 9:7
(B) South Africa 86. According to CERB, What is the rank of Bangladesh
(C) West Indies economy among the largest economic in the world in 2019?
(D) New Zealand (A) 43
79. When was the Bangabandhu I satellite launched (B) 42
successfully into the space (C) 41
(A) May 12, 2018 (D) 44
(B) May 10, 2018 87. Which one is the natural satellite of earth?
(C) May 14, 2018 (A) Atlas
(D) May 16, 2018 (B) arial
80. What is the average life expectancy in Bangladesh? (C) Oberon
(A) 65 years (D) Moon
(B) 68 years 88. Who invented the computer mouse?
(C) 72 years (A) Douglas Engelbart
(D) 70 years (B) Alan Kay
81. Which sector has highest contribution to the GDP of (C) Ted Nelson
Bangladesh? (D) Vennevar Bush
(A) Service 89. Which is known as the Land of the Rising Sun?
(B) Agriculture (A) New Zealand
(C) Maufacturing (B) Korea
(D) None of these (C) Japan
82. Which one is known as the mangrove forest Bangladesh (D) Norway
? 90. When is Human Rights Day observed?
(A) Lawahcara National Park (A) December 15
(B) Ratagul Swamp Forest (B) December 10
(C) Sundarbans (C) December 3

163
Collected By: Md. Raihan Ahmed
Job Information For All Students (Bcs & Bank)

(D) December 2 96. Who is the father of modern computer ?


91. Who is popularly known as Gandhi' of Sri Lanka? (A) Charles Babbage
(A) Patrick Medis (B) Alan Turing
(B) Glen D. Paige (C) Simur Cray
(C) A.T ariyaratne (D) Augusta Adamin
(D) Sulak Sivaraksa 97. What is the GDP per capital of Bangladesh in USD?
92. How many stipes are there in the US flag? (A) 1751
(A) 13 (B) 1650
(B) 7 (C) 1675
(C) 6 (D) 1505
(D) 12 98. Nobel Prize in economics in 2018 is awarded to
93. Who was the first governor of Bangladesh Bank? (A) Richard Thaler
(A) Mohammmad Nurul Islam (B) Paul Romer & William Nordhaus
(B) ANM Hamidullah (C) Oliver
(C) Khorshen (D) Peter Diamond
(D) Latfur Rahman Sarkar 99. What is the name of the currcency of Myanmer ?
94. DPI stands for (A) Kyat
(A) Digits per inch (B) Ngultrum
(B) Dots per inch (C) Rupee
(C) Dots per inch (D) Dong
(D) Diagrams per inch 100. The Padma bridge will have a total of ____ spans.
95. WWW stands for (A) 36
(A) World Whole Web (B) 41
(B) Wide World Web (C) 39
(C) World Wide Web (D) 45
(D) Web World Wide

Job Information For All Students (Bcs & Bank)

164
Collected By: Md. Raihan Ahmed

Potrebbero piacerti anche